Sie sind auf Seite 1von 259

NBME

Form 1 Section 1

7. A SO-year-old man has a 10-year h istory of poorly controlled hyperrension. Vital signs are:
Pulse
Respirations
Blood pressure
Left a,m
Ri ght arm

96/min
16/min
2261120 mm Hg
2181118 mm Hg

With the patient in the left lateral decubirus position. a late diastol ic sound is heard best with the bell at the apex. Which of the following is
the most l ikely explanation for this auscultatory finding?
O A) Aortic insufficiency
O B) Aollic stenosis
0 CJ Mitra! insuffic iency
O D) Mitra! stenosis
O El Open ing snap
O F) Pulmonic insufficiency
O G) Pulmonic stenosis
0 H)S3
0 l)S4
0 J) Tricuspid insufficiency
8. In gout. hyperur icem ia resulling from overproduct ion of urate is most l ikely to result from a lack of feedback inhibit ion by ADP or GDP at
which enzyme?
O A) Adenosine deaminase
O 8) Dihydrofolate reductase
O C) Glucose 6-phosphate dehydrogenase
O DJ Nucleotide phosphol)1ase
0 E) Phosphoribos','lpyrophosphate S'/flthetase
9. A female neonate has protound hypotoria. The only known complication of pregnancy was p otiflYdramni os. The mother has a tack of
faci al e,,pression and weak muscles; she says she did no! have any problems as an infant 0< chld. Which of lh8 following best explams
the difference in preserurions in the mother and chi ld?
0 A) Anticipation
0 BJ Delayed onset
O C) Geneoc heterogeneity
O D) Incomplete penetrance
0 E) Ple1otropy
1O An atcohodependent 40-year-old woman ""10 is e health-care worker has had episodic sympt0<ns of llypogl)'cema for the past year
During one of these eposodes, plasma glucose concentranon 1s decreased. plasma insulin conc;-atl on Is Increased. and serum
CpepOde concentration is not detectable Which of the folowing is the most lkefy cause of the hypoglycemia?
O A) Acute hepatic failure
0 B) lngesbon of a suttonyuea
O C) Pancreatic I> cell neoplasm
0 0) Retrop8tltoneal f1bros.l'c0<na
O E) Surreptitious use of insuMn
11. Double-stranded DNA can be denatllfed by heating and renarured by cooing. RenaOJration under conditions of high stringency has
which of the following effects?
0 A) Considerab le base rnsmatching 1s permitted
O 8) GC base pairing is irhbited
O C) GT base pairing 1s promoted
O D) IVlsmatched DNA t,,tlridization is prevented
12 A 22-year-old woman who is pregnant Y<i1h her first child c0<nes to the physician's office for a routine visit et 24 weeks' gestallon The,
size other uterus has Increased very lltlle during the past4 weeks. Ul trasooography shows htlle fluid in the armlotl c sac. Which of the,
following fetal organ systems is most likely to be mauncboning?
O A) Carclovascular
0 6) Ceroat nervous
O C) Gasuointestlnel
0 O)HepatJc
0 E)Renal

13 A 35-year--Oid woman comes to the pt,,,sician for an initial visit She refuses to fill out a questionnaire lonn about personal data and her
medical history, then sits sllentty in the waiting room with her fists c lenched.When the physician greets her in the e.Xamination room. she
says, "Let's get this overwith." Which of the following questions is most liketyto facilitate communicationwith !he paient?
O A) "Are yoo al ways such an angry person?"
0 BJ "I can see that you ere upset Please tell me what ,s bothenng you."
O C) "01<. Teti me what bnngs you in."
O DJ "Should we reschedJe your appoimnent for when you are in e bener mood?"
0 E) ''Wm/ are you angry with me?"
O F) "You knOw. you do no1 have to be here if you do not want to
14 An SO.year-old woman comes to the physician because or a 2-week hislory ol m ild constipat ion end occasional inability to comp letety
empty the bladder. She takes over-the-counter medications frequently. Vital signs are normal. Evaluation shows bladder distention and
mi ld confusion Which of the follOwing drugs is the most likely cause ol her symptoms?
O A) Cimetidine
O B) Dtphenhydramine
O C) Docusete sodium
0 0) Gua,rene51n
0 E) Ibuprofen
15. A 27-yeer-old man is round to have a sol itary thyroid nodule dur ing a routine health mainte nance examination. A thyroid scintiscan shows
uptake of radioactive iodine by the nodule. Which of the following is the most likety diagnosis?
0 A) Chronic autoimmune (Hashimoto's) thyroiditis
O B) Folli cular adenoma
O CJ Granulomatous thyroidi tis
O 0) Medullary carcinoma
O E) Papilla carc inoma

I
16 A 32-year-old man with AIDS has a &.week h istory of progressive weakness of his ri!tlt hand. expressive aphasia. and a mild
headache. A CT scan of the head with contrast Is shown. Which ol the follOW1ng Is the most likely causal orga111sm?

0 A) Clyptococcvs neoformans
0 B) Cytomegaloirus
0 C) Pneumocys/lS cann11
0 D) Toxoplasma gondu
0 E) Treponema pa/lidum
17 A SO.year-Old woman is eva luated for occul t blood in tile stool. Colonoscopy shows a 5-cm sessile mass 'nitll a velvety surface that
pro1ec1s abOll 2 cm above the mucosa of the rectOS1gmoid. Microscopic exam,nabon ol bssue obtained on biopsy of the mass shows
papillae composed of delicate hbrovascular cores covered bydyspiasllc columnar epithelium. This paoert 1s at increased nsk for which
of the following colonic neoplasms?
O Al Adenocarcinoma
O B) Carc lno l d
O C) Fibrosarcome
O DJ Leiomyosarcoma
O E) L'!""phoma
0 F) Squamous carcinoma

18 Relative 10 Ille normal values for pare meters Iha! govern cepilaly exchange. which ol lhe following is Ille mos1 l iketycause of the edema
that resuls from use of a venous occlusive 1oum1quet?
O A) Decreased mtersnt,al flld hydrostatic pressure
O B) Increased mean capi llalyllydrostatic pressure
0 C) lncreasoo p lasma onconc pressure
O 0) Increased cepille,y filtreoon coefficient
O E) lncreasoo capilla,y reflocllon coefficient for plasma proteins
19. A 72-year-old man comes to the physician because of an 8 -month histo,y of feeling l ight-headed when he stands up. Blood pressure is
130185 mm Hg while supine and 94/64 mm Hg whi le standi ng. High-dose Rudrocortisone therapy is started. After 5 days of treatment.
laboratory studies are most likely to show which of the following changes?
Serum
K
0 A) L
0 B) L
0 C) L
OD) r
0 E) f
0 F) f

Urine

Renin

K excretion

f
f
L
f
L
L

f
I
t

20 . A sexualty active 18-yea r -old woman comes to the physic ian's office because of fever. lower abdom ina l pa in. and a purulent vag i nal
discharge for the last 3 weeks. A tender mass is palpated in the right lower quadrant on bimanual examination. Which of the following is
the most l ikely site of the mass?
O A)Ova,y
O B) Fallopian tube
O C) Uterine tundus
0 D) Uterine bocty
0 E)Vagina
0 F)Bladder
0 GI Rectum
2 1. A 13-montf>.ol d grtl has a history of recl.l'Tent bactenal mfecnons. Producnon of o, .- H20,. and OH by neutroptols 1s normal and
prockJclion of HOCI is decreased fol owing
stimulation byopsonized bacteria. The most liketycause is deficiency of
l
O A) catalase
0 B) gluta1h1one perox idase
O C) peroxidase
O 0) NAOPH oxidase
0 E) superoxide dismutase
22. A 50-yoor-o l d man with angina! pain takes nitroglycerin The pa in is rapid!/ re lieved becalse of
O A) inhibition of phosphOipase C act i'lity
O B) sbroolabon of cyclic AMP formabon

O CJ stirru lation of cyclic GMP formation

O 0) sbrrulabon of mos ,tol 1.4.5-trisphosphate formation


O E) slirrulalion of protein kinase C acli'lity
23. A 62-year-old man is brought to the physician for eval uation of a 6-month h isto,y of progressively strange behavior. He has been
impulsive and distractible. has stopped bathing and combing his hair . and has become uncharacteristicalty rude and unkempt.
Occasionally, when eating, he has stuffed food into his mouth with his fi ngers. He has had some mi ld problems with memory duling this
period. His onti-medical problem is a 1-year histo,y of hypertensi on well controlled with atenolol. Neurol ogic examination shows no focal
signs. but the grasp and snout reflexes are prominem. Which of the following is the most likely cause of this patient's condition?
O A) A1teriosclerosis
O BJ Dement ia. AIZhe imers type
O C) Huntington's disease
OD) Normal aging
O E) Pitk's disease

24 A 4-mon!ll-Old boy is admitted to the hOspital with d isseminated candidiasis He has had s_a, episodes olupper respiratoryviral
infecbons and has a h1sto,y of tetany. Two months ago. his serum calcium concentration was 2.0 mg/dL. Truncus aoteriosus was
diagnosed. He was gr;en.,,tamin D end celc1um supplelll8llS end was discharged et 1 morai of age. wr11ch oftile following s81S of
laborato,y findings 1s most like I)' at this bme?
Gamma
globul ins
O A) Decreased
OB) Noonal
OC) Noonal
OD) lllereasad
OE ) lllereasad

Per ipheral
hotytes
decreased
decreased
decreased
increased
increased

Mediastinal
shadow on
x.rayhlms
narrowed
widened
narrowed
widened
narrowed

25. The destruction of 1-12<>2. which is a mildtitoxic reduced oxygen species. is catalyZed DfWhich of the folowing enzymes1

0 A) Calalase
OB)Cytochrome oxidase
0 C) Nitric oxide synthase Onduci ble)
O D)Superoxide dismutase
O EJ Xantlline oxidase
26. The preva lence of breast cancer is compared in two groups of women based on parity. The following daca 8'e obta ined at age 70 in
both groups:
....

NO

120

180

1380

820
1000

Based on these data. what is the re le!JVe risk (risk rabo) for development of breast cancer m chi dl less women compared withwomen
who have children?
0 A)0.67
0 B) 12
0 C) 1.5
0 D) 1.8
0 E)22S
27. A 55-year-old man has had patcl'rf leukoplakia of the anterior f loor of his mouth for 4 years. He now has a raised, firm whi te lesion in the
same area. Examination of tissue obtained on biopsy of thi s new lesion is most l ikely to show
0 A) aphthous ulcer
OB)Burk itt's lymphoma
0 CJ Candida infection
0 DJ Kaposi's sarcoma
O E)m ixed tumor of the parotid 9land
O F)squamous cell carcinoma
O G)Walthin's tumor
28. A 31-year-old woman is admitted to the hOspitat in labor at 28 weel<S' gestation Administrat ion ol...ciich of the fol!CW<ing will most like!)t
enhance production of smactant by fetal type D alveolar ceDs?
O A) Corticosteroids
0 B)Glucagon
OC)Growth hormone
0 D)lnsuin
0 E) Somatostatin
29. A previously heah<t 56-year-old man has the sudden onset of severe headaches that become worse in the aitemooos. His blood
pressure is 180/110 mm Hg. Renal arteriogaphy shows 85% occlusion ofthe right renal artery. The laboratory findng that best
predicts whether an operalion that relieves the obstruction wil reduce the blood pressure to normal is increased rerin activity in the
O A)lei! renal arteryvs Ii renal arteJ\I
O BJ left renal vein vs right renal ve in
O C) rigtt renal artel\' vs left renal artery
ODJ rigtt renal ve in vs left renal vein

30 A neonate has ambiguous gootaha. In lhe firSI 48 hours of life. Ille neonate feeds poorly and becomes del'r)'drated Which ol lhe
following ,s the most fikely d1agnosis?
O A) Androgen msensibvity syndrome
O B) Sa-Reductase deficiency
0 C) 17-Hydroxylase deficiency
0 D) 21-Hydroxylase delic1ency
0 E) True hermaphrod itism
31. A 4-year-ofd girl is brought to the emergency depa11mem because of a S.day history of fever. diffuse abdomina pain. nausea. vomiting,
and diarrhea. Examination shows petechiae. hE;1Jatomegaly. and splenomegaly. Hemoglobin concentralion is 8.7 gldl and platelet count
is 60.000/mm3. A peripheral blood smear show; el)llhrocyte fragmentation. Which of the following renal finding; is most likely to be
associated with her cond ition?
O A) Glomerular crescent formation
O BJ lgG-assoc iated glomerular basement membrane antibody
O C) Membranoproliferative glomenulonephritis
O DJ Narrowing of the capillary wall by fibrin depos!ion
O E) Periglomerular granuloma with gi ant cells

32 The developmental anomaly shown in this stillb orn fetus is mosr commonly CherecreriZed by which type of inheritance?
O A) Autosomal dominant
O B) Autosomal recessive
O CJ Mtochondriel
O D) M.Anfactonal
O E) Sex-linked

33 A 30.year-old woman with type 1 d iabetes mell1tus has mcreasmg .uvar pruri tus and a vaginal discharge. A wet mot111 preparaoon of
Iha discharge is shown. Which of the follOl'llng ,s the most hkelycausal organ ism?
0 AJ Cafldida albicans
0 B) Chlamydia traehomafis
O C) Herpes simp lex vinus
O 0) Human pepillom8Vlrus
0 EJ Trichomona svaginalis

34 The presence orwllich of the following is characteristic ol the infecti ous agent of bovine spongifomn encepllatopatlly?
RNA

ONA

Protein

+
0 A) +
0 B) +
+
OC )
00) +
OE)
+
F)
0
OG )
0 H)
+
35. A healthy 30-year-old man is pallicipating in a study of the effects of a compet it ive inhibi tor of endothelial nitric oxide synthase. After
intravenous injection of the inhibitor. blood pressure immediately increases. Supplementation with which of the following amino acids is
most appropri ate to reverse this increase?
O A) Arg i nine
o Bl Aspartic
acid
OC) Glutamic acid
OO)Leucine
0 E)Lysine
O F) Methionine
0 G) Tyros ine

36. A 48-year-old man w itll a long histo,y of alcohol abuse has painless vomiting ol age amounts ol bright red blood. Heart rate is
110/min and blood pressure is 80/0 mm Hg. He dies despite rapid transfusion olpacked eiythrocytes. At autopsy, the source of the
hemorrhage is round in the lower esophagus. A photograph ol the site is sh<Wffi. Which of the follol,ing mechanisms most l ikely
caused the hemorrhage?
O A) Alcohol-induced esophagitis
O B) Deficiency in hepatic S)<lthesis of vitamin K-dependent coagulation proteins
OC) Mechanical tearing of the esophageal roocosa during vomiting
O0) Pepoc utceration olesophageal mucosa due t o reflux of gasttic acid
O !:) Transmission ol portal twertens,on to esophageal veins
37. A 25-year-old woman has had generalized bone pain for 9 years. She has frequent episodes of loose bulky stools. Examination shows
muscle weakness and a distended abdomen. X-ray films show general ized demineralization of bones. absence of the lamina dura. and
fracture-like lines in the bones of the feet. Which of the following sets of findings in serum is most Ii kely in this pat ient?
Ca lcium

OA)
OB)
OC)
00)
0 El

I
f

Phosphorus
l
f
f

Alkaline
phosphatase
f
f
f
l

Parathyroid
hormone
f
l
f
l

38. An 18-year-ol d coll ege freshman comes to the clini c at her parents' urging for eval uati on of a 6.8-kg (15-l b) wei ght loss. Over the past 4
months, she has had increasing di fficulty concentrati ng on her schoolwork and has begun to miss cl asses. She avoids friends and often
skips meal s wllile watchi ng tel e\'ision al one in her room. She sl eeps fitful ly and often awakens at 5: 00 MA and is unable to go backto
sl eep. Her wei ght is currently in the l ow-normal range. She agrees that something is wrong and feel s guilty about her academi c
pelformance. Whi ch ofthe followi ng is the most li kel y diagnosis?
O A) Anorexia nervosa
0 B) Bul imi a neivosa
O C) Conversion reacti on
0 0) Major depressi ve disorder
O E) Obsessi ve -compul sive disorder
O F) Schi zophreni a
39. A 42-year-old woman has dizziness and ri ngi ng in her ears 7 days after stalling a drug.Which of the folla;.;ng drugs is the most l ikett
cause?
O A) Acetaminophen
0 B)Aspilin
0 C) Auranofin
0 0) Meltlotrel<llte
O E) Predni sone

"

s.,
:,:


WEight (kg)

40. The scatter di agram shown records the hei gtt and w01ght of a random sampl e ol mal e coll ege students. The correlabon (r) between
height and wei ght i s ctosest to Whi ch of the follOINing?
0 A) +1.00

0 B) +0.60
O C)O
0 0)-0.60
O E) -1. 00

41. Serious. possiblyfatal. adverse effects ola new drug mayfirst appear only after the drug is approved for ma11<eti ng by the Food and
Drug Admi ni strati on. The fai lure to detect these eff9Cls is best expl ained bythe use ofmch ofthe foll owing in the premarkeong cli nical
trl ats?
O A) Heallhy rather than ii subjects
O B) Lower doses of !he drug than are approved for ma11<etmg
O C) Patients with pos1 ove outcomes only
O D) Placebos
0 E) Too few sooiects to detect rare events
42 Wilch of the follOwing Jl(Ocedures improves the immunogenici ty ofvacci nes that use capsular polysaccharides as eniigens?
0 A) Heat dena11sat ion
O B) Uposome mcorporanon
O C) Oligosaccharide reducti on
O D) Protei n tQr4ugabon

43. The venebral angi ogram shOwn is from a 40-year-o l d man. The abnormal ity is marked by "X." Whi ch of thefollowing cl ini cal findi ngs is
most l ikel y to be present?
O A)Aphasia
O BJ Ataxi c limb movements
O C) Resti ng tremor
O D) Spastic hemiparesi s
O El Vi sual fi el d defect
44.
!),Go
v - w -4.0
w - x +0.4
X - Y -3.4
+5.T
Y-Z
Fromtheval ues of 6Go gi ven in the tabl e. v,nich compound in the overall metaboli c pathway v -w-x- Y -Zwi ll accumulate in the
largest amount at equi libri um?
0 A)V
O B)W
O C) X

O O) Y

0 E)Z

45. The table shows the distri buti on of spinal cord injuries and deaths byiype of accicleot
Twe of acci dent
Highwaytraffi c
Fals
Recreati onal
Occupational
Assaun
Olher
Total

54

,
,

6
3

69

Fatal

All

%'

%=

78.3
5.8
1 .4
8.7
1 .4
4.3
100.0

101
20
12
11

65.6
13.0
7.8
7.1
3.2
3.2
1000

s
s

154

N =number
= % of all spinal cord injuries
" = % of an deaths
i

Based on these data. What is the case.fatal ty rare for recreational spinal cord injuries?
0 A)1/12
0 8) 1113
0 C) 1A>9
0 D) 1.4fl.8
0 E) 7.1/8.7

46. A 4yeer-ol d man....tlohas fi ve chi ldren comes to the physicianto di scuss a vasectomyand asks aboutwhalto e:,;pect. Which of the
following 1s most l ikel-/to occur duri ngthefirst 4 weeksfolowing the proceaxe?
O A) Atrophy ofthe tesres
O 8) Erectile disorder
O C) Numbness of the posterior scrotum
O D) Presence ofviable sperm in ejaculate
0 E) Significantly decreased vol ume of ejaculate

47. Activation of T lymph<>C){es is ini tiated v.hen processed antigen bi nds co Ille T lymphocyte receptor on the eel membrane. and
phosphol1 pese C 1s ac1JV8ted Thi s resulls in producti on of whi ch of the folowtng componencs of signal transoo<:bon?
0 A)CydcAMP
0 8)CydcGMP
O C) Oiacytgl)'cerol
O D) lnteeukln.2 (I L-2)

48 A 70.yeer-ol d woman Ms increasi ng weakness. weight l oss. generehzed lymph node enl ergemenl. end hepetosplenomegely.
Laboratory studi es showabsol ute l ymphocytos,s with the maionty of cells bei ng CDS. The pabert also has a mild aMglobuhn-posi bve
hemol',1ic anemi a. l<aryo!ype is normal. Which of the following 1s the most li kely dagnosi s?
O A) Acute 1-frnphobl asti c leukemi a
0 8) Acute promyel ocyti c fel.kemi a
o C) Chtoric lymphocyti c leukemi a
O 0) ChtORc myelo1d leukemi a
O E) Fanconi's anemi a
O F) Hai ry can leukemi a
O G) IVtyelod\lSplasti c S)'lldrome
O H) Myeloid metapl asi a with myelofibrosis
O I) Polycythem1a vera


49. Neutrophi ls produce reactive oxygen compounds followi ng the phagocytosi s of bacteria. Which of the following enzymes protects bocly
ti ssues against these hi ghly reacti ve oxygen compounds?
0 A) Cycl ooxygenase
O 8) Lipoxy'genase
O C) IVlyel operoxi dase
O D) Superoxide di smutase
O E) Xanthi ne oxi dase


50. A yeer-old manwith a long hi slory of poooy controlled hypertension suddenly devel ops severe retrosternal chest painthat radiates
into the t.f)per back between the scapul ae. On exami nation he has di aphores,s and severe shortnessof breath. Pulse is 125/min and
regular. and bl ood pressure is 130/60 mmHg inthe l elt armand 80/50 mm Hgin the ri ght arm. The<eare diffuse. wet. inspi retory
crackles m both lung fields. di stant heart SOtllds. and a pre111ousl-/ undetected grade 216. blowing, eattdi astoli c nurru in the second
ri ght end third l eft imercostal spaces adjacent to the sternum The most likely Cllagiosi s is
O A) di ssecting aorti c aneurysm
1B
21C
O 8) myocardi al infarcti on
2C
22C
38
23E
O C) pericardi tis wi th temponade
4E
24C
O 0) pulmonary embol ism
5C
25A
0 E) rupn,-ed chordae tend1neae
6A
26E
O F) ventncular septal rupture
27f
7)1

SE 28A





9A
JOE
1 10
12E
138
148
158
160
17A
188
19C
208

290
300
31D
320

33A

3411
35A
36E
37A
380
398
408

41E
420
438
440
4SA
460
47C
48C
490
50A

NBME
Form 1 Section 2

6. Ei ght hundred motorvehicl e colli si ons occurred over 2 years in a city with a popul ati on of 600,000; 600 of these acci dents were
al cohol -related. During the same period. there were 400 fatal ities due to motorvehi cle collisi ons. of1Ntii ch 75% were alcohorel ated.
Which of the follovnng best represents the 1-year incidence of al cohol ,rel ated motorvehicl e fatal iti es in thi s communi ty?
O A) 150 per 600,000
0 B) 300 per 600,000
O C) 450 per 600,000
0 D) 600 per 600.000
O E) Unabl e to determi ne from availabl e infom,ation
7. Which of the following hormones acts through receptors that contain tyrosine kinase?
O A) ADH (vasopressi n)
O B) Al dosterone
O C) Cal ci toni n
O D) 1,25-Di hydroxychol ecal ci ferol
O E) Epi nephrine
O F)lnsul in
O G) Parattryroi d hormone
O H) Thyroxi ne (T4)
8. A 39-year.ol d woman wi th rheumatoi d arthriti s devel ops protei nuria altertreatment wi th penicillami ne. Exami nation of tissue obtai ned on
renal bi opsy shows di ffuse fi ne granular deposi ts ofl gG and C3 in the gl omerul ar basement membrane. The most likely diagnosi s is
O A) focal segmental scl erosis
O B) Goodpasture' s syndrome
O C) l gA nephropathy
0 D) membrc,nous g1 omerut onephri ti s
0 E) mi nimal change di sease
9. Antibodi es that can pass from the mother to the fetus through the placenta are of1Ntii ch class?
0 A)lgA
0 B) l gD
0 C) l gE
O D) l gG
0 E) l gM
10. Latent infecti on with which of the following vi ruses is associ ated with expressi on of some (but not all) viral genes in B t-JmphocYtes?
O A) Epstei n.Barr vi rus
O B) Hepati ti s B vi rus
O C) Measl es virus
O D) Si mi an virus 40
O E) Vancella-zostervirus
1 1 . A 28-year.ol d woman is broughtto the emergency department alter a motorvehi cle accident in whi ch her chest struck the steering
1Ntieel. Pul se is 125/mi n and bl ood pressure is 90/60 mm Hg. She is di aphoreti c and has dyspnea on exerti on. Head and neckvei ns
are di stended. Heart sounds are di stant and there are no mum,urs. The radi al pul se pressure decreases abnom,al ty duri ng inspirabon.
The most l ikely di agnosi s 1s
O A) cardi ac tamponade
O B) hemorrhage from a ruptured descendi ng aorta
0 C) mi tral valve prol apse
O D) pul monary embolism
0 E) ruptur of a papi llary musct

12. Alter a full course of immuni zati on with a new vacci ne consi sting of a recombi nant polypeptide. 10% of adul ts fai l to make anti body to the
polypeptide. The nonresponders have an increased frequency of one HLA type. What is the most likely expl anation for the fail ure of thi s
group to respond to immuni zati on?
O A) Fai lure of B t-JmphocYtes to recogni ze the pol ypeptide
O B) Fai lure of T l ymphocytes to recogni ze the polypepti de
O C) Lack of cl ass I MHC presentati on of the polypepti de
0 D) Lack of cl ass II MHC presentati on of the polypepti de

13. A neonate is born with the abnormal ity shown. Abnormal t1 robust acti vi ty ofwhi ch of the followi ng processes is the mostl ikelf cause of
thi s mal formation?
O A) Angi ogenesi s
O B) Cell adhesi on
O C) Cell migrati on
O D) Gaso-ul ati on
O E) Somitogenesi s

14. A 73-year-ol d woman has a 1-cm. seal/. rai sed lesi on on the right side of the face. A photomi crograph of tissue obtai ned on bi opsy of
the exci sed l esi on is shown Use of whi ch of the following 1s most l ikelyto have prevented the devel opment of thi s l esion?
O A) Conjugated estrogen
O B) lsoni azid
0 C) Sunscreen
O D) Treti noi n cream
O E) Triamci nol one cream
t S. V"1li ch ofthe followi ng is the most likely consequence of impai red synthesis of interteukin.1 (l l-1)?
O A) Enhanced phagocy1ic activi ty of ci rcul ati ng macrophages
O B) Enhanced production of antiboclf by B lymphocytes
O C) Impai red antigen processi ng by ci rcul ati ng macrophages
O D) Impai red function of T lymphoC)'tes
O E) Impai red producti on of l eukotrienes by neutroph1 ls

16. The drawing is of a cross secti on through the trunk of a human embryo at 5 weeks' gestati on. Whi ch label ed structure contains muscl e
precursor cells deri ved from the somi tes?

0 A) 0 B) 0 CJ O 0) 0 E) 0 F) 0 GJ O HJ O I )
17. A 17-year-ol d boywith moderate mental retardati on has macro-orchi di sm. Cytogeneti c analysi s shows a fragi le si te on the long arm of
chromosome X. Which mol ecul ar defec1 is most likely associated with thi s abnormali(Y?
O A) CGG repeat expansi on
O B) Exon del eti on
O C) Gene amplifi cati on
u D) Point mutaoon
0 E) Spl ice si te mutation
18. A full-term 1-week-ol d mal e neonate wi lh a crani ofaci al deformi ty and pul monary hypoplasi a has not produced uri ne. Labor and delivery
were uncompl icated. Hi s mother has hypertensi on and tookvanous promotional sampl es of antilrypertensi ve medicati ons throughout the
pregnancy. Whi ch of me foll osong anti t()'Pertensi ve drugs most l ikel y caused the abnormali ty?
O A) Atenol ol
O B) Di lti azem
O C) Enal apri l
0 D) Hydrochl orothi azide
0 E) Methyldopa
O F) Verapami l

19. Examinati on of me l ungs al autopsy shows nodul ar areas of chroni c pneumoni ti s wi th necroti zi ng aeri tis and focal collections of
epitheli oid hi sliocytes. The most likely di agnosi s is
O A) desquamative intersti tial pneumoni a
O B) Goodpasture' s syndrome
O C) hi sti oosi s X
O D) idi opathi c pul monaryfi brosi s
0 E) pul monary alveol ar protei nosi s
O F) sarcoi dosi s
O G) Wegenet' s granut omatosi s
20. A so.year.ol d woman comes to the physi ci an because of a vi si bl e pai nl ess lump in her neckforthe past 6 months. Her pul se is 80/mi n,
respi rati ons are 14/min, and bl ood pressure is 120170 mm Hg. She has an enlarged. nontender thyroi d gl and. Ini ti al lhytoi d function
studies are normal. Her serum thyro1d-somul abng hormone (TSH) concentrati on is 15 Ulml. and serum thyroxi ne (T4) concentrati on is
3.5 g/dl. Whi ch of the followi ng descri bes the most l ikel y pathol ogic fi ndings in her thyroid gland?
O A) Absence of thyrogl obul in
O B) Heavy t,,mphocyte infi ltrate with germi nal center formation
O C) Noncaseati ng granul omas
O D) Presence of an antibodythat si mulates the acti on of TSH
O E) Regenerati ve nodul es wi lh lakes of excess thyroi d proteins
21. A 65-year-ol d woman devel ops a proxi mal deep venous thrombosi s 1 week alter an el ecti ve 1otal hip repl acement. What is me
mechanism of intravenous hepari n in treating !hi s disorder?
O A) Acti vati on of pl asmi n
O B) Faci litation of inactivati on of mrombi n by antithrombin Ill
O C) Inhi bi tion of acti vation of ti ssue pl asminogen
O D) l11l1i 1Ji t.iv1 , vr vi l<:lm i,, l<IJ1::;1t,1 1'-.Jt,1 1l 1,,.vc:1yul c:1li vn rc:i1.lv1
0 E) Reversi bl e inhi bi tion of protei n C and protei n S acti 'iity

22. The anti codon of yeast serirfy1-tRNA is s-AGC-3'. Whi ch of the followi ng is most li kely to be a serine codon?

o A) s-AcG-3'
o B) s.ccG-3'
o q s-Gcu.3
0 D) s.uAG-3'

o El s-ucG-3

23. Whi ch ofthe followi ng drugs is a skel etal muscl e rel axant that woll<s by a di rect acti on on exci tation-contracti on coupli ng rather than an
effect on the central neivous system?

O A) Bacl ofen
O B) Benztropine
O C) Bromocripti ne

0 D) Dontrol ene
0 E) Oi azepam

24. V"1li ch ofthe followi ng cells share a common progenitor cell with macrophages?

O A) Astroc;,tes
O B) Epen(fymal cells
O C) Mi crogri a
O D) MOlle(s fibers

O E) Ol igodendrocytes
25. A cl one of cancer cells is resi st2m to vi ncristi ne. doxorubi ci n, and dactinomyci n but not 10 methotrexate and vari ous al kyl ating agents.
The mechanism of resi stance is most li kelyto involve

O A) absence of superoxi de dismutase


O B) altered DNA pol ymerase
O C) enhanced drug transpol! out of the cell

O D) inabi lity to form polygl ut2mates


0 E) increased DNase

26. A 70-year-ol d man has severe atheroscl erotic disease of the abdomi nal aol!a that obstructs the osti a of the superi or and inferi or
mesenteri c a1 teri es. He has intermi ttent episodes of abdomi nal pai n and bl oody di armea. V"1li ch ofthe followi ng regi ons of the bowel is
likelyto be most severely affected?

O A) Ascendi ng col on
O B) Descending col on
O C) Hepati c fl exure
0 D) Spl emc flexure
O E) Transverse col on
27. Alter taki ng an overdose of propranol ol , a 3S.year-ol d woman devel ops si nus bradycardia and syncope. I n order to stimulate the same
second messenger system that is activated by -adrenergi c receptors. whi ch of the followi ng drugs will be most effective to increase her
heart rate?

O A) ADH (vasopressi n)
O B) Atri al natriuretic pepti de

O C) Cl oni di ne
0 D) Gl ucagon
O E) Pnenyl ephline

28. A 35-year-ol d woman at 18 weeks' gestation has an increased serum a-fetoprotei n concentrati on. Whi ch ofthe followi ng has the
greatest infl uence in determi ni ng the predi ctive val ue of this test for neural tube defects?

O A) Concentrati on of a-fetoprotei n in maternal serum


O B) Maternal hi sto,y of monozygotic twi n pregnanci es
O C) Preval ence of neural tube defects in the popul ati on in question

o D) Sensi nv,tyofthe test

O E) Specificity ofthe test

29. A patient wi th the lesi on indi cated bythe arrow i n the figure is expected to have whi ch of the followi ng types of hemi anopi a?
O A)Bi nasal
O B) Bi temporal
O C) Homorrymous
O D) Monocular
30. The ability of endotheli al cells to inhi bi t pl atelet aggregation is medi ated in part by the metaboli sm of cell membrane-denved arachi doni c
aci d bywhi ch of the following?
O A) Anaerobi c gJycol ysi s
O B) Gycl ooxygenase pathway
O C) Gl utathi one reductase pathway
O D) Lysosomal enzymes pathway
0 E) Mi xed function oxidase pathway
O F) Tricalboxyl ic aci d palhway
31. A 36-year-ol d man has acute pai n in the nght upper arm and shoulder. Acute supraspinatus tendini ti s is suspected. Whi ch of the
followi ng physical exami nati on manewers of the nght shoul der is most likelyto reproduce the pain. thus supporti ng the di agnosi s?
O A) Abducti on
O B) Adducti on
O C) Extensi on
O D) External rotati on
0 E)Fl e>aon
O F) Internal rotati on

32. A physi cian is in an el evator wi th three resi dents and ovemears thei r conversati on. "How coul d you make such a mi stake?" one asks
another. "You tqiew Mr. Morfis was allergi c to peni ci llin!" he conti nues. ' 'You know that the anti bioti c cross reacts wi th penici llin. Now he
has a rash everywhere! You' d better hope he doesn1 develop respi rato,y probl ems. We are all going to get sued!" Whi ch of the followi ng
is the most appropriate response for the physi ci an?
O A) Advise the resi dents to report this as an adverse drug reacti on
O B) Ascertai n whi ch ward the pati ent is on so that he can revi ew the chart
0 C) Ask the resi dentwhi ch anti bioti c was used so that he does not make a si mi lar mistake
O D) Get off the el evator al the next floor
O E) Ignore the conversali on
O F) Remind the resi dents that discussi ons about patients shoul d be keptprivate

33. A 23-year-ol d man comes to the physi ci an's offi c.e for a health mai ntenance examinati on. He mentions that he is having di ffi culty meeting
new peopl e at graduate school because he feels inadequate. He has always had di ffi culty soci alizi ng with peopl e because he feel s they
will ridi cul e hi m. He is sensi nve about negati ve cri ti ci sm. The most l ikel y personal ity di sorder ,s
O A) anti soci al
O B) avoidam
O C) borderl ine
O D) schi zoi d
O E) schi zotypal

34. Whi ch oflhe following human cells is 1N and hapl oi d?


O A) Genmi nal epithel ial cell
O B) Pri ma,y spenmatocyte
0 C) Spermati d
0 D) Type A1 spermatogoni um
O F) TypA R i:.pArmtognni11m

35. Which olIlle rollowi ng cOIT'!)Onents or the second-messenger cascade is di recutresponsi bl e tor release or intrecellAar stores or ca2?
0 A) Cycle AMP
O B) Diacvtglycerol
0 CJ G-Jl(otei n
O DJ Inositol 1.4. S-tri sphospha.te
O El Nitric oldde

O F) Protein kinase

36. A 35-year-ol d man who takes street drugs inuavenousty develops fever. cough, and hemoptysis.X.ray film of the cllest shows bi laieral
infillrares. Echocardiogram di scloses a 1-cm mass Illar, s anached to the Uicospi d valve. Blood culture grows yeast The mostl rkel)'
causal 0<garnsm ,s
0 A) ,git/us flavus
0 B) Cafldida atbicans
0 CJ Cf}IPlococcus neofonnans
0 D) Hisloplasma capsulatum
0 E) Mucorspeci es

37. A 2-year-ol d gi is brought to the emergency department 90 mi nutes alter ingesti ng a drug from her mother' s purse. On arri val. the chil d
is sedated; respi rations and bl ood pressure are 1101111al. Whi ch ofthe following drugs is most li ket)'to reverse the sedation?
O A) Acetyl cystei ne
O B) Amyl ni trate
O CJ Apomoll)hi ne
O D) Atropi ne
0 E) Fl umazenil
O F) Hal operidol
O G) Nal oxone

38. Co111x1red with a person who follows a l acto-ovo vegerarian diet. a personVihose vegetalien det does not permitintake of mi lk or eggs
is moSI ikely to have a defic,ency of
0 A) fola:e
O B) .,;ramin A
O CJ "1tamin B12 (cyanocooalami n)
O D) .,;ramn c
O E) .,;ramin D
39. The bindi ng specificityfor extracellular matJix protei ns. such as fi bronecti n and lami ni n, to eel membranes depends on whi ch ofthe
fol lowmg cl asses of pr<>1e,ns?
0 A)COlegen

0 BJ Elaslin
0 CJ llltegrin
O D) M1crofibri llin
0 E) Proteoglycan

40. A 68-year-ol d woman has breast cancer that is metastatic to the axi lla,v lymph nodes. The primarytumor is excised and chemotherapy,
incW ng cycl ophosphamde and tamo>0fen. 1s staed. 'M'lch of the following mechanisms best expl ai nsIha bkel)' benefi cial effect of
tarno>0fen ,n thi s panenc?
O A) Competition with c\'(oplasmi c estrogen receptors
O B) Emancement of c')'dophosphami de C\'(otoxic effect therapy
O C) lnhbl1lon of residual estrogen synthesisbythe o'lanes and adrenal gl ands
O D) Retardation of tumor i;,owth by Its anctogemc acnvny
O E) Somutauon of membran&,bound progesnn receptors
41. A 45-year-ol d man has nausea. vomiting. and abdomi nel pein. He has tachycardi a. absent bowel sounds. and invol untary guardng and
rebound tenderness ofthe abdomen. The most l ikel y cause of these findings is
O A) cholecystiti s
0 B) gaso'itis
O CJ gastroenteriti s
O DJ pentorntis
O El sm"'1 bowel obstrucnon

42. A 49-year-ol d man has a 1-week hi story of shortness of breath wi thout fever. chi lls. or cough. Breath sounds are decreased in the l elt
lower lobe. and there is decreased vocal and tactile fremi tus and dullness to percussi on in this area. The mostli kely di agnosi s is
O A) bronchi ectasi s
O B) bronchospasm
O C) emptrysema
O D) intersti ti al infi l trate
0 E) lobar consoli dati on
O F) pl aural effusi on
O G) pneumothora:x
43. A 23-year-ol d woman >'<ith a historyof asthma comes to the physi ci an' s office because she has had progressi ve wheezi ng over the past
2 days. Whi ch of the foll owing drugs is most likel y to induce bronchodi lati on in this patient by activati on of adenylyl cycl ase?
O A) Al buterol
O B) Becl omethasone dipropi onate
O C) Cromol yn sodium
O D) l pratropi um bromide
O E) Theophyll ine
44. A full-term mal e neonate is born wi th a 4x4-cm mass posteri orto the light sternocl ei domastoi d muscl e. Deliverywas uncompli cated and

Apgar scores were 9 and 9 at 1 and 5 mi nutes. respectivel y. The mass is unil ocul ar and fi lled >'<ith wateryfl ui d; no other abnormali ties
are present. The most l ikel y di agnosis is
O A) branchi al cyst
O B) cysti c hygroma
O C) preauricul ar si nus
O D) ttryrogl ossal duct cyst
O E) Zenka(s di verti cul um

45. Mutati ons in the cysti c fi brosi s transmembrane conductance regulator (CFTRJ protei n al terwhi ch ofthe followi ng ion transport
mechanisms?

0 A) Ca"'-activated Cl secreti on
0 BJ Ca"'- activeted c1 -uptake
0 C) Cycl ic AMP (cAMP)-sti mul ated Ct-secreti on
0 DJ cAMP-sti mulated Na secreti on
0 E) cAMP-sti mulated Na uptake

II

m
46. In the fami ly represented bythe pedi gree. affected persons have a di sease characteri zed by mi tochondri al ty inheri ted l ate-onset optic
atrophy. Assumi ng that the fami ly and personal medi cal hi story of the partner is unremarkabl e. the ri sk for optic atrophy to deval op in the
mal e chi ld or I V-9 is cl oses1 to
0 AJ0%
0 6)25%
0 C) 50%
0 D) 75%
0 E) 100%
47. A 47-year-ol d man undergoes resection of hi s enti re stomach for intractabl e bleeding from a gastlic ul cer. Alter recovery, he is at

increased liskforwhi ch of the followi ng?


O A) Copper defi ci ency
O BJ Vi tami n A defi ci ency
O CJ Vi tami n 136 (pyri doxine) defici ency
O D) Vi tami n B12 (cyanocobalami n) defici ency
O E) Vi tami n C defi ciency

NBME
Form 1 Section 3

7. Sudden withdrawal from whi ch of the following substances is most l ikely to produce a l ife-threateni ng syndrome in a person dependent on
that substance?
0 A) Amphetami ne
O B)Cocai ne
CJ Heroi n
O D) Methyfphenidate
O E) Secobarbi tal

8 A 2- year- ol d chi ldhas a hemorrhage in the l ower gastrointesti nal tract A Meckel's d1 vert1cuun is surgi cally resected. What ki nd of
ectopic ti ssue is mostl ikel y to be present in the di vertleulum?
0 A)B1 l iaoy
O B)Col onic
0 C) Duodenal
O D)Gasll'ic
0 E)Spleni c


9. A 74-year-ol d man has a cough. On ausculation of the nglltposteri or chest, the pati ent's spokenee" sounds mo<e llke " 8'{." Whoeh of
the fo'"'1ng cond1t1 ons on the nght ,s most bkely?
0 A) Bronchopl eural fistula
O B) Lowe< l obe consol idation
0 C) Mddle l obe atel ectas,s
O D) PletKal effusi on
O EJ PlelKal pl aque
0 F) Pneumothorax
O G) SeQUestretl on


1 0. A SO-year-ol d woman with a hi stooy of hypeftensi on suddenlf develops a severe headache and cOlapses while shOpping. She is
intubated and mechanical venti lati on is started because of irregular respi rations. A CT scan of the head shows a massive ontracerebral
hemormage with bl ood eldeodi ng into theventricl es and subarachnoi d space. The followi ng morning she is unresponsive to all sti mui.
Her pupils are fi xed and dll ated, oculocephal1 c reflexes are absent, and she has no spontaneous respirati ons. An EEG confirms
el ectrocerebral inacti-.ity. Whi ch of the following best describes her level of neurologi c functi oni ng?
O AJ Bran death
0 B) Cexy
O C) Limbic encephal opalhy
0 D) Persistent vegetative state


1 1 An asvmptomatl c 42-year-ol d man With essential IYypertension is started on an angl otensln.convenl ng enzyme inhi bi tor. Which of the
followng shows the most hkel y effect of tr$8l!Tlent on circng levels of aldosterone. renln. and bradykln1 n?
Aldosterone
O A)
O B)
O C)
O D)
O E)
0 F)

Rerun
I

t
t
t

t
t
t

Bradyklni n

r
r

no change
t
I


12 A 55-year-ol d man is exposed to mal athion whi le worl<ing on a term After successful tre81/1l0/lt of chol iner!JC symptoms with ettopi ne.
he cooonues to haYe difficul ty breathing and muscl e weakness. Whi ch of the followi ng i s the most appropnate treatment?
0 A) Gallamone
O B) Hexamethonium
0 C) Mecamytami ne
O D) Pitocarpine
0 E) Pral1do>0me

13. Duri ng contracti on of striated muscl e, the thick and thi n fi laments al ter thei r rel ationship wi th each other and the sarcomere shortens.
Which of the following bands is mostl ikel y to maintain a constant l ength during muscl e contraction?
0 AJA
0 B)H
O CJ I
O D) M

14. A 32-year-ol d man has a fami ly history of bl eeding. Pallial thrombopl asti n and bl eeding ti mes are prolonged, aggregati on of pl atel ets
with ristocetin is absent. and factor VIII (anti hemophi li c factor) concentrati on is decreased. The most l ikel y diagnosi s is
0 A) defi ciency of antithrombi n Ill
O B) di ssemi nated intravascul ar coagul ati on
O CJ hemophi l ia A
O D) hepati c fai lure
O El von Willebrand's di sease

15. A 35-year-ol d woman has had twelve 0.5 to 1.5-cm lesi ons on her neck. trunk. and limbs for the past 5years. A lesi on on her arm is
shov,n. Whi ch of the followi ng is the most likely diagnosi s?
O A) Chroni c hepati c di sease
O BJ Cushi ngs S'y!ldrome
O C) HI V infecbon
O DJ Mel atoni n excess
O E) Neurofibromatosis

16. A 56-year-ol d man has had fever and pt euriti c chest pain for 1 day. He was admi ned to the hospi tal 3 weekS ago because of an acute
transmural anterior myocardial infarcti on. He has taken a -adrenergi c bl ocking agent and aspi ri n si nce being discharged. A fri ction rub
is heard over his precordium. Whi ch of the following is the most l ikel y cause of the current findings?
O AJ Adverse drug effect
O B) Aollic di ssecbon
0 C) Autoimmune reacti on
0 D) Postviral inflammati on
O E) Recur<ent ischemi a
17. A 23-year-ol d man who recenlly emigrated from Costa Ri ca is admitled to the emergency department with sei zures. Fi ndi ngs on
exami nati on of hi s cerebrospi nal fluid are within the reference range. A CT scan of the head shows numerous cortical l esi ons with
cal ci fications; some ha;e surroundi ng edema. The l ife cycl e of the parasi te causi ng these l esi ons invot-ves
O A) the bi te of a bl ackfl y to human
O BJ the bi te of a mosquito to human
O C) the bi te of a sandtlyto human
O DJ passage through cows
O E) passage through pi gs

18. A 22-yoor-old woman comes to the emergency depanment because she has had fever. (fysuria. and flankpai n for thepast 2 days Vital
si gns are:
Rectal temperature
Pul se
Respiralions
Bl ood pressure

39.4 C (103 F)

13(){mi n
25/mi n
90/55 mm Hg

She has anuri a. Blood CUitures growEscherichia coli. Intravenous broa<l-spectrum antibi oti c therapy isstaned. wtjeh of!he following
c','tol<Jnes contribute to her wrrentillness?
O A) tnterfeulon.2 (I L-2) and ,merferon.y
0 8) IL-3 and I L-7
0 C) IL-4 and I L-5
0 0) IL-10 and I L-12
O E) TllllOf necrosis factor-a and I L-1

DNA source
Fathor

B T

--

Mother

B T

Sister 1

B T

--

Sister 2

B T

Brother

B T

Proband

B T

19. A 3-year-old boy is suspected of havi ng an X-l inked immunodeficiency. To establ ish the pattern of transmi ssi on of thi s geneti c defect.
DNA is extracted from the T and B l ymphocytes of hi s parents. hi s two si sters ages S and 7 years. and hi s 3-month-otd brother. Alter
restricti on enzyme di gesti on. the digested DNA is subjectedto gel el ectrophOresi s and probed wilh a DNA markerthat disti ngui shes the
maternal and paternal X chromosome allel es. The results are shown. Based on thi s DNA analysis. whi ch of the followi ng statements is
correct concerni ng the inheri tance of thi s geneti c defi ci ency?
O A) The brother has inherited !he defecti ve gene
O 8) Both parents can transmi t !he defecti ve gene
O C) Sister 1 is a carri er
0 D) Si ster 2 is a carri er
O E) Both si sters are carri ers
O F) Neither sister is a carri er

20. A 2yoor-ol d woman. a,ida 1 . para 0.who is Rh-negative. deli vers a ful-tenn Rh-positi -ve neooate. The mottle( is gi ven propltj4actic
anti-Rho(D) immune globuli n immedi atelypost partum. Dunng her second pregnancy 3 years later. she 1s screened each tri mesterfor
Rho(D) antibodi es. An mdirect antigl obum test done di.ling !he thi rd tnmester indi cates the presence of anb-Rho(D) anti bodies m her
serum. Whi ch of !he following is the moSI ikel y mechanism forthe occurrence of these maremal antibodi es?
O A) Anamnestic production of maternal anti-Rh0(0) immunoglobuli n
0 B) Intrauterine transplacertal fetal-maternal hemorrhages <llring the second pregnancy
0 C) Residual ci rcul ati ng prophylacti c anti-Rho(D) immunoglobui n
O D) Transptacental passage of fetal tgG anli-Rho(D) anti bodies

21. I n rruc:Olipi dosi s II (eel dsease). the erlJl8 catalyZi ng fonnation of mannosa 6-phosphate,Ille signal for transfer to lysosomes, is
mi ssing Protei ns destmeo for lhe lysosome are most likelyto be found mwhich of the following locati ons in patients wilh this di sease?
O A) Endopjasm1 c reticuum
O B) Extracellul ar space
O C) GOlgi compl ex
0 D) Mtochondri a
0 E) Nucteus

22. A 00.year-ol d woman wilh osteoarrhri tis has recurrence of pain and stiffness in herjoints. She i s afebrile. The most awropnate therapy
is
O A) allopunnol
0 8) aspirin
0 C) colchlcine
O D) esuogen
0 E) probenecid

23. A 32-yeer-ol d woman with pul monaryfibcosis is most l ika, 10 minimize the wort< of breathing bf using whi ch ol the followi ng breatii ng
patterns?
Respiratory
freqiency
0 A)
0 8)
O C)
O D) Normal
0 E) Normal
0 F) Normal
O G)
r
H)
0
t
0 I)
r

Tidal

vokme

normal

r
I

normal

normal
t

25. During an experiment on the cough refl exi n humans. a subject imal es ai r containing di fferent amotns of parri cl es tll8Iwill impact and
adhere to mucus pri marityin the trachea. Blockade of which of the following afferent pathways would most li kely pr8"8ft thi s subject' s
refl exto ini tiate a cough?
O A) Glossopharyngeal
0 8) Lal)119881
0 C) Olfactory
O D) Trigeminal
0 E)Vagal

26 A 38-year-ol d woman unde<goes surgical repair of e hi p fracture She hes been treated for Iha past 20years wilh hi!11 doses of inhaled
end oral medl ceil ons for -re asthma. Dunno surgi cal repair. severe osteoparos1s 1s noted. L009-term admi ni stretJon of whi ch of the
follOW1n9 drugs is most hketyto have caused the os1eoporosis?
O A) Al buterol
0 8) Cromol)'n sodi um
O C) lpracropium
O 0) Pre<hsone
O E) Theopl1y11ine

27. A 23-year-ol d woman has had interni nent amenorrhea si nce the bi rth of her first chi ld 5 years ago. She recei ved 1 O uni ts of bl ood duri ng
her del ivery. Her skin is thi ck and doughy. She has no energy and is depressed. Which of the following is the most li kely cause of her
symptoms?
O A) Adrenocorri cal insufficiency
O BJ Chroni c fatigue syndrome
O C) Hemochromatosi s
O D) HI V i nfection
O E) Pitui tary necrosis

28. A marri ed coupl e in their mi d-20s comes to Ille physi ci anfor eval uati on of inferlil ity. They say they have had sexual rel ati ons 2 to 3ti mes
perweek for Ille past year. Which of the following is the most appropri ate nextstep for the physi ci an?
O A) Ask the coupl e whatlfley mean by " sexual rel ations"
O BJ Perform a gynecol ogi c exami nati on on the wife
O C) Perform a testi cul ar exami nati on on the husband
O DJ Refer Ille coupl e for malital counsel ing
0 E) Suggest !flat the husband undergo ferlili ty eval uation
O F) Suggest that tile wi fe undergo fe11i l ity eval uati on


29. A SO.year- ol d man who recendy immigrated to the USA from Russi a devel ops fever. swelli ng of the neck. and di fficulty swallowing. Hi s
r.r.i nation hi story is unclitrtai n. E:v.ami nati on of th throat shows a gray m.ambrana across tha pharynx. Tha pathogan rasponsible for this
infecti on most l ikel y acquires its vi rul ence through whi ch of me followi ng geneti c events?
O A) Acquisi ti on of a transposon
O BJ Chromosomal mutati on
O C) Conjugation
O DJ Recombi nati on
O E) Transducti on


30. A 53-year-old woman comes to the cl inic because of a 1-week hi sto1yof pai n and burning on urination. She is homeless and has a poor
di et. The physi cian gi ves her enough tnmelfloprim-suamethoxazol e to l ast 2 weeks. Because of her inadequate diet. she is at
increased ri sk for w1lich of the following adverse effects ofthi s drug?
O A) Cerdi omyopalhy
0 B) Megal obl asti c anemi a
0 C) Ototol<ici ty
0 DJ Sei zures
O E) Systemi c l upus el'/!hematosus


31. A 22-year.ol d coll ege student who has been tfeated for anxiety and dep<essed mood for 6weeks develops episodes in wtiich she feel s
l ight-headed and al most fai nts. Pul se is 1 1Olmin. and blood pressure is 110/70 mm Hgwhile tying down and 85160 mm Hg whi le
standing. Whi ch of Ille following drugs is most likely to have caused the Ollhostati c hypo!eosi on?
O A) Alprazolam
O B) Amitriptyl ine
O C) Buspirone
0 D) Diazepam
0 E) Fooxetrne


32. A seri es of experi ments is performed to determine the mechanism by whi ch a pharmacol ogi c agent traverses cell membranes and
accumul ates within target cell s. The rate of transpolt depends on the concentratron of the drug only. When the extracellular

concentrati on of the agent exceeds 1 O mM. no further increase in the rate of uptake is obsed. Strucrurally si mi lar compounds pass
+

through the cell membrane. but at a l ower rate. Ouabain. an inhi bi tor of Na1 1< ATPase. fail s to inhi bi t transpon. Whi ch of the foll owing
is the most l ikel y mechanismi by whi ch this agent enters cell s?

O A) Anti polt

O BJ Faci l itated diffusi on


0 C) Ion-gated coupl ing
O D) Si mple di ffusi on
0 E) Sympo11

12

14

2 4 6 8 10 12
Elimination half.hour (h)

14

10

33. The bar chart shows the di stribution of el imi nation hal f-l ife of theophy11 ine for adults (sol id) and chil dren (striped). Whi ch of the followi ng
characteristics of chi ldren best explains these data?
O A) Hepatic metabol ism is faster
O B) Oral absorpti on is poorer
O C) Renal secretion is l ower
O 0) Theophy11 ine is l ess potent
O E) Vol ume of distribution is larger
34. A 2-year-old boy ingests an unknown quanti ty of iron sul fate tabl ets that his motherwas gi ven for anemi a followi ng chil dbi rth. The most
l ikel y consequence of this ingesti on is
O A) gastri c bl eedi ng
O B) metabol ic al kal osi s
0 CJ polycythemi a
O D) respi ratory depressi on
0 E) uri nary retenti on
35. An experi mental drug is admi nistered to a human subject with moderatel)' severe congesti ve heart failure. Wi thi n mi nutes of intravenous
administration. there is a measurabl e decrease in cardiac prel oad and cardi ac aftertoad. and an increase in gl omerular filtration rate.
The effects of the drug are most si mil ar to those produced by whi ch of the followi ng?
O A) ADH (vasopressi n)
O Bl Al dosterone
O C) Angiotensin II
O DJ Atrial natriuretic pepti de
0 E) Insul in
O FJ Norepi nephri ne

36. A 56-year-ol d man whO has colon cancer is bei ng treated with 5-fluorouracil. Whi ch of the foll owing is the mechani sm of the
anti neopl asti c acti on of thi s drug?
O A) Bi nding free di hydrofol ate
O BJ Inhi bi ti ng cait>amoyl phosphate synthetase I
O CJ Inhi bi ti ng nbonucleobde reductase acti vity
O DJ Inhi bi ti ng thymi dy1ate synthetase activi ty

37. A 35-year- ol d woman has had type 1 di abetes mellitus for 20 years and mi croal bumi nuri a for 3 years. Whi ch of the following findings is
most l ikel y on examination of tissue obtai ned on renal biopsy?
O A) Corti comedullary cysts
O B) Diffuse glomerul osclerosi s
O C) Endotheli al cell prol iferati on
O D) Glomerul ar crescents
O EJ Glomerul ar leukoeyti c infil trati on

38. The concentration of sodi um in the bl ood is affected by a hormone secreted by cells in whi ch label ed zone in the photomi crograph?

0 A) 0 B) 0 C) 0 0) 0 E)


39. A 27-year-ol d woman has a 1-week hi story of increased uri naiy frequency. dysuri a. and intermi ttent suprapubi c pain that have become
worse over the past 3 days. Exami nati on of her urine is most l ikelyto show
0 A) aci d-fast organi sms
O B) buddi ng yeast forms
O C) fungal hyphae
O D) gram-negative baci lli
O E) gram-posi tive cocci


40. Pol iovi l\ls can be grown in cul tured monkey ki dney cells dlrough repeated passages. resulti ng in virus that is no longer neurovirul ent.
Thi s virus can be used as a vacci ne because it has become
O A) attenuated
O B) defecti ve
O C) inactivated
0 D) latent
0 E) recombinant


41. A 26-year- ol d woman has a 3-week hi story ofjoint pai n and a photosensi ti ve rash on her face and other exposed parts of her body.
Which of the following tests is most useful in confi rming the diagnosi s?

0 A) Anti bodi es agai nst nati ve DNA (doubl e-stranded ONA)


O B) Anti bodies agai nst rubella virus
O C) E,vthrocyte sedimentation rate
O D) Leukoeyte count
O E) Rheumatoi d factor assay
O F) Serol ogi c testfor syphili s
O G) Total hemolytic compl ement concentrati on


42. A bacteri um hal1lori ng a 15.000-base pai r (bp) pl asmi d that encodes resistance to tetracycl ine and ampi ci llin segregates mutants that
l ose resi stance to ei ther drug. Each independent mt/lant pl asmid appears to increase in size by approximately 5200 bp. Whi ch
mol ecul ar event best expl ai ns these findi ngs?
O A) Formation of pl asmi d concatemers
O B) Formation of pyrimi di ne di mers
O C) Homol ogous recombination
0 D) Insertion of a transposon
O E) Sl ip.stranded DNA repl icati on errors

43. A 24year-ol d man is brought to the emergency department after bei ng stabbed in the chest. Exami nati on shom a stab wound in the
fourth intercostal space on the left. immedi ately l ateral to the sternum. Whi ch of the followi ng structures is most l ikely injured?
0 A) Pul mona,v trunk
O B) Ri ght allium
O CJ Ri ght ventri cl e
O D) Superior vena cava
O EJ Thoraci c aorta

44. A 61-year-ol d man has a 3-month hi slo,v of dull . aching pai n on the lelt side of hi s chest Whi ch of the foll owing is the most appropri ate
next step in eval uabon of tins patient?

O A) Ask hi m about his past medi cal histo,v


O BJ Ask hi m about hi s famit)' hi stoiy

O C) Ask hi m for addi tional detai ls about hi s chest pai n

O DJ Order an exercise stress test

0 E) Perform a cardiovascul ar examination


45. A pharmaceuti cal company has devel oped a drug that prevents egress of Mg from the N-methyD-aspartate (NMDA) receptor. What
is the most l ikel y effect of thi s drug in a patientwi th an acute stroke?

O AJ Decreased risk for sei zures due to hypomagnesemia


0 B) Faci l itati on of
entry into the neuron and enhancement of excitotoxi n-medi ated cell death
O C) Hyperpol ari zation of membrane and conducti on bl ock
0 DJ Preventi on of ca>< ent,v into the neuron and reducti on of exci totoxin-medi ated cell death
0 E) Rapid depol arizati on of membrane with lowering of the sei zure threshol d

ca2

46. Nine monlhs ago. a 60-year-ol d man had a myocardi al infarcti on of hi s left anteri or ventri cul ar wall. Recoverywas uneventful. A
photomicrograph of the area of infarcti on is shown. Whi ch of the followi ng macrophage products is primant)' responsi bl e forthe
dominant feature in the area surroundi ng the X?

O AJ Fi brobl ast growth factor

O B) Formyl ated peptides


O C) lnterferon-v
0 0) Leukotri ene 04
0 E) Neutral proteases
O F) Ni tric oxi de

47. The 17-year-ol d ooy shov.n ,n the photograph 1s brought to the physician alter having a tonic-dom e sei zure. wtich of the following
additional fi ndings is mos1 l ikely to be dsdosed on de1ailed hi slo,y.taki ng?
O A) Episodes of hypomania
O 8) Episodes of severe depressi on
OC) Famity his1ory of alcoho sm
O 0) Special education beginni ng in first gade
O E) Special education beginni ng in tenth gade

48. A 42-year-ol d man with pneumococcal pneumoni a has acute fibri nous pl euri ti s.Which of the following proteolyti c eniymes is requi red to
el imi nate the exudate and restore notmal pl eural anatomy?
O A) Collagenase
0 B)Pl asmi n
0 C) Stromeolysin
OOJ Thrombin
0 E) Trypsi n


49. A neonacewith swelling ofthe neck has a decreased serum 110l<ine (T4) concemati on and an increased serum lt'oid.stimul ati ng
hormone concentrati on. Which of the fol lov,ing agents chronicaly ingested by!he mother duri ng !he pregnancy is !he mos! l ikely cause
of thesefindi ngs?
0 A) CO<ticosteroids
O B) Propranol ol
OC) Propyl!hiouracil
O 0) Th','roX1ne (T,)
O E) Triiodothyroni ne (T3)


50. As the resull of a fall on10 a hard surface. a 20-year- ol d woman sustai ns afracture of the medal humeral epicoOO)'le. and a nerw is
injo'ed.Which acti on wiD be most affectedbythi s nerve i11ury?
O A) Adducbon of all d1 grts
O B) El<!eosi on of all digi!S
O C) FiexJon of all d1g11S
O 0) Pronation of theforearm
0 E) Supination of !he forearm


18
2C
30
4C

218
228
23G
240
25E

5A

68
7E
80
98
IOA
IIC

260
27E
28A
NE
308
318

13A

33A

328

12E
14E
15E

44C
4lD
46A

34A
350

16C

liE

18E
190
208

4lC

360
378
388
390
40A

41A

420
43C
420

4i0
48B

49C

lOA

NBME
Form 1 Section 4

5. A 60-year-ol d man devel ops cirrhosis. He had mul ti pl e bl ood transfusi ons foll owi ng severe trauma in 1985. Ha has no history of jaundice
or hepatiti s. Serol ogi c tests and polymerase chai n reacti ons on hi s bl ood will most likely show evidence of infecti on with whi ch hepati ti s
vi rus?
0 A)A
0 B)B
O C) C
D) D (del ta)
0 E)E

n
becomes
and
her
sk
a
has
had
several
episodes
of
ner.ousness
in
which
her
heart
pounds
6. <>,er the pesl 6 months. 54-year-oldwomen
i
cl ammy. Sile has al so had di fficul ty steepi ng and recently devel oped hypertension. The mostikety cause 1s a neoplasm in whi ch structu'e
label ed on the cross-sectional drawing of Illa abdomen at Ille lavet ofT-12 to L-1?

O AJ 0 BJ O CJ O D) 0 E) 0 F) O G) O H) 0

".,

>E

50

.,
"'

Q.
C

E -50

.0

Q)

-100

Time (msec)

7. The graph shows the actoon potenbal on a typical nerve eel. Whi ch of the following bast describes the relationshi p between membrane
permeabilities (P) in tile hatched region ofthe cU1Ye?
0 A) Pco: < PN..
0 B) Pco: > PN..
0 CJ PN.. < PK
0 D) PN 1s approlClmately equal to PK,
0 E) PN > PK

8. A 40-year-ol d woman has recurreni cal cium- conlaini ng renal cal cul i i ch of !he followi ng strategies i s most tikety to decrease her ri Sk
for form ng add1bonal cal culi ?
O A) Adm1mstrabon of a th1azide di ureti c
O Bl Al kal inization of Ille urine
0 CJ lngesbon of a hi -oxalate di et
O D) Ingestion or a hlprotein diet
0 EJ Restricbon of ftui d intake

9. Whi ch of the following best describes the movement of Na and K+ duri ng one cycle of the Na.1< pooip (ATPase) in neurons?
Na

O A) 21n

0 B) 2in
O C) 3in
0 0) 2 out
O E) 3 out

K+
2 out
3 out
2 out
2in
2in


10. A 30-year-ol d woman has a 1 O-month hi sto,y of a vuhi tish discharge from both breasts. Her l ast menstrual peri od was 9 months ago. She
had a bilateral lllbal l igati on 4 years ago after the bi rth of her thi rd chi ld. Palpation of the breasts di scl oses no masses. Whi tish fl ui d is
expressed bil aterally on compressi on of the ni ppl es. The most l ikel y diagnosi s is
O A) bi lateral masti ti s
O B) intraductal carci noma of the breast
O CJ metastati c carci noma of the breast
0 0) pi tui ta,y adenoma
O E) premature menopause

11 A 4S.year.Qldman devel ops a fever 4 days after an abdominal operation. Blood cultures grow Sfaptryfococcus aureus. Whi ch otme
following is the most l ikely si te of infection from nhich the organi sm gai ned access to the blood stream?
O A) Amenor nares
O B) Sputum
0 C) Stool

0 O) Unne

0 E)Wound

12. A 1 a.year.ol d gi rl has fever. fati gue. and easy brui sing. Laboraro,y studies show:
Hemogl obin
Hematocrit
Leukocyte count
Pl atel et count

6.6 gldl (N=11.8-15)


20% (Na36-45%)
13001mm'
12.000/mm

A photomicrograph of bone marrow obtai ned on bi opsy is shown. The most likel)' di agnosi s
is
0 A) al eukemi c l eukemi a
O BJ apl asti c anemj a
0 CJ immune thromboCjtopeni c purpura
O D) rnyel ofi brosis
O E)vitami n 812 (cyanocobal ami n) defi ci ency

13 A 22.year.Old woman 1s pMi cipeti ng i n a studi' mat measures renal function. Eval uati on of serum and uri ne Shows the following
Na (mEqll)
K (mEqlL)
Glucose (m!}'dl)
Creatini ne (mg/dl )
p.Arrlnohl ppl.lic aci d (mg/ml)

Serum
140

Uri ne
100

50

100
1
01

0
100

50

Assuming a mne flow of 1 ml.hni n, whieh of the following best represents herfiltration fraction?
0 A) .10

0 B) .20

O C) .30
0 0) .40
0 E) .50

14. A new analgesic drug lhatwi l decrease pai n at si tes of acute innammati on is to be devel oped. Thi s action can be achi eved by creati ng
an antagonist to whi ch of the followi ng infl ammatory medi ators?
O A) Braclykini n
0 BJC5a
O C) Hi stamine
O DJ lnteeuki n.2 (l l-2)
0 E) Prostacycl in
O F) Transforming growm factor

1 5 A 40-year-ol d woman discovers a breast mass during seH-exami nalion. At a routine heal th maintenance exarnnati on 2 months ago. her
physici an had told her all findings were normal. After having a mammogram that is suspi cious for breast cancer. the paU ert begins to
wonder if her phySici an fai led to noti ce the unp (luri ng the exam1naoon 2 months ago . She is most llkety to sue the physician under
ooch of thefollowing ci rcumstances?
O A) Occu,rence of breast cancer
O B) Poor communcati on between the pauem and physi cian
O C) Potential fi nancial reward tor the peli ent
0 D) Slrong pressu-e from ram1ty members

16 A 28-yeer-ol d woman devel ops temperatures to 39 4 C (103 F) anel substemal chest pain 3 days after bei ng treated byher Clenti st for
g1 ngMDS. An x-ray fim ofthe chest shOvS an en arged cardiac s1 lhoueae On pencardocentesi s. 250 ml of cloudy fk.ld contai ni ng large
numbers of neutrophilS ,s removed Which of thel fol kMlng 1s most llketyon exami nalion of the flu id?
O A) Aciel-fast baci lli
0 B) Budding yeastfOllllS
O C) DNA 'lirus
O 0) Gram-negaove baalli
O E) G<pos1 ti ve cocci

1 7 The leoti de sequence encodng four amino acids near the amino te,rninus of a 10- kd protei n is s. ACTGAT TGC GIT -3. Thi s
sequence 1s mutated by 1nse111on of a si ngl e nucleoti de (A) in the second codonand becomes 5. ACTGAA TTG CGT -3. Whi ch of !he
following consequences of this mutaoon is most bkety?
O A) Alered cel cycle checkpoi nt
O B) Decreased rate of transcri pti on
O C) Protei n D'unce11on
O D) Single ammo aoel substituti on

18 A 3-year- old boy has an abdominal c,wtorcili d testis that canno1 be brought down into the scrotal sac. The testis shoul d be removed to
decrease his risk for which of the following?
O A) Abnormal hOrmonal secreti on
O B) Infecti on
O C) Intestinal obstructi on
0 O)Mali gnancy
0 E) Psychological impairment
19. An 18-year-ol d man attempts sui ci debytaking 15 g of acetami nophen. Fourhours alter the overdose. high level s of acetaminophen are
found in the pl asma. The mostapproriate lherapy is
0 A) hemodi atysi s
O B) oral admi ni strati on of aspi rin
O CJ oral admi ni strati on of N-acetyl cyste1 ne
O D) respi ratory support and watchful mi ting
O EJ uri na,y all<al ini zati on anel forced di i.resi s

20. !3-Hexosami ni elase is made up of a and subunits. Tay.Sachs di sease is Clue 10 a mutation in the a-subuni t gene. and Sandhoff
di sease 1s due to a mutati on in the 13 subuni t gene. I f a carrier of Tay.Sachs disease marri es a ca,ri er of Sandhoff di sease. what is the
probability that their fi rst chi ld will have ei ther disease?
0 AJO%
0 B) 125%
O C)25%
0 0) 50%
0 E) 100%

21. Actively dividing fibrob asts in cu ture are transfe,red from a 37 C incubator to a O C ice bath. and 10-6 M cof.ch c ne is added to the
l
l
i i
cul ture medi um. Alter 30 minutes. many cells have been arrested i n mitosis. Whi ch of the foll owi ng cytoskel etal prote
i ns is most l ikel y to
be involved?
0 A)Acti n
O B)Desmi n
0 C) Oynein
O D) Keratin
O E) Ki nesi n
0 F)Lami ns
0 G) Myosin
0 H) Neurofilamentproteins
O l)Tubul in

22 Which of the following anti mi crobial agents is mostl ikely to be admini stered orally for the treatment of potentially serious systerrrc
intectJons because of ,ts predctable bi oavai labtUy?
O A) Amphoterici n 8
O 8) C1prolloxeci n
0 C) Gentamic,n
0 D) Potymyxin B
0 E) Vancoi n

23 A 1 9-year.old college student devel ops hemeturi a and proteiruie. Approximately 3 weeks ago, he had superfiaal celluhos or the n(t
proxi mal thigh and buttocks. 11s condi tion ,s most li kelyto be secondaryto ,nfecben wi th
O A) Eschsrichis coli
0 8) Pseudomonas aeroginosa
0 C) Staphylococcus aureus
0 0) Staphylococcus eplderm1d1s
0 E) Strepfoooocuspyogenes (group A)

24 A 27-yeer-ol d men wi th reculTent peptic ul cer disease has a small pancreatic neopl asm that produces large amounts or gastrin While
awai ti ng operaoon. the pati ent shoul d be treated wi th
O A) chenodeo)('/1:hol ic acid
O 8) methachol ine
O C) metodopralTllde
O D) omeprazol e
O E) sucralfate
25. A 5-year-old boy wi th a hi story of recurrent ear infections recei ves hi s preschool booster immuni zation agai nst
di phtheria-tetanus-pertussi s. He is parti cipati ng in a communi ty.sponsored studyto determine the humoral immune response to tetanus
toxoi d (It). Hi s response is well bel ow nomnal forage. and sex-matched chi ldren. Peri pheral B l ymphocyte count and T lymphocyte count
and functi on are withi n the reference range. The anti body he makesis posi tive in both the passi ve hemaggl uti nati on and
compl ement-medi ated tysi s of It-coated erythrocytes. Hi s antibodi es do not opsoni ze It-coated l atex parti cl es for phagocytosis and do
not di rec!lf preci pitate n efficiently. Thi s chil d most li ketf has a defect in whi ch of the foll owing processes?
O A) Affi ni tymaturati on of immunogl obuli ns
O B) lmmunogl obuhn isotype switching
O C) Recombi nati on of he0Vf chain vari abl e regi on genes
0 D) Recombi nati on of light chai n variabl e regi on genes
O E) Somatic mutation of immunogl obuli n genes
26 A 32-year-ol d woman has a 10.year hi story of chronic paranoi a. audi tory hal kleinations. and tf1e bel ie/U1ateharacters i n tel evisi on
pro<)"ams are tallong direcUv to her. Drug therapy is most bkely to di rec!lf involve wtuch of the tonowmg neurotransmitter systems?
O A) Acetyl chohne
O BJ v-Aminobutync acid (GABA)
0 C) Dopamine
0 D) Glutamate
0 E) Norepi nephri ne

27. An 85-year-old woman whowasadrrunedto the hospi tal 3 days ago for myocardi al infarction has a radi onucli de scan that shows
damage to the ante nor wall of the left ventncle Which of the foll owi ng coronaryvascul ar structures was most likel y occl uded?
O AJ c,rcumtlex Mery
O B) Coronaiy sinus
0 CJ Left arteri or descendi ng artery
O D) Postenor descending artery
O E) Right coronary artery
O FJ Right marginal artery

28. Over a penod of 3 years. a 40-year-old man develOps generalized ct,,stoni c. hyperlonenc movements and progressive dementi a. His
mother and one si ster have a si mi lar di sease. buthosfather and another si ster are heal thy. Which oftile following lesions is most likely
to be present in me central nE!IVOus system of me pati ert and his affectedfamily members?
O A) Atropt,f of the caudae nudei
O BJ Loss of pi gmented cells in the substanba ni gra
O C) Mi crogiel noool es and multinucl eated giant cens in thewl'lite matter
O OJ Neurolibri llarytangles in the cortocal neurons
O E) Sponi;jform changes in the cerebral corteX

29. An outbreakof acute pneumoni a ocCIJ's in mifitary recnits li ving in one barrackand only in those personsl ocated near the air
condi ti oner Epi dernologi c surveillance results in isolanon of the causal organism from the patieru and from the dnp pansof the air
condi ti oner. The organi sm is resistantto pemc1Di n. susceptibl e to erytl1romyci n. and wealdy gram-negati ve. The most likelo/ organi sm is
0 A) Borde/el/apertussis
0 B) Haemophilus influenzae (nontypeabl e)
0 C) Klebsiel/e pneumonise
O OJ Lllgionella pneumophile
0 E) Mycobacterium luberculos,s

0 F) Mycop/asma pneumomae

0 G) staphylococcus aureus
0 H) streptocoocus sgalactiae (group B)
0 I ) streptocoocuspneumoniae
0 J) streptococcuspyogenes(group A)

30. A heal my 32-year-ol d man has seasonal rtliniti s that produces mi ld nasal congestion. He feel s fullness in hi s nasal si nuses and ears. The
most appropri ate oral therapyfor the nasal congesti on is
O A) al buterol
O BJ de:xiromethOrphan
O C) gl yceryl guai aco.l ate
O DJ ipratropi um
0 E) isoproterenol
O F) prostagl andin E2
O G) pseudoephedri ne
O HJ theoph')'ll ine

31. The increase in the intracellular concentranon ofAMP in muscl e alter 15 minutes of strenuous exercise has whi eh or the fall owing
effects?
0 A) Increased acti vity of adenylyl cycl ase
O BJ Increased breakdown of cogen
O C) Increased release of insuin
O 0) lnh1bitton of phospholructol<inase activ,ty
O E) Inhibiti on of pyruvate kinase activity

32 A 42-year-ol d woman repolls fattgabi hty. nervousness. irri tabil ity. and heal intolerance She has a diffuse goiter. exophthatmos. and
prebbi al edema. Ci rcul abng antibodies to whi chof the following are most hketyto be found m thi s pabent?
O A) Mi crosomal protei n
O B) Reverse triiodothyroni ne (rT3)
0 C)ThyfoglObul in
0 0) Thyfotropm
O E) Thyrotropin receptor

41. A 32-year-ol d woman who is an e di tor for a publ ishi ng company comes to the physici an because of inremni crenr diarrhea for the past 10
years. whi ch occurs three to four times peryear. lasts 2 to 4 days, and is associ ated with deadl ines at work. Duri ng these epi sodes. the
stool is loose and contai ns no mel ena or bri ght blood. Be,veen epi sodes of diarrhea. she occasionally has one bowel movement every
4 to 7 days: the stoot is hard and "pellet- l ike." She dri nks one gl ass of wine dai ly. Ph),Sicat examination shows no abnomnal ities.
L aboratory studi es show.
Hemogl obi n
Hematocri t
Leukocyte count
Erythrocyte sedimentation rare (WesrergrenJ

14.3 g/dL
43%
8100/mm3
7mmlh

Which of the foll owing is Ille most likely di agnosis?


O AJ Crohn's disease
O Bl Col on cancer
O C) Diverticuliti s
0 DJ Irri tabl e bowel syndrome
O EJ Small bowel l ymphoma
O FJ Ul cerative col iti s

42 Excess accumulati on of urate. agg-essi -.e behavior. sell- mutil ati on. and mental retardati on are seen i n pati ents with Les.:h-t'lyhen
syndrome Which of the followi ng enzym&s is most li kely to be deficient 1n these pauents?
O A) 1-i',poxanthi ne guanine phosphooboS'/ftJansferase
O BJ Nucl eoside monophOsphete ki nase
O CJ R1bonucleo!lde redoctase
O DJ Urease
O EJ Xanth1 ne oxl dase

43 A 4S.year-ol d men has fever. chil lS. cough, and pleuri llc chest pai n Cul rure ot a rust-col ored spurum specimen'yiel ds manya-hemolytic
colonies of l ancet-shaped. gram.pos1 bve cocci in pairs and short chai ns. The most hkely causal organi sm ,s
0 Al Kt11bsiella pflf/umoniae
0 Bl Allycobacterium tuberculosis
0 C) Allycoptasma pneumoniae
0 D) Slaphylococcusaureus
0 EJ Slreplococcuspneumoniae
0 Fl SlreptococcusP','Ol,8nes (group A)

44 A 50.year-ol d men Who has had amJOCardi al infarct was subseq.,enlly diagnosed as h!l\ling fami lial hyperehol esterol emi a. an
autosomal dominant disorder. Hi s son has not been testedfor tta,erchol esterol em,a. What is the probabili ty that the pati ent's
granddeu!11ter. through his son. wil hevetryperchol esterol emia?
0 A) Less tha-i 1%
0 BJ 10%
0 C) 25%
0 0) 33%
O E)50%
45. A 25-year-ol d woman at 32 v-.-eel<s' gestation begi ns taki ng a drug to del ay tile onset of pretemn labor. After the first dose. she noti ces
tremul ousness in her hands. Whi ch of the foll owing types of receptors is most li kely to be invol ved in thi s effect?
0 A) 1-Adrenergic
0 B) 132-Adrenergic
O C) Dopami nergic (D1 )
O D) Muscari ni c (M1)
0 E)NMDA

46. An adul t's hemogl obi n is 98% saturated wi th oJ<ygen. bur arterial oxygen content is decreased. The mostl ikel y cause of thi s condi ti on is
O A)anemi a
O BJ an arteri ovenous shunt
O C) hemogl obi nopalhy
O DJ high-al titude hypoxi a
O E) pul monary di ffusion defect

47. A 14-montl\.old gn is brought to the physician's offi ce by herfalller for a well-clti ld visit. The father is concerned because hi s ol der
daughter started to wal kwhen shewas 1 0 months ol d. and this chi ld has Of#!i JUSt started towal k. She can sayfive words. pi ck up a
piece or cereal willl her lllumb and index finger, scri bbl e. and pole twO bl oci<S. Whi ch ot the tOllowi ng b8SI descri bes her developmenc?
Cogniti ve
0 A) Del ayed
0 B) Del ayed
0 C) Del ayed
O DJ Del ayed
0 E) Nonnal
0 FJ Nonnal
O G) Nonnal
0 H) Nonnal

Motor
delayed
delayed
noimal
noimal
delayed
delayed
noimal
noimal

Soci al
del ayed
normal
del ayed
normal
del ayed
normal
del ayed
normal

48. A 20-yeer-ol d man has had increesi ng!)t severe muscl e weai<lless and incapaci tati on. He has used a wheel chair for ambul ati on si nce he
was 10years of age. Exami nati on of ti ssue obtained on bi opsy of a cal l muscl e when he was 6years ol d showed a mixed pattern ot
muscl e fiber necrosis alternating wi th hypertrophy and regeneration ot resi dual muscl e fibers. At age 18 years. bi opsyfindi ngs showed
virtually compl ete repl acement of muscl e 'rPf frbrofatty tissue. Whi ch of the followi ng cellul ar proteins is most l ikel yto be mi ssing from thi s
pati ent's muscl es?
0 A)Acti n
0 B)Desmi n
O C) Dystrophin
0 D) Spectri n
O E) Vi mentin

49 A 5-yeer-old gin from sub-Saharan Afri ca is brought to the heallll cl ini c hermother because of diarrheafor the past 4weeks. Her
pulw is 150/min. and blood pressure is 80/40 nm Hg. Physlcal exami naoon shows edema of the face. abdomen. and Jeos She is
l osdess and has reddi sh hei r. Serun al bumi n concentrati on , s 1 2 gldl. CO!ll)aredwoth a healttrychild. wlieh of the following fi ndi ngs is
most fikely inJhisgrl?
O Al Decreased interstitialfl ui d-.olume
O B) Decreased ,ntersaaal hydrostaoc pressure
O C) Decreased plasma oncotic pressure
O D) Increased capolary hydrostaOc pressure
O E) Increased pl asmavol ume
50 Which of the foll owing drugs for mai ntenance the<apyin pati ents with moderate to severe asthma al so has positive cardiac inotropi c
effects. prowces vasod1laoon. has d1 ureoc acbon. andstimulates di aphragmatic contracaon?
O A) Atropi ne
O BJ BeclOmelhasone
0 C) DeXtrOmelhorphan
0 D) PropranolOI
0 E) Theophylline
IA

2B
3A

4F

5C
611
7C
8A
9E
IOO
I IE

120
130
14A
150
16E
17C
180
19C
20A

21)I
220
23E
240
250
26C
27C
28A
290
30G
31B
32E
33E

34E
350
36A
37A
38C
39E
400

410

42A

43E
44C
450
46A
4711
48C
49C
50E

NBME form 4-1


1 . During a study of the components of regulatory protei ns, an investi gator wi ll most likely find that zi nc finger motifs
facilitate which of the foll owing functions?

0 A) DNA binding
0 B) Hi stone binding
OC) Membrane binding
o D) Serine/threonine kinase activi ty
o E) Tyrosine kinase activi ty
2. A 17-year-old boy is brought to the emergency department because of severe cramps in his calf muscl es that
began after he ran a 1-mile race. His uri ne is a burgundy col or. Physical examination shows tender calf muscles.
i
Laboratory studies show an increased serum creatine kinase act vity and myoglobinuria. Followi ng exercise of
an ischemic l eft forearm, his serum lactic acid concentration does not increase in blood obtained from the vein
proximal to the exercised extremity. This patient most likely has a deficiency of which of the following enzymes In
muscl e?
O A)
O B)
0C)
O D)
o E)

Debranching enzyme
Fructose-1 , 6 -bisphosphatase
Glucose-6-phosphatase
Glycogen phosphorylase
Glycogen synthase

3. A 23-year-ol d woman has skin bli stering that results from a point mutation in a single gene. Epidermal cells in
these patients become fragile and rupture. Which of the following is the most likel y effect of the geneti c
mutation?

0 A)
0 B)
OC)
o D)

Defective transport of vesicles from the centrosome to the cell membrane


Disrupted assembl y of keratin into intermediate filaments
Lack of GTP required to fonm polymers of tubul in
Phosphorylation of actin monomers to promote polymerization

4. Rapid efflux of newty secreted bile from canaliculi into the hepatic intersti tium can be caused by structural
disruption or functional alteration of which of the following components of normal interactions between
hepatocytes?

0 A) Adherens junctions (zonulae adherentes)

O B) Desmosomes (maculae adherentes)


o C) Gap junctions

O D) Hemidesmosomes
0 E) Tight junctions (zonulae occludentes)
5. A coupl e seeks genetic counsel ing foll owing the recent diagnosi s of cystic fibrosis in their youngest child. Their
two older children are healthy and have normal sweat chlori de test results. Whi ch of the followi ng is the likelihood
that each of the unaffected chi ldren is a carrier of cystic fibrosis?
0 A)
O B)
OC)
O D)
O E)

1 in 2
1 in 3
1 in 4
2 in 3
3 in 4

NBME form 6-1


1. A 75-.year-old man devel opsa burning sensation Ollerhi s 1i gttflank. One day later, he develops an enc>tion otcllsteredvesides on erythematous bases ,n the same area.
Pa,nassoci ated w,lll the rash ,s sharp, Slabbi ng, andsevere A <*1JO that 1rhbots whi ch ol lheJoloYllng actions Is mos, app,opna1e?
0 A) eel wan syrC'leSIS
'# B) DNA repltcalion
( C) ONA I Op01S01T1erase a<*Vlty
o OJ Ergos1er01synthesi s
o EJ Prolease adMty
f: FJ Reverse transa1 p1ase adlvtty
t GJ Ribosomal protei n syrlhesl s
2. D<tnng development. neuralprecursors m,grale fiom Ille bowel mlo agans thal d"""l op fiom lhe primoraal go.t.As a result, which ollhe following organs 1 s mostl ikely to
conraingangl,a innervated by enrenc n8\l'ons?
A) Adrenal gl3nd
B) Kldney
o C) Paocreas
e D) Spleen
O E) Ure!et

Brain

PeripheraJ
tissues

Levodopa alone

Levodopa+ drug X
3. The bar graph shows the results oran expe1i menl mwhi dl levodopa was a0010ni slered 0<alyalone and m the presence old1ug X. OI\IQ Xis most lkelywl'lch ol the
t0101nng?

,. A) Benzll'op, ne
f" BJ 8<0mocnpt1 ne
CJ carbodopa
D) Dopami ne
E) o,Melhylcjopa
F) PerQOhde
,. GJ Seleg,li ne
4. A 43,.year-old woman has hypertension that has been unresponsiveto di uretics, an!jotensi n.corrlefting enzyme i rtii bi tors, and calcium channel bl ocking agerts. Adrug that
mimi cs !heautoi nhi OilOfYeffects orn0<nephrinei s staed llis drug acts atwtieh of lhe tonowi ng lettered sites on lhe schemati c or adrenergi cand ChOl inergi c nes
innervating an organ?

D E F

O A) @ 8) O C) O 0) O E) O F)
5. A healthy 28 year-ol d si ngle motherofttvee healthy chi ldren recentfy started empk>ymertas a secretary at a klca1 corporation. She has a vari ety ofheal th care pl ansfrom
wtieh10 ChOOse. Wrich or lhe tonowi ng 1ypes OIplans is most likel y10 have the IO'll'estoot.-Of.pocket cost for !hi s patient and her !ami ty?
o A) Feeor-service
e 8) Health maintenance otQani zatlon
o C) Personal lax- exempt health care account
o D) Poi nt--Ol-service
@ E) Preferred provider organization
4

6. A 31-year-old manwi thAIDS i s recei'Ying hi gliy active antiretr0\1ral therapy (HAART). Laboratory studi es show si gnificant myel osuppression anda deaeased pl asma HIV.
mRNA burden. To counte,ac:1 tr.s toxi city (,athe,tllan m()(lify hi santiretrovira1therapy), he tiegi ns t reatment wi th a hematopoi eti cgrowth factor. Thi rty mi nutes later, he
devel ops dyspnea, severe musde pain, vomiting. and si nus tachycarcia; his bklod p{essure is 80/40 mm Hg. Whict!I of the followi ng hematopoieti c growth factors mostl ikely
caused these newfil'Klings?
C A) Erythropoieti n
@ B) Fil grasti m (G-CSF)
() C) Pl atelet- derived growth factor
C D) Sargramostim (GM-CSF)
e E) Thrombopoieti n

7. A newborn i s born wilh a hypopl astrc mandible. Whch of lhe lol loWlng strucll.res I s also lkelyto be affected?
-' A) Cricad cartll age
J B) Greater coroo oflhe hyo1 d bone
, C) Malle<JS

0) Stapes

E) Thyroid cartil age

.0
20
30
"0 60
60
Pl a.sma concentrati on (moltL)
8. The dagram represents the di stributi on of pl asma concentrations for isoni azjd 6 hours afteroraJ admi ni stration to a large gro of sub ects. The bimodal dsfribution i s due tc
j
genetic variabi lity in which of the followi ng?

e A) Gastrointestinal pH and motiity


@ B) Hepatic acetyltransferase activity
e C) Hydroxylase polymorphi sm
e D) Pl asma chol inesterase acti vity
E) Pl asma protein structure
9. A 62 -year-old manwith a hi story oral cohol depel'Klencei s admitted to the hospi tal fortfansurethral resecti on of the prostate. Thefolowing morring,while being transported
to the operating room. he hastwo general ized toni c- CIOnic sei b.Jres withi n 5 mi nutes. Neuro1 ogi c exami nati on followi ng the second sei zure sh01,s no focal abnormal ities.
Whi ch ofthefolowing i s the most l ikety dagnosi s?
ct A) Alcohol withdrawal
e Bl Hypooatremia
e C) Korsakott syndrome
() D) Parti al compl ex (psychomotor) sei lll'e
e El werni cke encephalopathy

10. A 15-year-dd go hasv1111izlng adr enal hjperpl as,a. -.creased serum ccncentrab<lns or 11-deOX)'oortlCOSlerone ha reS<Aed ,n hypematrerr,a, hypokalerni a. Md
h)'l)efV()l em,a SenrnACTH concentratr<)n , s ono-eased, b" serumaldoslerone concentra1100 ,s <leo-ease<I. Admonl stratr<)no! dexamelhasooe normaizes the ACTH, 11.
deoxycorticosterone. aldosterona. and androgen concentrations. 'Michofthe folowing l abeled enzymes is mos1li kelyto be defeaiw inthi s patient?

o r-,........,,.....,_

...I

Eff r'
r.,.,..,_
11-0..........
"" 11010.,,...

11. A45--year-old man with an acuteirferior wall myocardial infa,ct hashypc:(ensi on secondalyto si nus bradycaldia. Whi ch of thekllowing drugs , s most appropriatekif ttis
pati ent?
e A) Acelyfdloine
BJ Alrop, ne
CJ Methaeholine
e DJ P1 1oCarp1 ne
e El PyridoSll!JTOne

12 A35-year-old woman comes tothe physician because of a 2.week hi story offever and headacheslhatseem to occur eve<y otherday.She recenly returned froma 1-tnO!ih
campi ng exf)e<)1>0ni n Carr()Odi a He<temperature i s 40.6'"C (105 1F). Physi<:al examinati on SllOWSdoaJ)l>Oresi s and $l)len0me0aly. A penpt,e,alDI OOd smear ShOWS
Plssmodium vivax. andtreatment WJthcNoroqui nei s started and followed witha c.oc.rse ofprimaqune. Whi ch of the folcMing is the best explanationforthe inclusionor
pc1 maQui ne inths patient'sme<11cation regimen?
e A) ce, eblal iOYOlvement
BJ Hemol ysi s
e CJ Hi gh paraS1te loadon 1he DIOOd smear
e 0) Mi ni mal toxioty
e E) Prevenbonof uo.nence or malana

13. A45-year-Old man with p00<lycomoled di aDetes meatus devel ops,...,.,shaki ng cliffs, tr,potensi on. and general ized malaise shortly after1 rtuSlon ofan an li"'1gal agent
fOt a l arge necrooc JeS1on on ttoe -pal at e WhCIIof 1he fottovng an11funga1 agents , s most kel y respons,t,le?

A) Afll)hoencm

B) Fliconazole
CJ Fl,cytos, ne
D) 1 vaconazo1e
e E) Terbinafine

14. Whi ch of thefollowing is the major di sadvantage of nal oxone as a treatmerifor opi oi d overdose?
6 A)
e 8)
6 C)
@ 0)
6 E)

As aparti al agoni st. it may produce depressi on ofthe central neNOUS system
It does notantagonize the effects ofheroi n
lthasaklwtherapeutic i ndex
It has a shorter durati on of acti onthan most opi oids
Tolerance developstoi ts antagonisti c effect

15. A 14. year-ol dgirli s brougl1 to the physician because offever and pain in the k:lwerextremi ties for the past week. She had a sore throat that resotYed without treatment3
weeks ago. Her temperature i s 38.9c(102F). Exami nation shows swelling ano t enderness of the knees and ankt es. three 1.cm nooui es on the rl gllt e1 Do11. and a new
grade 3J6 crescendo-decrescendo systol ic munnur. Whi ch ofthe foDov,ing antimicrobi al agents is mostappropriate to prevent recurrence of thi s disorder?
o A) Acycl <l'<ir
e BJ Doxycycl ine
e C) Fllconazol e
e D) l soni azid
E) Peni ci ln
() F) Trimethop<i m,st.MamethoxazOSe

16. A55-year-Old man comes tottoe physician because toe has had a 3.6-l<g (8-1>) vreigh1 loss ruing the past 3monlhs. He i s a conslructi onworl<e<. Hehas smoked 2 pad<s ol
cigarettes <lalo< 35 years. He has no history of-1 ,11nesses Vi1al Signsare nonna1 Pt,ys,ca1 exarrtnabOOshOws S\4){al)<C)oCtenderness t.Nlnal)'Si s shOlvs3
RBC/hpf, OW801'l>(, and no baaeria. Uri ne cytologyshows malignal'leels. Whi ch ofthekllowing i s thestrongest predi sposing risk fae1orfor ltispatient's condition?

A) Age

BJ
C)
D)
E)

Ci garene smoking
Gender
Heredi ry
Dcc<4>ation

17. A 13-year-ol d boy i s brought to the physici an by hi s parents for a routi ne wel khild exami nation. When al one withthe physi cian duri ngthe i rtelView, the p;at entsays thathe
i
occasl ona11y smokes cigarettes afte< school with hisfri ends. He says that he smokes fewerthan 10 ci garettes weekl y. When the pllySician inibatesa disa,ssi on about
smoki ng cessati on, the patient says, "Smoking isn't a problemfor me. " Whi ch ofthe followi ng respons-esby the physici an is most appropriate?
e A) "At wha1 point woLlcl smoking become a probl em foryour
@ 8) "Did you know tllat smoking has manyl ong.term health consequences?"
() C) "Do you wan1 me to tell your parents about yoursmoking?"
() D) "How Oda ni ce ki d like you get started smoking?"
() E) ' Why don'tyou ustqui tnow before it getsto be a bi gger problem. "
j
18. A 52-year-ol d woman ,comes tothe physi ci an because ofpain in her right knee.The knee i s red, wann, and swollen. Serun uric acid concentration is 11.4 mgfdl. Joint fl ui d
aspi rate shows negafrve1y bi refri r)Jelll crystals. klOOmetnacintoerapy i s starte<I ano sne begi ns a punne,restncte<I Oi et One weekl ater. her24,hour unnary\li e aci CJ
excretion is800 mg/day (N = 300-600 mg/day).Whi ch ofthe followi ng drugs is most appropriate to decreasethe serum uric acid concen irat ion in thi s pati ent?
A) All opunnol
e BJ Aspi ri n
o C) Colchi cine
e D) Probenecid
() E) Sulfinpyrazone

19. After 6 days of therapy with a psychotropic drug, a 35-yea,.ol d man i ritiatesmovemerisl owly and has a shufling gait. Muscl e tone is increased syrrmetrically in al
extreiriti es. WhiCh ofthe following drugs is the most li kely cause?
e A) Amttril)lyline
@ B) Chl orpromazine
e C) Di azepam

o DJ Fwxebne

e EJ PhenelZine

20. A 76-year--0l d man wi th a 15- yearhi story of type 2diabetes melli tus and hypertension has had dryskin, markedcold i rtol eiance, and constipationfor 8 months. He is 178
:
cm (5ft 10 in) tall andwei ghs 85 kg {187 l b); BMI is 27 kg/m . Treatment is startedwi th a full- repl acem.enl dose ofthyroxi ne (T4). Thi s patient isat increased risk for
devel opi ng whi ch of the followmgduri ngthe first month of therapy?
6 A)
6 B)
6 C)
@ D)
e E)

Angi na pectori s
Asthma
EXOl)hthalmos
Goi ter
Jaundi ce

21. A 19-year--0l dwoman comes tothe physi ci an requesting an oral cortraceptive. She i s sexual ly actrvewith one partner. She undeswent a spl enectomy 3 years ago after
sustai ni ng a brunttfauma iry tothe abdomen He< vital si gns are wi thi n normal l imi ts Pt)ysical examination shOws a we&,.hea5ed, mi dline abdominal scar. Laboratory
studi es sho'I/the presence of Howell- Joly bodi es in the erythrocytes. Thi s pati ent i sat greatest ri sk for mortali ty if she ,devel ops an infeai on with whi ch ofthe following
causal organi sms?

e A) Candida albicans
6 6)
e C)
@ D)
e E)

Moraxella catarrhalis

Mycoplasma pneumoniae

Neisseria gOIIO(fhoeae

Sl!eptococc.uspneumoniae

22 A 43-year- ol dwoman has sl ow retaxat lon ot strelch renexes ond a hoarse voice. She hos a de<:reosedpl asma thyroid- stimul ating hormone conQ!ntratlon thot Increases
onershe I s gi ven t hyrotropi n.rel easing hormone The mos1 likely <1/agnosl s Is hypothyroi di sm causad by whl ch ol t he following?
o A) Abnormally in the hypothalamus
B) Abnormally in the p1lui !ary gl and
6 C) Abnormally in the thyroi dgl and
D) Di etary iodine <leficl en,:y
E) Resi stance to thyroi d hQr mone

23. A45-year--0l d man comes tothe emergency department after inadvertentl y taki ng an overdose of metoprol ol. Hi s pul se is40/rrinand bl ood pressure i s 80150 mm Hg.
Gl ucagon is a<1mi ni ste,ed i ntravenousl y. Fi ve mi nutes tater, his pul se is 70/min and blOOd pressure i s 130!80 mm Hg, Whi ch ofthefOIOlling beS1Cle:SCri beSthe mechani sm
of acti on of gl ucagon i nthi s patient?
e A) I nhibi ti on of cardiac muscari ni c 2 (M:) receptors
e B) I nhibition of pllosphodi esterase
6 C) Open ng of K channe s
l
i
D) Potenti ation ofdrug- i nduced hypoglycemi a
@ E) Sti mul ation otadenyly1 cycl ase

24. A 19-year--0l d woman wi th a5-year history or poorly controlledtype 1 di abetes melli tusgives birth to a pl ethori c mate newbornwho wei ghs 5 kg (11 l b). Twelve hoursl ater
ttoenev11>om has hyPOglycemia. Whi ch otthe followi ng is the mos, l ikery cause ofthese findi ngs in the newtwn?
e A)
e B)
6 C)
D)
() E)

I ncreased fetal pancreatic seaetion ofmsul in


I ncreased fetal producti on or c0f1isol
I ncreased tetal producti on or grovAh hormone
Maternal anti- i nsuh n antibodies. wlli ch cross the placenta
Maternal insulin. whi ch crosses the pl acenta

25. A24-year--0l d medi cal student recei ved hepatitis B vimsvaccination 2 years ago. Which of the followi ng findings i s most likety onseroklgi c studi es at thistime?
6A)
6)
OC)
6 D)
6 E)

HBsAg

HBsAb

HBcAb

HBcAg

26. A 32- year--0l d nulli gravidwoman is being eval uated for in fertility. Her last menstrual peri od occur1ed 6 months ago. Menarche oc,curred attheage of 13 years; menses had
ocCIJ're<I at regctar 28- day interval s. Physi cal exami nation shows a mi lk)' <Jischarge from l)()(h breasts. If pharmacot herapy is ind icated,the most appropnar e treatr ner11i s a
drug wi th whi ch ofthefolo11i"ng mechanisms?
(') A) I nhibi ti on of O:adrenergi c receptors
6 B) I nhibition of dopami ne receptors
6 C) I nhibition of estrogen receptors
t, 0) Sti mul ation of0z--adrenergi c receptors
E) Sti mul abon of<IOl)arrine receptors
o F) Stimul abon oCestrogen recept(l(S

27. A patient wi th a peptic ul cer is treated wi th d1ug X. Several weeksl ater the patient is al so gi ven diazepamfortrealmeli.ofanx.iely, tensi on, and insomni a. The patient fa.ls
asl eep much moreQui ckly and steeps f(l( a tonger penOCI or ti methan normally exl)8(i enced wi th di azepam, Drug Xis mostlkel yto be whi ch otthe followi ng?
6 A) Atropi ne
" B) Ci metr dine
( C) Ornepratole
0) Propanthellne
o E) Rani bdi ne

28. Bacteri al strainXi s resi stant to ampicillin and sensi tive to kanamycin. Bacteri al strai n Y is resi stant to kanamycin and sensitive to ampi ci lli n. Bacteri al strai ns X and Y are
g,own in mi xe<I cullu(e in me<llum without an1i biotics, thenthe cunll'e is plate<I on medi um containi ng both ampi dllin and kanamyci n. Bacteri al col oni es Q(Ow on the pl at es.
I n a second e.xperimenl, ONase i s added tothemi xed cllu(e medi um. Whenthis mi xed cothxe is plated on medi um containi ngboth anti bioti cs, no coJonies grow.
Assumi ng that bacteri al cells areimpermeabl e to ONase, whi ch ofthe followingprocesses bestexplai ns these obsecvations?
C A) Conjugation
C B) Mutati on
@; C) Transduction
C D) Transformati on
C E) Transposition
29. A 32-year--0l d woman and her31- year- old husband cometo the physi cian because they wantto have a babybut are concerned about the impact thi s will have onthe wife's
emo1i ona1 s1a1e. She has haO lhree ma or depressi,e epi sooes andnumerous pani c attacks wi lhi n lhe paSl 10 years. She di sconbnued herantidep,essan1 med,cali on 6
j
n-.orihs ago and has been asymptomatic forthe past year. She i s concerned that she may become deiP{essed while pregnant and will be unabl e to take any medication
because ofthe pregnancy. Whi ch ofthe followi ng is the mostappropriate response forthe physi cian?
e A) "Thedepression associated with pregnancy is ll'lrel ated tomajordepression; therefore, you areno1 at greater risk for bei ngdepressed whi le pregnant."
@ 8) ''The risk for depression is greates1 afterdel iveryand depressi on dtri ng pregnancycan often be safelytreated. "
e C) ''The risk for depression is greatest during pregnancy, bU: electroconvul sivetherapyi s qui te safe. "
6 D) "Si nce you have been S)mptom- free forthe past year, you sholt:I not be at greater ri sk fordepressi onthan the normal populati on."
e E) 'Wewill need to followyou cl osely si nce the suici de rate is hi gherforvromenwhoarc pregnantthan forwomenwho are notpregnant."
30. A 51- year--0l d man is admitted t.othe hospi tal fortreatment ofadvanced pi..tnonary fibrosis. Hi s IQ is 78, and he has a tistory of major depiessi ve disorder. When
quesli oned, he says lhalhe no IOnger smol<es. bul his wi fe i mrneoia1eyens a1 hi m ano says 1 ria1 he conii nues tosmoke oespi le tier besl efforts loten hi m<Jaily no1 10 do
so. Throughouttheinterview, the patient continually opens his wallet and counts his credit cardsto check that they are al there. Physi cal exami nati on shows no other
abnormal ities. Whi ch of thefolo1ng psychosoci al factorsi s the strongest contraindicati on tol ung transplantati onfor thi s pati enl:?
@ A) Continued cigarette smoki ng
6 8) Current marital discord
e C) Hi story of majordepressi ve di sorder
tt D) Mi ld mental rctardati oo
e E) Obsessi ve-compul si ve tisorder
31. A 32-year--0l dwoman recei ves a dose of warlarin 01al lyfor a large deepvenous thrombus ofthe rightl eg. Shei s curenttytaking an oral contraceptive. Her pr,othrombin bme
is 12 sec (I NR 1. 0). Tv1<1 hours afterlhe admi ni strati on or wanarin, he< pro1hroml)in bme remains 12 sec (I NR 1.0). Whi ch ofthe ronowi ng bes1 explai ns wily 1 his pati enrs
prothrombi n ti me dd not change?
() A) Thelarge thrombus increases the dose ofwarfarin requi ted
() 8) Oral contraceptives accel erate the cl 'earance ofwartari n
C) Preexisti ng circul ati ng coagulati onfactors have not been c eared
l
() 0) Prothrombin time is OOl a measure ofthe efficacy otwartarin
() E) Walfarin should have been admini stered intravenously
32. A27- year--0l d woman has had mensesonce every4 to 6 months si nce age 16 years. Menarche oca.nred at age 12 years, and she had regul ar 28..<fay cydes until she
o ned lhehi gh schoo11rae1<learn a1 age 16 years. She now runs 3010 40 mi tes wekJy. She ea1s a healthy di et She is 170 cm (5ft 7 in) 1an anct wel gns 61 kg (135 ll)J; 8Mt
j i
s
i 21 kgfm2. Thi s woman has an increased risk ror deveJopi ng whi ch ofthe following?
6 A) Anorexi a neivosa
8) Endometriosi s
@ C) Osteoporosi s
0) Ovarian cancer
6 E) Vi tamin 812 (cobalami n) deficiency

C
0

j\_
NE
I

DrugX
o.__-=----

Ti me

33 Thedo agramsi-s con1rac1, onofa l>IOO<lvessel segmen1 in responseio"' "'"oadmnsuabonof norep, nei,tnne (NEJ anct drugX Drug x,s mostlkely1obewfl1Chdthe
following?
A) Cocai ne
B) Guanelhidine
C) l soproterenol
0) MOIOj)(olol
E) Phenoxyt,enzamuiE>
F) Reserpi ne
e G) Tyrami ne
34 A 16-year--Old boy comestoIle physi aankJra physicalexamnation requredfor a bfeguud,ob al a local pool. PhyS1calexami nabon shows no abnonna.Mties. Whi chdthe
following a<MCe by the physooan,s mos1 afll)(opri ale?
A) Advise tom 10 wear a wt0e""1m01ed llal SIMlgiasses. MCI sunscreen
e, 8) Suggestthal he weargoggles when swvrmng , n the pocl
e C) Ten tom he coold make mote money mowing lawns
0) Ten t.m that he 1s l1 ne anctto erjoy llmsel at the pool Illssummer
E) Warn hlm1<>wa, 1 about 1 hou1 alle1 eabrci before goongsw,mml ng

35. A 42-year--0l d man has pancreatibs associ ated wi th hypertri gtyceridemi a. Hi s brother and father have a si mi lar condition. The patient's serum VLDL concentrationis
icreased. Whi ch olthe followingis the most aPl)rOPriate therapy for the hyperi gtycenoemi a?
o A) Chol estyrarr,ne
@ B) Gemf1 brozil
o C) LO\lastah n
o D) Probucol
o E) Psylli um hy(lroplilic muci llOld
36. An l8-year.old womanwho has hepatocellul ar carcinoma i streated wi th a nevr chemotherapeutic agent. One week after i nitiation oftherapy,exami nation oftissue obtai ned
on Oiopsy ofthe iver snows that many neoplasti ccells have decrease<! in size. Many fragments of memllrane, bO\JnO Slrvctures cO<llai ni ng conclense<t nuClear chromatin
andorganelles are identified. Electrophoresi s on agarose gel of DNAi sol ated from this tissue shows a "l adder" consi sti ng of rrM.Jlti pleregularly spaced DNAbands.
Acti vation ofwhi ch ofthefolcwnngenz)'mes is most liketyto be responsibl e for thealterati ons in ONA?
o A) DNAl igaso
o B) DNApolymerase
@ C) Enclonuctease
o D) Phosphol ipaseA,
o E) Transglutami nase
37. A 55-year--Old man i s gi ven atropine and ITidazola.m before undergoi ng hip replacement surgery. Anesthesia i s inducedwith thie>penal and maintained wi th nitrous oJOde
and sevoruane VeCU'ooi 1,m isadmi ni steredto Obtain musde relaxation One hOI.I"after comprienon otIJ'I& operanon. SPOntaneous respi rati on haSnot retll'Oecl, and
stimul ationat1he ul nar nerve irdc.ates contiooed neuromuscuar blockade. Which ofthe followingctugs is mostappropriate for restoring neuromusaJarfi.n:tion i n ttis
patient?
A)
0 B)
,. C)
a D)
-. E)
O F)

Atropi ne
Bethanechol
Carbachol
Noostigrri
Nicotine
Pral iooxime

38. A49- year--0l d man undergoes radical neck dissection fora maignancy. AJI fotX parathyroid glaOOs are iemoved. He has hy-pocalcemi a postoperativety. Cal ci um
concef'trati on re1ums10 the reference range wi th cal ci um anc:s vitamin O sp1 emef'tati on WhiCh ofthe ronowi bes! describes he result of ross of para.thyroid h0tmone on
vitami n D metabol ism?
(') A) Decreased cutaneous synthesi s of vi tamin O from ste<oi dprecursors
() B) Decreased hepati c conversi on of vitamin D to 25,hy'droxycholecat ciferol
(') C) Decreased intestinal absorpti on of di etary vitamin D
o D) Decreaso<l ooml>erol 1, 25,di hy(lroxydlotecalciferol receptors
E) Decreased renal conversion of 25-hydroxyvitami n Oto 1,25-dihyd(oxychol ecalci ferol
F) I ncreased hepati c catabol ism otvitami nO

39. A 25-year--0l d man comes tothe physi ci anfor a follow-op exami nation. Two weeks ago, he undeJWent ingui nal orchi ectomy and operative removal of the retroperitooeal
l ymph noctes because of ameastati c nonseminomatous ge<m cell h11no< of thetes1es. Al tOday svisi t. the physi cian recommeix1s adjuvaot cnemotherap.y incu:ting
vinbl asti ne. Vtlhichof the followi ng bestdescribes the mechani sm ofaction ofvinblastine i n this patient?
o A) Alkyl ates DNA
o B) Causes si ngle, and dodllE!,strancled DNA breaks
o C) Destroys asparagine
() 0) I nhibits <ti hydrofotate redue1ase
@ El Prevnts rni aotul>l.11! Olymerizati M
40. A 12.year-0l dQJ is brought to the physician by her parentsbecause Ola S<>re neckfor 4 days. Her temperatur e ,s 37.7'C (99.9'F). Physi cal examination shows tender
swelli ng anteri or to ancl Just above the thyroi d notCh ofthe neck. The physi cian explains thatthereI s a cystI nthetract at ong whi chthe lhyroi o gl and descended All of the
teact ti ssue must be removed between whi ch ofthefolo,nng two structurestotreat thi spatient's conciti on?
(' A} Left IObe olt hyroi d gt and ancl tonsilarfossa
f> 8) Ri ght l obe of thyroid glancl and epl g10tt1s
c C) Ri ght l obe orthy<Old glancl ond hyoi d bone
(0, D) Thyroi d I sthmus and foramen cec,m
E) Thyroi d i sthmus and pi ri rorm recess

41. After bei ng gi ven an infusi on of manni tol (400 mM),a healthy person is most likely tohavewhi cflof lhe foll owi ng changes in plasmaosmolallyandpl asrnaAOH
(vasoptessi n) concentraUon?

280

290

300

31 0

P&asma osmol alily (mOsmol /L)


O Cont rol bek>,e itlfus.ion
() A)

0 B)

@> C)

0 D)

0 E)

42. The dashed line on lhe graph shows aM!<>lal ventilabon (V4) as aftndion ofPaco,(arteri al co,tension) under nom,al condi ti ons. Whi ch otlhefollowingcurves best
represents Ille rel abonsh.pbelWl!<!<l Paco,and v. aMer equ,!bratmwi th 1 8 MAChalolll8ne?

PaC02
A)

o B)

<!l C)

DJ

E)

43. Certain chromosomal abnormalities can reslli n rudi meriary develop of the cerebral hemi spheres. Which of the rollo'Hing l abel ed structures in the aoss section of a
normal brai nstem is expected to be mosi underdevelope<I as a resun of Suell 81\ anomaly?

O A)

@ B)

O C)

O D)

0 E)

44. A68year--0l d man comes tothe emergency departmentbecause of chestpai n for 30 mi nutes. An ECG shows a regul ar rate and rhythm and invertedTwaves. Whil e
l abO<atory SI\K1i es a,e bei ng done. he i s gi ven oxygen and aSl)irin ASpirin is most apptO()(i a1e inthe tteatmelll (l{ttjs patient because otthe inhi btlion ofwhi ch 01the
folowing enzymes?

A) Cydooxygenase
o B) Lipoxygenase
o C) PhosphOl ipaseA,
O D) Prostacycl in (PGI ,) synthase
o E) ThromllOxane A, syntllase
45. A 30 year-<1l d woman i s bei ng treated with leuprol idefor estrogen-dependent breast cancer. After 2 months of continuous treatment, wtichof the followi i s the most likely
effect on tile secreli on of-.rei ni zinghormone anCI f011c1 e.stimu1 ating hormone from tile pituotarygtano?
Lute1 1"1z:mg Honnone
Folde- sti mul ating Hormone
A)
I
I
0 B)
I
OC)
o D)
t
4

OE)

..

...
..,

46. A 48- year-ol d man comes tothe physi cianwith tiswi fe because of a 10montll hi storyofheadache and daytime sl eepiness. Hi s wifesays that he snores at night and often
stoPS breathi ng wtil e steepi ng. He has no history of anymajor me<J1 ca1 i llness anCltakes no meoi catl ons. Hei s 173 cm (5 ft 8 in)ta and weighs t 15 kg (253 l b); BMI is 39
kgfm2 Hi s blood pressure i s 1501110 mm Hg. Physical examination shows no ,other abno,malities. Overni ght pofysornnog,aphy confinns the di agnosis. Nocturnal treatment
wi thnasa1 bi- level posi tive ai tway pressu(e is most appropri ate fort l'ls pati ent to accomplish whiCh of the following?
@ A) AJl owi ng the ai rway to (emai n patenl ror appropri ate venblation
o 8) Decreasi ng thewO<'k ofbreathi ng du<i ng the re51>1ratory cycl e
o C) Reduci ng a irway secretions duri ng the venti latory cycl e
e D) Stimul ating i ncreasedl ung compliance on inspi ration and decreasedrcx.g oompllance on expi ration
o E) Stimul abng , ncreaseCI respi rabons by spli nbng the mouth and nose

47. A 20,year.<i ld woman comes tothe physi ci an ror an examination prior to partici pating on the collegevarsi tyvol leyball team. She has a 1.monU, tistory of mild racial acne.
Physi cal exami nati on shows no otherabno<maliti es. Treatment wilh a topi cal agent is begun. Three weeks l ater. she returns to U'le physi ci an because ot!leg swelling and
wei ghtgain. Physi cal exarrinati on shows severefaci al acne and acne on the chestand back. There is 2+ pitting edema ofthel ower e.xtrerrities. Laboratory studi esshowa
mil dly increased serum direct bi fi rubi n concentration and serum transami nase activities. The most ikely cause of her condi tion i s self. admi ni stration ofwhich ofthe follO'lvi ng
slJ!>stances?
O A)
o B)
O C)
o D)
@ E)

ACTH
Ethi nyl estradiol
Growth hormone
Hy<liocorlisone
Methandrostenol one

48. An irrvestigational antiviral drug blocks acthlityofa virus- encoded, RNA-dependent ONA polymerase. Thi s drug is most likelyto inhi bit gepli cation ofwhi ch ofthefolovnng
viruses?
o A) Adeno,,rus
o BJ Herpes si mpl exvirus
@ CJ HI V
o D) I nfluenza wus
o E) Poliovirus
49. A00.year.ol d man comes tothe physi ci an because ofa 3-montt, tistory of difficulty urinati ng, constant loss of small amol111s of urine, and the sensation or incomplete
bl adder emptying, He has hypertensi on, type 2 di abetes melitus. and !)Olien alle<gi es C\l'rent me<11catl ons incl ude metopro1 01 , h)'drochl orothl azide. insulin. magnesi um
hydroxi de, and diphenhydrami ne. Hi s bl ood pressure is 176198 mm Hg. Physi cal exami nation shows no other abnOf'ma!i ties. Whi ch ofthefollow ng medi cations is the mag
i
l ikety cause ofthe urinary probl ems in tt'is pati ent?
@ A) Di phemydrami ne
O B) H)'drochl orothi azide
O C) l nSltn
o D) Magnesium h\'droxide
o E) Metoprolol
50. Snakevenom isa potevasodi latorofsome pof1ions ofthe artefi a1 tree, parti cul arl y in the arteri ol es of skel etal muscl e. Thi s effecti s unaltered by '3.adrenergic bk>ckade
but is markedl y exace<bated by pretreatment with capt()l)(i l. venom. i nduced release of whiCh of the fol lOWing substances ls most consistent with these findi ngs?
o A)
@ B)
o CJ
o D)
o E)

Adenosi ne
Bradyki ni n
Hi stami ne
Pl atelet- activating factor
Serotoni n

Form 6-2
1. After sustairirg seveie burns, a 4S.year. o(d man becomes hypo!eosi ve to, se1e1al hours. Hi s U"i ne outpul subsequently decieases l o 150 mlJday for 3 days. Microscopic
exami nati on of oone onthet hi rd day Is mostli kefyto which ottoo ro1owl og findi ngs"/
@ A) Casts contai ni ng renal tlbular epi theilm
O 6) Large num1:>ers ofocytes
O C) Large numbers ofuricacid aystal s
o D) Nu me,oos eosi noptii ts
O E) Nu rooroos oval rat t)Odi es
2. A 55.yea,. ol d man comes to the physi cian's office because ofa headache and a feeli ng of ful loo:ss i n hi s head and neck. Physical exami nationshows dil ated chestvessel s
and p,Je,thoramatI s WOl'Se\\flet'l00 liesdown. X- rays oft he chest ShOwa righl upperIObe massand a wi OOnedmt>di al num. The most kely cau se of! hi s patient's
symP(omsi s c.omptession ofwhi ch ofthefolowi ng struwxes?
O A) Extemal j ugular vei n
o 8) Hemi atygoswin
o C) tirerl otvooa cava
o D) titema
l j ugularvei n
o E) Ri ghtlymphati c duct
O F") Ri ghi mainstem bronchus
O G) Subcl a'Jianvei n
@ H) Superiorvena cava
3. A 41. yea,. ol d man has had di anhea, wei grtl oss, and di plopi a for 3momhs. Physical exaffinalion shows hyperpigmenta!i on and synovili s ofthe knees and ankl es. Stoolfat
cooten! ls increased. Small lltestinal tis.sue stai ned wi thperi0d1 c acl d-Schllf (PAS) sllOws macrophages containi ng di astase.resl staOI PAS-poSlti\'e sl ekl e- form parti cl es;
electron microscopyshowstheparti clesto be rod- shaped bacili. Which ofthe followi ng isthe mos1 lik.ett di agnosi s?
O A) Amyl oi dosi s
o 8) Bacteii al orgrcvh'ltl
o C) cellacsprue
o D) Crohn disease
o E) Hypogarm139lobuli nemi a
O F") tltesti naJJlPha.ngi ectasi a_
O G) Pancreati c insuffici ency
O H) Systerric mastocytosi s
o I ) Tlcpical sl)fue
1, J) Whipple di sease
4. A 61. yea,. ol d man has had progiessive hoarseness overthe paSI8 months. He has smokedlwo packs of cigarettes dai lyfOf40years. Cervical lymph nodes are pal pable.
Laryngoscopyshows a 1- cm! rregtJar, ul cerated mass! nvoMog thetruevocal cord. M1stol oc exarmnabon ot tlswe:fromthemassI s most kely to show \\'hidloft he
following?
:l!l A) Adenoca,ci noma
o 8) Lymphoopi tOOlioma
o C) Papl lloma
o D) Pot/!)
o E) Sc:vamous eelcarcinoma

5. A 35- rear- ol d man has hact burni ng epi gastric painfor 1 year. I niti al ly,the pai n occtnecl a few hoursaftermeals. bt.-rete11Wi t has become persi stert. The pai nhas not
responded to H:-<eooptorartagoriS1s antibi otictreatment for HelieobaClorpytori. ACT scan ottoo abdomenSh0'11s a massi n tile head ofthe pancreas; the surgi caly
removed massi s shown. Whi ch oi too rol lO\'nl'IQ paocreanctumors is mol iketf?
O A) Aci nar carcinoma
O S) Ductal adenocarci norna
@ C) Gastrinoma
O D) Glucagonoma
o E) VIPoma
6. A 50.yea,. ol d man comes to the physi cian because otsevere chest pain fot lhe past several months. Pulsei s 74fmin, and bl ood piesslJ'e is 160198 mm Hg. Further
eval uation shows aMl c stenos! s. Coronary bl OOd flowi s greaterthan normal. Wtieflort he followi ng cottri butestotoo increased conary blOOOnow In thi s patient"/
O A) Decreasedl eft\entric.ul aroxygen constmP(i on
O 6) Decreasedl efl ,enlricul arpressll'e
@ C) Decieased l eft \enlri cul arv/Ofk
o D) fn<:1eased adenosi ne concent ra.!i oni n <:atdi ac tissi.e
o E) tt<:reased oxyoen concentration 1n cardiac tisS\Je

7. A 70.yea,. ol d man comes to the physi cian because ota 2. yea1 hi storyofuri nal)' hesi tancy and r,equency. Hestates that hewakestwo to threet imesa ni ght to YOi d, and thal
dll'log the paS1 year00 has Md lnte,m1 ttent pai nvrit h u,l nat! oo. Physi calexaminati on soows oo aboo,ma.Hles. Re-suns a url nafySl s arewithi n too reterooce range.
Ultrasonography ofthe abdomen shows bi lateral di lation otthe renal oolectl ng system. Whi ch olthefol lowir.gi st he mostl ikelydi agnosi s?
O A) Adenoca,cinoma oft he prostat e
jj B) Betli gn Pf'OS.mie: hype,fl)l asi a
o C) Neph,obl astoma (Wims tun()(}
o D) Rooat0011 carcinoma
o E) Urerolthi asis
O F") UrewopeMc;.inction stricture
O G) Ureqhral vaNes
O H) UrOlheli alcarcinoma ofthe bladder

8. A 35.yea,. ol d womanhas a 1- day hi story offevei, coo and ri t. si ded ches1 pai n. He, cou!jl is Pfodudi ve drus1, col ored sputum. She has dul1r.ess alt he posteriOf ri!jll
IO'l1er llrlg, andltlete are increased b,eath sounds c,..er the same a,ea. Vi tal si gns a,e:
Tenl)erattM'e
PIJlse
Respi rati ons
Bloodpressure

39.1.C{102. 3.F)
108/mi n
24/min
11 0J80 mm Hg

Which ofthe fol lO\ng addi ti onal findi ngs indi cates a consoli elative procoss in thO posteriork:met rightl uog?

o A) Decreased tactilefremitus
O 8) Oecieased YOcal frerrilus
jj C) Egophooy
e O) Trnl"hol<:hi fl T ht ho riOhl
El wr...zlng

9. Thel eftkidneyShownwas obtAlnedat autopsy \'Aa 00-year-old man. Thesetis.sue allerati l)nsm:>stl ikety represeNI which ofttlefOllowi ng proces.se-s1
8 A) Ctronic analgesi c abuse
O 8) Ctronic P)'el onepf'lriti s
O C) Di abetic nepliropathy
0 D) Hype1tensi ve f\eS)hfOScte{OSiS
o E) urewaJobSlf'Udion

10, A26 Y9$r Qld mtn wi lhta 15 Y"' hi $t<>ry QItype 1 di .ibexes melli tvs ha.s protei nuri a, A phQ!Qmi Q'<>gt$phoft issue obt.ai ned QnrentI biopsyi s shown Whi vh Qith9fol!Qwing
i s too most kely dlagoos.s?

o A)

O 8)
O C)
O 0)
li) E)

Chronic OMl>hri1is

F"ocal segmertal glomen.fioS(l erosis


Merrt,ranoproli fera6ve gl omerul onept,riti s
Merrbranous gcmenJoneplvitis
Nodula1 gome,JUoscle,osi s

11. ReabsQfPli on ofthe la1gest fracti on oflhefile1edl oad ofHCO>-i s most likely to occur al whi ch of lhefo!owi ng labeled regi onsi n the ciaam of lhe neph100 shown?

@ A) O B)

o C) o D)

O E) O F)

12. An 8.yeru..old boyhas had generaized edema for2 mooths. Laborat orystudi es of se,umshow:
GIJCOS8
C<eati ni ne
Uiea ni trogen (BUN)
Albumin

89 mg/<IL
0.9,no(cfl
11 mgi'dl
2.8 gtdl

Urinalysis shows 4 p1otei nand oval fat bodi es. The most fikety cause is a di sorder irrroMng,mich ofthe following slrudmes?
o A)
o 8)
8 C)
O D)
o E)

Affe<ert anerioles only


Affe<ert and efferert arterioles
Glomerular capil laryloops
Glome1ula1 mesangial regi on
TubulO-i ntefitial <e-gi on

13. A42.year.old man comes to the physician because ofa 2.yearfistory of heartburn, wlichi s worse al righl He has smoked 2 packs ofcigarettes dai t,'fot the past 30
yeats. Upper enooscopyshOws esophageal eroSlons. 'fl ssoo Obtained on biopsy or U'le iower esophagus IsSf\O'lm. \.'Vhictl ol t he ronowu'IQ Is U'le moSI likelj compli cat! oo 01
tis disease?
I) A) Adenocarcinoma
B) Kaposi sarcoma
o C) Leiomyosatcoma
o D) Malignant hOma
o E) Sc:,.,amous eelcarcinoma

14. A37. year.ofd man comes to the physician because ofproessi ,e he.artblm overt he past6 mooths. The S)mploms are most severe after eating la1ge orspi cy mealsand
are parti al'/ reliQ\'Odwith aritaCidtherapy. An endoscoptc bi opsyspecf meri oftll8l oweresopllagusSfl<WIS esophagitis. The mostliket)' cause 01the esophageal changesI s
a substance secreted bywhi ch ofthefotowi ng cell l)Pes?
O A) Ctief cel ls
o B) G (ga,i n) cell s
O C) Gobtet Cell s
D) Mucous cells
E) Parietal cel ls

15. A60..year,ofdwomani s undergoing surgicalexcisi on of a renal cell carcinoma lhrombos thal extends into the 1i g renal vein. Ou1i nglhe procedure, t he \'ena cava is
temporarlt, occk.lded abOve the ,eoal \lelns.I ncreasedvenous pressurei s mostli kel y tobe noted Inwtil ch of the tol lOwi vei ns duri ngttle period of occusl on?
@ A) External iiac
o 6) Hec,eoc
O C) '1feri ot mesenleric
0 D) Pata!
e E) Spkl,l\i:C

16. The foll owi ng pumonaiy pJessu,es (in nvn Hg)were meastJ'ed in an arimal who was bfealfli ng spontaneously:
Hydrostatic capi lary l)fessure
Oncolic capillary pressure
Hydrostabci ntersti tial press,xe
Oncotic intersti ti aJpresst.1e

12
20
- 5
15

The fillfatl on coeffi cient01 t he pulnooaty mi crovascularure i s 1 mUmi fV100 gHssuelt"t'tn Hg pressure, and the ,enectlon coefficient1 0<protei n1s 1 Whi ch Of th& lolOwi ng
best represents tile magnitude and di recti ooofflui dflux inthi s arimal ?
8 A)
0 6)
o C)
o D)
o El
O F)

Di recti on
Magni tude (ml/mi n)
cao! Harytol nte<strtial soaoo
2
irterstitlal space to capllary
2
capillaryto1 nte<sbtial space
12
i11ers1Jtial space to capi eaty
12
capillaiytoi nta-sbtial space
22
irersHlial space10 capilaty
22

17. A 27.year,ofdwoman has a sii>1aseH ar craniopha,yngi oma., wlich leadsl o pressure. i ncllced atrophy oft he posleriOf pi tuilary. \Alhi ch ofthe foll owing responses is most
hkel y Inttle kidney?
O A) Decreased production of1. 25-di hyc.-oxvcholecel ci ferol
O 6) Decreased sodi um reabsorpti oni nthe proxi mal COfl'IOk.rted t ubule
O C) '1creased calci um reabsorp!i on in the PfOXi maltubule
o D) tic,eased wate, reabso.plion by!h e thi ck limboft he k>op of He1
E) PrOducti oo01 a hypotor'lc urine
18. A 60..year,ofdwoman comes to the physi cian because ofvagi naJ bl eedi ng. Menopause occunedat age 52; she has not hadvagi nal dryness or hol flashesand has nol
been on honOOOc repiaoomenttherapy. She riastype 1 diabetes melitus and v1e1ghs 114 kg (2SOl b). PhySical exami nation shows absence olvagi nal at rophy and scart
bl ood fromthe cervix. An increase inthe serum concentration of\..tlichoft he followi ng hormones mos1l ike accountsfor thesefindi ngs?
O A) Cortisol
o 8) Dehydfoepiandfosterone S\Jl fate
:ti C) Es11000
o D) Progesterone
o E) Probcti n
19. A newbomwho has not yet beenfedi s having respi rat Of)' di stress. The abdomeni s 1e&.1.li vely Ra.I , and l heiea,e no bteatllsounds 01e1 lhe l horax ont he left. F1ontal and
lateral plain x.-rays oltheCf\eSt sllOw a shi fttothe nght oft he medi aSli nal struc:turesand opacillcatl ooorthe te-ft hem1 1tlomx exceptror several sau saoe- Sflaped ai r
col leai ons posteriortyand inferiorly; the right._,ng i s aereted normatf. Whi ch ofthefol'.owi(9 abnormal itiesi smostl ik:etf?
O A) Absence oflhe perica1dhm
jj 8) PosterOlatemloo,nia oft he leotll'IE:'1nidi aphragm
o C) Pul monary sequestrati on1 nlhe leftl owerIObe
o D) Reuostemal herni a
o E) Tracheoesophagealfistul a
20. A 31.year,ofdwoman has anenla1ged ri ghl ovary de!eded on peMc examination and confi1med on endovanal ullrasonog,aphy. At laparoscopy, an 8.cm cystfiffed wi lh
thi ck b<own ma1 erla1 I s removed. lil stol ogic secti onsstv::t11that t he cystI s HOOd by macrophagesoortai nl abundanthemOSaderin. Whi ch 01tile tollowt Is t he mostl ikely
di agnosi s?
O A) Adenomyosi s
o 8) Betlign cysticI E!mtOO'la
:ti C) Endome-trl oma
o D) Pot,<ysbc o,,ary
O E) SIIUl11a ovarii
21. Comparedwith the contents offreshly secre{ed hepatic bile, the PfOporti onofwhich dtile following is decreased in bile storedi n the galbladde1?
it A) Bi ln.t>J n
o 8) Cal cium
o C) CholeS1erol
O 0) Pocessi um
O E) Wales
22. A 31.year,ofdwoman has had several epi sodes drehosl ernal chest pai n while al ,est di.ring tile past 6 monlhs. The most ,ecenl episode occurced while shewas asl eep
and prOduced several mi nutesot ST segmetll el evabon ooa home cardi ac moni tor. She has no known cardiac risk !actors. Whi ch ottheIOIO'l,l ng events In al arge
epicardi al arteryi s most likely responsibl eforthechest pai n?
O A) Endo\llelial ceb dysfuncti on
o 8) Fonnation oca ll'li nli brous.capped plac;ue
o C) NeutrophiticJ ntltrabon wi thin ttleJ ntirna
it D) Pl aque rupn.re and ooronary emb'zat1 on
o E) Pl aque ruott.re and nual hemorrhage

23. A3S.year, otdwomani s b1oughlt ol heeme,gencydepart:rnett after lwling a stroke. Eval ua!ion shows i nlemii tlettty decreasedl eftveliri cu1a1 fil ling. Whid!ort he followi ng
be-st e)q)lai ns hercondi tion?
@ A) Atrial my.coma wi thsystemi c emboli zati on
O 6) Carciac rhabdomyosarcoma wi lhmetastasistothe central nervous system
O C) Di ssemi nated i nlravasOJl arcoagul ati oo
o D) Pulrnona,y embo! us,'lit h ftagmentati on
E) Tfi cusoi dvaM? encsocardltis

24. Two days afte1 sustai ni ng an ao.te myocardi al infarcti on, a 65.year.o)j man develops anaclte a! ri ovenlrioJa1 bl ock. Despi te resuscitative efforts and the p1acemen1 ofa
pacemaker, tile pa
bent di es.An autopsy is requested. I SChemi c i njurytottleatnoveitri clJar nOde is t he suspectedcause otltll s pati ettrs alflovoolficliac bl ock. Eval.latl on or
a secti onfrom which ofthefolowi ng locationsi s most appropri ate toevaluate this patienl's atriO'Jeitri cular node?
O A)
B)
o C)
fW D)
c, f:)

te,atrial seplum nea1l heattactvned ofthe sepia! cusp ofthe tri cuspi dvalve
Supooorborder 01the muscul arportion 01the il'ieMttri cu1ar sep(u:n
Wall or1ne lefi atri um Mtween tile oofioos or1ne ri !1Jt and lefi pul monaryvei ns
Wall oftne ri !1Jt atn um al ongtheori fice oft he inferiorvena cava
Wl:111 uf Uu: 1i yli1l <1t1i um i;$1 UUl:J Un:u,i fit.:t: Qf un:i t;U<::l iUf wnu..w

25. A54 year, ofd man has ao.rte pai n se\eralhrnrs after rubber band lgati onora prol apsi ng hemorrhoi d. Whichoft he foll owi ng i s lhe moSI fkety expl analion for the pai n?

o A) The tigat! oo mvolvedtoo mucousli ni ottoo analcanal


o 6) The ,gati oo stopped bl ood s pJ)tftothe hemorrhOi d
u

O C) The tigati on was done atthetevel ofthe anal coh.rnns


@ D) The figati on was done i nferiortot he pectinate (mucocutaneous)l ine
O E) The figati on was doneon an i nlemal hemorrhoi d
26. A29.year, ofd pri mig1avid womanal22weeks'gestalion has arociety and poor concenlJali on. She has gai red 2.3 kg (5l b)during the pregnancy. Her pul se is 1 10/mi n and
bl ood pressureI s 132165 mn Hg. She has a tine uemorand warm, moi stSki n. She has had lntermttent pa! pilatl ons af'ld tlw'eel oose bOwel mowmerts pet daylor too pasc
6 weeks. Fi ndings on prenatal visits up tothi s ti mehave been ll1remal1<8.ble. Hemogl obi n and serumgl ucose concentrati ons are withi n the reference ranges.Themos1
app1opriate ne);f step isto assess lherund on of,m ch ofthe fo lo\\ing?
i
i
l
o A) Central oorvous system
o 6) Gastrointestinal traa
O C) Heart
0 D) Kidney
O E) 1.i ve,
@ F) Thy1oi d gland
27. Afood handle, prnpares a cuslaid piefOfa pi cni c andl etsi isital ,oom temperaturefOf 8 00\l's before baki ngi t. The pie i s lheot homughly baked and 1efrigerated unlili ti s
sel\lOO at too p1 01l c. W1 tlin 2 to 4 OOLl'S alter eatillQtoo pi e, sevoo people dewk>p severevomi ti ng. Which olttle1011owl ngI s the most kely cau sal organi sm 01tl"ls i11 ness?
O A) Campy!Obacrflrjejurn
0 6) Ese/JMC/Jia COIi
o C) Lisuirla ,nonocytoge00:s
o D) Salrnot1Clla &1'1lcritidis
E) StaphylOCOCCCIS 81JreUS
F') VibrioJ)8rahaemotytkus
) Ye,rsini enfli!ic

28. A 3, mon!h. ol d boyi s b1oughlt ot he physicianbyhis mot herbecau se offaihl'e to gai n wei ght His di el consi sts of form.il a,,\fli ch he constmes vigoroust, at reeding limes.
Me has tourto e;ghlbowel movements dai ly, andttlescoots are beJkyand toul sme1 . Al bi rth, hewas atthe 25th percentile I lengthand weight, but 00 ls oowbelowtile
5thpercentile. Physi cal exami nati on shows pal lor wi thitUe slbcuaneoos fat. There is mi ld abdominal dstention and active bowelsoll'lds.A compl ete blood court a.rd
senmel edrot,ie concentralions are \\il hi n the reference ,anges Test ofthe stool for fati s posi tive Hi s sweat dlloride concenl1ati oo is 78 mmollL Whi ch otlhefo!owi ng
pathOl ogac processes, s ltle mostlkely cause 01 the poorgrO'Nltl ,nltll s intant?
O A)
O 6)
O C)
D)
1, E)

Bacteri al o>Jergrowtti ofthe small intes1i ne


Hbrousstenosi s ofthecolon
l.ymphocytic inflarrmati on oft he panc1eat ic is
Miaovesi cul a, steatosi s tilel ive1
Paoc,eati c duct Obfucti oo

29. A 72.year, ofd man has a 1.week listory of r1equenl headadtes. He has moderat e swelli ng and lenderoess i n hi s righl temporal 1egi on. Exami nation ofti ssue oblai ned on
lol kmillQ?
bi opsyof the nght 1empora1 arterySh<:wlS focal 11anuoma1ous lnnanwnatl oo. nll1treated, he 1s at Increased riSktorvml ch

atoo

O A)
8 6)
O C)
o D)
E)

Loss olhearing
l.oss of,.;sion
Mellngi ti s
Patatysfs Offacial nec\>eS
Ul ceratioooft he ski n

30. A p1e-Aous!y healthy 51.year.old woman comes tot he physician because ofa 9.kg (20.t>) wei glil oss over the pas1 6 monlhs. She has smoked 2 pack.s ofcigarettes daily
to, 20 y&ars. Sile takes nomedi cati ons.Vitalsigns are normal. Physi cal e.xaml natl onS00'/1s noab<lonnaliti es. Laboratory studies show:
17 gldL
Hemogl obin
52%
Hema1ocr11
Laikocyte c.ount
5100/mm
Platel et count
250, 000/mm3
5"rll!1
Urea ni trogen (BUN) 17 mgldL
Cteatini ne
1.0mgldL
Urine
14/tl)f
RSC
1/hpf
WBC
Wtichofthefoll owi ng i sthe mostilcety ctas?

o A) Endri al carci noma


o 8) Lurig catci noma

o C) Potf(ythema "era
@ D) Renal carcinoma
O E) Renal vei nthrombosis

31. A 23-.year.oldwoman has a 6-yearhi storyofinlermi tlert epi sodes offever, crampy abdomi nal pai n, andfrequenl,l oose s1ools.. A sedi on of bowel reseded after a recent
seve<e epi sOde is sncwm. v'Vhicti01 t he ronowi ng ,sIM moSIl ikety diagnosi s?
O A) Acute appendi citis
O S) Adenocerci noma
O C) Cel iacdi sease
lii D) Crohn disease
o E) UJ cerati colitis
32. During an investi gati onal study, a ne\\ff S)l1.hesi ted dn.g (DrugX)is foU1d lo sel eclively bl ocklhe effecls ofi ntibi n. A 1esearcher amli risters Drug Xt o an e.xperimenlal
ani mal. Wl'id'I orttle1011owi ng1 s most kely to oocur?
O A) Decreased serum fol ide- Sli mulating hormoneconcentra6on
O 6) Dec1eased sen.im gonac:folropi n. rel easing hormone conc:erCration
tiJ C) '1cieased serum folticle. s1i mulali ng ho1mone concert,ation
o D) '1c,eased serum gonado! rOpi n,rel eaSiog hoononeconn11ation
o E) LQ)'di g cell atrophy
o F) Leydi g cell hype<1rop/rf
33. Wedi on byl heroomps ,.;,usi n men may ,esun in decreasedfertili ty because of..;1al damage to which oflhefollowi ng shucttJ'es?
o A) Ejaculatory c)Jcts
O 8) Epidi dymi s
O C) Prostate
e D) Semi nal VP.Si rJes
@ E) Testes
34. A heallhysubject ieceives an infusi on of2 L ofisolonic sali ne. Assuming no Lrine e.xaeli on, whi ch ofthefolowi ng i s the expected i ncrease i n lhevolume (in L)oft he
extracell!Jatl tu1 d comparuoort?
@ A) 0.3
O 8) 1,0
0 C) 1. 3
O D) 1.7
0 E) 2.0
35. A 30..year. ofd pri nig1avid womanat28weeks'gestation comes to the physidan for a rooline prenatal examination. Labo1atorystl.di es showa fasti ng senmgl ucose
oonceruation ol 135 mgtdl al'ld a 2hOOrpostp(andl al sen.mgl ucose concentrati on ol 151 mgl<IL One weekl ater, repe.at labat()j)'tes.1i ngsnows si mi lar resuls. Ifltli.s
pati ent receives notreatment and deli vers at 40 weeJc.s' gestati on,,\fli ch ofthe foUov,;ng is mosttilcetyto be presentinthe ne\'lt>om?
:l!l A) Macrosomia
o 8) Respiratorydi sltess syt\d(of'l"I&
o C) Si renomelia
o D) Spana brfida
O E) Ventfi CIJa.f sel)tal defect

36. A62.year,ofdwoman comes to the physi cian because ofa 4week his.1ory of prog,essive shortness ofbfeath. He, temperat ure is 37.sc (10<rF), pulse is 96lmin,
respi rations are 28fm1 n, and blOOd pressure ,s 162186 mm Hg. Merj ugular venouspressure Is 20 cm MP, carolld upstrokes are decreased, and tile tracooa ,s mh:li oo.
The(e i s dlllness to peu::ussi onatt he base or b(l(hI I.Ilg fiel ds, b1eat h sounds a te decreased al bolh bases, and t he,e are aaekleshat.vay upb(l(hI t.fig fields. Carclac
examination sl'IOws a nonna1 S1 ands,, an Sigallop, Md a grade 2t'S l'IOl osystOlic rrumur Matd a1 too apex. Whi ch otthefolowi ngi s tile mostl1 koly di agoosis?
o A) AJelectasi s
o 6) Broochospasm
C) Congestive heartfail ure
o 0) Consoli dation
O E) Pul monary hyperte nsi on
37. A2. yea1, ol d boy has hi bl ood presstJ'e int he righta,m lhani n thel eft arm. decreasedremoral pul ses, and a systole ej ection muumr hea1d best bebwthel eft davicle.
Wtleh ofttlel ollowi ng I s too mostlkely cau se ol this congenilalanomaly"/
@ A) Abnormal de-.-el opment ofthe aorta
O 6) Oextrocer<:fia wi th sitvs irr1ersus
O C) fncomplQto partilioni ng oft ho -alri owntrict.liu canal
o D) PetSi stentfetal ci rculalion
o E) Transpositi on or lhegreat vesselS
38. A sexual ly active 18,)'ear.oki woman has an erythema!ous maail opapul ar rashover he1 chest. back. abdomen, and di stal ext,emi ties, incklding her patils and sol es. She
al so has weepi ng pl aques on he1 external geni talia. Exami nati onor tissue obtai ned on bi opsy Of a pl aque showsa dense chroni c 1n11ammat0ty infinrate oehi n plasma cells
and swelli ng orendothel ialeels obli terati ng klmi na ofsmall vessels. Whi ch ofI.hefolowi ngi s the most fikely causal organi sm?
o A) Chlamydia trochomtJtis
o 8) HetpeSVirus
o C) Mu rnan papi aomavJ rus
o 0) Neisseria g(){l()rrfloeae
E) Treponema pal/idum
39. Cell s oflhe adrenal medull a seael e ca! echofamineswhent he sympat het1c nervous system is powerful ly stimulated. Which ofthe foDowing neu,otlansmi t!ersi s usedt o
aCIJvate too cttomaffi n cells?
@ A) Acetylcholi ne
O 6) Epinephri ne
O C) GUamate
o D) Histami ne
e E) N0<opi n<>phiino
40. A 6.yea1, ol d gi r l is btOUJl1: to the physi cian because ofda1k urine and pe1i ort>i ta1 edema, noted i n the momi ng afte1 waki ng, fo1 the past 3 days. Physical exami nafion shows
mild hypertensi on and ankle swelill!, UrlnatfSl s SOO'l,s too p,esenoo ot red blOOdcelS, red bl OOd eel ubular casts, and J ncreased p<otein concentration. Ove<tneoo:xtr ew
days, uri neoutputi s di mi ni shed despi te adequateflui di ntake. Whi ch ofthe fol lO\"ng is themostl iketf IOcation athe c:isease process?
:l!l A) Glome1ula1
o 8) titerS!itiaJ
o C) Postrenal
o 0) Prerenal
o E) Tubur
41. Whichoft he foll owi ng bestdesc,i bes peptide hormones?
O A)
jj B)

0 C)
O D)
0 E)
0 F)
O G)
O H)

RecePtor Location OnsetofAction


Cell memb1ane
rapi d
Cell memb1ane
rapi d
Cell memb1ane
sl ow
Cell membrane
sl ow
Nucl eus orcytosolic
rapi d
rapi d
N\lcl eus orC\1osoli c
sl ow
M.lcl eus orc','tosoli c
sl ow
Nucl eus orC)1osoli c

Exampl es
l
insuli n
a c:losterone
l
insul in
a1 c:loster(;('le
insuli n
a1 00sterone
insuli n

a dosterone

42. A 40..year,ofdwoman devel opsfe\er, sweat s, COt,;Jh, and wei glil oss 0\1er a 1.monthperiod. Hi Slcxy includes ci ga1et!e smoking and heavy akohol use. Hersputtmi s foiJ
smeli og; Gram sta n ShOWSl eukocytes and miXed bacterial nora. Smearfor aci d.last baal Is negati ve. X-ray otU'le Chest sl'IOwsa cavitaryl esi on In tile rlghl tower IObe.
Wtchorthefoll owii ng i sthe mostimportantfactor in the pathogenesi s ofher iU ness?
O A) Acqui red dysfuncti oo ofchemotaxis by leukocytes
8) Aspi ralion Ofotal sec.-etions
o C) Oetioency 01 a p,oteolytlcenzyme 1 nt1bi tor
o 0) Destruction otC04 T lymphOcytes
o E) Reacti vation ota previouslyl atenti nfection

43. M 81. ye.ar.ol dman,ttl urin31)'h'CQucncy and incompl e<c bladderORl)tyingundCfgocs a transurctlYal prostatic resection. At>hol()lllCfOQrat>h clthe rcsectoCI speci meni s
shoYm. Metastasi s ofthe neoplasm"ilmost characteri sl.ic.a if'fV())Je wticb of! he foll owi ng struct\Wes?
o A) Adrenal glaoo
o, 8) Booe

" C) Brain
O 0) Liver
0 E) Lung

44. Whdl cllie loll owi ng bestdescribes plasma and wre osmolafltym a 27.yea,.oljwoman 'Nf'lohas neplTogenicciabetes Ircsi pi dll!i?
t.raneOsmolali tyMe<
Unoo Osfl'lOlallly.Mer
Pl&srnl'I Osmol aHty MildWater OeprivaDon Admini strati on ofAOH (Vasopressin)
r
no change
I
nocnenoe
no change
nochonge
45. A 55,.yea,..oldman develops shorlness oftxealh. Examnabon shows a harsh systolic ejedi on rrunuatIfie base lhatraciales tothe neck. ECGshows evidence cl
promJletlleft...entrl cularhyperrophy. \Al?lld'l01100fOIC:JIMfO cardi acalies, s most kefy?
, A) Aortic:6sseai on
J 6) BKUSP1C11oni cwtve
C) onNficl ency
D) .., stenos s

l
o E) Trkusped1 nsUff
1 dency

48. A46-yea,.<*Jwoman wilha ctwOl'lc ilness affed!Qg hef carehopulmona,yli.nctlon di es. ThephOtog,aph shOws hefheart asseen atalAOPS)'. Thisappearancei s most
ooflSIStfflwithwt.enofthekllowing di sorders?
0 A) .AlaJhoic card; OOlj<)p..,,
o B) Alnalsep1aldefect
e C) f.ilrat stenoSi S
, 0) Pnmo,ypulmonary ,--on
., E) S)'Sleffle hypeoons, on
47. A 16.yeai..oldgi ni s btougtttolhe physician byher molhefbecauseshe ha:snevei had a menstrual period. She is 140an(4 ft7 in)laJandweighs 45 kg {100 b); BMli s
23 kg'nr Physical examinationshowsli ttle developmerj ofsecondarysexualcharacteri stics..wtichofthefdlowi nglindings i s mosll iketf onexami nati onoft heota.ries?
0 A) AUOl)trf
0 8) Corrxxa .....
@ C) -stoma
O 0) cwariandi sea.se
O E) Tera1ana

48. A20..year,otdwoman has had J)fog1essi ve weakness, anorexia, i nl ermitten1 vorril ing, and a 6-kg (13. 2. l b)weighl loss aver thepast 6morths. She is thi n, he1 blood
pressure Is 88.'60 mm Mg, and too-re as di ffuse brown dai1Cer-.og ofthe et>ow ccease,s and In an 0kt appeooectomyscar. Which octoo lollOw!ng sets of laboratOty findi ngs is
mostl ik:et,, in ttis patien1?
Sen.m
Uri ne
ACTH
CM/sol
Aldosterooe
K
Na
O A)
!
I
l
O B)
norma
n<lfmal
nounal
!
l
"' C)
l
O D)
norma
l
l
0 E)
I
49. A 24 year, otdwomanwhoi s a fa,mwo1ke1 has had progressi ve dyspnea and i nlermi tlM fevers overt he pasl 6 moolhs. An x-1a yofthe chestshows diffuse intersti tial
nodulardens,ties. Examinati on otbssue obeai ooel onl oog bi opsyShows accUO'lll atl on ofd'lrool cl nllammatory cell s Inthe 1Mersbti um and noncaseati ng,l oosef)'rorme<I
granul omas. W'hich ofthe following isthe mostl ik.ett, di agnosi s?
O A) Acute respi raloty cliS1ress syndrome
o B) Asbestosi s
C) Mypersensdivity poeunooi ti s
o D) Pr,e(Jmo<;ys1is pneumoni a
o E) Si kosis

50. A 3S.year, ofdwomanwilh pri marypulmonary hypertensi on unde1goes anu,comp! ica! ed 1i gltl ung 1Ja nsplan1at1on. One month la! et, her FEV, i s normal , and an exe,ci se
stressteS1 shows no abnormali ties. Wtich ofthefoll owi ng i s most tikely to be increasec:ti n thetransplaniedl lfl9 compared wi ththe leftl ung?
o A) AUway resi stance
0 8) BI OOd tlo\Y
o C) Cough reflex
D) Lul'lQ compli ance
O E) Mucoci1i arycl earance
O I=) Vastular resistance

Form 6-3
1. A healthy26,:year. oldwoman ccmestolhe physi ciantora health maintenance examinati on. She says Chathe1 moods changeeasit, andt hat shehas seen n11Tierous
psycr.amstsandltlef'apl stsfOf "'emotional p<obl ems."PhysacalexamnanonShOwS muni ple hootQ, sel-inl'lctedWOll'ldS. Al end otthe ex:ai-..nanon, she thankSthe
physi ciantor"bei ngthe mostsded.compassi onate.ard\llderstandi ng pt,r,sicianI haveever krlcM1l"Whi dl ofthe folc:Mino is the mostl ikety<iagnosi s?
e A) S.polal dsorder
B) Bordednepersonali ty clsorder
o C) Oyst:hJmte di sorder
0 0) ttSIJ'KlBCpetSOnalfly dlSOfder
O E) Majordell<essive di sorder
2. A 52.yeai.oldman has cavita.rypmnonarylesions. ExaninaSon oftissueobtainedonbi opsy oft he bigshowsgranul omatous inllammalion with caseous nec,osi s. T
fll>hOCyteS extractedtromttlisttss:ueare most kely fOhaveanincteased concerwratlooorwhi chtolowiog?
0 A) 8<o<t,1cirin
O B)
C) 1 11erleron..y
0 D) 1.-...-.a (IA!)
0 E) Myoloj>6loda"'
3. Multiple merrt>ers aan extendedfaniJy are diagnosedwitha specific auloi mrnre disease In an atten\'I,to iderti fy a mark.er IOassess the 1i sk for disdsease in Olher
lamt, rneni,QrS, theexp<esstonawt'ld\011hetall owi ng ShOcJdbei dentified1neactirarr.ly member?
( A) ,nm,noglobuin
t B) . .., lel.f<ocytes
o C) CD4molecules..,Tt,mllhoq<es
o D) C03>molecules onTt,mlJhoc)<es
ti' E) HShXOrfl)atibili ryarci oens

Hi-

4. A 70.yeat.oldman is beirg trea!edwith a newantifU'lgaldrug for fllngal sepsis. The d1ug is achelyseaeted by Ille bctleyand has a vol.meddist1i bt.ti onof35land a
cte-.aranceci690 mUm1 n. The hal-lfe 01 the au,;J(inmin),s clOsesttoWhChcime,loHowiog?
@; A) 15
0 B) 35
0 C) 65
O D) 105
0 E) 205
5. An 1S..yea,.ald man has anappendectomy. Twoweeksliter, the abdolllnal incisionhas healed wellandhasa pirf:. sliny, g1an.lal appea1ance. VV!jchoflhe foll owi ng
findings,smost llketf on e:xamnabOtl Ollissue Obla1 nedromme heal ing .aroo?
@ A) Arvogenesi s
0 B) El>itheic,d cells
e C) Fi brinoidnecrosi s
o D) Gc811JomatoL1Si nnammation
O E) 1-,rpy

6, ThecondlnAbon reacti on betweensucci n)'l,CoAandoit,one ist he taleJimingreacti oni n the tMos,nheSIS ofwhi ch dhilolowing colll)ounds?
t, A) C,_ol
B) Heme
O C) Urea
0 D) Vit,rrinD

7. Anirwesigatof has desi gneda mgthat is t ransportedi fllolhe cell lflra membrane channeland remai nsisolatedinthe q1oplasm.Ti.s drulJ will most ikelJdicecltf
i nnuenceWhCh otltletOlow, ng cetUarproce-sses?
o A) DNAnnscripoon
O B) Elemonuanspon
0 C) GlyaJlrsis
@ D) Pr-..lol:l ng
o E) Tooi l actorbf
8, Duri nga meaJ. lhevagLIS ne,yestim!Jat es secretionofacetyldlolne, whichstimul ateslhe siomamto sec,ete HCIin a second.messengerpattlwayderi \'ed citedlyfrom
membrane pt'IOSphOlipi as Whdto,thetOlowi ng secOM me-ssengers1s1
( A) CIWI'
( B) cGMP
(' C) Cldne
0 D) Gp,"'"
E) lnosilOl 1,4, 5-tri phOsc:,l'late

9. A4S.year.Ofd man wi lha hi story Oftype 1 di abetes memtus is bl'ought to tllE:' erne<gency OOpaitmenl becau se of Chest pain radi ati ng 10t hejawfO(t he past 10hOu rs. He di es
Sh0rtly after adniS'Sion. Apl\Otograpl\of a todnoyas soenat autopsyi s Shown. Wtlich ofthe fol lO\ng types of necroStsi s thO mostli kely caus& ofttiesefiOOi ngs?
O A) Castion
B) Coagulation
C C) Fat
O D) Genous
o E) Uquefaai on

to. A specimentom a pal ienl's sa.rgica1wound is slre.akedforISOiation on an agar pla.1e.Ar.erovemigtiaa.e al 37c, t he plate hasbacterial growth asshowni n the
claam. WNd1 Of the follOwtlabetedarrows poinlS lOa CdOnythatI s most ketfto be al aboratorycottam1nant?

ae.

A)

B)

0)

C)

E)

t1. A 28.year.old woman whois at 12weeks' gestationand herhusband ,eq.,est genelicevam1ionbecau set heyhave a son wi th cysticfibrosis. Genetic irbgeanalysi s is
performed Results r;Jgee
l()lypil
igto delett at aa 5pomorphi c.loci linked iot hecysijcibrosis1ransmembritnerato,ygene ( are shoffllTherisk ot
recononanon ootweeneamdthel oci and CFTRas knownto beleSStttan 1%.
MOlhef Son With CF Fetus
Geneti c Marker Fathe:!'
I
1, 2
1,2
1,2
2,2
II
4,4
D
3,3
3,9
3, 3
3,3
1, 8
V
1,1
1,1
1, 8
IV
4,8
5,8
5.8
5.5

Whichdthefolowi ng bestrep-esents the aJeles present on the falhef'sctw-omosomethat hasa CFTRmutaliQl'I?


C A) 12,14,119,N1, V4
O B) 11,14.119,IVt, V4
<I' CJ 11,14, 113,IV1,V8
CD) 12.14. 113.N1, V4
0 E) 1 2. 14. 19,Nl. V8

12. Si xty. t hree cases ofsmal ox are confirrred wi lfli na 15.mil e radi us ora townwith a popu\at ion of 200,000. The outbreak is beieved to be the result ofa bi o!errorismattack
10 days ago.l f'itiation ol wl"IOO orltlefollcwl ng 1 s ttie, best methOd forcootro1 ng the spread oft hi s di sease?
f) A) AcydO'Ar therapy
C 6) Almr<o<ine theropy
O C) 1 rnerl euki n. 2 (IL.2) (he1apy
c O) l n!,avenousi mroonegl obulinthe,apy
o E) Ribavilint herapy
<t: F) Vacdf'ia i mroonl zat! on
13. A mani ed 78.yea,.ol d mani s broughl l ot he eme,genc:ydepartment afteri ngesting ten IS.mg temazepam tablets. Afte1 hei s stabi ized medi cat,, heLfldergoes psychi atri c
eval uation. Wtll ch 01 the tollowmgI s most 1.)0rtar InasseSStng hi s nsklo, sui cide?
o A) Ear morni ng awakeni ng with C/ecmse<J appeti te
f) 6) Fami ly history ofS\icicle
O C) Mal e gender
c O) Ma,ilal stat us
E) Pa11en1s beker ltlat temazepam would ki ll hi m

14. A 25- yeaf-Old man deVE-IOps revet, muscl e pal o, aod headache 1 week aner being bi ttenby several ti ckswtii campi nearthe Grand canyon. Thetever remi ts and he
feels better in 7 days. Two to four daysl ater, tis sytoms rear.A Gi emsa stain of a peripheralbl ood smearshowsthe bacterium itustrated. Whi ch ofthefolo'lt'lingi s the
most ikely expl anati onfor the reamenceoft he s)TTlptoms?
o A} EY.pressl on or endOloldn.lke S1J\JC1ures ont he surtaoo oc the 0<ganl sm
B) Mi 118,tion ottile organi sm froma ti ssue si te to tile blOOdstream
o C) Production
of a tonthat had become suppressed dt.firg the infeai on
(> 0) Secondaryi nfeai on bya different organism ofthesame species
O E) Vari ati onof a major surface proteinanti gen ofthe organi sm
15. A 3.year.digirti s brougtitolhe emergencydepal1menlt.r1Coosciousafteri ngesti ngan uri.nown qua.Jil ly ofethanol. Hersenmglucoseconcenlrati oni s 30mg,'dL
Th''lffiieh OSthe folleiwumechani sms arelhe pcOduetsoralCOhOImetabOlsmOOl'lflt:uiogto 100 h?
e Al Increasing the conversiondpyn.Nate to oxalo3cetate
(> 6) Increasing the formaDonalpyn.N3tefrom ac.ecyf CoA
C) Increasing the reduction ofpyruvateto ladate
c O) lrfibifing pyn.rvate dehyctogenase
o E} ll'ftblbnglhetoonationdpyn,vaterromafaftl

16, Whi letnWeing byshi pfrom h:lato Mexi co. a 52.year.otfman devek)ps malaise. chi lls, and spikirglefll)ef'81 ures to 39 4c(103"F) Overlhenexiseveral days, he
deYelOpSprogress hyp and muniorganraue.He LStakento a l'IOSpUI, a compl e{ebloOdeelcount shows a hefnog10ban concentrationol4.0g.'dL A Wriglt
stained penpheralbl oOCI smeartSShCMn Whi ch ot!hefolc,Ningi s the most iketydiagnosis?
O A) Leistmariasis
@ B) Malaria
e C) Me,...,.,goc
.,,,,,
occ
=
e.

11ia
o D) S<lwslooomasl s
o E) TnebOOSos
17. A 4 .year.old boyi s broughttolheph)'sician byhis parertsbecause ofglct>aldateiopmenlaldeCays and odd behaviors such as hand ffapping. tis mdher hasa teau"ing
di Sa.billy, and hismatemal lncle riasmental cetacdallon. Physi cale.xamnaoon soows macroces:,rw,, Wgeears. and ap,ominent Cflin. Hehas:a3-wOCd vocabula,y. A
di agnosisoffragile Xsync:tomeis rm.de.Afterthepareras are irtormed offieresuts. they askiflheirhealthy 12-year-dddaugtter."1lo hasmeuJ cSevetopmef'UIJ
rrilestooes,,shotJtj be tested todelennine hercarrier status. Whichofthefdlowing respooses bylbephysi ciani s mostappropriate?
o A) ,reoo.1111eucJteS(i ng he<ro,,sothat soo can 00p,epacecf10<\'ttlat mayle ahead1 1 she haS ctatten. "
o 6) 1 ,eoo111nencJwai ti ng l.lllll Sheisolder and man.reenoughto decidefherSelf if she wouldl1ceIObetCSii:Cd:
o C) ItW'Qidbebei.1eri fshewereatleast 16 ye.arsofage, ctoserto her dikl:>earing )ears, bef0<etes6ng wasi nitiated.
@ 0) '"l.ersOSOJSsthe situationwr.hher, reviewthe prosand cons oftestirg.andthen ask herifshel iketo betested nc,w
O E) '"lersbc*:foffont hat andciscuss lhe optionswillherffllef'I shei s plarrirgto marry andhave dlildren "
18. An HIVW\1$ ha$ been isolatedthall tSnoti nhi biled by$OljJle C04 Muti,tions are rnos;tEkely to have oco.ned i n the ,.;m1 gene era,ding\mch or t he folowlng?
A) Cott
8) Glyooclroletn
0 C) tegrase
'0) M31rix
, E) R...,... ..nscrip..se
19. Embfyoric:mDnd1ogenesi s repfes the formationofmesenchymal condensa!ions althe sit es oftta.e carti lage. Duri ng!lisdevelopmenlalstep cA.MP concenlratioosi n
ttiecetssu)Q 100 cOOdenS'alionI ncrease si. Treatrne wiltla membra.00- penneat>le&onn OIcAMP (eg,. cll:lllyryl cAMP) alOw'Sil'Ollldual meseru:hymill
cellstoc:trierettia.iei nto chonckocytesootfirstfonrirgthese conoensations. Wlibifion ofwhichdlhefollowit9 enz,meswoud mos1 di reat, irtibitdi b(.(yryl c.AMP
mediatedi nduction ofchOl'Oogenesisfrcmmesenc:fl!tme?
o A) Gua()'(la.se
o 6) Ptlospnodlesteraso
O C) PrClleaSOOle
@ 0) Protein kinase A
O E) Tyrosinekinase

:,,
20, Twodlittenm afamily have a disease charaderized bJmertal retardab:onandmid dysmorphic feaues The famly wasew.luated by standardka.ryotype and lhenbf
ONAITICtosatelbte polymorptwsmS1l..Oes rortwo autI markers. Apedqee lS s00\'111. 1Nh.d'I Oll'latOllowi ng1 s ttW?mos1lkely cau se orrteMal C11 sorder?
0 A) Miopalion
., 8) AtJto5c)rNJrecessi vei nhencante
.i C) Genome u'1)rl nbng
D) M1todW::lnCtlal imen1anc:e
21. A20..year,otdwoman comes to the emergency departmentbecau se ofdi fficulty breat hi ng fOf2 hot.rs. Aucibl e wheezing i s heard. Physical exami nati onshows
overdi stenti on ofttleCM'Sl and ma,ked restrlctioo ofe.xpansi on. LabOratory tests show an 1nae.ased bl OOd eosi nophil count Md eoSinophl lS anmespUlum. Whi ch ot!he
followi ngi s the most ikely seqoonce oteverns for iritial sensitization to the arergen cavsi ngthesefiOOi ngs?
o A) Cytokine reAease->l gE prol)Jction-,-T-l ymphocyte inducti oo
o B) lgE production -, e.ITJ>hocyte activation T..t,mphoc)1e i nduction
:ti C) lgE production T.t,,nphoc)1e i nducti on, cytoki ne rel ease
o D) T.lymphocyte ion cytokine release l gE proc:l.iction

'

22, A 2.ynrgir1 11 adml t ted t olhll'IOSP'lil forl he secondlimenee bi11h becauseGfpneimonla end lfll*etolydi sl ress for1 weetc She appears poort,dweloped and
l hi n Mor rtspuanons oro -41/mln.. Physteal exami nati on SN:lWSm1 1Cf 10spl ra1oryClsnss Ari x.ray ccthechrKI ShOwi hypQflntlationMlhi nllkfBto;i nl hQngtamldele and flgf'lt
lower k:>besrJlhe h.l'IQ, 0\11,ng hosl)l&eiuu,on. t he patiert hJs greasy bowel wit h a pungent odor Asweal chlori detesl cortirmslhe di agnosis, endthepati enti s
startedona speci al fOOTVJl a withorallemenlat1 on dlat..soltble vtlamns Ifthemodence oft hi s geneticdsease 1 s 112500amongwhr les, wllch dlelolowingi s tile
approllimatecame,frequency dri.sme-ndeli antraitI nn.spopLla.tion?
0 A)
, 8)
, ci
D)
E)

1/12
1/25
1
11125
1/625

23. A 1S.year,ofd gi ni s btoughl tot he ptrysidan' s office by he1 mothe1 because aa 3-.daytii storyofconstant dt.d suprapubi cpai n. The molher leaveslhe room befo,e t he
examlnalion, and too paliefll says, "My mottler has noidea tha.1 1 am haVtllQ sex. and she would ki ll me i f shefound out. Pleasedoo'Itell tiec. " Examinabon showsmarked
l ende1ness on cervical motion and a copious ye'IJ.<.111di schargefrom the cE!Mcal os. Cuttu1 es ofthe <ischa,ge grow Neisse,iagono(lhoeae and Chlamydia rrachomatis.
Wtlefl ofmefollo>Nl ng I slhe mostapp<opriate next step inpatl et1t care?
O A) Refuseto treatthe pati ert un1ess she promi sesto abstai nfrom sexual activity until shei s oScSer
O 8) Refuse totreatlhe pati ert un1ess shetels her mother
8 C) Tleal the palient and do not tell lhemot herbut urge the pati ent to do so
o D) Treat ti pabenl and tell IM1nolhe<i n ftol'II olthe patient
o E) Trea1 the paben1and tell lM mottler pri vate-

24. Ari 82- year- ol dman has had bl eedi ng gums, several l oose teeth, and brui seson hi sl egs ewer the past 3 weeks. Hi sl ov1e< eldfein1 bes rue shown. He Ms prepared hi s awn
meal s si nco rii s wifodi OCI 4 monltls ago. Whi ch ofltlefOlowi ngi s mostl1 kolyto boabnomKtl i n this pati oot?
o A) Booe m! n(lralzabon
J 8) Collagen sy,..hesl s
O C) ONA metabOsm
o 0) Gamma cart>oxylatl oo
E) Thi ami ne ptiosphOrylation

25. Anirtanl1llil1aninheri teddefectinomi thine t,anscarbamajta.se ismosl fikefylohave an increased plasma concenlralion d-.tichofthe follo'Ning?
OJ A) Anmna
O 8) rine
0 C) Citruline

O D)
0 E) Urea

26. VlrlJeftstrains ol Nelss<JnanningJ tJdtS, Streptococcuspm,unr,m'(J(J, and Haomopl'Wusinlll.Nlnlf:KJ each cOf'Uln\\flleftollho IOI surfaoocomponectS?
.
l)ii
A) a....... forlTing
B) e.-o,ons
' C) F1agela
D) Hemot,so ns
E) Polysacchan<ltcapstJes
27. A 1S..year. <*J itsustai ns a mno, bLm onlheforearm. Wrhn severalmn.rtes, a red (hyperemc) areadevelops ontheSki nsuroundi ngthe site ct l'J.lry. Whdloft he
IOIOWilnQmoo, ato,si s themos1lketycauseotmehyperenu?
A) C3b
8) 5-HETE
8-l C) Histami ne
D) lr<ttleun 1 - 1)
E) LMotnene B.
-, F) Thomboxane
28. '1an animalmodel of myocardial ischemi a, morphoc and biochemical changes in myocardaleels are studedafterigati oo ota branch oflfle axonaryartery
Oegractanonormembrane Pld btlayers andorC)'IOSkel ecal protetns occll'SfOIOwtng aal vabond, ruacelltr.11 pnosphOlipase A,andproteases. AAi ncreasei nWhChalhe
tolowi ngi smostdi rectly respon51ble tor aCIIYabon Of UlOse en;eymes?
A) Cat c:11.rntenconcercrauon
6) C1eat1ne bna,se ael:i>Aty
C) Enseactivity
-111 O) Sodi umionconc.emration
E) Waler conleri
29. wtichoft hefollO'Ning bestexplai ns\WI)' prcteins synthesizedin theroughencloplasrric retie.Am and sec,etedfrometl:aryolic cells haYeashorterl ef9h !flanwotJd be
predi cted by thetrfespe,cll\<e mRNAs?
A) Cleavage of 5' of nascertpotn,epbdes bei ng syr1hesized on Pofy(ibosomesSIthe endoplasmi crec.o.A.rn
() 8) Cova.ieri binding dnascent polypepCidestospeci fic membraneproteinsduri ng their syrthesis
""" C) Nonspedlicpreaseactivity cbi.ng passage ofthese proteinsttxocghlheendoplasrricreticu\Jm membrane
0) Sh0rteni ngofthec:atboxy te-rmirusbecausehydl'ophObi c 8l'l'inoaddS 1emai nVtill'linthemembrane
30 Wl'id'I of1he following genebephenomena most lkely eCCOI.IU IOf t hevariebi fftyinbiobansfOfT'Oltion dcon,poi.nc:ts bya l iverh)Q"oxylese?
A) -exwslon
8) Fragile si tes
C) H)pe<Vanallle ONA
D) t,U.ple polymorl)hsms
"I E) Muliple pseooogenes

'*"'Oli1

31 The graph Showsll'lerelatlonshl p l)e.(Wffnmee>i>ressi onof recwes,orpraei n and l l'leexpression (l rtu1Scri r,c1on endtianslallon) da 'z gene, I n wllcJ.rype E:hendlra
mutati ons in elements oftheJoeoperon cOIM:S accourtfo,Un 1oloti onsh1 p?
cot1. Whi ch d the
A) 11:JCz promote,'MIinoal1i nl ty lor RNA p()IJme-rase
B) 11:JCz protetn"'"1 naeasee1 affini tylor StA>slrare
C) lacz proteinMtlnoaffini ty forsubstnue
0) Repressorw,m ancteased affitlityfor opec-ator
E) Repressorwi d'I nc,affil'llyforoperator
32. A 37. year, otd man returns to the physician because ofpersi slelt pai n in his dorrinanl hand ,esutti ngfromt enosyncwilis oflhe abductor po!li ci sl oogus. Hei s upset because
Ile Is nol on!)E'r able to play tenni s and sofiball, wtllch he used to do regularly. Me has been treated i..nsuocessfully <Ner the past7 month.S wi thI ce, exerase, l mmobl zatl oo,
si ca1 therapy, and 2 corticosteroi di njections. He says, "You are ;..stshutlJng me around through all these worthless treatmenis! " Whi ch ofthe fo!I0\\1ng is the moSI
1phy
.appropriate
response?
o A) "veryep otmeway I M\'8 presoobed too appropr! a1 e.standard tteatme-fls. "
o 8) "I real ty can'tdi scuss ttis,wti you urtiil you cal m Clown. "
o C) 11thati s ho'llyoufeel now, perhapsi twould be best to ;.istchange doaors."
@ D) "Ithas real'/ beenveryfrustrati ng toget solttle he p from everything we've tried."
o E) " Perhaps you need to seea counselor to work onlyour piobl ems wi tha nger control"
O F) "You are goi ng l o!lave l o learn howto play wi thyourlefthand."
33, Inrucleotides, thesugars areattachedto nit rogen.con1airingbases bywtichoflhefoll owi ng?
A) E18<1rOS1aoc 111eracbons
8) H)Qogcn bonds
,I) C) Ni di c bonds
D) Phosphodi es,e, bonds
E) Stadong lotlfll<2lons

34. A4S..year,otdwoman comes to the physi cian because ofa 6.moothri9.ory of mouth ul cers, IJid(ening dtile ski n, and tlobbing pai n in her handsVlhen theyare exposed
totrle oold. Sheal so has had a 1- moolhr.swyol sevete headaches aoo difflc:uty swalewilog. Pttystcal exami nation shows smalled ie:Slons a<oll\dltle mouth and
general ized musdeweakness. laborato,y s1udi es show a hi ghtiter of anti.Sci 70 (ruti -topoisomerase f) antibody. Abi opsy specimen ofthe skin shows thinning ofthe
epidenni s, hya1i nizalion and obliteration ofarteriol es, and dermal tlickering. These dermatol c changes are caused prima1i ly by the rse ofchemoki nes fromwhich d
the I Ofl owl ng l nll ammatory cell types?
@ A) Endotheli al cens
o 6) Eosi nophi1 s
O C) Lymphoq1es
O D) Mast cell s
o E) Pl atel ets
o F') Se,gmented neutroph1l s
35. A37.year,otdwoman devel ops pancytopenia afte1 exposure toa bentene,ootai ni ng agent Tissue obtai ned on a bone marrowbi opsyis mos1 fikety to show replacemen!
of normal hematopo! et:! c cells by\\flld'l ot1 00 lo!lowing eels?
8 A) A<!poc11es
o 8) lymphoc\1es
0 C) MOOOC)1es
0 D) OsteOblaSIS
O E) Re(i CulOendot heli al OOI IS
36. A2. yea1, ol d bcr, is broughl tot he ptrysician' s office for a follow. up exami nati onbecau se of recooen! bll ate,al o!iti s medi a. He is at lhe 50th percernile for hei gli, weigh!, and
OOad cl rcl.lllte,ence. MeI s aI mo tollec trai ned, pretends to hel p ca,etortisyounget si bli ng, and can tom, trtee to four-word sE!t'llenoos. Wtll ch of the tol lOvti beSI
descri bes hi s development?
Cogri1i ve
Molos
Soci al
O A)
Del ayed
001ayed
de.Jayed
O B)
normal
Del ayed
de.Jayed
O C)
Del dd
n..:itrnal
li:lyt':'d
O D)
Del a)'ed
normal
normal
0 E)
delayed
delayed
Normal
0 F)
Normal
delayed
nonnal
O G)
normal
Normal
del arI
normal
$ H)
Normal
normal
37 Wlictl al1he follOWlnQ steps1 ncollagen syrthes:i s Is 11 edI npebenls wi thscu,w?
v A) Cieava.ge otIha N and Ctron1nal pep'8Sfrompcocollagen Inlh8 extraceuar matnx
6) Foemation dttieprooollaoenuipte 00Din coo cndOplasmic reio.Jl t.m
C) Gly(:osylaUon c/t,yctoxyly$ne$ ont ilepro,o col lagendlll, n
! D) Hyckoxylati ond selected protnes andt,sanes on1he pro-CJ collagenchain
., E) Se1-assen1)lyda coll agen molecl.6e RO a col la:genfibril
38 A68-year.Ql:j llll"i s b10!41 tolhe emergency c,epanmerc beciluse da 24ayhiSlo,yor lever, heaclacbe, mu5deweaknes:s, end confusi on. tislelll)ef8tUe is 38 6"'C
(101 5'1') Pny<,cal examl no-$1\0W$QOOOmllzO<I l)'ITC)ha-atl>y ano -kness ot tno ni,,, iowor extr1mty. lal>Orltory,,_ $llOWa.-oios 10a Vll\/9 that,.
n"\ai ntalnedi n wild bi rds, can'ledt o hi.man$ by Cuftl.xspeciesmosq1.11oes..and hHa IOw falall')' ,ale. Wtlic:hOlthe: tolOwlng is lhe most liket, causal virus?
o A) CoronaV1rus
8) Denguovirus
C) No,o,,rus
D) PolOY1rus
E) We,,;A,Ni1evirus
., F) Yelow reverwus

,oes.

39. A50..year.dcfman comes tothepbfsician becauseda 10. month hiskwy dmuni ple syrrciomsi nc:llcing fatigue. shortness clbealhwith exerioo. ringi ngi n hi s ears. a
He appea,sbland pal e. Labora.toty
90re tongue, abdOmlnal d1 SiCOr'l'IOfl,mrld penpheralmusdeweakness.bunu sensabonson hi s harw:ls, andbting 1 n hi s
studi es show:
10gtt.
Hemogk)l>, n
Hematoait
41%
let.fcoc::t-es
4000.mn'
Retimocytes 2%dredcels
Hemogl olline!earophQf0$isi snormalA pcri i)ltffill>loQQ smear $llQwSOl'oi matroqocaiisocytosis anosome PQi kil oc)IQSisQIerytnr0<:yte$. He$aYSt!la1f<llOWS an
adequate da tis: condi tion is mosi like)y caused bfa ct,'Sfunctiondi c:: aaM1ies: in which ofthefolowingceH l)'pes?
A) FOl Ctlat Qpll'Jelial cells Of 1MflyrOld glaOO
o 8) liepatocyteS
C) MacrophageSott>onemarrow
0) Parietal eelsdthe.s:1omach
E) Sc::hw3nneelsotcoronaryarteries
40. Ma rry humangenesencode mo,e t han one po!)pep!i de sequence. Wlidloft he followi ng mechani sms accoonls for tile produdi on olmore l han one piol ein by ooe tunan
geneIn most cases?
O A) Allemati\-espfidngolthetranscri pt
8) Producti onofa polyc::i stronic mRNA
O C) Readi ng mRNAi n bolh di rections
o D) Readi ng mRNAi n mOt'e lllanonereading frame
t> E) Transmpt1 on otbothsuands 01 the gene

41. A6Q..year-<*l man w1 tnSignsands o,an ot>strucbon Inlhe llfge bowel llldergoes wgical resectiOfl The presenceor a malgnanl neoplasm Is moststrongly
ste>Ported bywhtch ofthek*>wl"9feanxes seen inthe photographdthe reseaed segmertdthe cclt?
e A) Fibrosisott,ewal
B) .,.,,iion d._sUbmUCOSIand musclilt laye,
C) NanowlrgdltwHmen
o 0) Sharp oomatcat1onfromIlenormal m.icosa
"'I f) UICCrati ondIlle mucosa
42. WHu::h or1he kllowi ng inlraceacu si gnal nlolee:uJes ra1 esChe phoSphOrylatlcm dfruelose15.phosphate?
A) Cabum
e 8) cMIP
C) Cerami de
e O)
1.6-l>sphosphote
E} r-n.r::1ose 2,6-bisphosphate
F) MalanVI CaA

""'"'

43 A5S.yea.-old manhas a 2wee:k l'lsaoty of intense bf.ming pamm hi s c:at;es whenhe walkS1aPfdfy lhe pa,n1 s reerw!d by rQS:bng H&has smoked 2 pack$ Ofcigaiettes
daityt0<40 years. The sl<W'Ion hi srowere.xb'emitiesi sdl'y, sca!y.aoo Shiny w1 lhnonnal C<*>fand tef'll)eratuce. Femoralputses areweak. HemoglObi nconcemat1on ,s 14
O'dl \'llwtll ofIllelolowi ng best describos u,e collt.lar changes ,n l he mrocytes in hi scONeS d\JringltlO poi mAopisooes1
A) Cytoplasmi c S"Welng
c, B) Fonna.bonotlal'gedensltlesinrrrloehoncria
Cl KaryonhOxl s
o 0) Lys1 soCendopta.smi cre11CUum
E) Rupt11e olly50S0mos
44. A64. year, ofd man comes to the pbysician because ofa 3.month tistory of bl eedi ng gums anda tendencyt obmi se easi l'f. He has chrooi cpanc,eatilis and a 40. year
l'llsto,y ot al cohOt abuse. Mis pr()(IVOrrt>l n ti me , s 21 secontts {lNR = 3.2). Ouari tatlve-anaJysl s ot stootsnows Increased tat. wtil ch oi the tot lO\'t'l Is most liketf responsi ble
forthedevelopment ofthese syrll)toms inthi s pati ent?
O A) Felic acid deficiency
o B) Thi ami oo fici ency
C) Thrombocytopenia
o D) Vitami n B12 (cot>al ami n)defidet1Cy
E) Vit ami n Kdeficiency

45 Wl'lld'I ofthofolowt ng taboraro,y tCSCSi s mostuseful ,n difterci at 1ng SlQPl,ylococcusaureus! remotherstaplrJtococcl?


A)
B)
C)
0)
c E)
( F)
G)

Bacitracin"'-di sk)susceolibili ty
Caiatase tes1
CoagtAasetest
LIKIOH ft1mentaOploctjn (P di sk) susces,obolty
0-SOlesl
Ureooo tesi

46 A 2S.year. otiwomanhas a peMc neoplasm composed olmahYe glamtJar and chondroi delemertsand cysts inedbystratifiedsquamousepithelium, Y1'hichofthe
1o1ow, ngI s thomost Ike dlav,os?
o A) Adenocorc,nomo
( B) Adenoma
C) Cho.-.11 omo
t' D) Chond1 osarcoma
E) Tetatoma

ATPX XP,
ADP I
R - OH

Enzyme 1

R-0-

R= Re11u1Med 11111! f!lln

P-o-

Enzyme2
H20

47. The reactions sho'\m CQnStitute an illl)Oftani regulatory S\Wichforthe activity ofal arge numberofprotei ns in eukaryoti c cell s, Enzyme 1 transfers a phospt,a: e group from
ATP toparlict.Aar amino aci dsi n regulatOty PfOleins. E.n?)me-s lhat 1fotm thi s reacti on are mbers ofwhi ch Of rot:::wling gros or en2:ymes?
o A) Adeni ne phosphorylases
0 6) Adeny1 cyclases
O ,C) Adenytyl transferases
O D) Protei n kinases
E) Protei nphOsphatases
48. T1ealmenl of 1 espuaJory syncyulvirus wi th 70% ethanoldes11oysitsui'edivity The loss of1 rlecbvity islhe restAata dared effectclthe treatmert onwflid'Iofthe folc:Mlng
viralllttlions?
A) Enoyoflt1 e ""leoc8psi d; ,..1tle ""leus
6) F'USKIOof !tie vinon membranewllh the taroet eel menaane
C) Repkabonofthe'111usgenome
-, D) Transoi:ptionofmRNA
teui blie
49. A4S.year.ofdwormn is am'llled to thehospi talfOfttea.t mertda perforaled ulcer She1efuSesto speak lothe nu,seassignedtohe1. Shecats the nurse.. a
pet"SOd"andtelSlhe phyStoanthat he ,s Che best dOC.10f she haS everhad. Thi s beha>aor 1s best deS'Crlbed as whi ch otftf
A) Defial
6) lde!Sfictlon
C) Prqecti on
-o O) Rationai?>tion
, E) Rea.aionlonation
F) Rt1><ess1 on
G) Somabzabon
H) 51>la,<9
I ) SutJlimati on
J) Suppressaon
50. Transgeric mice aregenerat edlhat ate incapable ofexpessi ng BnAontyrosinekinase These mi ceare ati ncreasedrisk for sew,re i nrecfions caused bywhich ofthe
I OIOWlf'G mcrooroal'ISffiS?
A) Aspp,Jlus fulr.gatus
6) -vin,s
"I C) LoalOa
D) sc,ap:e p1i ons
E) Slreplococcus _...

Form 6-4
1. A 19-year-old man has had coi nuous pai n in hi sl eft knee sioce a motorcyde colli sion 10 days ago. Physi cal exami nation shows excessive posteri or movement orthe til>i a
rel ati ve 10lhe feO'llr. FJexionand extensi on are normal , and mere
is noevidence ofante<i or. medi al ,or1 atera1 insta bili ty, I nj ury to whi chof the to11 owi n9 rabel ed structll'es in
i
the sagittal MRI ofa normal knee is mostl ikely associ ated w th these findi ngs?

Anterior
6 A)

Posterior
6 B)

6 C)

:) D) 6 E)

2. A 68-year-old woman i s brougN.to the emergencydepartmeri by her husband 3 hours afterthe onset ofsevere pain in the mi ddle of her Sower back alter ca,ryi ng g,oceries.
She takes no medi cati ons and has nevet receive<J hormone rep1 acemen1 therapy. Physi cal exami nation shows tenderness overthe 1 umt:>osacra1 spi ne. Neu,01 ogi c
exami mati on shows nofocal findings. Wtich ofthe followi ng findings i s rnos1l ikely on an x-ray ofthe spi ne?
@ A) Compression aC!lKe ofL4
o B) Facet joint di slocationat L3-4
o C) Fracture of a late ral J)(ocess ol L1
() D) Herriation ofthe LS-S1 i nteivertebral disc
() E) Spi nal stenosi s at T12
3. A 22-year-old woman who i s a marathon runner has a stress hacture ofthetibi a. She parti ci pates in a study in whi ch bone densi ty is measured and is found to have
decreased oone oensi ty. Whl Cll ofthe followi ng is Ille most l ikel y cause'>
o A) CalCitoni n defioency
0 B) CC>lbSOI <Jeli ci ency
o C) Estrogen deficiency
o D) Growth hO(mone <lefi ci ency
o E) l nsum deli ci ency
o F) Parathyr0<d hormone defi ci ency
@ G) ParathyrO<d hormone excess
o H) Thyroi d hormone defici ency
o I ) Thyroi d hormone excess
4. lightdi rected into a patient's right eye reslAs in constricti on of the ri gtipupil but not the left pupi l Light di ,ected into the patienrs left eye results i n constricti on of the right
pupi lOut noc the 1 enp"'I, Whi ch ofIlle f,()IOwing neMlUS stn,crures on th,e ien is the most1i ke1 y si te of a le sion?
o A) Laleral geni rulate nucl eus
o B) Oculomotor ne,,,.,
@ C) Oi>ticvac,
o D) Pretectal area
o E) Supeo orceMcal gangrion
5. A 49-year-old woman comes to the emergency department because she has not slept for 2 weeks. She deries any previous hi story ofsl eeping difficulty. She says that she
has drunk one WIie of wine dai for the past week Out has noc drunk wi ne in the past She demancs to oe seen Oy the head of the department. Her speeCll is rapi d and
presstHed, and she is i rri table. A review of her records sho'NS a di agnosis ofmajor depressi ve di sorder 6 years ago. Whi ch ofthe followi ng i s the mostl ikety diagnosi s?
() A) Alcohol halluci nosi s
o B) Alcohol wi thdrawal
o C) Alcohol ism
@ D) Bi polar di sor<1e<
o E) Cydothymi c disorder

Number of urinary tract


infections

6. Whi ch olthelolowing is the median nurrber of episodes ol llinarytract irtect,onslor ctlildren (n:100) intho sample shown mthe graph?
0 A) 0
w BJ 1
(, CJ 2
D) 3
E) Cannor be determi ned fromthis graph

7. An investi gatori s eval uating the oltcome or coronaryartery bypass grafti ng coOOucted by two surgeons in the comrooni ty in the past year. Surgeon X, who operates
excl usi vel y at a t erti ary.care faci lty, has a 30-<Say O!)erati ve mortal ity of3%. Surgeon v. who Ol)erates excl USiVely at a commurity care faci l ity, has a 30-0ay operati ve
mortal ity of 1%. The difference i n operative mortal ity between sl.l'geonX and surgeon Y is significant (p<0. 05). The investigator concludesthat patients should have care at
the communi ty care faolity. Whi ch Olthe followi ng ra, ses tile most concern regaroing the vali<!lty ofllli s con<Mi on?
e A)
e B)
e C)
() D)
@ E)

Lack ofcont,ol of case oofTl)l exi ty


Lack of control or patient cholesterol data
Lack ofcont,ol of patient follow- up
Lack of control or patient recuperabon
Unequal sampl e sizes between surgeons

8. Four days after adrrission to the hospi tal wi th m!Ai plefractures sustai ned i n a motorvehi de colli si on, a 27.year-Oki man has the sudden onset of shortness of breath. His
reSl)i rations are 301min, Oopple< .-irasonography of the1 0\,-et extJeml ti es Sh01vs ceep venousthroml)osi s, ano a Sl)i ra1 CT scan ofthe CheS1 shows e><ioence olmuni pl e
sltlsegmental pul monary emboi.Anticoagul anttherapyi s begun. One week tater, a follow-up CT scan ofthe chest is done to eval uate atypi cal chest pain. Results show no
abnormaities, and the pai n was considered to be muscul oskel etaL Collateral ci rculati onfrom whi ch ofthefolowing bestexpl ai ns the lack ofidentifiabl e pt*nonary
parenehymal infarctsin thi s pati ent?
e A) Bronchi al arteries
e 8) Bronchi al vei ns
e C) Pulmonary arteri es
@ 0) Pulmonaryveins
O E} Superi or vena cava

es after bei ng hi t by a speedi ng motori stwhil e crossi ng the street. On arri val, he i s unconsci ous. Hi s
9. An 82- year-old man is broughttothe emergency department 30 mirut
tempeiature i s 37'C (98. 6'F). pul se , s 75/mi n, 1espi ratl ons are 30/mi n. ano bloOO pressure i s 140/90 mm Hg. NecrofOgi c exami nabon shows that the pabent is
norwes:ponsi ve with hyperrefle.xi aonthe right Pupils are reacti ve. An x-ray ofthel eftl ower extremi ty shows a fracture ofthe left femll'. A noncontrast CTscan otthe head is
shown. Blood is mostl ikelyto be seen in whi c:ll ofthe followi ngl ocati ons?
0 A) Epi c!uial Sl)ace
8) l ntraparenchymal h?ft parietal Jobe
e C) Subarachnoid space
@ D) Subdural space
e E) Wi thi n the ventricl e

10. A Syear-ol d woman comestothe physi cianbecause da 3-month hstocy of ctvoric back pain and diffict*ywalki ng. Netrol ogi c examinati on shows tenderness ofthe
spi neand footdrop Lumbar myelograms 81e shoWn. As thelumbar <isc herni ates inlo 1hespinalcana which ofthe following igamenlS 1s pushed into the spi nal root
nerves1
o A)
, B)
C)
D)
:, E)
o F)

Anlerior longitudinal igannenl


Crua!orm l igannenl
l nle<spi noos lgannent
llgamentum lla1l\.rn
Posteri orl ong,Wnal lgannenl
5'aspi nous ligament

11. A 55-year-ol d woman has had progressive tremor of her hands al rest, di ffictay ini tiating movements, and rigi dity of aD extremi ties. The most likely cause ofthis di sorder is
t
p
1 oss of neurons inwhi ch ot he toH owi ng &abel ecl strUC1ll'es in the photogra h ofthe brai n?

O A)

O B)

@ C)

O D)

O E)

12. A6-year-old boy has the mt.Cli ple l esi ons shown in the photograph. Whi ch of the followi ng is most likelyto be seen on mi croscopi c exami nation ofthe expressed contents of
a lesion?
e A)
o BJ
@ C)
o D)
e El

Buddi ngyeast
Fil amentous branctong hyphae
Molluscum bOdies
Multiple smal yeast and sllor1 hyl)llae
Scabi es m, tes and eQ!IS

13. ti a 24---year. olcfwoman, which ofthe folowmg rs most liketyto1nduc:e a shi ft in the normal balance ofbone accretion and resorption toward net bone loss?
e A) Bodybt.ildi ng ar<I use ofanabolic steroids
e B) Eating a tigh-calciundi et
e C) Living on a space slafionfor the past 6months
@ 0) Metaboic alkalosis ar<I chroni c vorriting
e E) Rtmng 110 2 niles, 3 times weekly
14. A 30 year--0l d man comes tothe physi ,ci an because of an 18- month tistory or frequentty falli ng asl eep dllXi ng the daywhil e at work. He reportsvivid halluci natorydreams
nght aherfallingaSleep at ni ght, ar<I o:n several occasions he momentan1 yfelt as though his tegs were paratvzed when he awoke In the moming. He recall s that his older
brothef frequentlyfeU asl eep at school He does notdrink al cohol orsmoke ci garettes. He is 178 cm (5 ft 10 in) tall and wei ghs 72 kg (160 l b); BMI is 23 kgfm2 Physi cal and
neurotogi c exami nations show no abnormaliti es. Whict. ofme followi ng is the most ,kel y diagnosis?
@ A) Narcol epsy
e B) REM Sleep IJehavior di sorder
e CJ Restless legs syndro
e 0) Sl eep apnea syr<1rome
e E) Sl eep terror di sorder
4

15. A24-year.()l dwomonw, th a2.year IISlolyol AIOSar<la 4-monthhi sto,yolhbet'cul osi scomes to the phys,oonlOra lollOW-14> examnation. C\Jrreitmeocationsrncule
amretrov,ral llle<apy ar<I anti tubercuos,sUle<apy Her wat sagns are witl'onnormal lrrots Pllysacal examl nabon Shows noabnormatruesexcE!l)l lor n-..iscutarwasll ng
Re'Sl.t,s of a PPO skinlest are norveadive.The presence ofwhi ch otthefolowing condi ti onsinttis patient isthe most likely cause ofthe PPD skinresults?
A) G<anuloCytOC)<!Ra
B) Hypocomp-emi a
C) Hypogammaglc)IUnerr,a
0) Lymphocytopena
E) Thrombocytopma
16. A case- control sttKfy i s performed to determi ne the associ ation between brai n ll
.. and cell phone use. The results show an odds ratio of 4.3 (95% confidence i rteivat
mo,s
0.9-13 2) foran associ ati on betweenbrai n tumorsand cell phone use. Asslllle that a 99%confidence interval was computed rallle< than me95% cO<llidence interval WliCh
ofthe fol lowi ng statements best descfi: bes the si ze ofthe 99% coOOdeoce intetv.11 compared to the 95% confidence interval ?
e A) Larger
e B) Smaler
C) The same
() D) Cannot be de1e<mi ned fromthe infotmati on gi ven

17. A24-year-<>ld man recentlydr ag,osed with a non-Hodglon lymphoma olthe 11et.mcomes tothe phys,oan for a kllow-14> exanination. He has a llelong histo,yol eczema
ar<I has had rocutrer1 pnoumcna ar<I 0111,sme<lra 1rleC110ns w,tto HHmOl)Mus rnDuonzae, Strop,ooccuspnoumoniao, and Slaphyfococcus .....,.,., Ploys,cal
exam, natJonshowsanerythematous, butgrng ngllttyfll)&r>t menuane. Laboratory stuces shOw:
Hemogl obin
14. 1 g,'dL
Hematocrit
42%
Mean corpUSOJlar volooie 87 l'ffl'
Leukocyte count
4320/mm'
Segmerted neutrOl)lils 79%
4%
Bards
6%
Lymphocy,es
11%
Monoeytes
Platel et co..c
29, 800.'mm'
Senn,
476mgldl
9S4mg10L

11 mg,'dl
lgM
Thi s patient most fikety has whiCh olthefolowing conditions?
A) AIDS
B) Commonvarl able,mmunodelooency
e, C) OI George syr<1rome
e 0) SjOgren syr<11ome
E) Systemc Sde<OSIS (SCle<Odemla)
F) Wisl<ot1.Ald1 1ch syndrome
18. carbon dioxi de produced by tissaue metaboksm , s earned in lhe bloodp,edom, narily on whrch olthe folowrn11 forms?
AJ Carbammohemogl obm
BJ co, dissolved i n pl asma
o CJ co, oneiythrocy1es
DJ HCo,- in eiythrocyles
EJ HCO,-inplasma

19. A50-year-okf man has hearing l oss due lo cen.men impaction i.n hrs right ear. Whi ch ol the lolowil'(I abnonmali li es are most lkely on Weber and Rime Jesting?
Weber Testi ng
Ronne T
esbng ol Righi Ear
AJ Lateralizes to the left
air conducbon > bone conduction
#r B) Lateralzes to the tefi
bone conduction > air cond!Jesion
CJ Laterozes 10 the ngt,1
arr condut:11on > bOne condiJCllon
" 0) Lale<aizes lo lhe rigtt
bone conduction > air cond!Jction
e EJ
Nonlatoral izing
air cond\Jclion > bone cond!Jesion
FJ
Nonlaterah:ong
bone conduc11on > a11 cond!Jcnon

20. Which ofthe folowing functions ofthe sl<in wil be most directly affected if the Merkel cell l ineage is illl)aired?

e AJ Arti gen processing

BJ Formabon ofvitami n D

e CJ l rrrnunologi c dcfe,,.-,c

DJ Prevention ofdenydration
EJ Production ofscll1.rn
FJ Protection from ultraviolet radiation
.,. GJ Regenerauon ol supelfiaal skin l ayers
HJ Tactil e sensation
I J Thermo<egwtion
A

21 A J.day-ol d newborndev feverand a stiff neck. Exa1r1nauon ol cerebro,pnal flui d slloWslUl'Mlrous ne<#OPhi ls and....., o,'am.negalM! rods. Whcll ofthe followi ng
,s lhe most lkelycausal orgarosm?

e A) Candida albicans
B) QyptococcU$ nootormans
C) EIICh&riChtBOOliK1
0) Gr oup B S0el)IOCOOCUS typeI
e E) Hasmoph1/us1nlluenzastype b
e F) Ustaria monocy1ogel)Qs
G) Neiss<His llWHlifl(litidis.serOQo'oupA
H) Pro/eus trllf9bll,s
e I) Streptococcuspneurnoniae
e J) Toxoplasmagondii

22. A 50 year-<1 d man comes. tothe physi ci an because offevet, rooscle pai n, sore Uuoat. and nonproducti ve cough for 7 days. He has had two episodes of si mi ar symptoms
l
l
duringthe pa$! year. He has smoked 2 paCl<S ordgarenes dai ly for 30 years. Hi s temperature is 38.9c {102 F). RhOnchl and crackl es are rieard on auscultati on Ollhe
chest Chest x-ray shows consoli dati on ofthe right towerl obe. Whi ch oflhe following mechani sms is the most likely cause ofthese ciinical findi ngs?
e A) I nacti vation of muramyt di pepti de
e 8) I nduction of ac idification ofeodosomes in alveol ar macrophages
ct C) l n ur)' tothe roococi lar)' escalator
j
e D) Sti mul ation OIlgM to lgA switchi ng
() E) Suppression of cough reflex
4

23. A 25--year-ol d man has a respi ratoryfrequency of 10/mi n, a b:dalYOl ume or 550 ml. anda dead space of 150 ml. Whi chofthe foDowing is his a>.<eolarventil ation (in l.hrin)?
() A) 1.5
@ B) 4.0
e C) 5.5
e 0) 40
e E) 55

24. A pie'\liousl y healthy 42- year.-okl woman has a 6.week hi story of fatigue, lightheadedness, and shortness of bfeath . Laboratory studi es show:
Hematoai t
Hemog obin
l
LeUkocyte count
Platel et COUnl
Reticul ocyte count
Total serum bilirubi n
Direct sernm bi irl.Clin
Uri ne ll'obil inogen

25%
8. 3g/dl
10, 0001mm'
250,000/mm'
4%

3 mg/dl
0. 3mg/dl
2

A per1i phera blood smear shows mi crospherocytcs. Whi ch ofthefollcw.ing i s tho mostl ikety cause ofthe ancmja?
l
e A) Ac."e hemormage
@ B) All!oi mml.lle hemolysi s
e C) Hemo ytic-uremi c syndrome
l
() D) Hypersplen
i sm
E) Throrrtioti c thrombocytopenic purpura

25. A 17.year.ol d g:11 comestothe physicianbecause of a 1.monthtistory of progressivegeneral ized headaches and episodesoffading \'ision i n eithereye for several
seconds. Vi sual aclity is20/20. The pupilsreactwen tol igtt andaccommodation. Visual fields areflAon confrontati on testing. F"unduscopic firdngs ofthe right eye are
shown. Whi cll ofthef<>lowmg is most Ike!(?

A) Optic atrophy
() B) Optic neuritis
@ C) Papi lederna
() 0) Retinal inlardion
e E) Retrooolbarnel6itis

26. A25-year-ol d woman (type A, Rh-negati ve) gYeS bi rth to a typeA, Rh-posi tive newborn. Which of the followi ng shot.*:t be admi ni stered tothe mother immedi atel y afterthe
bi llhtopreven1 hemol ytic Clisease ofthe newt>om in futurechi lcttefl?
@ A) Acti ve 1nvnul"ltation to induce anti- Rhl gG anti bodies
e 8) Acti ve i mmurizati on to induce antJ. Rfll gM antibodi es
o C) Acti ve , mmu,vzatioo lo remove Rh anhgen
e D) Passave imm.mi tabon to 1 emove Rh antigen
o E) Treatment wilh corti coste<Otds
27. Whi chofthe following OOfadreoeigi c receptor dassesis fol.l'ld in both pre.synaptic and pos1synapbctermi nals?
6 A) a,
@ 8) a,
() C) .
0 D) ,
OE) .
28. A newborn who 1 s born at 27 weeks' gestabon has respi rataydistress syndrome. Tbs most ikely restts from i nadequate secretion fromwtich olthefolo'Hing cell types
labeled , n the ,nustrall<)n clOle alveolar wal Shc)lvn?

A)

0 8)

C)

D) ( E)

29. A 53-ycar-ol d man who i s a professi onal golfer has a 1. 5- cm red scal y nodule on the hel ix of his l eft ear. Abiopsy ofthe lesion i s shown. Whi ch of the followi ng is the most
likely IP{ecursor to thi s condition?
@: A) Acti ni c keratosi s
e 8) Basal ceD carci noma
o C) Dysplastic news
e D) Fibrous hi sti ocytoma
e E) Seborrhei c keratosis

30. A60 year-ol d woman has the sudden ooset of midthoraci c back pai nwhil e doi ng housework. She has had di ffuse aches and pai ns for 3 months. She had a renal cakul us 1
year ago. Physi calexami nati on Sflows kyphOsis wi th tenctemessi n the mi dthoraci c ,eg,on, The remai nder otthe 8)1:Qminati on shOws no abnormalities. Laborat ory studies
show:
4

Hemogl obi n
Hemafocrit
Seam

Ca""

Urea ni trogen (BUN}


Creatini ne
Phosphorus
Total pJotei n

12. 9 gldl
38.9%
11.Smgldl
12nigldl
0.9mgldl
2.9mgldl
7.2 gldl

Which ofthe followi ng is the mostl ikely di agnosis?

o A)
@ 8)
o C)
o DJ
o E)

Ankyl osi ng sporodyli tis


Hy,rathyroi d1 sm
Ostettis del ormans (Paget di sease,)
Osteomaiaoa
Osteoporosi s

31. A 744year-ol d man is mabl e to repeat phrases and name objects after ha\1ng al eft-si ded stroke. Although readi ng cormpfehensi on is preserved, he has di fficulty reading
a1 oucs and writing, His spe,ech is fk,ent,anet hi scc:>mprehensaon i snormal. Whi chof the followi ng S1Nctures, s moSIl ikel y damaged?
o A) Arcuate lasawus
o 8) Al>d1lol)' associati on area
@ C) Suppl ementary mow area
o DJ Unanate lasci cul us
o E) Word recogrotion area
32. A374 year-ol d woman has a 3-year history of progressr,e dementia and wotflng movements inal fOll'extremi ties. Her fatherdied da si mi la.J Ill ness. Whch of the followilll
sttuc:lures is mosa lkely 10 be if'IV(Wed?
o A) Cetebelar hemsphere
o B) Qngulate (kmboc)motor cortex
CJ Neurons ofltiestlal\Jm
o OJ Plgnented ne..ons ol the substanla n,gra
e EJ Pnmal)' mot0<tor1ex(area 4)
F) Subtha am c nucleus
l i

33. A 27 year-0ld woman is brougtt to the emergency departmert 30 mmutes after bemg 10V'CIM!d in a motorvetucl e C<Jlisi on. Physi cal examnanon shows exqui si te
tenderness ol lhe l ell upper eldremrty. Anx-ray ollhe aected area Is shown. llis pabent ,sat greatestnskfor ,njtry olwhi ch ollhe folow,ng neM!s?
4

A) Medial

B) Median

C) Muscuk>ct1.11neous
D) Radial
E) Ul nar
34. A 35-year-ol d man has the sudden onset of headaches and parti al loss ofvisi on. He has a hi story orrheumatic heart disease and atri al fibril lati on. He has a l eft
homooymoushemi anopl a votnparti al macuiar spnng. The most lkel y e><Qlanatioo ror tnese fin<:li ngs is occlusion ofwhi ch ofIlle IOIOwingarteries?
o A) Lenbcuostnate
@ 8) Mi ddle cerebral
o C) Posterior cerebral
o DJ Thal amoperto,ate
o E) Verteb<al

35. The table showsSUIVMII of patients who had an operati on for a par1ic(jar form ol cancer.

tnteMI
0.1 year
1.2years
2-3years
3-4 years
4-Syears

Number ol Pab<>nlS
at the Be111nni ng
of the 111ter,a1
300
185
143
124
106

Nlfltler of Patients
Who Dred D<rnng
Illel nteMJI
115
37
24
18

25

Percent of Pabents
SL1VMng
ThrSlnteMI
62

80
84

85

76

,
I f a pati ent survives 2 years ane the ope,atl on. wlll eh of the lolOwi ng rs the probabtbty of SUMvtng at least 4 years?
t- A) .80
(' BJ .85
CJ .84 x.85
0) .62 X .80 X.8<1 X 85
c E) .85- 80
36 A25-year-dd man has btlaleral artlrri tis ollhe sacrooiacjoints, i nllammation ollhe shoulderjoints, and early i!MllYefnent of the llmbarspi ne There is norash,
s\bQJ!aneous nodul es, or urethrlbs Wl'leh olthe followi ng Is the most bkel y dl agnos,s?
8 A) Ankytos, ngsponctytrtis
c B) Gouty ar11'fr ti s
C) JtNer,le rl\e""8tood ar1Ml1 S
OJ Osteoar!Mtis
e E) Psonabc alllv,bs

37. The openi ng ofwhrch of the following types ol ioo channel rs reQUi red for release ol neurotransmi tter from a synapticending?
f" A)
"' B)
t. CJ
D)

Acetyl<:hoine receptor iOOOl)hore


Ca' Chaooel
ca>-.adlvated K' chamet
Delayed redifiertype of K channel

C E) Na Channel

38. Appro,oinately 87% ofdeallls resulbng korn kllg cancerare dl red!y related to ogaret!e smolong. Whi ch olthe folovong best represents ti.s find, ng?
e A) Tho attri bl.13.ble risk is 87%
" BJ The rnodence 1s 13%
" C) Tho incidence is 87%
e OJ Tho positive predictivevakle is 87%
E) The prevalence is 13%
"' F) The prevalence 1s 87%
39. A42- year.ol d manwho i san accountant comes to the physi ci an because ol an 8 -month hi s1ory of anxiety during company meetings si nce bei ng promoted to area
manager In tis new POSiti on. heis required to make weekly presentations to U'le five otherarea managers and t he boafCI of direct0<s. He says he i s " gri PC>ed by anxi ety"
whenever he coOOuctsthe presentations oreven ttlnks about them. He describes feel ings of dizzi ness, pal pi tations, sweati ness, and general "dread." He tries to avoi d
making the presentations and is concernedthat hi s reacti ons are noticeabl e and embarrassing. He is afrai d that he mi gh1 fai nt from anxiety duri ng a presentation. Whi ch of
the ronow, ng is the most likelydi agnosi s?
e A) AoJ..e stress di sorder
o B) Agoraphob a
i
e C) Facti tious di sorder
@ D) Generalized anxi ety di sorder
e E) Maj or depressi ve disorder
F) Soci al phobi a

40. A6-week-ol d boy is bfoughtto the physi ci an by tis mother for a routine wel-chikf exami nation. He is atthe 50th percerii le for length and wei ght and the 90th percentil e for

head circumrerence. ACT scan ofme heM shO,is an eo1 argec posterior Iossa conrai ni ng a large mi dl ine cyst that is assooatec with absence olthe cerebellar veoris
Whi ch ofthe following is the mos1l ikely diagnosi s?
(') A) Agenesi s ofthe corpus callosum
@ B) Chi ari rype II maormati on
o C) Dandy-Walker syndrome
o D) Down syndrome
o E) Hol oprosencephaly
o F) Hydrocephalus ex vacuo

41. A27.year--0l d man has a pai nf i nfected abscess on the posterior aspect ofthe scrotum. Whi ch ofthe following tylll)h nodes i s most likel y to be tender and swol len?
6 A) External iiac
6 8) I nternal il iac

<11; C) Luni>ar (aorti c)

6 D) Sacral

6 E) Superfici al i nglinal

12. A 50-year--01d woman has a I-year tastory of re<urert hemonhages and thrombosis. She has a brn,se on 1hefo,ehead secondary to m,no, lra<ma. Evaltabonfor undeil)tng
1rllammatory and neopl astic di sease , s negatove. laboratorystu:1,esSholY:
Hemoglobin
Lel.l<ocyte colJlt
Segnented nel.Croptals
Bandlorms
Lynl)hocytes
Monocytes
Platelet count
Er)llocyte mo<pl1ology

12gldl
7000lnm'

60%
5%

30%
5%

2, 000,000/nvn'
normal

Whidl ofthe folowing is the most ikefy diagnosi s?

t A) Cllroni c mye4ogeoousl eukem,a


,. B) Essenti althromt>oc1111ema
l
C) Myelofl boosls
o 0) Pol)'C)'them,a ve,a
o E) React, ve thron'C>Oeytosi s

43. Addition or OOfepinephri ne to a sol uti on bathing an isol ated cardi ac papi llary musde l eads to an increase i n the amount of work perfOfmed by the muscle at a given load,
wi thOul changi ng lhe musdE!:'s ini ti all ength. The i netease in w0tkdOne bythe muscl e at i 1s new steady.statel evel is the result ofan increase rn'whichof the followi ng?
@ A) overlap o,tllcl<. and t lln filaments a1 111e ini tial l englh
B) Sarcopl asmlc ca conoontratlon wNl e lhe IOad Isl ifted
t C) Sarcopl asmlc phosphocreatlnc concentration
b D) Stiffness ofthe series el astic el emeris
44.
Test

110

Pos1tive
Negati ve

1890

2000

The performance of a new noncul ture test for infectious disease Xis evaluated by compa1i ng its performance to that of a perfectly accurate cultU1e in an 1,11sel ected
po,pu1a1i on or 2000 ,nd, vldualS. The resul ts or m, s eval uabon are snown. Whi ch olme IOIOwing , s lhe Sl)ecifietty oltl'ls newtest relati ve to that OIcul ture?
0 A) 66.7%

B) 83.3%
C) 90.9%

O 0) 94.9%
"' E) 99.5%

45. A 3,.month--0l d boy is brought to the physici an by hi s motherfor a weJ.-chi ld exami nati on. He i s atthe 80l11 percenti le101 lergth and the 60th percentil e for wei ght Physi cal
examination ShoHS no abnormali ties. A Babi nski sign is present tlil aterauy. Whi ch olthe ronowi ng best e,p1 ains this findi ng?
o A) Axons olmotoneurons have not yet , meivated the for.4 muscles
@ B) CC>obcospi nal tracts i n the Sl)mal cord are not yel myelmated
o C) Dorsal rooc gangia or the l umbar segments have not yet differentiated
o 0) Monosynaptic ,ene,es from the l eet are not yet eslablished
o E) MC1Scl es lhet flex the toes ventral are not yet mature
46. A 22 .year--0l d woman participates , n a dirwcal studyof a pu:ative protem hormonethati s synhesized in the liverm response to a deaease in the blood platel et
concentra11on lrle(llon ofthe hormone causes an increase on platel el pr-on Wlliell oftheloll o1Yi ng is the most lkel y1 oca11on of1he recepto, that accounts rorthe
increase I n platellll eotn?
A) tn 1he cytopl asm olhepatocytes
B) tn 1he cytopl asm olmegakaryocyteS
C) tn 1he nudeus ofhepatocytes
0) tn 1he nudeus ofmegakaryocytes
E) On the cell merrlltane of hepa1ocy,es
ol megakaryoq,ies
F) On the cell

menmne

47. A studyi s done todelemi:ne the efficacy olfish oil in preventillJ progressi on ofrenaldisease in patierts withlgA nephropathy. One hundred conseo.tive patiem with newly
OoagnoSed lgA nel)IWOPathy are entered orcothe Stud\' The Odd runbe<ed pabentSare treated w, lh fisn OIi and the even nurrbe<ed pati ents aretreatedwith ol ive o, an
inactiYefatty aci d. o..ierfhe next 5 years. alpatients ace folowedforthe developmer1 ofend-stage rem!dsease. Results areshown:
=

Fish oilgroup (n 50)


D1veoolg1o14> (n:50)

NIIOOer Reachi ng End-S1age


Renal Di sease

17

The orodence olend-S1age r enal diseaseon 1hefiSll oi l gr0<4> was si g,.li cantly-lhan on 1he olive<>IQJOl4> (p<O.O1). Whcllolthe followi ng besldescnbes Ills stwy?
e A) case-control
o B)
series
C) C-Ohol1
0 D) Cootroled!rial
e EJ Ctossove<
e F) C<oss-sedional

case

48. A 37-year--0l d man comes tothe physi ,ci an because ofsevere back pafn for 2 days. The pai n radi ates downthe buttock, posteriorthi gh, and posterolateral l eg. He atso llas
numbness on the si de of hi s ren foot. On p/lySical examination. sensation 1 0 pain i sdecreased over t he 1a1 era1 si de oflhe 1en foot. Deep tendon reflexes areabsentatthe
l eft arnkl e, and there i sweakness of dorsiflexi on ofthe k!fttoot Compression ofwhi ch ofthetollo'Mng ne:rve roots is the mos1l ikely cause ofthese findi ngs?
O A) T12
OB) L2
oC) L4
@ D) $1
o E) Sl
49. A28year-ol d woman comes tothe physi ci an because ofa 5-month history of sevefe, throbbing headaches followed by exhaustion. She says thatthe headaches are
sometimes f)(ece<Jed by sl owl y enl argi ng, C-shaped fl ashi ng lights. She is a 1eacher and onen spends moch l ime reao,ng and prepari ng examina1, ons on he< compu1er.
Sh" oiso consi ders herJob eX1remc,1y sessful. Neurol ogl c examl nali on $hows no abnormal ili es. Which of 1he followi ng ls 1he mos1 lkel y di agnosis?
() A) Acute si nusi ti s
o B) Cl usler headache
o C) Eyesirain headaehe
" D) Mi grai ne
( E) Tensi on.iype headache

50. A 17.year-ol d boy i s stabbed i nthe ante,i or axma. He has loss orsensation on the anterol ateral sllface ofther01earm and loss of the bi ceps 1eflex. Whi ch orthe followi ng
nerves ,s most lkel y 10 be damaged?
e AJ Axi llary
o B) l nte<costObrachi al
o C) Lateral pec1ora1
o D) Long thoracic
o E) Medial anteb<achi al cucaneous
o F) Medi at bractoal cutane<1us
o G) Medial pectoral
o H) Medi an
@ I ) Muscul ocdaneous

NBME form 5 1
1. A 55-year-Oktman with hyperteflsi on hasthe sud<feo ooset ofweakness oflhe righl l ower extremity. The ri ght uppere.xtremi 1yi snot affected. The most lik ely cause is
ocdusi on of vlhi ch of the folowi ng arte<i es?
o A) Anteri orcerebral
O B) Basi lar
o C) Mi O<JJe webral
e, D) Posteri 01 cerebral
O E) Vertebral
2. A 75- )'<lar-ol d W()man Is broughl to the emergency depar lment seml comatose 24 hOurs aner hi tbng the back dhet head In a fall. No symptoms w,irepresent after Ille fall , but
she devel oped bl urred visi on and weakness of thel eft extremi ti es 18 hours after the faJ. Exami nati on shows left hemi pares.s; the nght pupi l i s dil ated and unresponsi ve to
tight. A CT scanwi thout cont,asl shows a di ffuse ext,a. a.xial hemouhage over the 1i ghl cerebral hemi sphere. Whi ch of the following is the most likel y mechanism of i
A) Contusion oflhe brai n
e Bl Laceration of a cerebral bn<l!)rng ve, n
e C) Laceration of o smalt eptomenl ngeal bl oOd vessel
e 0) Laceration of Ille ml ddkl menl rogoal ane<y
e E) Sheari r,g IO<C<l$ damaging wtltte matter
3 A 27-)'<lar-<>I dman is evablted lorvertigo and hearing loss i n the right ear. Wl1ch olthe folowing labeled crani al ne,vesi s most ikely damaged?

A)

r B)

C)

- 0)

E)

F)

t G)

4. A 65- year- ol d man comes to t he physi ci an because olshortness of breath on exerti on for 3 years. He has smoked 2 packs otcigarettes dai ly ror 50 years. Hi s artenal

oxygen saturati on I s 84% (N 95-98%). Ttis patl enrs lnollillty 10 mai nr.,l n adequate ti ssue oxygenation wr ll most l ikel yt ead 10 lncteased producti on ohvhl c:tl of thefott owl ng
I n hi s eryttvocytes?
e A) 2,3.8,sphosphogtycerat e
O B) El)'lllrOPoi ell n
e C) -Hydro,cybutyrate
e 0) Lactate
e E) Methemoglobi n
e F) NAOPH
5. An mtox, c:ated 24 year-oldwoman ,s lxougN to the emergencydepartmen because of a femoral fractll'esustai ned m a fal from a second ffoor balcony. Two days after
i rt.emal fixation ofthefracture. she i s agi tated, appears cortused, demands tol eaveme hospital againstmedi caladvice, and accusesme staff c:lrorturi ng her. PLtse is
120/mi n. and blood press'-'i s 160/1 10 nm Hg. WIich ofIllelotlowirog best explains thi s beha,aor?
c, AJ ACI.Ce stess disordOI
Bl AlCOhOIWllhCfrawatoeti rhm
t C) Bi pofar cisorder
t, 0) Oeluslonal dr sorde<
" E) Post.tral.lTlalic stress clisordef
6. In a suty of 100 slqeds wi th coble potyposi s, 20 develop colonc malignancies c,.iera 10-yearfolow...-up peri od Toesti mate the relative ri sk ofCOO'li c potyposis forthe
oevetopment of COioncancer, wtoc:tl ollhe fonow, ng ooruot orows WOUid be best?
e A) Heallhy me<lrcalswdeots
e B) A listorical corlrolgroup
O C) A simi largrcq, olsubjeds wi th coloncpolyp05is
e 0) &.c>,ectS den ved ltom a oastroe11ero1 ogy referral Clmc
o E) S<bjects willl no4Mdence of colonle pOlyposrs
4

7 Wlich o11rie fonov,;ng ne1.ro1ransmners i s rel eased in excess following i sdlemi a and can i ni tia1ea recepto1- me6ate<1 ca irtlux that leadsto netJrooal dean!?
A) v-Ami noootync aod (GABA)
" BJ Gl t.Camate
O C) Glyone
e 0) Hr starnne
E) NorepinQphrine

8. A 47- year.ol d main comes to the emergency departmeri because herelt the sudden kiss or sensation and abi ity to move the leftsi de oftis body. Hi s fathei di ed ofa stroke
at thi s same age. Physi cal examination shows no OOll'oklgi c defects. The pati enlseems lllConcerned about tis symptoms. Whi ch of thefalowiog is the most
fikely di agnosi s?
o A) AJ::J.A.e stress disorde1
o B) Conversi on di s0<Cfer
e C) Generalzed anxi etydsorder
e D) Hypochondri asi s
O E) Pani c di sol'Cler
o F) Somat,u,tl on di sorder
e G) Sil eo! ce-rebral i nfarcti on
9. Whch olth<l tollow,ng c:or<l1 bons 1 s mos! likely to bel)<esent whenransversehlstologi(sections ctIle COMcal spinalcord at c.o5hoWdegeneno'<,n of bolh c0lbe0$l)l nal
1rae1Sand atrophy oflhe ventral roots?
e A) Amyotrophi c lateral scl erosi s
e B) Mt.liple scl erosis
o C) Pdlomyeilll s
o D) Sponat cood transedlon at c.2
E) Spinal cord transedlon at T-8
e F) Tabes dorsals

10. A2e-year.old man comes tothe physiaan b<!ause cta c,..mon111historyctIOsl ng consciousness When oo lat91S. He s 12 to 14 hOtl$ every nigh!, has\llvld dreems
and ni ghtmares, andfeels tired du nngthe day. He was recently ll1YONed ma motorvehi cle cofli saon. Findi ngs on physical exarr1nation ace twema.tka.ble. Ylhlch ofthe
-wing i s 100 mostl ikely diagnosis?
e Al Absence sell!Jles
o Bl CompieX pan,a1semses
f'i C) Ma ordepressiYo di sorde<
e DJ Malij ngenng
() El Naroolepsy
e F) Pos1. numa1, c stress dlS()Cder
G) St eep apnea
e H) Sl oopterro, disorder

11. A 58-year- oldwoman dies after a 2- year hi story of progressi ve memory kiss. disori entation to ti me and pl ace, alld i ocreasi ngfyfrequent epi sodes of l ethargy and cortusi on.
Therewere no focal neurol ogic deficits. Which ofthe followi ng findings i s mostl iketyat autopsy?
e A) Decreased concentrati on ofy. ami nobutyric acid (GABA) in the striattm and the substantia nigra
B) Degeneration ofcorti cal and basalforebrai n neurons
o C) Formation ofLewybodi es in the pa1s compada of the slbstantia ni gra
() O) i ncreased concentrationor acetyl cholinestecase In the ce,e1:>ra1 cortex
E) Presence of anti bodi es to the ni coti nic chofinergi c receptor

12. A 16,.yea,.old boy has pa, n rnthe ngtt knee i mmedi atelyallerlwl 1 1 v.tlleplang soccer. E>ami nobonshOWS excessi vepasteri or slod1 ng or lhebbi a wllen ,ti s pusooc
backwardwith the knee flexed. Whi ch ofthefollowing li gaments ismostl ikelyi njured?
e A)
e B)
e C)
, 0)
o E)

Arveri<lrcruoace
Postenorcruciate
Coronary
Lat eralcollateral
Md1al conate<al

13 Two daysaner a hi ke in thewoods, a 26,year-old man has a rai sed, red,vesia,l arrash onhi s righl lower extrenily. Wlich ct the lollow,ng eeltypes is responsible lo<
sensrul/lty 10 Ille relovanl armgen?
A) Basopt;l s
f., B) CD4T iyn1)hogtes
( C) Eosi noptil s
e D) Neutrophl s
E) Plasma Cllls
14. When a movemert is planned butnot executed. whi chofthefolowmg l abeled regi ons of the normal brarn shown WilshcrNan maease m activity in a ftn:::bonalMRI?

A)

B)

C)

0)

E)

15. A25-yea,. ofd woman whohas beentaking anl.lialcnvn drug for 3 weeks ,s having a veiy heavymenstrualpenocl. Petedli ae are pJeseri.on her legs. She 1 s anemic.
Platelet co<Jtis 3000/nm'. Whi ch oflhe follOI findi ngs indicates lhatherthrombocytopenia is a re..a oftoxic s,wession ofplatel et proructioo by lhedrug instead of
an autol m"""8 <11sorder?
() A) Decreased numbers dmegakaryocytes in thebone marrow
o B) Enlargement ofthe spleen
C) Fai l ure 10respond10platel et tra0$1u$lons
o 0) Gi ant plalele!S on a penpheralDIOOd smear
o E) Apositiveantiglobufintest
16. There 1s no ve vacctne against which of the follO'HtrlQ Viruses?

0 A)
,, B)
-, C)
r, D)
O E)
f'> F)

Hepabli S 8 "1rUS
MeaSles virus
M1611ps virus
PolioYirus
Rl.lJela vnus
YelOw leve<v1 1\JS

17. Five separate analyses for sen.m gl ucose are done us,ng a new melhod on a si ngle sample. The vau.s obtained are 102, 100, 96, 101, and 98 mgdl. These types of
data are used t0 determne WIICII ofthQ following charactensllCS ofthQ test?
<!: A) /lasacy
B) Precisi on
t, CJ Pre<lldlve value
0) Sens,tMly
t El Speafic1 ty
18 I n o,gan tronsptaraliOI\ g1aft-versus-hosl disease may be i nduced by dono, Tlymphocytes Whdl olthe folowing disease-speci ficorgan lranspl aru is most rkely to
denve some benefit fromthi s graflNersus-host response?
t A) Bone marr01v uansplan1 fo,aclfe myel ogenousl cuk"'11ia
, B) Heart transptartforYirol cardi omyopa!hy
c C) Isl e! cell lransplanl for type t diabetes melltus
t 0) Kidney 1ransp1 an1 lo, membranous glomenAonephri1s
f' E) Lung uanspl antfor primary purnonruy hypertension
19. A 27.year. oldwoman comes to the di ni c because of a 7. year hi story or intermi ttent headaches, abdomi nal pai n and bl oati ng,l owback pain, di arrhea, pain during sexual
intercourse, decreased interest in sex. and di fficulty wi th balance. She tells the physi ci an that shei sfrustrated because she cannot wotk because of her symptoms. She has
seen 12 physi ci ans in the past 5 years, none ofwhom has been ableto di agnose her concition. Thi s patient mostl ikefy haswhi ch ofthe fol lowing disordel!S?
o A)
o B)
e C)
e 0)
O E)
o F)

Autoi rrvnune vasculilis


Chroni c cadmi um toxicity
Fadi ti ous di sorder
Fi bromyalgi a.
Paraneopl astic synd,ome
Somali uillon di sorder

20. A 49-year-old man co,nes lodlo physi cian because of inaoasinglysevere pain in hi s righl llunb <Ming thepast 2 days. He has a 10-year lislo,y oftype 2 di abeles
mel1 1uS. Physical exami nati onsh01vs-111ng ancs eryJhema of the l!Urb. X.fays otlhe rlghll l'Unb show osteomyehbs at me arrows shown. Which olthe loll ow,ng
SIIIICt..-es i s mostl ikely affected by the pathologic process?
A) Ad<liiclOf pQaci s br8'1s muscle
B) Ad<llicl0< poacls rT<JSde
t., C) EX1"'1S0f pci!ICIS loogus tendon
f 0) FleXOf po41icis longus tendoo
El Muscul ar branch of medi an ner,e

c
e

21. A 32-r-<*l man 'lltlo is seropos,bve for HI V has a decreased CD4 T-lymphocyte c:ount. Which ol 1he fol lowi ng 1 s most ikely decreased In th,s patient?
.} A) Anhbod)'.def)ende,,t cy!OIOOty
B) lnlllrlet1<1 n.2 (11. 2)
, C) Myelop<,roxi dase
' 0) NAOPH oxidase
v E) Tumor neoos,s lactor

22 A60-ycar-<*J woman has had progressive weal<ncss over!ho past 8weeks. Conl)lclo blood eo<n shows
Hemoglobi n
Le11<ocy1ecooni
Segme(lled neUlrOphlS
Lcy!es
Monocy1es
Erytt<ocy!es
P1alelet <Xl<l1I

8. 0g/dL
15,0001mm'

65%

25%
10%
rooleaux present
60,000/mm'

Sooy pcrcertof bone marrow cells have feat1Xessim:l ar to those shown Whi ch orthefolowing adclbonal findi ngs 1s most llketyto be present?
..., A) Anb:nudearartibodi es
O B) HelelOPhde anli bodi es
C) Increased letJkocyte alk8'ne phoSpholase aaMIY
0) Light chai n prote1 m11 a
,.. E) Reciprocal chromosomallranslocabons (9:22)
23. Astooy is donetodetermine the relabonshpbetween use oforalcontraceptives endteMcal cancer. Study StC,fects mdOOe 50,000women who are usmg OJal
contracepti ves and50, 000 womenwhohave had a lubaJl iganon. After 2 years offollow..up, the rate cl1 n situ cervical cancer is 18per 10, 000 i n thecnl contraceptive
group and 3 per 10,000 (p < 0.05) i n thelloaJ figati on grcq1.W'tlch ofthe folowing isthe esti mated relative risk ofceMcal canceramong users oforalcontraceptives as
comparedwithwomenvmo have had a h.bll igati Ol'I?
A) 3- 18 0. 17

B) 183=6
C) 18-3= 15
- 0) 18 3 = 21

E) lnt1et (Hmi nablefcm


r the data gi ven
24. NI 1S-monllMJid boy whO 1s "chubby" 1s b<oughlto theph)'SIOans omcefor a 1001Jne examnabon. HisweigttI s above 1he 90th pe,cenble. Laboratory studi es Sh0w:
Hematocri t
Leukocyte COll'II
Mean corpuscularvol ume
Plalotel COin
Whi ch ollhe foll ow,119 l aboratory vuIsmOS1 ikely to be abnormal ?
, A) Cllolesle<ol
., Bl Fol ate
., C) I ron
0) Vi tami n e,, (cobal arron)
") E) Vi tami n E

30%
9000/mm'

60m,

650,000Jmm'

25. An80-year-okt woman comestothe physi ci anbecause ofa 10.year history ofsl owt,, worsening pai nin both knees. She has a 20- year hi story of hypertensi on andworked
as a housekeepermost ofher adultl ife. Physi cal exami nati on showsirregul ar bony promi ne-ncesaround the li bi al pl ateau atthe kneejoi nt line bil aterally and moderate
bi lateral kneej oi nteffusions. X-rays ofthe knees show thi nned cartil age and extensi ve osteophyte formation. Whi ch oftheroHowi ng i sthe earli est stage i nthe development
ofthi s conditi on?
e A) C<acki ng ofthe carti lagi nous surface
o 8) Margi nal osteophyte prolireration
o C) NeOYascul ari zation
o 0) Seepage of articul ar flui CI into the bone
e E) Subchonelral bone cystfonnati on

26 A 19.year-otl 1wiman comes 10 Ille physl aan b<!caus<a ot rerurrenl heaclamas for 3 w..eks An MRI ot1he brain ,s shown; Ille arrow1nd1 ca1eslh sll ot an ,cpanclng
l esion. Which ot!he folowingad<ltional findi ngs i s most likely in tlli s patienl?
t' A)
t, 8)
l C)
t' D)
t, E)

Babinski sign that is presenl bilale<ally


O.ab<!tes ,ns,p,dUS
Impaired bilatraleye &d(lJCII OO
Left-si ded hemineglea
Ver11calgaze palsy

27. An 18-year.old man 1s brougf1 to the emergency departmertafter riLIUlQ l'ls l eft shot*:tcr whi le playmgl acrosse. He has pain 1 nthe shoulder and dt#iculymowng the
hMlM!d exlremly An ar1eropos1eno, (A) and an axi llary (B)x-rayol the shoulde< are sll<Wm Wloch ot the following nerves1s mos1 likely 1t'J)rec!?

(' A) Axlllary

t BJ Medi an
r C) Muswocuaneous
t, 0) Radial
28. A newtest has been devel oped to deled the presence ol a too,or.speofic l)<otein in 5ellffl. The ini tial evallati oo ot!tis test
TestRosult

:::;.t;:,

T'""'

Present

Absenl

11--"'

;;....--+-..;
a;-1 :
100
50
50
0

Which of the lolowing is thel ikeihood lhilt a patient with a negalM! test does not have a t11t101?
A) 0.25
8) 0.40
C) 0.60
" 0) 0.75
"E) 0.80
29. A 35 -year- okt man hasthe sudden onset of excruci ating headache and losesconsci ousness. He has nuchal rigidi ty and bi lateral extensor posl\lirg. Whi ch of thefolowing
is the mostlikely cause of hi s condi tion?

e A) Cerebral embol ism


O B)
o C)
e OJ
O E)

Laa.mar infarction
Large vessel thrombosi s
Parenchymal hemormage
Subatachnoid hemorrhage

30. The'1(eval ence cta diseasei s tl'lJeetimes g,eates 1 n nonwhi tes than ,n whi tes, blJ the i nci dence clthedi sease , s nod1 fferer1: inwtltes than 1 t 1s, n nonwhi tes. Y"ltld'Ithe
f-ng bestexplains these findings?
o A) The case fata tyrate i s hi ghecamong nonwtites
li
e B) Theduration ofthedi sease i s shoner in wtites
c C) Nonwhi tes use medical care foolles more often than whiles
c D) Thentmier ol newcases I n nomtlotos IS l rceasmo
o E) Theproporti on dasyrtl)tomanccases is hi mnonwtvtes

31. Apatient willl blood grOIC) D undergoing an ope111tion forrepai r ofan aortic anellYSffl 1 s mistakenly transfused willl a !10\C> A111i t or packed red blood eels. He
immedi ately goes into shock. A blood sarll)l e is markedly hemolyZed. Wllich clIlle foll owi ng i s Ille prima,y mechani sm for Ille hennotysi s?
A) Action by prope<dn to - C3
B) Action ofC5-9 CO"l)l ex ofcomplement
!) C) AcirVation of facto, XII (Hageman factor)
., D) PrOOudlon of CSa to Sbrrdate p11agocy1osrs
., E) Release oftistami ne by1he acti on cl

csa

32. A newdug has been developed thatseledr,elyblocks cal citrnchamels atthe SJnOl)Se. Whi ch ofthe lolowing is the step ,n chemical tranomssion that i s mostil:ely to be
di snc><E!d?
A) Binding ofthe neurotransmi ttec to the poslSyf'lapti c receptoc
c B) Propagobon oftheadlOn potentl al lrtothesynapse
c CJ Releese orcal ci um om,.,.,_oell ul arSIOfes
c D) Release orthe ne..Olransmtter Into the deft
O E) Syrlhesi s and packaging ofthe net.rOlransmtter

r.story

olfeeing dizzy Worm wat er instilled mto th e ten earw hileIlle patient is 5'4)me elicitsn ystagmus with a quick phase lo the lefl.
33 A 29-year-old man has a 1-week
warm water mi lled u,tothenght ear ebats no eye movements Whi chof the folow1ng 1s themost lkely site ofa leston?
t A)
t, B)
(' C)
(, DJ
t E)
Fl

Left kootaleye fields

Rigl-t frorul eye fields

Leftponl1ne gO!.e oente,


Rrgt,tpor11ne gaze center
Leftvestibuar apparatus
Riln vesllbular ODOaratus

34. A 37-year-old man has nurroness overlhelhenar emi nence. Wtich of the folowing findi ngs wiUconfinm the mostlkety tJag,,osis?
A)
.} B)
C)
., D)
., E)

>

Aoohy of the lnte<osseous museles olthe hand


Decreased sirenglll dunng e111ens,on ol the ttunb
Decreased strenglll of opposi non of the thtrnb
Loss of sensabon to the slon cove<l ng the dotsum olthe hand
Loss of sensabon to the slon olthe IUle finger

35. A 16-year.ofd boy partldpates,n a study or exercise and musde hype,1rOPhy.Allera p,etrarrang biopsy speomen ofthe Quadriceps musde IS olJCained, hebegi nsa
wei gtt-ifting J)(ogramto i naease the mass and strength ofthequaateeps irusde. Mer 3 months dtrai ni ng. a second musde bi opsy speomen 1 sla.ken forhi stologic
assessm ent ofskel etal muscle charaderi Slics Compared w1th thep,etrai11ngsample, the post.trai ning safTl)le i s most likelyto sl1oW whi ch ofthe following findi ngs?
,
C
t
0
r

A)
B)
C)
D)

Numbe, Of
Mitochondria

AmolR OIAdln
per Myocyto

To4al l'ltmbe<
of Myocytes

i
nochange
nochange

no change

no change
f
No d1ange
1
No change
no change
nochange
No change
(' G)
nochange
no change
No change
t, H)
36. A full- term mal e newborn has enlargement of head circumference {3 cm greater than 99% of age range). Bodyweighti s appropriate forgestational age. The aarial SUures
mi
are separated. Ultrasonographyotthe head shows enl argemertotthe lateralventri cles and thi nni ng of1he cerebra con
l ex. The ne\'ltlom's maternal unde had si l ar
abnormali ti es. Further anatorricstvdies are mostl ikel yto show ,,mi ch ofthe foll owi ng?

E)
(, F)

c A) Absence ol lhe torarrona orLuschka

o 8} reen,ir ,>Wom,>

c C) Holoprosencephaty
e D) Neurofib!omatosi s
E) Stenosi s orthe aqueduct olSyMus
c F) Turous sclerosi s
37. A 6-year-Old t>oy has a hstory ofCl'lonic rec<trerc ,nrectionsor the ston and respiratory rract by Stap11y1ococcus aureU$ Ulborar ory SIU<l, es sr.Ma eficiencyi n
phagocyle NADPH o,idase at1rv1ty wt.ch ollhe tollowl ng bacterlal p,oducts COffllbutes to the patlenrs 1naease<1 susoephb1 htyto S sureus,ntecti on?

Capsular potysacchande
Catalase
a-He<nolysln
Hyal uror>dase
l.JPOP<>l\'S3(Xflaride endOton
38. A 25-) ear- okt man is bfooghl to the emergency department after falli rg and di sl ocating tis shoulder. Physi cal exarrination shows markedly extensi bl e skin and hypermobUe
o rts. He i s a1 i ncreased riskfor development of which ofthe follO'Mng?
j i
A)
Acanthosis ni gricans
o
e B) Col oni c angi odysplasi a and hemorrhage
o C) Hepatolenti a.4ar degene,ation {Wilson di sease)
o 0) Mal i gnant melanoma
o E) Osteopetross
i
6 F) Progressi ve ndi focal l cukoencephal opathy
O G) Rupt\l'e ofthe colon andl arge arteries
c A)
c B)
C C)
t, DJ
o El

44. A 33- year-okl man vitK> is seropositi vefor HI V comes tothe physici an because of a l esi on on hi s forehead for 4 weeks. He has a 10- year hi story of intravenous drug abuse.
Physi cal exami nati on shows a 1-cm, irregul arly shaped, rai sed, indurated 5esion on the forehead. Aphotomicrograph of a bi opsy speci men of thel esi on is shown. Whi ch of
the followi ng is the most ikely causal organi sm ofthi s l esi on?
e A) Aspergillustumigatus
o B) Cytomegal O\/irus
O CJ Herpesvirus type 8
O 0) Mycobaderium avium-intraceflu/aro
e E) Pneumocys#sjiroveci (formerty P. carini1)
O F) Toxoplasma gondii
45. Astudy is conductedto compare two dmgs (X and Y)fortreatment of arthritis pai n. l=i fly sub;ects are selectedfOf the study: half are assigned to take drug X alld hafto take
drug Y. One month l ater, pai n reli ef is assessed. Followi ng a 1-weekwashout peri od (no drug), those who took drug X are reassigned to drug Y, andthosewho took drug Y
are reassi gned to drugX. One monlh later, pain rehefi s again assessed. Pai n reliefwi th the two drugs i s then compared across the 50 subjects. Whi ch of thefolowi ng best
describes ttis study desi gn?
O A) Gase-control
o B) Gase series
O C) C<ossover
o 0) Qoss. seCbonal
o E) Hi storical cohort
O F) Prospedi vo cohort
O G) Randomized cli rical tri al

....,..,,..,,r:

46. A 28-yea,--Old woman comes to the physi cian because olthe lesionsshoYln. M x-ray ollhe chesl shows hilar iynl)hadeoopathy. Which ofthefolowingfindings are most
lkely on IIS1ol og,c exam, nauon ol a skr n lesion and a hrlar lylrc)h node?
o A) Atypical lymphocytes and germnal cer<e<s
B) Gtaruomas witl foreii,,body giant cols
, C) Nec,..,,ng vascul ilis w,lh fibri nood necros,s
c, O> NeUIJ'aphil ici mlrate and roamytwsti ocyteS
E) Pl asma cells, giani cets,and exiensive amy1oi ddeposi ts
47 A 10-ye,u-old gut r s brought t o theemergency depam,enl 1 hour afte<she fen fromher bi cycl e Pl!yslcal exami Mllon showsa 2-cm, deep l ace<abon owr t he left torearm.
thofolOWtng plasma proce,ns most lrkety bol.rld cl oni ng laCl<lf XI (liagemanfaaor) and lor,nogen, aloYilngautocatalytrc
As a result of tho b.-tralllla, wlld>or
a"1)11fication ofthe nlammatO<y m It-is pabenl?
o A) Braclyki ni n
., Bl Fibnoogen
o C) lrMU>OglobUtln
O OJ Ka&krei n
O E) o-Macroglobulin
o Fl Plasm, n
o G) ll<ombi n
" HJ van Wll lobrand faaor

A 17-rear-old boyhos onesihesi a cllheiUle fingerend atrophy cl the lnterosseous musdes 3- eltef receMng a 'llclenl bloWto theonteoore,i le whi te ploying
looeball. Based onIlle dr agram ofIlle bradllal plexus, which ollhe IO,Ow,ngl abeledSIies 1s mostlkely10 be i.,.-ed?

CJ
D)
E)
(, A) t, B)
49. Atandomzed control tnal CA'Jnl)ares the efficacy of melhotrexate wi th that cta pl acebo for mai riai ning rerrissaon in patiertswtth Crohn<isease. The rest.tsare shown:

Nurl>ol pal Of'IS


rnswdy gr0<4)
i
Nt.mbef remai ning ,n remssi on

t.le!hotro,ate
40
26(65%)

Placebo
36
14 (39%)

ThetiOOllfounct anabSOlJle ri sk redUCbOO ol26.1%(95% confidence1111erva1:4.4% -47.8%);p.va1Jers0.04. lftrle sna,randomzecl 1000 patiettsinstead of 76.
whi ch of the folowing is the ed impact on the confidence intervat?
A) Very likelyto increase
B) Very hkety10decrease
C) Equat,,lkety to increaseordecrease
50. A67. year. CJkt man has had rapi dly progressi ve memory kiss. agnosia, and apralOa over the past6 months. He had a corneal t,ansplanl 9 months ago. He has myodoni c
j erki ng movements. Whi ch ofthe folowingi -s the mostlikely cause of thesefindi ngs?
O A) Al umi numtoxi city
0 B) Head i rjuy
o C) Sl cnvvirus(pnon)di seose
o 0) SOke
o E) Vi tami n B1: (coba1 ami n) deficiency

NBME Form 5-2

1. A 54.year.old woman has ch'or,c symmetncswelmgand tenderness o nher proxi mal orterphalalll)E!al and me1aca1pOl)hal angealpr1S. She has .-...sde adlES and S11 ffness.
especiatyoo awakeni ng.Anatysis ofserumshows arti bodi es abletobind hlf'Mnl gG. These afli bodi es are most ikelytorecogizewhich ofthefolowing pairs of labeled
domains o nthe sdlemat1c .-ogoam ol anl gG molealle?
A)
B)
0 C)
0 D)
O E)
F)

1 and2
1 andS
2and3
2and6
3and4
Sand6

2. A 17-month-o dboyhas a h S1ory of recurrent bacte< a infect ons. Neutroph ls st mu a ed with opsooized bacteria show decreased ptoduct on of 02-, H:,,0,- HOC, and OM.
i
i
i l
i
i
i l t
I
l
Whi ch of lhe followi ng enzymes is mosl likely delicienl i n ttis patient?
o A) Cataiase
c, B} Gt utat hi one pe<o:xi dase
e C} Myel operoxidase
O D) NADPH odase
() E) Superoxi de di smutase

3. Tay- Sachs di sease is an autosomal recessi ve di sorderforwhi ch the carrier frequency in individual s ofAshkenazi Jewi sh descent is abou 1 in30. On the basi s ofttis cani 1
frequency, whi ch of the fol lowing is the best esti mate of the probabi itythat a chi ld bomto two parents, both ofwhom are ofAshkenazi Jewish descent, will be affooed by
Tay-Sachs di sease?
A) 1 In 15
B) 1 ln60
( C) 1 in4SO
6 D) 1 in900
" E) 1 in 1800
F) 1 ln3800

a-

pani ciesc:on1alns a kagmeni ottheE coli


During beCl<!!lophage T7 inlect1 on ol e si ngle Escl>oficlvacollcot. 1000 beCl<!!lophage pan,cl es are released One 01
ch'omos<>me rather a,anu,e T7 ch'omos<>me W1 llllis phage be al>leto In ect itS DNA lrcoan E. COIieel. and, K so. hOlv many T7 parbdeswll be P<Oduce<I by thi s eel?
j
Allie tortect DNA? #otn Parti cleS Proouced
"'A)
Yes
1000
B)
Yl>S
100
C)
1
Yes
D)
Yes
0
e EJ
No
0
5. An 11- year- oJd l>oy has been passi ng large vol umes ofui ne and has been extremel y thi rsty r the past 10 days. He hasl ost a consi derabl e amount of wei ghtdespi te
ha\ltng a900d appeti te. Duri ng the past 2 days, he has become increasi ngly drowsy, lethargic> and di fficultto arouse. Respi rations are deep and rapi d. His blood pressure i !
100160 mm Hg vthile supine and 40 mm Hg/undetectable while si tting. I n additionto gb;ose, whichofthe followi ng i s the pri nci pal fuel oxidi zed i n hi s brai n?
o A) Alarone
o B) Gl utamate
e C) Glycogen
O D) fl.Hydroxybulyri c aci d
6 E) Pall'nitale
o F) Tnacy1g1 ycell
G A blosyrthebcen0< In readi ng lhe geneti c code gi ves nse toan oozyme lhal fol ds o n an abnormal mame< Which ollhe folowlng os mOSI ikely to happen10 ll>S enzyme?

* A)

Denal\Jlabon and refoldi ng


B) Fasier de!,adntoon
C) lnaaivation by phosphbOn
D) Transportaoon oi.c orthe ceP via lhe Golgi omplex

7. A61- 1 r- old woman has had prog,essivet,, seveie pain i nthe rightroverthe past 24 hou's. She has had type 1 di abetes melli tus for many years and has mi ld renal
failure, retinopathy, and neuropathy. She has no hi story ofdrai nage fromthe ear, upper respi ratocytract infecti on, fever, or chil ls. She is afebril e. Heari ng i s normal bil ateral ly.
The external appearance ofthe eari s no,mal , but touchi ng Of mo'li ng the pinna produces seve,e pai n. The external audi tory canali s inftamed. The tylll)ani c membranei s
cl ear, and no flui d is seen. Whi ch ofthefollowi ng is the most likelyaagnosis?
o A) Acoustic schwannoma
O 8) Acute oti tis medi a
O C) Labyrinlhilis
e O) Ma11gnan1 otltis extema
C E) Polychondri tis

8. A geneti cist has artifici al ly produc,ed a microorgani sm that contai ns a new coni:li nation or genes. tiewantsto knowi f recombi nation or transposi tion has occurred. The maj or
di fference betweenthese two processes is that recombi ooti on irwoNe-s which ofthe followi ng?
0 A) DNA repair
o B) Endoooclease sites
() C} Gene rearrangement
o D) HomOIOgous s,tes
e E) Sites on t he same chromosome

9. A 40-)ear old woman has a 3-month hisuwyofshortness of breath. chest pai n, fever, aoo nigtt sweats. Anx-raycithe chestshows t,nl)hn ode enl argemef'l. Tissue obtai ned
on a lyn'l>h node btopsyi s sOOwn. Theserirdngs are most con51slef(wrthw hich ofthefolowmg types of i rtlan'mahon?
o A) f1 bnnou
:, B) Gra1UOmatoos

C) Neo:Oli zing
0 D) Serous
o E) SuPl)crative

10. A 74-year-ddwomani s broughttothe physdan byfamily members who areconcemed because she has been losang weiglt She has a historyof chroni c an:oety and
dysthymicc:isorder. Anx-1ayci the cheSI showsa large cEf1raJmass. Unreseaablel ung canceris di agnosed. Thewomani s capabl e of informed decisi on-making. but her
famil y askslhat she not beloldthe di agnosasbecause theyleelthe newswil h8S1en her dealh.Whi ch ofthe folowmg is t he most appropri ate response?
A) Tell t helam ti I s uncomonable con1Jntif'9as he< phys,oanlnderlhe,eorcunstances and,wggest alemawe phys,clens!hey ml ghl c..,...
B) Tell l hepatientaboU the x..-ay fi ncll ngs, reCer her to an c:>na*>gi S1. and canst.I: wi th a ri skmanagement attorney
C} Tell thepatientthal heftes1results are available for ciscussion ifshewishes
O} Tell thepatientthat she sholJd be referredtoan oncologistlora second opirionand i nfocmthe oncologi staboutthe di lerrrna
E) Tell tilepabent that tilex.ray 1s abnormal and 1het silo sholldsee speoallsl
1 1. Whch ol lhe followi ng laboratory resisIs mos,use/ul I n d, llerentlabng Streptococcuspneuffl()lll"9from olhe< a,hem<:llyli c streptococci ?

Sacitracin (A <isk) susceptibilty


Catalase teS1
CoagtAase test
lactose lermentabon
Optodln (P <llsk) suscep1>bi lty
Oxidase produclion
urease produai on
12. When adipocytetri glycerides are mobili zed,. the nodatty aci d domain of thi s mofecule can be used by thel iverforgltconeogenesis. Whi ch of the folowing enzymes initiates
tllis p<0cess?
e A) 1, 3-Digcerate ki nase
e B) Fructose 1, 6-bi sphOSl)hatase
e C) Gl uookinase
e D) G1 yce<aldehy<1e, 3, phosphat e dehy<1rogenase
ki nase
e E) Glycerol
t) F'} Hoxoki nasc
13 01y heatat lOO'C l oreo .,...-..es Is used toste<1 lte lnslnlnel15 at a smel tcorrrnunlty 01.Cpatle<t lacl lty. Usirqtllisteehnl que, ""Ch otthe1-ng 1s most likelyto survive
as a viabtecortami nan1 dlhesteri hzed 1 nstn.mens?
A) Baal/us sublilis
B) Eschendll8cot,
C) Psou<lomonllsBMJgt/lOS8
D) Slaphyloooccus aureus
E) Slreptoooccuspyogenes(group AJ
A)
B)
fl C)
0)
E)
F)
G)

14 Whch ollhe followi ng SinJCIU'es aremosl lkely tol>ld wi ththe sll\JClll'eS labeled 1" on the pholomiaograph?
A) MW!soon pl aques
8) Gap j<nt1ioos
C) Hemi desmosomcs
D) Myosonfi-S
, E) To Joocuons (zoruae ocelldertes)
15 A 55-year-dd N&ove Amencenwomanwho l'Ves alone rsb<oug111 lo Illeemergency depar1mert after a ,.;no, fall o ntoe,home. whiCh toe,molher wllnessed She tels her
physi ci anthatoorhusband, whotied 6weeks ago, comestovisrt her fNef'/,..g1 and gives herg.adance. She alsostates that he cns otbei ngl onetf andiswai ti ng
for he, to,Oinhm.She states that herappeti te and energyare normal anddenies sui ci dal ideation. Physi cal ex.arrinatiooshows a bn.isedarm andl eg. Neuologi c
exami nationandCT scan ofthe heads:howno abnonnaflies. Whi chof theng is tOO mostappropriate nextsrepforthe physician?
A) Ask her retawes dspeokl ngtodeceased1 0\/lld ones os common to lheoraaure
8) Ask her rela"""5 o f there is a farnlyhistory of ma,or<1e1><ess1 ve d1 sordef wolhpsychotic symplOmS
o C) Ask hef rela11eS 1 flhe1eis o farnlyhslory of Sthzop<e1u
A D) Order EEGto det ermi ne the paen1 Is sie
E) Order anMRI cl1loe head

an

,r

16. A48-year-old man has a painless enlarged alliftary lyrrc)h node Ewsi onalbiOl)syshoWsrepl acemenl olnodal arChi ledure by neoplastic foades composed ofsmal
cleaved !Jrrc>hocyles. C)logeoellc stucies show a 1(14;1 8). Allerallon ofwtoch of the follow,ng processesmostcoosi stenlly l>lde1 esthe palhogenesas clIlls les,on?
A)
O 8)
C)
D)
E)

DNA repair
Growth factor proo.Jdion
Growthlaclor receptor oeglAotion
Nucl eartranscnf)ll on factor eJ<l)(ess,on
ReglAibon ol apopcosis

17. A 69- year-ol d womanis brought tothe emergencydepartment 1 hour after the SOOden onset of crushi ng, &tlsternal chest pai n and nausea. Physi cal exami nation shows
pallor and di aphoresis. AA ECG and an eval uati on of serum cardi acenzyme acti vity confirm a di agnosi s ofa myocardi al i nfarction of the anteri or wal. Activation ofp ate ets
l l
and segmented ne\Arophi duri nglhrombogenesi s inthi s patient caused lhe rel ease ofcy\.oki nes, includi ng platelel-deri ved growthfactor. The milogeni c signal proviljed by
this cytoki ne to smooth musde eels at si tes ofvascular injury in this pati ent ismos.1 likelyto be inhi bi ted bywhi ch of the following intracellul ar enzymes?
A} Aminotransferase
e 8) Gl ycosylhydrofase
e C) Ki nase
c, D) Lipase
E) Peroxi dase
e F) PhO'"
.;phatase

ll

;r;
I

18. A 23-ycar-old woman (rn<lvodual 11. 1 111the podol)"oo) seeks gc""11c CO\l'lSdlngbecause her b<other has Duchetm muscuar dyslroplly (DMD Her brother (1.2) has a
del ebon on lhe DMD gene Alhough her mothe< (1.2) does nol hall<l the detebon, 1nc1Mdua11.1 ,s tol d lhal hel oSk IOI' car,y,ng Ille DMD gene is greater l han that of the
general populanon. Whch cllhl> follow,ng best exi,laons ttos increased ri sk?
A) De novogene nua.tion
o 8) Locus helerogene,ty for DMD
C) Maternal germlne mosaocl sm
0) Promotermlf:ati on
El Suspcaed parental consanguini ty
F) X inactivati on
1 9 . A 55-ycar-ddwomanIs tree.led with 500 mg cl drug X U.-ee hmeS daily for cancer pain. She ,s pain.free. b<.C shede-elOl)s nausea. vom, ng. and dlarmea, requi ri ng
cessation of drugX. Drug Y is staned at20 mgtwi ce daily. She is pai l'Mree, her appeMe i"l)roves, and she gai nsweight Whi ch ofthefolowi bestdescribes the
maiomal efficacy Ollhese drugs?
A) Drug X, s more ellcaoous lhan cloug Y
"' 8) Drug V Is more efficaoous thandrug X
C) Drugs X andY areequallyefficacious

20. T1 eatrn<!f'II ot human l)ITC>l,ood tissue W11ll en e"l)e1i mer1&I drlJg that ceusesO"'lrelll)<essoon or the bd2 prote,n I s moslkkely to ,e,s,a ,n a dec1ease 1 nwhich ottheIOl!owlng
i nB es?
A) Apop1osis
8) FAS protein
C) kilraceu.1 adhesi on molecul e- I
0) NAOPH ox1 <1ase
E) U1i caod
21. Sequence.Sl)eci fic deavageor protei ns plays e tole i n which of the rolowing processes?
"' A) AcWatJon OIG protei ns by hormone 1eceptors
B) Assa,ni, of rr.ctotut>ul es
C) Celuliar 14)(llke of iron
0) Initiat on olcoagulation
-. E} Transport of protems into the nucleus
22 A 1.mora,.Ol(I boywith hydrocepha has suspected T
oxop/asma gondinnrecti on. wtidh ofthefollOWUlg routes of i rlect1oni s most l ikely in this pati ent?
A) Allttopodbi le
B) Exposure dunng vaginaldeliYo,y
o C) lnges!JOn ofcows ,nlk
CJ 0) lngeSttOn ofmolhe(S mi lk
E) Postnatal1ra11Smission froman infectedfami lymeml>er
F) PostnataltrallSmission froma fami ly pet
G) Transplacental transmi ssaon
23. Which ofIlleIOll<Y,Yl ng molecules rrusl, nterect to Induce 111"'1l.lle cell aCIMlbon by the 514>eranbgentoJOc shodi synd1ometOJOn.t?
A)
B)

C)

0)
E)

F)

Molerue 1
ClasslMHC
ClasslMHC
ClasslMHC
Cl asslMHC
Cl asslMHC
Cl asslMHC

Mol ecul e2
CO!ll)l ement receptor I
lgM, lgO
T- cell 1eceptoc
CO!ll)l ement recepior 1
lgM, lgO
T- cell 1eceptoc

24 The spl iong ofpnmary genet1anscripts i n euka,yotic cells is most l ikely to havewlwch ofthefollowing effects?
- A) Deaeased Slabi itf ofmRNAs
B) GenerabonofhybOd (ch, menc) mRNAs
C} W:reased repertorre olgene products
o D) ureased si ze ofmRNAs
E) Rearrangement of celul ru genome
25. Alerabon ofwhldh ofthe fol lowing inl uenza viius proteins 1 s mostlkelyto 1esult m alered reactivityto nelAralz,nganbbodies?
A) Hemaggl l.Clni n
v B) Mamx p1otel n
C) NOOS11UC11nl protei n
D) Nucl (l<Jl)rotcin
... E) Pol)merase

26. A 54- year-ol d man with al coholsm is broughttothe emergencydepartment becauseofa 10-0ay hi story dweakness, confusi on, aOO shortness of breath. He drin
approximalet, three fourths of a iter ofli quordai ly. He appears wasted, and t here i s an odOC' of al cohol onhis breath. Hi s respi rations are 32/mi n. Physical exami 11ati on
showsjaundi ce, abdomi nal di stention, spi derang omata, and Tanner stage 2 gynecomasti a. LabOJatory stucies show hypoalbumi nemi aand an increasedserum estrogen.
to- androgen rati o. The devel opmen! otgynecomastia in thi s patient is medi ated pri mari ly by whi ch ofthe bllowi ng signaJtransduction pathways?
O A) Activati on otphospholi paseA:and generati on ofarachi doni c aci d
c 8) Ac11 vati on otphospho pase C and generation of1nos 1 0 1.4, 5.tri sph0sphate
o C) G prorein-coupled actilivationofadenylyt cydase i 1
6 D) Receptor di meri zation and autophosphorytati on oftyrosi ne resi dues
(') E) Transmembrane di ffusion and biOOi ng of si gnal ng mol ecul e to transcription factor

27. A 6.fl"IOl1h-ol d bo,tsbrought to the emergency dep811mefd.because ofpal lor. He has been bfeast.fed excl usivelysi nce birth by hi s mother, who is avegan andfollows a

stri awgetari anciet that exCDX'S all meat. fish,and dairy products. Physi cal exami nationshows tachycardia and pallor; he appears mamurished. Laboratorystudi es
showanemia. Plasma studi es showan, naeased concenlrabondhomocystJne and a low-normal melhioni ne concentration. Urine orgaric acid screeni ng shows an
increasied concenrabon of melhyrnak>tlc aad. Treannen1 with whleh of the folowing is mosll ikely 10 eliminatethe methyl matonic aci<kna ,n ttlis paue,nt?

A) Cysa,ne

B) Foicaci d
o C) Methoni ne
- D) v, 1a,,..n B12 (cyanocobalall'ln)
E) Vitemin C {OSCOl1>i c 8Cld)
28. A 17-year-ol d girli s eval uated because ofa .,boil " on hes left sh<>l.tief. E.xarrination shows an erythematous, pairltA. ftJduant lesion; a yelow-green liQlid material is
C11a1ned"o,cal ly The pai n andtl)'lhQma g,adually S<.C>sule Whldl of the fOIIOWlng cellsconinbutes mosttolht Dssut changes p1esent"'dlQl esi on Sdays aftt< t11i nage'
A) Eosmophi ls
B) Lyffl)l1ocytes
C) MacrOPhages

D) NeUrophl lS
E) Plasma cells

29. A 12. year.ol d man oomes to the physi ci an because ofa 10A,eek hi story ofweakness and shortness of breath wi th exerti on. Physical examination shows marked pallo, and
a l ack oferythema ofthe nai l beds. Laboratorystudi es show:
He-mogklbi n
Leu<ocyte count
Reticul ocyte count
Ef}'lhrocyte se<limentati on rate

5.2 g/dl

54001mm'wi th a normal di ffor,ential


0%
IOnmll

Uri nalysi s shows no abnormal iti es. A peri pheral bl ood smeari s shown. Whi ch ofthefollowi ngl aboratorystudi es is mostl ikelyto showthe earli es.1response to treatment in
ttis patient?
o A)
e B)
O C)
6 D)
o E)

El)'throcy1e sed,mentat, on rat e


Hemogl obi n concentrati on
Leukocyte count
Reticul ocyte courrt
Urinalysi s

30. The 45S !RNApreansori s cl eaved i nto 185, 5.85, and 285 RNAs inwhi ch of Ole folowing?

,,. A)
B)
C)
.., D)
E)

Enclopfasrroc relJQJUTI
EncloS<lml!
Golgi complex
Lysosome
NucJeol.os

31. A 30-year-<>ld man has a 3-day llstory of diarrheaWith 6-3 watery stool s per day. Tests of arteri al blOOd are most hkel)' to snowwhi chor the folOwi ng sets ofvalues?

Pco,
6 A)

B)
e C)

D)
E)

pH

7.30
7 30

740

750
7.50

HCO,-

(rron Hg} (mE)

28
55

40
30
47

15

27
24

22
33

32. Duri 11;1 DNA repli cati on i n eukaryoti c eels, whichof thefolowing occurswhen topoisomeraseI cleaves a phosphociester bond inone DNA strand?
A) Free rotation ollhe IWO sect,ons of the DNAhelx on erlher si de ofthe nick
B) A reoombl natl on ew,nt be1teen adj acent -heli ces
C) Removal and su,sequ(lf'I!transfer of a small piece of ONAto another positi on on 01& dYomoS<lml!
D) Removal olll'ismatched nucleotidesUlatwJd result indeleterious nuations
33. A 32- year.ol d woman comes fora routi ne health maintenance exami nation. She has had intermittent consti pation and flat\Aence. She says that there has been i ocreasi ng
tension with her husband, and she is uncertai n about the\liabili ty of hermarriage. Exami nation shows an ecc:hymosis oothe right upper arm. Pelvic exami nati on shows
vagi nal abrasi ons and i s di fficultto pefform because ofthe patient' s di scomfoft. She descri bes her mood as occasionatty sad wi th no persi stent depressed mood. She i s
moderatelyaroci ous and has a dysphoric affect. She has no suici dal ot horricida1 i deati on orplan. Whi ch of thefc:6:lwing is theappropri ate next step in management?
O A) Sched!Ang a returnvisit in 2 weeks
O 8) Wonring the patient abou domesti c violence shelte rs
O C) Psychologi cal testing
o 0) lndMduatpsydlOlherapy
O E) Mari tal therapy
e F') Tricycli c antidepressant therapy
34. A 66- year.ol d man who i s undergoing chemotherapy for letdtemi a has a 3-day hi story of progressive feve.i, headache, nausea, and deteri oration of mental acui ty. Analysi s
of cetebrospi naJfkJid showstymphocyti c pleocytosi s, a nonnal gl,cose concentrati on, and a sl igttl y increased protei n concentrati on. The causal agent: is p(esumptively
i dentified by a positive fndi a ink test andl ater confirmed wi th culture. Whi ch of the folowi ng is the major\lirul ence factor ofthe irlecting organi sm producing thi s di sease?
() A) Antigericvari ation
o 8) Exotoxi n
o C) rnmunogl ollt.6n protease
6 D) Leukoci dn
6 E) Polysacchaii de capscle

35. A 70-year-dd man is brought to theemergerqdepartment because offever, cough pr<><lldive ofblood-ti nged Sj)U\ITl, ard shortnessof breath for 12 hours. His
IQmp&ronn , s 30.S-C (103.1'F), and resp,rabOns are 36,mn Cractcs are h<lard a, t he ngh1 k.ng bate Cunure ofsputumi,ows a-hemolyti c, gram-pos.i, va dopl ococc,
WIich ofthe lollo1Yi ng thoractenSlocs ofthe causal o<ga"51Tl best e,optams the clevelopment of pne...,_a in thispabenl?
A) Exp1eSS1on of a polysaccharide capsule
Bl 1n1raca1uar su"'1val in macrophages
CJ Produt11on of endolOxon
6 Dl Production of a pote<tADP-ribosytating exotoxin
E) Production of a --.nligen
36. The erythrocyte is pronelo 1 1'py byreduced oxygenmetaboli tes llat are produced afterspcqaneoos oJOdation of oxyhem(90bi n. Vt'hi ch dthe folo'11i"ng reacbons or
pcocesses is most ai 1i caJi n pre1ec1i ng the erytt1rocy1e from injlry by 1hese oxida1i on pcodUCIS?
A) O.ssociatJon d methemogl obm i ncometheme and gl obln chains
"' Bl Gllcose6-phosphate dehydrogenase-catafyzed generation ofNADPH
Cl lneai vaon of hydloxyth 1&dicals by jl.alrotene
Dl Shesisof ublQul none
El Vitami n C-Oependen1 reduc1ion ofmethemoglobin to hemoglobin
37. Which ofthe fofl t>wtng ceaJarj l.11Ct1 ons blockthe bacldlow ofglucose horn the basal si de ofthe ontesbnal ei,ithelum into lhe unen of the gut?
A) M>enng,cbons
BJ Desmosomes (maQJfaeadhere11es)
C) Gap jcndi ons
OJ Hernl desmosomes
El Tig,t jl.llcti ons (.zoruae otaldentesl
. A 19,year-ol d woman comes to lhe emergency department because She Isafrai d thatshe w\11 kill herself, She has had recunent epi sodes olsel-rroti tatlon. mood
ins1abi 1i ty, and poor impul se control overthe past5 years. Transi enl relief from these symptoms has been associated wi th muh1 ple lreatments, i rrl.1ch ng anti dep<essants,
anxiotybcs, neurol eptics, and group the1apy. She reports brief. intense romantic rel ationshi ps wi th men and women. These lindi l'lQS rue most cof'ISlstentwi th whi ch ofthe
followi ng?
,-, A) Acu te ci'ug reaction
f" B) Ad(us.tment di sorderwi th mixed di stllbance otemotions and conduct
Cl Atcohol wl thdrowal
Dl Bi polar dl SO<der
E) Borderli ne personalr tydisorder
6 Fl Dyslhymic di soode<
6 G) Major depressive di sorder wi th psychotic features
o Hl Poswoumabc svess di SO<der
l l SchlZOl)llr8"1 a
Jl Scparobon anxi ety dosorde,
39. A5-day-old femal e newbomwflo is breast.fed is brought tothe emergencydepartment because ofa 1- day history ofpoorfeeding,vomi ting, and letha1gy. Physi cal
exami nati on shows decreased responsi veness, hypotoni a, and dehydrati on. There i s a caramel- li ke odor inthe di apet. Serum studies show a decreased gkJcose
concentrati on, ketooes, and metabol ic aci dosi s. Metabotic studies of1 eukocy1es show no corwersi on :>frq l eucine to 1co2. An increased pl asma concentrati on ofwhi ch
of the followi ng substances i s mostl ikely i n this patient?
o A) Atarone
o Bl lsolralericadd
e C} Ml?thyl mal onic acid
6 Dl Phenyl al anine
e E) Proo1oni c aci d

o f) Vatine

40. A:JO.year-ol d woman has tefVical Pap smears showi ng cyfopathi c changesi ndicative of tunanpapillomovi,us type 16 irlecbon. wtich ofthe foll isthe most lkely
1ong,,1enn conseque,,ce?
A) Endornetri ol carcinoma
B) Papolary adenocaranoma
() C) Serous carcinoma
D) Srna1ce11 caronoma
EJ Squamous cell carcinoma
41. A54- year-ol d man with squamous eel carci noma orthel ung comes to hi s physi cian' s office because or constipation and gastroi rtestinal discomfort. He appears confused.
wtich ofthe followi ng se<umval ues is mostl ikely to be increased?
6 A) Cal ci um
6 Bl GIUC<lse
6 C) Magnesium
o Dl Ni tric ode
o El Potass
i Lm
6 Fl Sodi um
(') G} Uri c aci d
42. A25-year.old pnrngrevtd woman at 3morh' gestationcomesto the phys,aanfOJ herfirst prenatal visi t She requests an amrn s "to make me feel betterthat
everything is all righl -.,ththebaby. "The<e is no fami ly historyofgeneti c di sorders. Physi caJ examnation and laboratory studi es showno abnonnaltles. Thel ikelihood of
fi an abno,mailyistow. The physioan needs to balance whi ch of the following ethi cal principleswhen deci<ling,methe, to granttlis patient's requesl?
A) Al.tonomy versusmalefi cence
Bl Al.tonomy versus iaMananl sm
C) Beneficence wrsusdeonol ogy
6 0) Beneficence versusmaleficence
(') E) Oeontol ogyversus\Mtariani sm

43. I n a popul ation, thefiequency of the HlA-A loo.ls allele (A'01) is 0.20 andthef1equency of the HlA-B l ocus allel e (8'08) is0 .20. fn the same popul ation, the fiequency of the
hapkltype A"01, sos i s 0.16. wtich ofthe followi ngterms best expl ains the popul ation cha,acteristics ofthe observed frequency oftheA"01. B..
08 hapl otype?
e A) Linkage di sequi libri um
o B) Random assomnent
o C) Somati c mutation
o 0) vanable e,p1essivl1Y
44 Prostacycin (PGl 2)syrlhOSIZe<Iby the endotheial eels promotes wtich olthe foll OMng?
-N
B)
C)
"' 0)
E)

Aehlsion of pl atel ets


Express1 00 of seledlns
Primi ng ofleukocytes
Production of nitricoxi de
Vasod1 latloo

45. A 2-year-ol:I boy devlllopoo thevesiaJlarl esionsshown 2 dlays ago. Which ofthe followi ng besldescnbesthe viruscausingthe lesi Offl?

A)

BJ

C)
0)

E)

Nucl eicAo<J
Orulle-stranded RNA
00<.tlle-strancle<J RNA
Oolble-stran<Jed RNA
Single- suan<Jed RNA
S1ngle-S1ran<Jed RNA

Nudoar C4psid
Symmec,y
ocosahe<tal
helical
IGOSOheaal
icosahe<tal
tCOSahe<Yal

EnYel ope
Preserc
ye<;

no

no

46. AJ .year. okJ chi k1 wi th mental reta1dalion is seen because of seve,e behavior p1oblems in school and self-di rected aggressi ve behavior. The patien t is diagnosed wi th a
complete deficiencyofhypoxanthi ne- guani ne phosphoribosyttransferase. Thi s defect mostlikel y results in the aCO.J'Oul atioo ofwhi ch of lhe follo'lling metaboli tes i n serum?
A)
Adeni ne
e
B) Cytosi no
C) Guani ne
0) Urea
C) E} Uri cacid
47. A 1.ye,,r.o1c1 boy ,s brougtuothe Ph)'SIOan by hi s rnoa,e, t or a -.chi ld eJ1J1rronation. She says he ha5no ,nte,est, n tOlleCVal ni ng He I s al lhe 30th per tor he,gnt
and wei ght Hehas avocabda.ry of abolt 10 words aoo recenttybegan wal ku-g on tis own. His motherconsuned akohol "severalt1rnes" dunng herfirsttnmesaerof
p<egnency. WIich ofthefollowing beSIdescnbes thedlevel opmeq olthi s boy?
MOW
Language
A) De aye<J
detaye<J
l
B) Do a
normal
l
C)
delaye<J
Normal
0)
Nom,al
nonnal
43. A 40-year-ol:I man has had p1ogessiYe sllOl1ness ofbreathfor 3years. He has smoked 2 packs ol agai ettesdai ly foi20years. Examnabon ofthe broncllal ITTJCosa
obtai ned on biopsy sto.vs cil iated. pseudosarati fied ul arepi1hetium withi merspersedfoci ofnormal ly mah.ling, stratified squamous epitheltm overtying an intaa
basemeftmembrane. Which ofthe fol lo>.,,ngprocesses is the moSIikefy explanation forthese findings?
N Gl an<JUlor <Jyspl&Sia
B) Gl an<Jular hype<plasoa
C) Gl andularmetaplasia
0) S<iuamous <Jysptasia
El Squamous hyperptesia
FJ Squamous metaplas,a

NBME Form 5-3


1. A 13-year-oldgill i s brought to the eme<gency departmentby herlather ar1er he roond her in a semi comatosestate wi th an ety botUe or anti hi stami ne lleside her bed.
Pl asma di phenhydrami neconcentratloos measuredhOurl)' tor 6 hours are Sllown,
Time Plasma Drug Concentrati on (ng/ml)
240
1: 00 PM
2: 00PM
202
170
3: 00PM
143
4:00 PM
5: 00 PM
120
6: 00PM
101

i
Approx mately howl ong after thefirst measurement (i n hours) will i ttakefor this pati ent's pl asma concenrrati on to drop below 10 ng.fml?

O A)
0 B)
O C)
0 D)
o El
O F)
0 G)

10
12
16
20
24
30
38

2. A62-rear-old man comes to Ille physlaon With a 3-month hlslory oflabgue anddel)lessron. He Ives alone and has been dnnkmg m(lfe (up to 6 mixed alCOOOlc dnrl<sdruly)
since tiswife of40 years died 6 maf'lhsago. He says, "Yeah. I know rve been driricinga loll ately. but I'm so down inthe dumpsthere' s nothi.ng else Iwan1 to do. I miss her
so rooch.' ' Wlldl ofIlle lollowrng r"-"ses by the pt,ysi cian 1s moslapl)<ol)lial e?
(. A) "An)1)0dy In'(WI arccmslance WOIAcl be depromed. T,me wll hell) yoo overcome Ille IOss ot your w,te."
., 8) "Dnnkl ng isnlgo.ng 10 bOng yourwi let>ack You need toget on ..lll ycu life and put !his sad expenence ll<!hndyou.
C) " I knowyoo hall<! been ttrough a lot recettty. Hawthi ngsbeoo so badrecentlylllat you havellloughlabout kilng yourself?"
D) " I'dke tosa yoo on some medicati ontor del)<essl on kSO!nO<lmes lakes a lelv Wfft<s t o-1<.''
o E) ''You seem to bedevelopong analcoholp<0blem Youneed toqurtdri rlang. I canrele, you toIlle local chapter orAlcoool ics Anonymous.' '
3. A 52year-0l d man is admi tted toO'le hospital because of an aa.te myocartial infarcti on.. Corooary catheterization shows a lesi on i n th eleft arteri ordescending coronary
&nOfY. AAar a ballOooangi oplaS1y,s pQfformocl, atx,x,mab ,s a<tnin, siered Blockade of wtodlol1118 fonow,ng is moo,hkQly 10 occur ininis pauont?
,. A) y.Cart>ox)1ation ofcl otting lactOfS
B) Factor X, (octivaioo Stuart fact0<) tormabon
C) Flbrinogento fibrin COl'HersiOO
D) Pl atelel glya)prOk!ln llbdla receptorfunction
t., E) Pl ateletthr001boxaneA, prodJCti on
4. A52- year -oldmanwith akohoism comesto thephysician for a kllol,v.up exarrination. He hashypertensi on, type2 diabetesmeltus, osteoarhllis, and ctysfipidemia.
anentmedications inclJde meifonni n, gtyburide, metoprolo andsildenafil Tiree months ago.he began to folowa di et recommeoded by theAmerican HeartAssoci ati on
because ofan inaeased senm LDL-chol esterolconte008tion. He has beena<llererc wi th thisnewregimen. Hisbloodpresstl'eis 135/85 mnHg. Physicalexamination
shows spi der angiomata onthetam. Fastingstudies showa senm LDL- chaleslerotconcentration of 148 mg(c1.Thephysiciani s considering prescri bing prayasrati n. This
<tug shoul dbe used with calAionbecause orad,erse effects associatedwi1hwlid> or the following conditions?
o Al Alcoholism
e 8) Erecti le <lySlurlcllon
( C) Hypenension
OD) Osteoarthritis
o E) Type 2 (jabetes meli tus
5. Shor1Jy after a surgical procedure. a 48-yea,.ofd woman devel OP5 "gnsand S)'ll'C)toms of neurorruscurar rntabr hty. rnctud,ng bngling pa,e5lheS1as. Gentlytapp,ng <M!Ithe
faci al nerve infrort ofthe ear results inreflex cottracti on oflhefacial musdes. Whi chofthe followmg el ecrrotyte abnonna1i ties1 s the mostl ikely cause ofthese findings?
I' A) Hypercal cenia
t 8) Hype,lcalcmra
t, Cl Hypemat, em,a
e D) Hypetph0Sphaiemoa
f' E) HiPOC3lcemia
F) Hypolcalcml a
t, G) Hyponabemi a
e H) HYIJl)llllOsphatem,a
6. A3Q.year.old woman comestothe physi cianbecause oflesions on herleftarm ror 3weeks. The lesions first appeared at the siteof a rosethorn prick.. Physicalexamination
shows aneaoticlJcer on theleft hand, withstt>cutaneousnodul es along the lymphaticsystem oftheleft t1er extrenity. A cu(ureda nodlle gows Sporothri:x schenckii.
mcona.zole therapy is i nibated Whim ofttie followi ng besldeScnbes thel'02Char'ism daction of i tracana.zol e in tris pabent?
c. A) Cel mentnnesterol syr<llesis reduced byinhibi tion ofC14 lanoste<oldemethytase
O B) Cel waUgkJcan synthesis reducedby irhbitionol jl-1,3-glucan syrtthase
O C) Cel waU irtegrity tofll)<orrised bypo.efocmation in Illecell wal
o Dl ONApol)mefase-v no,wroeblrie irhlllt01
e E) DNA 1opoisomerase II and rv cortl)etttive inhi bi tors
c, F) Ri bosomal23S subuni t binding si te intibitor
O G) RNA polymerase cofll)Etitive inhi bi tor
e H) Tubuli npolymerization i rhbito,

IAo!ion olanu1htsi t{h)


-Thi-

--,

7. Pati en1s are mairui ned at the samedepth ofanesthesia byinfusi on ofeitheittiopental or propotolThe graph showsthe recoyery time from anesthesia as afuocfion dthe
du'ation or aneslllesi a Whi ch ollhe,_,,mg dlaractensltcs ollh1 0l)enlal, btCnotof p1opolol, besle,q,lamsthe differeocebetweent hese two,nogs?
A)
8)
C)
<} D)
() E)

01Slri l>Wonollhlopental 1 nto adipcsetissue


F1 rn1,orde<hopabchmi no11ono4P<opolol
Saturable renal clearance oflhoperUl
Tolerance olP<Ol)ofol i nlhe cer*81 nerv<1us system
Zero.order metabolic el iminallon of thlapental

8. Whi ch ofthe following agents inhi bi ts both the producti on of ierl euki n.2 (ll- 2) i n resting T lynllhocytes andthe prodU;tion of y-intefferonin activated T lymphocytes?
() A)
e 8)
G)
() 0)

Azalhi Ol)rine
Cl spl alln
Cvdophosphami de
Cydospori ne

'E.) Mercaptopurine

.. l de?
9. Which ol111efolowlng labeled C0f1"C)()ne<1(S I n 1he <iagram represents the site of edlon olr osein

,[

CA)
80

:c

75
10

65

60

.'............ .
'\,

=\?

-. 130

:c

s 1 20
E

.;
t

110

...:;. 100
0

_{\_

90

80
10

+
X

C B) C C)

., 0)

-:, E)

e Fl

10. The graphs show the changes i n heart rate and mean bl ood pressure produced by drug X in a person under control conditions (sol id curve) and after bejng pretreated wi th
drug Y (dashed curve). The arrowshows VtOOO drug Xis admi ni Slered. Drugs X and Yare admi ni ered intraveoousty. Drugs X and Y are mostl ike,ty to be v.tii ch ofthe
fol lowing?
DrugX
OrugY
O A)
propranolol
Dopami ne
f') BJ
phentolami ne
Epi ne-ph'ine
e C) Notep nepru ne
prazosin
i
i
0 D)
al.Topi ne
PhenY'eph1i ne
f') EJ
metoprol ol
Tcrbutal ine
11. Which ofthefolowing isthe mos1 ikely explanation fo, the emerQence or HIV,,marts resistant to anlMral drugs?

e A) C04 Is expressed on many eels

8) HI V can bi nd manyco. recep1o,s


CJ HI V re..-,rse tmnsaipiase i s errorprone
., 0) Nef iscapable oldccreasi 'l! cel surface expressioo ofmany proteins
E) Rev effiaenlly exports viral mRNAs from the nucl eus
12 A 34-year-old Native Ame1i can woman recerGy di agnosedWith acute ledterraa tels lheph)'SIClanthat she waris to go to a traditional heaJer fortreatment instead of
lllderg01119 chemOlherapy. Which olthefollow,ng responses by tl\Q pr,ysician rs most apptopnote?
t A)
t, B)
C)
t. D)
f E)

, recommendthat you goto a tradi tional healerthat I haveworked with i nthe pas1 so that I win befarriliac withthetherapy you wil be receiving:
."
, vol referyou to a lherap,st so you can WO<!<ll'ls out. Please don't be ashamedabou1hs. Many people seek psymatri c hel p
,rn SOfl'/, b\AI can nol onger treat you because I do not agree with yourdeosi on 8" you have to do what you feel i s best
,n addi tion to nga trad, ti ooal healer, I would like youto consi der unde<gol ng chemothe<apy
"t.e4's set up a folow-<e> appoi rene11 and we can tal k abO<i thi s then."

13. The treatment of cance< with llloro,.-.al ,s effecti ve because ii preventswhch of the foll owing?
A)
f') 8)
' C)
, 0)
f') E)

Atndali on ol phospl1onbosylpyrOl)hospllale
Convers,onof nl>OroJCleOti desto deoxyr1 bonuc:l eoti des
0
l ncorparation ol a methyl group inlo N', N' tetraofolate
Methyl grouptmnsferto dUMPto tom, dTl.lP
Reduction of dihydrofol ate to tetrahydrofol ate

14. A 23-year- old man i s broughtto the emergency departmenl by fri ends after he had a generatized toni c- cloni c sei zure at a party and became tneonscious. His friends state
that he inhal ed a l arge amountof cocai ne 15 minutes ago. The pati ent is unresponsive and conti nues to cotMJtse violently. Lorazepam is given. Admini strati on ofl o<azepam
is mostl ikel y to have whi ch of thefollowing el edrophysiol ogic effects onthe neuron?
0 A) Decreased i nflux of ca:
0 B) Decreased i nflux of Na
0 C) I ncreased infk.oc of c1
0 D) I ncreased infk.oc of K
e E) I ncreased infllx ofMg'

a,,.,

15 A23-yea,.old men With sl el<le eel disease ls adrnttedtothe hospi tal bec:ause of acutevaso-occt usiv,,
and acute siddechest syndro,ne oxygen and navenous
fudsare started, ancfa bl ood transNS10n and an analgesic are aSlered. Fr ve days later, l eukocyte count 1s 6500/nm'and a pl atelet count 1s 120,000/rm,'. and he 1 s
dsdlarged wi tha prescriptionfor a to decreasethefrequency exacue ai ses. Sixweeks later, leu>cyte count i s 1800/mm and platelet count i s 60, 000krm. Which
olthefOIOwingdrugs rs mostli kelyresponsi ble fortheoeaeased let.*OC)1e and platelet COOOISi n ll'ls panert?
A) E,ytlvOl)Oi Qtrn
e B) Foic acid
e C) Hyo'oxyaea
e D) lr(ef1eron- o
E) 11rl d1 n11

I
16. A healthy 1- year-ol d chi ld i s i mmuni zed with an anti gen; the graph showsthe anti body responses measured duri ngthe next 2 months. Whi ch ofthefollowi ng types of antigen
was mos1l i kety gi ven?
e A) Bacterial polysaccharide
(') B) li ve attenualed measles virus vaccine
c C) Pol iovi rusvacci ne
e D) Tetanus toxoid
e EJ Unconjugated hapten

17. A previouslyhealthy 21- year- old man i s broughtto the emergency departmenl by fri ends 30 minutes after the sudden onsetof right-si ded chest pai n and di fficulty breathing
whil e pl aying basketball. He appears in acute distress. He is 183 cm (6fl}tall and weighs 65 kg (143l b); BMI is 19 kglm:?. Hi s temperature i s 3rC (98.6F), pul sei s
110/rrin, respi rations are32/mi n, anc:1 blood presStl'e is 120/70rrm Hg.Putmooary exami nati on shows hyperresonantsoundsaverthe rig.hi latera portion ofthe chest.
l
Breath sounds are decreased.A mi ld pleu-al rubi s heard. Achest x.ray shows a sharp ine runni ng parall el to the chest wall adjacent to a radi ol ucent area withOU. ung
l
marki ngs. Whi ch ofthe fol lowing is the mOSI appropriate next step in management?
o A) Spi ral CT scan of1he dle<st
0 B) MRI oflhe chest
O C) Bronchoscopy
o 0) 1 ntubationand meehani ca1ven111auon
o E) Need!e aspiration
18. Doong U1e f0\l1h throughe,gtlh weetcs ofgeslabon, thefe are nve pal r.s of aOllie arches. These w, 11 ..,matejy de-el OP ontocomponentsof the great a.rtenes. wtich ollhe
folovl\ng pai rs of aortic arches develops i l'1o 100 common caroti daJteries?
c, A)
B)
,i C)
D)
El

Firs!
Second
Thi rd
F0\11h
Si irth

19 Mer anOrthopaedicSl.fgrcalprocedUte.a 48,yeai-<>ld woman d<>esno1 begi n b<ealhl ng Sl)Onl8neously. TIAXIQnnne WM used fo, ....,scle relalratlon<U1ng thepr=dure
Sti mula11on ofthe '*'3.r ne,veshows comllJed netJ"omusciJarblockade. To speedrecoveryof motor fi.ncbonwtlle m1nimzing ll'M'al'ted effects,themoslappropnate
therapy1s atropi ne andwhich of the fokwwing othef drugs?
A)
8)
CJ
t, D)
El

Ca1badlol
NaOSllgni ne
Praildolame
Succin)1choine
Tri meOlaphan

20. A 67.year-old man -ps hypef1ensioo inIle recove,y room 1 toor afler a i.p replacemert He hos a histo,y of angina pedoris The most approp<iatetherapy to
decrease hts blood pressurew1 1houl causi ng a reftex increase m heanrate1s a drug that antagoni zes wh:ch dthe follovttng receptors?

A)
, B)
C)
D)
E)

a,Menergic
a,.AOrenerg,c

a.Aaene,gic and 11-adrenergic


a,Adtene<glc and deparnnerg1c
.Adrene,glc and deparrinergic

21. A60-year-old manwho1s admittedto the hospttaJ fortreatment orpneumoni a develops severe diarrhea t week after starting thefapy wtth dindamycin. Si doscopy
shows pa!dlesol ps<>udomemllranes. S1001 culturegrows a gram. posruve anae<ol>e. Oiagnosr s ,s based on finding ..,.ch olIlle folow, ng I n the pa1i 0111' s siool'I
t, A) Caps.Jar polysaccharide
6) Fimbtia,specific antigen
C) Hyphae forms
D) l nvasin
E) Toxin Aor B
22. A 14- year-<>ldgi n hasbecome reulant toea hos stopped balling, refuses tog<> toschool,and hos spent every overing al one in her roomfor1he past6 months. She
assens 111a1 herparerts andfcnner tri endS may be centarrinabngherfood Sheis dosuessed 111a1tl>ese 1o,ons are contron ing he<11>0ugh1s and causi ng he< 10 nearvoi ces
tellng hef 10ge evenw,lh herionnentors She appears disheveled.wspooous. and rnldly mano.,,shed. No otherabnormal ihes a.re noted. To.,col og,c scr een olur;ne I s
neganve_ Whi ch ofthefolowi ng medi c.atlOOS is the most appropnate treatmentforthlspati ffl?
f A)
f B)
'- C)
o DJ
EJ
f I')

Di azepam
FlM>xamine
l tnpranne
Meth""1enldate
Ri speridone
Senralne

23. In this metabolic pathway a songle enzyme catalyzes each step. Wtnch olthe followmgenzymes is i nhibited by aspui n?

__
y
"T"'

..,

-1 L-
Ul.lOlffiA-

Prtt.clllfldil:IO,!PtoU1gtw,ckl H,

ltfoobo:teno"'

A)

8)

C)

t D)

E)

v G)

H)

24. A 25-yeai- old man comes tothe physi cian because of nausea and vomi ting since begi ming phannacotherapy with cisplati n foJ testicul ar cancer 3 daysago. 000anse1ron
is presai bedfor his vomi ting. wtich ofthe followi ng i s the mostl ikety mechani smof theantiemetic effect ofondansetron inthi s pat ient?
e A) Activation of a2adreoergi c receptors
o 8) Acti vati on or . type opi oi d receptors
e, C) Blockade ofdopami ne,gic D, receptors
() O) s1 ockaoe 01 musearini c M, receptors
E) Blockade of serotoni nergic 5,.HT3 receptors
25. AOO.yea1- old man comestothe physi cianfora foll ow-upexalTination. He was di agll05edwith essential hypertension 4 weeksago. His blood pressueatthattime was
1so,oorrm Hg. He hasbeentreatedwith anangiotensi n- converting enzyme inhi bitor siocehis di agnosis.. tis pressuretodayi s 120180 mm Hg. '1addition 10
blockingthe conversiondangi otensi n I toangiensi n 11, this drugalso produces systenicvasodilation by ,vt.ch ofthefoll owing mechanisms?
() A) oeoeasedatriaJ natnu'ebc peptide release frc:m the atri a
e 8) Decreased Dradyklllnoegraoation
c C) Increased catec:holanine release fromtheadrenal medulla
e 0) Increased prostacydn(PGI,)synthesi s
e E) S6rra.llationof nitricoxidesyntheSis

20 A:n.yeor-old woman comes tothe PhY!IOM fo, a rO<Arnepl!ysl cal exsmrelion She and her husband have been lNble to COtlCffle for 10 years Pnor evak.etion for

den1 kty has shown no abnormab11 es in thewoman or her tl.lsband. They have sexual wecc:ou once every 6 weeks The wife tssatisfied With theirC:ll'fent trequency of
sexual aciMty 8M reaches orgasm on the lllajOnty of occas1ons. Her f'I.Jsband, howeYeS,would prefer moresexualad:Mty and he rs upset ab<:MJ.not having children yet. The
wife' s hi story does not includesexual abuse or assault. Shesays, TmW'Clll'OOdthere issomethi ngwrongwith me. lj.s don'tthnk about sex." Shesays she i s sadshe
doesn1 havedidren yet anddistressed thatsheand hertusbaOOargue about thi soften. However, she remai ns motrvatedatWOfkand takes pleasure in hercareer. Whi ch
ofu..fOIOwing, sthe most ll<Oty di agnosis?
t A) Adj ustmerccisordorwi th depn,ssodmood
e B) Oysthymic <isorder
C) Female seiwal arousal <isder
0) Hll)Oact1ve sexual desrredrSOteler
E) MaJordof)IOUIYOdi sorder
t, F) Sexual avers,oo di sorder
27. DI.ling an appendectomy, a 63-year-ol dwoman -ps paroxysmal sup!avenlria.lartachycarcia The armythmia is treatedwith a rapidlyacting agent whose di rect
effects lastorly seconds Thi s drug os mostl ikely to be wltich ofine folowlng?
A) Adenosi ne
u B) Amodarone
" Cl Oiscwanide
t, 0) Labelalol
El lidocaine
t., f) Procai namde
" G) Quinidi ne
4., H) SotalOI
I ) Verapami l
28. A3,ear- old manwno is planni ng to goona safan , n EaSIAlnca lo, 4 wed<$ comes to the physi ci an for P<OPh)'laJUsagainst malana. Whdl of thefollowing <111,gs,s most
app!opnate?
A) OM!lhylcalbamaz,ne

e- B) lvemlectin

" Cl MelloQ11ne
0) Praz,quarcet
E) TnrnCtthOl)nm-s-thOlolZokl

29. A 20-yea,-OldwomanWithanorex.aa 08fYOSa bves at home With he, parents. She has secondaryamenorrhea. Whi ch oflhe following is themost1orta11 step 1n restonng
reproductive and endocrine hn::lion to normat?
! A) Benzod1azepinetherapy
Bl Disc<l<fagi ng he<parencs i rNOt;emenl in he<life
I!, C) Encouragmg he< to gain wei
D) Helf)1ng he< resolve conlr clS overa<U sexual ity
t E) Shon eo<.rse ofestroge,,replacementthefapy
30. The tnad of signs and symptoms that ,s mostoften present,n a paUentstiof1!y after taking an ovetdose ol heroin inoodes >mrch or the ronow,ng?
Al
.., Bl
J C)
, D)
., E)

Coma. mi osrs,cyanosrs
Coma, mlosrs,LW11 carla
Coma, mydnasi s,llushing
Coma, mydnas1 s,gooseflesh
leg cra"l)s. mydri8SIS, cyanos,s

31 A4>yeat-oldwoman comes to the physici anbecauseof increas,ngly seffle epi sodic pai n inher rruscles and joinls si nce "1derg01ng a divo,ce Cmorctls ago She says,
ive bff!l suugging with these s)'l11lt(<)n1S for CM!r 10years, but they hawiust gonenworse sinceI ve bff!l on myown. "Sile also has had P<ot>lems sl e"l)lng duringthi s
peri od. Physi cal exami nabon shows no swell 1 '1J. tenden-.ess, orJ01r1. effusion; however, symmelnc tenderpoi nts weeliated pon ofthe laleral aspects dthe nec:k.
muscles. shoul ders, and innerthi ghs. Laboratory studi es show an erythrosedilTlel'lation rate of 10 rmvh. Whid'lof thefolowing i sthe most appropriate
pharmacotherapyfc:w thi s patient?
t, A) AmU,ptyl ine
o 8) Olazepam
f C) Hydr-ocodone
0) Mcthocarbamol
t El Secobarbllal

32. Rll>OSOmO!Sare 1econs111Utedom 30$andSOS subln1s p..-llled from suepcomyan.ser&IM! (S) and suepcomyon.res,Slllm (R) Eschenc/118 oo/1 These nbosomes are
then added to a phenyfalaninei ncorporation system. Incubations arecarri edrufflthorwi thout streptomyci\andthe alTIOUll ofradioac:liYe phenylalarioe incorporati oni s
meas<.<a<I (resubShOwn , n lhe table)
Procetns From
30S Sub<lit
S5
$12

s
R
S

s
s
s

S
R

s
s
s

Protetns From
50SSWuni t
S16
S14

s
s
s
s

s
s
s
s

R
S

S
R

l noo1po<a1ronol A<IMly (cpmx 100)


Wi lhoel
'Mth
Slrep<ornyon
Sueptomyon
63
41

62

61

59

31

42

27

59

37

42
61

40
40

The actionof streptomycin i s roost llkefy mediated by an mte,ac:tion with which of thefolowmg protesns?

1l A) S5 protem
., B) S12protel n
o C) S14 protei n
0) S16proiei n

33. A 10-year. okf boy has d'l'oric iron-deficiency anemi a.A test of stool for occult blood , s poo-bve. A bari \J'Tl enema with ai rconrastsho,vs no abnormaliti es.Fi nclngs on
,c,perQndoscopi caxa111nati onare nonnal His cOndi li oni nl)foves wrlll H2.recQpeoraniagonst lhOOIPY Whi c:11ofthe follow,ng Is d1't most lketycause ol d1't bleedi ng?
, A) AllcriovCMUSmaWonnation
o Bl C11tal megaool on (Hirsdlsprmg dosease)
, C) Esophageal telb<
-, D) Med<el <IM!rti ct.t.Jm
, E) Ulceralive coli tis
34. A 2Q.year-Ol<I man i s b1 oug11tto Ille emergency depem>en bye lnendbecauseOIcontusi on. ebd0mn61 pai n. and doanlleo101 4 hours. The SJIT1)toms began <U'lnge
party. The lnend saysthat the pabettmost Ikelook3 4.methyl ened1 0Xjmelhamphetami ne (MOMA)and LSD. C<.<rett medications i ndlde paroxeline for par,c <isorder.
Hts temperaMe i s39c (102. 2'F).Physi cal exami nabooshows myodonus andhyperrefl exi a. Anexcess ofwhi ch otthe folo'Mng neurotransmtters in the synapticdeft ,s
the mostikelycause ofthi s patieds S)f)llloms?
A) v-Ami .-.cyric acid (GABA)
B) Oopemne
C) Eplnepmne
D) Noreponepmne
E) Serotoon
35. A35-year- old woman w1 lha bloodpressure ol 15MK> mm Hg begmslaking h)'drocNo<olllazide. lnhi bobon olwhich Ollhe following i s most lkely resp<Jnsibletorlhe
natri <.<ecic effect olh)'drocNo<Oltiazi de in ttis pa1i ett?
-' A) Aldosterone receptors
' B) ATP.sensatrve K cllaMels
o C) Carbone amydrase
, 0) Ep1lllelia1 Na' Cllannets
O E) Na+
-Ct' transponer
-' F) Na-Ii' 1 ransponer
0 G) Na'-K' ATPase
, H) Na-K'-2Cr iranspor1e1
36. 4-22-year- old man i s brough tto the emergency department 1 hourafter being invOOed in a motor vehi de colli si on. Physical exami nati onshows intra-abdomi nal inju,i es
requi ri ng operati ve repair. During the operati on, he recei ves 10 units of whole blood. Poperati vely, hi s serum cak:ium concentrati on is 4.5 mg/dL Whi ch ofthe followi ng
features oftransfused whol e bl ood bestexplai ns the deaeased cal ci um concentration i nthis patient?
o A) Bl ocks endogenous para.thy,oi d hormone rel ease
o 8) Contai ns eryttwocytes ancsl eukocytes lhat a\lielty take up cak::lum
e C) Contai ns ion- chelating compounds
O 0) I ncreases endogenous calci toni n rel ease
O E) l naeases uri nary calcium cl ea,ance
37. An84-year-old man with demerli a, Alzhei me< type, is amii ttedlo the hospi a
l l lorlrealmenl olpneu'JlOl'ia At ni gl(. he becomes c-. yeting, "Where am I? How do I

get room seMce mthls pl ace?" He then sl apsone oflhe nurses and tnesto kicklhedoctor. He behaves1 0 a si m,tar mamerforlhe next 2 rghts. wtlch ofthefoll ow1 IWJ
actions i s most appropriate lo decrease thi s patienrs agtati on?

,:, A)
, Bl
C)
0)
E)

Request1h31 the pebenrs family hie a sl tte< ror hi m


Place lamolarOlljects anda ni ghUoghl near!he pa11enrs bed
Leave bnghl hgtrlSon ond lla\/G IlleSlaffctoed<on hm hO<r1y
Appo/restrai nts tolhe panett dlro lhe nigt
Medi cate the palenl with a long.adlng anxJOlyti c dlUg

38. A 24-ye01-old man who 1s lnlergoi ng generalanestheS1a lot an appendectomy develops a terrq,erature ol 40'C (104"F) and Sl)asms ollhe masseter roosdes. This
advers<! effeacO<Jd be ueated wllh a drug vnlllwhch ollhefollow,ng acti ons?
, A) Antagoni sm ol aootylcholino recopcors at Ille nOU'omuscwr uncti on
J
., Bl Decrease or Na-K ATPase m skelelalmusc e
l
O CJ Faahtabon or Ca' seQUeS!tabon , n sarcop1as111e rencu cmofskeleta rroscle
l
l
c> 0) I ncreaseof exatal ion--conracnon coupkng 1 0 skeletalmuscle
e El l mbition olacetylchol i nesterase at the neuroo-..scularj\11ction

39. A mal e newbom del ivered at38weeks' gestati on has contracted extremi ties and facial dysmorphofogy, incl udi ng dyspl asticears and flattenednose. Laboratory studies
show increased concentrati ons ofse<um urea ni trogen (BUN) and creati ni ne. Despite b ood transfusi ons, the newborndi es atthe age of 2 weeksof acute renal faiure. A
l
l
photograph of the kidneys, 1.1eters, and bl adder asseenat autopsy is shown. Whi ch of thefoll owi ng is the mostl ikety diagnosis?
o
o
o
O
O

A)
B)
C)
0)
E)

Oi George syndrome
Down syndrome
Nephroblastoma (Wi ms tumor)
Pottercomplex
TORCH complex

40. A bl COIIUlte .-eius most commonly res..s fromwni ch oftne following aooomial processes?
t, A) Bl llrceU on ol lhe genialridge
Bl llrcati on ollhe uroO(!(ltal s,nus
C) Bi llrcation oftile uterine canal
<"> 0) t ncorrc,lete fustonol lhe mesonephric (W<>lffian)rucls
t, E) t nco"l)letefUS1onor lhe paramesonepl'nc (nUerian) duels

,. B)

41. A 22-year- old manwith I-IV infection hasan eye infedlon c:lJeto herpes511'f)1ex-..rus 1 (HSV.1) that es resi stant to acydow. Devel opment ofecydov!r resi stancetsmost
ikerueto a nuatioo ofwtich o/thef-.'ling genes o/HSV-1?
A) GcoprO!QlnC
B) Majorcaps,dprotein
c C) Ri booodeotJdereductase
O) Thyml d, ne 1anase
E) Uraci l ONAglycooylase
42. A 6>year-old man comesto the pltysi dan 2 weel<s afternoti cing a n<><Ue on hs neck He is an avid gofer, bl.I hedoes notalways use sunsaeen wtite pl aying.
Elcami nat1on of1he neck slloWS a 10- nvn, pearl)' ponk, raised nocua An exc, si onal bioPSY speomen showS basal cell carci noma Whodl or thefolowtng is themost
appropnate treatment?
A)
B)
C)
0)
El

Tr ebno,nUlerapy
Qyoth<!rapy
Ioni zing radi ationtherapy
W irradati oo
Wi de exosionand fyrrc,11 nOdedlssedlon

43. WIichor Ule folowlng Sltlstances rncreases Ule secreti on of 1"5\.lrn omthe pancreas?
Al Cl ond,ne
.:, B) Glyburlde
C) Motformn
, D) Predni sooe
, E) Somatoslabn
44. Clomi phene ci trate. folcl e.stim.datirg hormone, and menopausal gonadotropi ns are admirislered to a 32.year-oldwomanto inducefolclAogenesisforan invitro
fe<11iza i onproce<Ue. The oocytes wnnin responding anal follicles rei ni ti ate mei oocdevelapmentas they are released from whi eho/the ro1oWi ng s,ages OImelouc
t
arrest?
c A)
Bl
C)
0)
c E)
, Fl

1st me,otrc prOl)hase


1SI metotrc mecal)llase
1SI meioic telol)nase
2nd merooc propltase
2nd mesotrc metaphase
2nd merot, c 1e10pnase

45. A60-year- oldwoman i s admitted to the hospi tal

witha pufmonary ent>oli sm. Treatment 'Ait henoxapari n is begun. The patieriseys, '"Thafs notthe heparin my other doctor
gave me. wttfam I getting thi s newfangled Sluff?" Compared wi th heparin,Ylhi chofthe foll owing best expl ainsthe appropriateness ofemxaparin for thi s patient?

A) Avai able oraly


l
l
() B) Larger oadi ng dose requi red
l
o C) less expensm foreach day's therapy
e, O) Lessmoni t0ti reQt.Jired

46. A 49,..year. Old womancomes to the phys:aaan becauseof a 1 .year history ofintenni nenc blOOdstained <liannea vttlaafll)i nginihe IOwer abdomen Resuts of
sigmoi doscopyare si-n Cultu'e of stool and biopsy of mucosa ,soowno oYidence of bederiol, amebi c, Yirat infedlon Thennos1 Olll)fopri ate mai ntenance therapy
lncbles adrronlstrauon ol v.!lichollhefolow1 ng drugs?
., A) Ciproftoxacin
B) OapS<lne
C) Do,cyqdine
O D) EUlambutol
E) Genlarroc,n
I') Slreptomyan
o G) &6adiaiine
, H) &6asalozme

:j r/;;:::;::::

----A(le6S 80teUS

--Aoe2s 4 V8m.

('er dlM01<1Jt(L/min) -

47. The graphs depi ctthe rel ati onshi p between cardi ac output and heart rate, end-diastoli cvollme, and stroke vol ume for healthy eldert,, men and healthyyounger men. Based
on the ,esul ts of theSe studies. whi ch or the followi ng age+related anatomi c changes contributesto wel-mai r1ai ned cardi ac oull)(A i nthe elderl y men?
o A)
! B)
t, C)
e OJ
o I E)

Oi lote<1 ascenc1, ng aona


I ncreased coronary collateral drcttati on
I ncreased nunber of myocytes
leftventricul ardil ation
Len ventricul a, hyperloaphy

48. A 30-yee,.otdwomanwllotaMs ibl.(J<olenf artl'Jotos has a m, ,,1<ed increase ,n mensloual crOlll)lng after Slarung a drug toprevM Pol)llc utce<. 1/Vhchol lhe foll
drugs mos,l ikely causedlhls adverse effect?
A) Allmirun hydroxide
B) Magnesillll hydrollide
C) Meched>oline
0) Sop<OSlOI
:, E) Omeprazole
F) Ranitiline
G) Sodi,mbi cartx>nate
H) Sucrolate

49. An 8-)'<lar-ol d boyis broughl toIlle physici an's officeby hi s mothe< becausehe has di lliaJly payi ng attenti on in school and compl eti ng his homeworl<. He has a hi story of
learni ng problems. Aer lunher evatl on, thephysl dan rocommene!S a 1 ria1 of a drug On 1 11<1first dayol l herapy 1he pat,enrs teacher call$ hi s m0ther to sayhe ha; had
,
"his best day In weeks. " He most Ike starte<1 thefapy wllh whi chof the following drugs?
,o A) Desi prami ne
o B) Dextromel horphan
o C) Di azepam
,o D) l rriprami no
E) Methyiphenl date
50. A 25-year-oldman comes to the physi cian for a routine exami nation. Duri ng the interview he says, 1feel l ike I am a woman trapped i n a man's body. I have ahvays erioyed
ac1l vi ti esthat are usual ly associa1ed with women and I am rruch m0te comfOflable in women' s clothes I have datedwomen before, but rm n01 anrncted 10 them. rm
attracte<1 to men, but not as a man, as a woman." Which the followi ng l s Ille most likely di agnosi s?
o A) Bi sexuality
o B) Fto<teur
i sm
e C) Gender identi ty di sorder
0) Hermaphrodi ti sm
o E) Transvesti sm

or

NBME form 5-4


1. A 64-year--0l d woman has a 2-year history ofprogressive shortness of breath. Blood pressure i s 171fl4 mm Hg with rapi d col lapse ofthe peri pheral pulse. She has a hi ghpitched, bl owi ng, di astol ic decrescendo murmur heard bestat the second ri ght intercostal space. Whi ch ofthefollowi ng is the most ikelydiagnosi s?
O A)
B)
O C)
o 0)
() E)
() F")
O G)
() H)

Aorti c regurgi tation


Aorti c stenosis
Mttral regurgi tation
M, trat steoosis
Pulmoric r09..-gitati on
Pul moric stenosi s
Tri cuspi d reg\lgitation
Trl cuspi d stenosi s

2. A OO- year--0l d woman develops pal pi tations O'Jer a 6- week period. A rhythm strip from her ECG is shO'lvn. Whi ch ofthe followi ng is mostl ikety responsiblefor the pa\:li tati ons
i nthis woman?
() A) Atri al flUter
e B) Atri oventricul arreentrant tachycardi a
O C) Si nus tach)cardi a
O) ventri cular fibril lati on
E) Ventri culartachycardi a

o
o

3 A 12- Y'!ar,ol d gor1dieshorn pneumona Shehad Chroni c matabSOl1)tion and rr,JIJpleec,,s OfC0\9' will C<lC)o()us SJ)IJll.ffl prOdilCl>on end occaS1onatt,emoptysl s. The
appearance oflhe U)Qat autopsy is shown.A a*ure of!he u,g alautopsy is most lketyto gaN whi ch of lhefolowtng organi sms?
Al Cocci</1 oicJesrmmms

c BJ
C)
0)
E)

Legione11a-hila
Myooplasma pneumonia,,
Pnoumocystisjinwoci(formetly P. catinil)
PStJudomonas-,,pinosB

4. A51- year-otd manhas e holosystoic ""'"',. !halls loudest c,;erlhe apice.ll f1'C)ul seandred, ates to the a,ollae. Wl'oCh Ofthe folcwMg Is lhe most like!)' d1egnOS1s?
-, A)
-o B)
C)
., D)
El
,, Fl
GJ
.a, H)

Aortic insuffi ciency


Aortic stenosas
Mitra! ins<Jfficy
Mitra! steno,;,s
P.-noroc o nsulldency
P\/rnoncstenosi s
Trtcusp, d 1 ns..Ciciency
Triruspi d stenosi s

5. ApreiAously healhy 24-year- ol d v1oman comes to theemergency department because of di arrhea, nausea, flatul ence, and bl oating forthe past 5weeks. Bowet movemeris
are occo:mponi ed by oll<lominal cramping. She has not trowle<I out olt toe Uni ted States. TYl?'Clani c tempe<ature i s 31c (98.6'F). Bowet sounds ore increased Test of stool
t or occut blOod Is negat, ve. Whi chof the following Is the most llel y causal organi sm?
t, A) BaclJlus cereus

o B)
o C)
o D)
O E)

CmpylobacterJefuni
Cryp1ospori</wm psrwm
Giard1 0 lamblia
Shige//a sonnoi

6. A 5- day-dll newborn hos been cyanotic s,nce t,,ru, and has a son systolic rrurrrur. AA x-ray ofIlle chest shows deo'eased pul monruyvasa.tar mar1<1ngs i nd1ca1mg a
decreas in ptknonarybloodflow. Which dthe following is the most lkely di ag,osi s?
( Al A!ri ll septal doloct
c, B) Bi cuspi d aofticvat.e
(' C) Coardatlon <tthe
'- D) DeJC11ocardla
( E) Patea ducrus arteriosus
f F) Tolralogyd Falot
(, G) Transposi tion dtheg,eat vessels
o HJ ven111eutar SEl)Cat detect

aooa

7 . A 14--year-dd gi rl witha 3-monh hi story dasthma comestolhe physician for a folow examinat ion. Ct.-rent medications inctKJe imaled corticosteroi ds. Her symptoms
occurless than onceweekly Thepnysi cianrecommends thatthepanerr add a new drug (drug X) tha wnen odmir>Ste<ed ao.ing an asmmaac ei,isooe, ,.,,,mmea,atefy
increase lhe FEV, 10greal er thin80% dIlle FVC Whl dl of Illefollowing bes1de5albes the mechanism ot acbon ol drug X?
o A) Constndl on ol iunonary b<onchl oles
( B) Constndion of pumonaryv.iscul ar smooth muscle
o C) Decreased pm,onarysurfaaant production
f" D) Di lafion dp<.tnonary brondiol es
c, E) Dllauon dp<Jmonary vasc1Aa1 smoodl musde
F) I ncreased pul monary sl.Wfaaant P<on
8. A 14-year-old boy has a ar l'istoiy c(type 1 di abetes meltus. He injects timselwith insul intwicedai. When hegoeslo summer cafl'). he stops taki ng hisi nsulin
because he does notwa11 to seem di ffetertfrom his peers. Afr.er24 hours withotf tis insl.Jin dose. hedevelops abdomnat pai n and vomitingand becomes ds:orien:ed and
offia,t 10 arouse. He isIBkento1he l ocal "'5pllal On admi SSIOO10 the hospila he IS mostl ikely lohrle an increased sen.m gl ucose concentrabon andwh1 dl dlhe
fonow,,,g findings i n a,...,alblOod?
HCO,() A)
B)
C)
I'> D)
c, E)

e
e

l
I
I
No change
No change

Pco,

pH

nocnange

nocnange
nocnange

9 WIich d lhe following signaling mechani sms 1s used totta.nsmtt 1he effects <tlriiodoltryronine (T,)?
A)
r, B)
r, C)
" D)
C E)

T, bi nding acri va1es a cet-sl.Wface adenylylcyclase recepto,


T, bi n<ing adi vales a cetl-sl.Wface guanytyl cyclase recept0<
T, bi n<ing acti valesIlle DNAbi nding actilllt)f d an 1ntracellwr recor
T, bi nds toa G p,01e,I\.COl4lled recep101 on the eel surtace
T, bi ndsto a l)Yosine ki nase.inke<f receptoronthe eel slWface

10 A 52,year-old woman has menswal i rregula1i lies_ hotffashes, and emoti onal labilly I nUls patient, deo'eased folcuar hormone ll(oducti on has most ikely resulted ,nan
1 11Creased senmconceruati onofwhi chd 1he folowtng smstances?
A) Estrogen
t B) Folcte-slinuall!'Q hormone
o CJ Heman chononi c gonadob01Nn
0) t nl'Obon
t El 0X)'!OCln
(, F) Progesterone
G) Proladin
(. H) Relalon
11. A state- mandated bl oOO screeni ng test conducted on day 2 ofl ife i n a healthy full- term newborn shows a decreased serum thyroxine (T.-l concentrati on and a normal serum
thytoid- sti mulati ng hormone (TSH) concentration. Tests onvenous blood confirmthese findi ngs. The venous free T. concentrafion i s normal. Whi ch ofthefoll owi ng is the
m0s.1 l ikel y exp1 ana1i onf0t these results?
c A) Oysgenesi s ofthe thyroi d gl and
t, B) Hypopi tui tarism
e C) nborn error ofthyroxi neS)ffhesi s
o D) II odi ne deti oency
() E) Thytoxi ne.t,1ndi r)1 g)obu!i n deficiency

12. An increase in the sef'\111 corx:ertralion orwhich ofthe following IS the mostcond.Jsive si gn thatowl ation has occurred?

Al
r, 8)
o CJ
0 0)
o E)

Androstenedione
EstracllOI
Estriol
Llleini zing honnone
Progesterone

13. A 45-year- old man wi th hypertensaoo is <iagnosed w,th pnmaryhyperaldosteronsm. Whi ch ofthe followmg setsolphysiol ogic changesi s mostl ikely in thi s patieflt?
, A)

B)
C)
0)
E)
F)
G)

Plasma Reni nAdi \/lty


I

Plasma K' Concervaboo

T
I
T
I
I
!

Extracellulai Flud Voune


I
I
!
I
l

14. Dunng the dlffe<enhahon or marrrna,ygtandalveOlarcets inlate pregnancy, the hoonOne thatactssynerg,Sllcaly w,th eSIJogen i swfli chol the fOIOWlng?
:, A)
o 8)
C)
0)
1l E)

ADH (vasopressi n)
Hlman dlononlc gonadOlropln
Hlman placerul laaogen
Lutemi zing hoonone
0,cytoci n

15. Th, smotif Ol a steroodhOrrnone recei,to, bonds Sl)eCi cal to whdl or the folow, ng compO<.lldS?
?) A) ATP
, 8) C)dcAMP
C) ONA
, 0) RNA
El Steroid hOnnone
16. A 37-year- old mandi es suddenly. He had a hi story offati gue, dyspnea, dependent edema, and a decreased ejection fracti on. A1 autopsy, the structure ofthevalves is
normal, blf all four chambers ofthe hean are enlarged. A mural thrombus is attachedtottriewall ofthel eft atri lJTl. Hi stol ogic exami nati on ofthe heart shows hypertrophi ed
and atrophi ed muscle cells. Whi ch of the folowing is the most likely di agnosi s?
A)
o
Amyl oi dosi s
o 8) Di lated careliomyopathy
e C) Hypenrophi c cardi omyopathy
O D) Myxedema heart
O E) Restri ctive cardiomyopathy

17. A41. year. 01<1 woman1emal nshosp,laize<l ancl lnbe<l 10< IOdays because olc:ompllcabonslolowing a hyste,eclomy for IM!nne lel omyomas On I lle lerlhday, she
suddenly d<Welops wvere r<15ptralOI)' di stress andcoll apses. SIiors resusc:r ialOd, but dies 2 days lator. Au,gu soon ataiaopsy is shown. Whi ch of11>11folowing is tho
most lkely di agnosis?

o A) B<onchol)neumonoa
o B) LOilar pnoumoria

o C) L1119 abscess
O 0) Nnonary 1nfard
c E) Nnonary me1asi.ses

18. A 56-year- ol d manwi th chroni c obstrucbve pulmonary disease is broughtto the eme,gency department because of severe shortness of breath for 1 hour. Two hours after
arriva hedies despi te appropriate li fesaving measures. He had smoked 2 packs of ci garettesdai lyfor40 years. A photograph ofthe l ung as seen at autopsy is sho,m.
These lrlQfindi ngs a1e most h kel y due towhi ch of the fol lo1Ang medlanisms?
o A) BreakdOIYn or el asticfi bers
o B) Constricti on of b<oochi ol arsmooth musde
tJ C) Excess collagen in the interstitial spaces
e> 0) I ncreased intrabronctiol ar ai r presswe
E) Proliferabon of alveol ar epi thel ium
19. A 60.year- oldwoman i s brour,.tto the emergency department by her husband because of chest pai nthat began 3 hours ago. She is dyspnei c. and di aphoretic, and she
appears anxious. HerptAsei s 110/mi n and reg\mr, respirations are 351mi n, and bl ood p(essure i s 190/110 mm Hg. An ECG shows an acute myocardial infarai on. wtich
of the folowi ng is the most likel y cause of this patient's pain?

o A) Abno1mal ities of ion transPort i n myocytes


o B) Accullul ation of metaboli c products in hean ti ssue
e C) I ncreased di astol ic pressure
e 0) I ncreased parasympatheti c sti mul ation

O E) I ncreased sympatheti c. sti mAation

o F) i ncreasedsys101o c pressure

20. A healhy 30-)"a<-Old woman isj awng. COmpared wtthvweswt.le she r s at ,es1, whc:11 olthefollowing ta1dlopul mooa,y responses are mosll ikely wllle she JOQS ata
eadyrate?
A)
Bl
C)
D)
E)

red Mirue Vonlaon


Unc119nged
Unchanged

Cardiac Clup<JI
uncl\anged

Aneri a1Po2
lllChanged
lllChanged
T
111ehanged
T

21. A newborn has eJ<temalgeni tala thatappear tobe female. However, thevagi r0 endsas a bindpouch. ChrOIOOSOme anolysi s sho'IIS a 46,XY ka,yolype. Wllc:11 olthe
fol lo'llingis lhe mosr ikely explanabon for lhepllysi cal fincings?
A) Abnonnal androgen receptors
8) Absence of the sex-dotorml rOQlon on theY Chlomosomo
C) Goredal dysgenes,s
D) l naeased fOlal conc:enlrahonalestrogen
E) I naeasedfetal conc:ertrati"ondtestostetone
22. A 26-year- old man Is acnl Ued toIlle l rtenslve care unrt because olbactenal sepsis. Laborato,y studies show:
Arle<ral bl ood gases (room e,r)
pH
7.2
POO,
32 rrrnHg
Po,
96 rrrnHg
Serum
12 mEqll
HCo,Whi chofthefollowing is 1he normal ini tial compensatoryresponse tothis pH?
A) Oeaeased exacbon ofHCo,' 8) Oeo:eased respiratory rate
e Cl naeased excreti on01 Hco,1
D) l naeased respo,ato,y mte

Inspi red oxygen conc.entration

23. The venbl at o,y reSPonse to decreasi ng i evets or Inspi red oxygen coocettrabon for an experi men1a1 animal , s shown. Wh! Ch or thefollowi ng l abel ed figures best represents
the response in ttis animal after removal ofthe caroti d bodies?

bLL
A

e A) e B) e CJ e D) () E) e F)

24, A mal e newbor n who Is deliveied at 39 weeks' geslat, on has respi rat ory di stress. The pregnancy was coml)Wcated by M!ve<e ol igohydn,rrnos. Examl nat, on oll he nellborn
shows a lowerabdomi nal mass. Further sludi es show pul monary hypopl asi a. Whi ch ofthe followmg 1s t he mostl ikely ooderly1 ng cause ofthese findi ngs?
() A' Congerita1 megacol on (Hi rschsprung di sease)
e BJ Meringocele
() C) Posterior u1ettwalvalves
o OJ PylOlic s1enos1 s
o E) Small bOwel atre,i a

25. Perforation orwtlch ofthe folow,ng structures wi l J)focbce reb'operitoneaJ gas?


,> A)
B)
C)
0)
;, E)

Ascending colon
Cecum
Jejl.llum
Tcrmi nali lelrn
Transver.;e colon

26. A 49-year- old woman wi thctuOl"IC coronary artery disease undergoes coronary artericqaphy that shows a 50% decrease 1 n the II.men ofthe leftanteno, descending
coronaiy ane,y F0<any ane<i OYllllOus pcessure g,adi erc, tileflow
!tis ane,y (c0<0pa1Qd wi th no<mal) w,Idecrease bya raaor ohmi cti ofme folowing?

llvougn

, A)
B)
., C)
0)
, E)

2
4
8
16
32

27. A 53-year- oldwoman has excessive thi rst and u1i nali on. lhere i s no detectable g))cose in the urine. All 11ater is withheld for 6 hours. Wl'ich ofthe f<Jllowi ng responses is
mOS1 consi stent withthe di agnosi s of nephrogeni c di abetes insi pi ctus?
O A) Decrease in serum osrnol al ity
O B) Decrease in serum sodi llll concenlration
O C) I ncrease in serum AOH (vasopressin) concentration
o 0) i ncrease 1n uri ne osmo1 a111y

28. A 55-year-oldwoman comesto the physlc:lan becauseofmcreased laoal hai r growthOYerthe past 8years. Menopause OCCU'redal age 45. laboratory stooies ofserum
show:
Testos1erone
Oehydroepi androsletone sulfate
Thyr0ldS11 mul 0bnghonnOnO
ESlradiof

Progesterone

80 ng/dl (N20-90 ng/dl.)


320 g.'dl (N:-350 i>'!L)
08UlnL

20 PO'n-1.

decreased

Whi ch ofthefolowi ng isthe mostl ikely cause oflhe i ncreased facial hairgrowth?
Decreased So-reouctase amity
Decreased adrenalandrogen produ:llon
Decreased androslenedione perlpne,al OC)(Werslon
Oecreaseo l ntlbl n pcocM11 on
I ncreasedovarian anttogen production
I ncreased testosterone:estrogen ratio

A)
8)
C)
0)
E)
F)

29. A 19--year-old man i s brought toIlle etnetgency deparmenl after hitti ng fishead on the pavement when he fell oilfismotorcyde. An MRI cl hi s head shows a transection o
1he pi 111tary Slalt wt.icti of thef-ng panems beSI oescns the secraoonof pi tui tary hOlrnones in lhl s panerf?
ACTH
l'SH
Growth Honnone
Profadln TSH
A)
e 81
C)
0)
El
F)
(ACTH= aClrenoco'1lCOtrol)lc hormone; FSH = ro1c1 e.st1nuaong hormone; TSH = 111yroi -sti mu1ang IIOrmone)
30. A61-year-oldwoman i s foll'ldtohavea 3-cm,asynl)tomatic,cysticadnexal massonrrui ne e,amlnaioo. U1trasosho1Ysa s,"1)lecyst. Whi ch clIllefoll owi ng
value$ meawed ,n serumi s mos:i:l ikety 1 0 aid 1nde1ermi ning 1tiemanagement ofltwS pati et1?
A)
e 8)
.., C)
0)
E)

CA 125
Carci noeni>ryoni c antigen
Estradiof
a-Fetopcotein
-Human chonooi c gona-op,n

31 AOil-year-oldman expene1ices lnc:reaS1ng 111nary fr-,cy and d1 ffi"*Y stani ng and stoc,pi ng Juflow of unne Elolmlnation clossue oblalned on bi opsy of the enl arged
prostate comnns b<,r1gnpcostati c hyperplas a WIich clthe follow,ng,. the most hkely hcaoonof h,s bas,c disease state?
i
A) ActJe bacterial pyetonephri ti s
8) 8actenia
C) Bactet1 alcyst1 tis
0) Chron,crenal fal ..-e
E) Mal irransformatian
32. Fud balance is stool ed ina bl ood;:lerfused ITIJSCle preparati on wi than intactlymphallc system rrom an expenmen&al ani mal.Mer a control peri od d..,.i ng whi chall
parameters arenormal, i ncreasing wtich ofthek>Howing is mostl ikto increase therate oflymph ftowwi thout affecting the protein contert of the lymph?
A)
e B)
,-, C)
0)
, E)

Capollal'fcollotd osmobC pressure


Capil lal'f hydroslati c pcess
..e
Conslncti on cl pcecapolary Sl)l1mcters
Endolheloal transcytosis
Wi dth ofj unaions be"""'n endolllelial cells

eu.a discharge. The urelt<al meatus is congested and edematous.


33. Four days aRe, selCIJal lntereo1ne. a 2yenr.old mandelleioPS <1ysuna and a muoopun.lent ..
Examination of discharge showsgram-negatrve cipl ococci wi thin neutrophi ic leukocytes Ifttis 1i rtectioni s no1 treated, Whi dl ofthefollowi ng i stoo most lkely
con'C)lcaboo?
(. A) Hemorrllag,c celulibs
f:' B) Cysti ti s
t C) Eptdr dymbs
t D) Orc:11itis
e E) Pyelonephnlrs
34. A 42,year.old mancanes 10 the pllysiaon becauseor swelli ng andten<lemess orlls nght el boW lor8weeks Nl ,.,,y otthe rigt1- extre1111yshoWsa destrUCIMI lyti c
massatthe right <MW radi us. biopsy olt homaS1a SllOWS mocasaabc cl r colcaranoma corrc,atrbl e vlh pnmary renal Ctl caranoma WIICh or the folloYtlng
-1 li nongs1$ most suggeshe ol hetedr laryrenal eel caronoma in thi s paberl?
o A) Extens, on oflhe roc,,ey tumor hough Geroo, rasoa
c B) J<idney 1urnor dtameter greater rian5cm
(I C) Presence olnmors in both kiooeys
e 0) Presence ofo..mort:tYombus i nh renalvoin
(I E) Pnmary necrorrc end cystic noII.re orthe ki dney hlnOr

35 A 38-yeor.old mon comes tott,e physi aon24 hours allefIIOITlbng coffee.ground.Ike mate,i al He has o O,mar1h hi story orintermittent. di.Ieplgostncpa, n that is
tanly rotevecl wi th ll>e useor antaads Physi cal exam, nat ion shows mi ld ep,gasiric tolldemllso. Uppo, gastrornteS11NI endoscopy shows o duodenal ulcef Serum
SUldoes showmarl<edly increased c rculabng oonceroabons otgastrm. Gastri n. meaaled hyperstirrualion orwllch or the lolowlng cell lJl>es has contr11Medthe mostlo the
deYelopment da duodenal inithi s pafieflt?
(I A) Acinar eels ol Brunner glands
(I B) Centroacl nar cells
o C) Ente, oendOCline cells
o 0) Gastric ch,elceus
o E) Mucous ned<cel ls
e F) Panell> cells
(I G) Parietal eels
o H) Su1acerr.,cousce11s
36. A healthy 25- year-ol d man has a sampl e or bl ood dravm 1 hour after he eats a baked potato. The concentration or which otthe following substances i s mostl ikety increased
in the blood sampl e?
e A) Amyl ose
e B) Gl ucose
O C) limit dex.tri n
o 0) Pecti n
e El Sucrose
37. Whi ch ol lhe lolowrng Is most hkely to causean inc1ease ,n fillraboo fraeboo?
A) Decr eased glOmer\Jar fillrahoo coelfldert
B) 1 naeased afferent aneno1 01 resisiance
C) l naeased effe rent arteriol arresistance
t, 0) I ncreased pl asma prote1n coocel'Vabon
E) l naeased renal bl ood now

38 An 18 year.ol dwcman deh.....s the !Wins shownbycesar8"nsea,ona1 8Pl)(matlll'f 30weel<S'aboo She has had no prenatal care Whlellollhe ro1ow, no rypes or
twin pl acenlabon1 s most liket,to be present?
e A) Oi c:horioni c di-arrnioti c
B) Di cllononi c dr aimolic (!used)
, C) Oi CIIOOoni c monoamnooc
t 0) Monochorioni c dlannobc
e E) Monochorioni c monoamriotic

39. Afemale ne-1,t>om born at32 weeks' gestati on has rapid, l abored breathing and mi ldl y decreased chest expansi on on i nspi ration. The most likefy cause ofher conditioni s
a deficiency in a seaetory product manufaCtl.J'ed by wtii cm ofthefollowi ng l abel ed cell types shown?

0 A)

0 B)

() C)

() 0)

40 A57.yea,.clclwoman withpemoci ous anema has absent gaslnc parietaleels An increase m"'1ich ofthelolowing is the mos, ikeaddi bonot findi ng?
A)
B)
C)
,,.. D)
E)

GaSIM Inblood
Protemogesi, oolnthestomach
Secretin inblood
Secretion dgastri c acid
Secretion ol intri nsi c factOf

41. An 8- year-Old ix,,isbroughtto the eme,geocydepartment by tismothef because he has had intermi ttertabdooli nal pain forthe past 24 houls. Witnn thepast2 hoursthe
pai n has become more seYere and constafl. and he has deYeloped a temperaturedC (100.4..F).. Physicalexami nation showstenderness and guaRing in the ri ght
lower quadrant. Adiagnosi s of aa.te appeooi citis is made and a faparotomyi s perlooned.The appe,,dix isfru,dto be normat however, an inflamed ilea! diverti cul um i s
remOYed. wtichmthefolloY/ing developmenlal defects i s mos,ikelyresponsiblefo,lllisfindi ng?
A)
o B)
() C)
e 0)
E)

Abnormalfixation ofthe intestinaJ mesertery


A defecti nlheloo!jtudinal taye< oflhei rtestinal muscuaris
Fail ure ofnormalrecanaffzationl ead1"9toduplication ofthe1 rtestina1 l umen
Fail ure orlherndgtJI IOOp 1 0 tate asd entersthe abdomen
Persi stence da rermant ofthe proxirnat poni on ofthe yolksac

42. A 13-year- old boyi s broughtto the physi ci an because he devel opedjaundi ce 2 days agofollowi ng a Boy Scout sll\'ival outi ng inwhi ch hewas requiled tofast for30 hours.
He has had inte<mi tteni mi kl aundice since ea"1' chil dhood. There i s no hi story ofmedi cation use or St.bst:ance abuse. Physi cal exami nati on shows jalJldice and mi ld
j osple00megafy Hts
Seleral 1 aerus. mere 1s no t1epa
serum total blll rubl n concentration 1s 3.a mgt<JL wt tn a Clrect COlll)onent or o. mg/OL I magi ng stoo1es ortne liver show
.
1
no aboormaities. Whi ch ofthefollowi ng is the most: ikelydiagnosi s?

e A)
O B)
C)
o 0)
e El

Aufoi rrrnune hemolytic anenia


Chroni c fami lial nonhemolyti c j aundi ce
Dubi n-Johnson synd1ome
Gi lbert syndrom
-Thalasserni a

43. A 32-year- old nulli gravid wc:man has had headaches. gal actorfhea, and amenorrhea forthe past6 months. Hermenstrua1 cycles have been i rregularfor many years. Serum
prol actin concentrati on i s 400 ng.ltnL All MRI ofthe head shows a 3-cm pitui tary mass. Whi ch ofthe following vi sual field defects showni s mostl ikelyi nthis pati ent?

VI SUAL FI ELD
Ai9htoyc

C, A)

()
()
()
()

,O B) C, C}

()
()
()
()

0 D)

0 E)

44. A previously healthy 35-year- oldwoman has had visualdi sturbances for the past 3 days and increasi ngt,, cold upperextremities and m.rnb hands fo, the past 3 weeks.
Physi caJ examination shows a bl ood pressure of 165/95 mm Hg, markedly weakened radi al and ri ght caroti d plAses, and ri ghl cefi nal hemorrhages. Angi ography shows
focal nairowing ofthe brachi ocephali c, ri ght carotid, and &t>davian arte1i es. Which ofthe follO'Mng is the mostl ikely di agnosi s?
e A) MucocUlaneousmph node syndrome (Kawasalo syndrome)
o B) Pol yaneri tl s nodosa
e C) Raynaud <isease
O 0) Takayasu arteritis
O E) Temporal arteritis
o F) Throml:>oang,i ti s obli terans
o G) Wegenerg,aoolomatosi s
45. A 3- year-ol d boy has a gadual ly enl arging scrotal mass. Transi ltmi nation and scrotal ultrasonography show a large fluid collection. Thetestes are normal. Abnormal
devel opment ofwhi ch ofthefolowiog structures i s mostl iket, to be related to these findings?
o A) Fascia oremasterica
O B) Gubernacul um
O C) Processusvagi na.ls
o D) Tuni ca al bug,nea
O E) Tuni ca dartos
Volume (l)
Lung

20 15 10 S O S 10 IS 20
Pressure (<:m H,0)

46. Slati c pressure.vol ume relati onships fort he lung and CheSI wall ota 30.year. okl man a,e shOwn inthe
graph. Which 0(t he foll O\vi ng
- i s the vol ume remai ning 1n Ill s l ungs at
the end ofa normal expiration (i n L)?

0 A) 0.5

0 B) 1

O C) 1.5

0 D) 2
0 EJ 2.5

47. A 60.year-old man comes to the physi cian because of a 6-month hi story of progressi ve shortness ofbreath. He has smoked 2 packs of ci garettes dai fyfor 40 years. His
respi ratio:ns are 22/mi n. Physi cal exami nati on shows hyperresona.nceto pe,ccussi on. Laboratory Sludi es show hypoxi a and hypercapni a Hi s gas exchange rate is 15% of
that of a healthy indi vidval. A CTscan or the chest shO'llS runeious obstructed small bronchi oles: wi th airtrappi ng. I n atveoti beyond the obstructi ons, whi ch of thefolowing
poi nts on the graph shown best represents the relationshi p between Po2 and Pco2 i f lhe patient is breathing room ai r at seal evel ?

---"
A

r"'

g ,,

3'

E
10

40

80 1 00 1'20
POz (mmHID

140

IO

N: t'-31!ttyi ndM<bllv.i! ll00 Sl! U::UOO

() A)

() 8)

() C)

() 0)

() E)

48. A no,mal male subjed hasa trans&ent Increase In blood p,essure to 14C/88mm Hg efte< rapid l rlus,on ol2 Lol nonnel seine. Whi ch ol lhe ronow,ng secs or findings ,n
sen.rn concemations is mostl ike.tj?
f" A)
t 8)
f" C)
t, D)
f" E)
t, F)

Alrial Natri tsl!llc Factor

R<!f'll1

Aldosaorone

Angi o1ensin AOH (Vasoprcssin)

49. A 12-year-<*1boy has hadslowly progessi ve renalCai Uefor thepast 2 years. He i s short and pale. Blood pressu-e is 150,"92 mm Hg. Sen.In caJc:11.mconcentrationi s
decreased and serum phosphate, urea Mfogen (BUN), and creatini ne concentrati ons areincreased. X...rays dthe bones show Sttlperiosteal erosi onsofthe phalanges,
osteopeni a, and coarsenng Ira- Whi ch olthefellowingsets ol l aboralocyindr ngs i n sen.m is mostlikely m thi s patien?
25-Hydroxydlolecak:Ke<ol 1.25-(),hydloxyc,olecal cllerol Parathyro,d Hormone
normal
( A)
1
( 8)
normal
Noonar
I
( C)
I
t
l
0)
nounal
l
E)
Noonar
l
50. A 36-year- old man has shortness ofbreath, hematufi a, and oliguri a. Serum urea ni trogen (BUN) and creati nine concentrations are iooeased. Antineutrophi1 cytoplasmic
anti bodyassay is strongly posi ti ve. An x-ray ofthe chest shows ndi pl e nodul ar infilrates inthel ungs. Exami nation of urinaty sediments is mostl ikely to show whi ch ofthe
fol lowing?
() A) Cellular debris and ePolheial cell casts
(') B) Many ovalfatbOCSi es and hyal ine casts
() C) Many red eel casts and el)'lllr,ocytes
O 0) Many white ceH casts and bacteri a
O E) Occasi onal tubul arepi thelial cell and oxal ate crystal s

Rd'o1ocdah

1--
0

UII Lengtctmln.i ,..i

6. The strategy for gene therapy summarized in the diagram is expected to be most useful for treating di seases
caused by mutations in which of the following genes?
O A)
0 B)
OC)
o D)

Genes for cytoskeletal proteins


Genes for essential enzymes
Homeobox genes
Oncogenes

7. A 75-year-ol d man has a pai nless mass in the left upper anterior neck. Examination of ti ssue obtained on biopsy
shows partial replacement of a lymph node by a neoplasm; the neopl asm is composed of cohesive sheets of
polygonal cells with granular eosinophilic cytoplasm. prominent intercell ular bridges, and pleomorphic round
nuclei. lmmunocytochemi cal preparations show a strongly positi ve reaction of the neoplastic cells to a high
molecular-weight keratin antibody. There is no reaction to antibodies against vimentin and leukocyte common
antigen (CD45). Which of the following is the most likely diagnosis?
o A) Adenocarcinoma
0 B) Mal ignant melanoma
O C) Non-Hodgkin l ymphoma
O D) Sarcoma
O E) Squamous cell carcinoma
8. A 27-year-ol d woman with alcohol dependence and major depressive disorder tells the physician that she has
been repeatedly physically and emotionally abused by her current partner. Which of the followi ng is the most
appropriate ini tial response by the physician?
0 A) "Batteri ng only grows worse over lime. For your safety and self-esteem, it is in your best interest to leave this
relationship."
0 B) "Do you feel it is safe for you to return home?"
O C) "I s there any way you can keep him from getting so angry?"
O D) 'There may be a correlation between your history of al coholism and the abuse. If you lake some steps to stop
your alcohol intake, the abuse may possibly subside."
0 E) ''Why do you think your partner abuses you?"
9. A 28-year-ol d man is brought to the emergency department after being lost in a remote wilderness area for 3
weeks. He consumed all ofthe food in his backpack 2 weeks ago and has had only wild berries and pond water
since then. Intravenous feeding is begun. Blood samples obtained before intravenous feeding was begun show
Increased serum concentrations of ketone bodi es and urea nitrogen (BUN); glucose concentration Is wi thin the
reference range. Use of which of the followi ng best expl ains the maintenance of a normal gl ucose concentration
in this patient?
o A)
0 B)
O C)
O 0)
O E)

Dietary protein
Hepatic glycogen stores
Skeletal muscle glycogen
Skeletal muscle protein
Urea nitrogen

10. Many aromatic hydrocarbons, such as benzopyrene found in cigarette smoke, are procarcinogens. They are
converted to more reactive, DNA-damaging carcinogens by cellul ar metabolism. This metabolic activation lo
reactive carcinogens invol ves which of the following actions?
O A) Conjugation with glutathione by gl utathi one transferases
0 B) Glucuronidation by glucuronosyl transferases
o C) Glycosylalion by gl ycosyl transferases
o D) Oxidation by cytochrome P450
O E) Oxidation by mitochondrial cytochrome A
0 F) Sul fation by sulfolransferases
1 1 . An 82-year-ol d man develops mild shortness of breath 3 months after beginni ng therapy that includes bl eomycin
for a locali zed squamous cell tumor of the mouth. Which ofthe followi ng pul monary function test resul ts are
most li kel y to be observed in this patient?
FEV /FVC
Total Lung
Di ffusing
Capaci ty
Volume
O A)
no change
t
l
No change
O B)
!
T
No change
0C)
!
!
O D)
!
t
!
no change
O E)
!
!
12. A 37-year-old nurse is accidentally injected wi th HIV-infected blood. After being counseled regarding the ri sks
and benefits of recei ving antivi ral prophylaxis, she begins treatment with combination anti retroviral therapy,
including zidovudine (AZT). Zidovudine is most appropriate for this patient because of whi ch of the followi ng
mechanisms of action?
0 A) Enhances host immune function
O B) Inhibits protein disulfide bond formation
O C) Limits viral caps id assembly
O D) Promotes hypermethylation of DNA
0 E) Termi nates viral DNA elongation
13. A previousl y healthy 42-year-old man has had postprandial epigastri c pain over the past 2 weeks. In the past
24 hours he has passed 3 maroon-colored stool s. Endoscopy shows a 2-cm diameter antral ulcer. Examinati on
of the ulcer ti ssue obtained on biopsy shows the presence of Helicobacterpylori. This organi sm is able to
survive in the stomach in part because it secretes whi ch of the foll owi ng substances?
o A) Bi carbonate
o B) Endotoxin
OC) Lysozyme
O D) Mucus
O E) Peptidoglycan
o F) Protease
0G) Urease
14. A previousl y healthy 37-year-old man is admitted to the hospital because of fever, chills, night sweats, and a
1.4-kg (3-lb) weight loss over the past 6 weeks. Physi cal examination shows a temperature of 38.5C (101.3F)
and a tender, raised erythematous papule on the bridge of the nose. An x-ray of the chest shows consoli dation
of the ri ght middl e lobe. A PPD ski n test is negati ve with positive controls. A bronchoscopy is performed, and
KOH examination of a sil ver-stained specimen shows a large, round budding yeast wi th a broad base
connecting the mother cell lo the daughter cell. Which of the foll owi ng is the most likel y causal agent?
o A) Actinomyces israe/ii
O B) B/astomyces dermatitidis
OC) Candida albicans
o D) Coccidioides immitis
O E) Histoplasma capsulatum
o F) Microsporum canis
O G) Paracoccidioides brasiliensis
O H) Penicillium marneffei
0 I) Pneumocystisjiroveci (formerty P. carini1)
O J) Trichophyton rubrum

15. A 45-year-old woman comes to the emergency department because of a 10-hour history of fever, chill s, and
abdominal pain that radiates to her right shoulder blade. She has a history of cholelithiasis. Her temperature is
39.6C (103.3F), pulse is 120/min, respirations are 24/min, and blood pressure is 90/60 mm Hg. Abdominal
examination shows tenderness in the right upper quadrant with guarding. Bowel sounds are decreased. Which
of the following cytokines is most likel y to be primarily invol ved in thi s patient's condi tion?
o A) lnterferon-y
O B) lnterteukin-4 (IL-4)
OC) IL-10
OD) Transforming growth factor-
0 E) Tumor necrosis factor- a
16. A 45-year-old woman has increased blood pressure, decreased urine output, and an increased serum
creatinine concentration 9 weeks after renal allograft transplantation. Which of the following elements of the
pati ent's immune system are most l ikely responsible for these cli nical changes?
0 A) B lymphocytes
o B) Basophils
OC) Natural killer cells
O D) Preformed antibodi es
0 E) T lymphocytes
17. A study is done to determine if raloxifene wi ll decrease the incidence of vertebral fractures in postmenopausal
women. The resufts are shown:
Number of women in study
group
Number of vertebral
fractures

Raloxifene Placebo
2557

2576

138
(5 4%)

260
(10.1%)

Based on these results, how many women need to be treated to prevent one vertebral fracture?
OA)
O B)
OC)
00)
0 E)

100/5.4 = 18.5
100/10.1 = 1 0
100/(10.1 +5.4) = 6.5
100/(10.1-5.4) = 21
100/(10.1 x5.4) = 2

18. A randomized, double-bl ind, controlled trial is conducted to assess the efficacy of a drug lo prevent osteoporolic
fractures in women older than 65 years. Si xty women are enrolled; 30 subjects receive the new drug (group X),
and 30 subjects rec:eive a pl acebo (group Y). Two years later, seven subjects from group X have had fractures,
and 15 subjects from group Y have had fractures (p=0.06). The investi gators conclude that there is not a
significant di fference between the treatments, and the new agent is not recommended. Whi ch of the foll owing
raises the most concern about the validity of this conclusion?

o A) Inappropri ate study population


O B)
0C)
o D)
o E)

Low external validi ty


Low power
Use of blinding
Use of a placebo

19. A 57-year-old woman wi th a ruptured appendi x has septicemia and symptoms of septic shock. Blood cul tures
grow gram-negati ve rods. Which of the following cytokines is most likely to be responsible for the shock?

o A) Interferon-a

0 B) Interferon-
O C) lnterferon-y
O D) lnterteukln-2 (IL-2)
O E) IL-4
O F) IL-7
O G) IL-10
OH) Transforming growth factor- (TGF-)
0 I) Tumor necrosis factor-a (TNF-a)

20. A 24-year-old woman is admitted lo the hospital because of a temperature of 40C (104F), general ized rash,
and al tered mental status. On admission, her pl atelet count, prothrombin li me (PT), and activated partial
thromboplasti n time (a PTT) are withi n the reference ranges. Duri ng the next 24 hours, her blood and
cerebrospinal fluid cul tures grow Neisseria meningitidis. Despite therapy, she becomes comatose and begins
bleeding from all venipuncture sites and from mucous membranes. Her PT and aPTT are now markedly
prol onged, and pl atelet count is less than 20,000/mm'. Which of the following plasma val ues is most likely to be
increased in this patient?
O A) Activated protei n C
0 B) o-di mer
OC) Factor VII (proconverti n)
O D) Fibrinogen
o E) Pl asmi nogen
21. A 61-year-old man has had weakness of the left leg for 6 months. Examination shows increased muscle tone
and reflexes in the left lower extremi ty, and an extensor plantar response on the left. Which of the followi ng
labeled sites on the spinal cord is most likely damaged?
J

ON

0 0 0 0

0 1)

O J)

22. A 51-year-old man undergoes an anterolateral cordotomy to relieve the pain of metastatic intra-abdominal
cancer. Thi s procedure is most likely to result in degeneration of spinothalamic tract axons that terminate in
which of the following labeled structures in the MRI of the normal head?

O A) 0 8) OC) O D)

O E)

23. During an experimental study, a researcher isolates a neuromuscular preparation and sti mulates the motor
nerve in the presence of a small dose of tubocurari ne to achieve a submaximal response. The resul ti ng encl
plate potential (EPP) is measured in both 0.9% saline (control) and in the presence of neostigmine
(experi mental). The EPP of the experimental group is increased in both amplitude and durati on. Which of the
following mol ecular targets is the most likely cause of this effect?
O A) Acetylcholinesterase
o B) Choline acetyltransferase
OC) Muscarinic acetylcholi ne receptor
<il D) Ni cotinic acetylcholine receptor
O E) Voltage-sensi tive sodi um channel
24. A 35-year-old woman comes to the physician because of intermittent tingling and numbness of her feet during
the past 6 months. She has a 30-year history of type 1 diabetes mellitus. Sensation ofjoint position is
decreased in the toes and ankles, and vibration sense is decreased in the feet. There are decreased ankl e
jeri<s bilaterally. Pathologic changes ofwhich of the followi ng most likely occurred in this patient?
o A) Dorsal columns of the spi nal cord
<il B) Large-diameter axons of the distal portions of peripheral nerves
OCJ Motoneurons of the lumbosacral spinal cord
o DJ Spinocerebellar tracts of the spinal cord
0 E) Spinothalamic tract neurons of the dorsal horn

25. A 72-year- old woman comes to the physician because of a 6- month history of blurring of near and distance
vision in the left eye and a small blind spot in the center of l eft-eye vi sion. Funduscopic findings are shown.
i
Whi ch of the foll ow ng is the most likely diagnosis?
O A) Glaucoma
<il B) Macular degeneration
OC) Optic atrophy
O D) Optic neuritis
0 E) Retrobul bar neuri tis
26. A 17-year-old girt develops torticollis and diffuse muscle rigidity 3 hours after starting a new drug for a
psychiatri c disorder. The most likely cause of these findi ngs is the antagonism of which of the followi ng
receptors?
0 A) a -Adrenergic

'

0 B) 13 -Adrenergi c
2

<il C) Dopaminergic (D)


O D) Muscarinic (M)

'

'

O E) Serotoninergic 1A
O F) Serotoninergic 2

27. A 16-year-old boy has a 1-week history of a pruri tic rash that has been spreading across his feet. He usually
wears gym shoes but frequentl y walks outside without shoes. Physi cal examination shows erythema, a few
tense vesicles and bullae with red bases, and mild scaling on the soles of the feet. The rash extends up the
si des ofthe feet to the ma lleoli. A photomicrograph of scrapings from the blisters and scal es is shown. The
most appropri ate therapy is topical application of which of the following?
O A) Betamethasone
O B) Diphenhydramine
OC) Metronidazol e
0 D) Miconazole
o E) Neomycin
o F) Pyrantel
28. A 40-year-old woman comes to the physician because of a 1-year hi story of chroni c l ow back pain. Exercises
to strengthen the abdominal muscles are recommended. Straight-leg raising consists of raising both lower
extremities while lyi ng flat on her back. Which of the followi ng muscl es is most directly involved in this
exerci se?

o A) Erector spi nae


O B) lliopsoas
OC) Latissi mus dorsi
0 D) Quadratus lumborum
O E) Vastus medial is

29. A 4-year-ol d boy has sl ow motor devel opment. He has a waddl ing gait and hypertrophy of the calf muscles. He
has difficul ty getting up from the floor and needs to use his arms to support himsel f. Serum creati ne ki nase
activi ty is markedly increased. Decreased concentrations of the muscle protein responsibl e for hi s condition
are most li kel y to result in which of the followi ng changes in muscle cell physiology?
0 A) Adaption of muscle protein isoforms ofthe contractile apparatus
o B) Decreased length of indi vidual muscl e fibers
o C) Increased muscl e bulk caused by muscle cell hypertrophy
0 D) Presence of thread-like structures within muscle cell s
O E) Replacement of muscle cells by connective tissue stromal cells
30. A 55-year-old man comes to the physician' s office because of a second episode of acute severe pai n in his
right big toe. Examination shows an erythematous, swollen, and exquisitely tender ri ght first
metatarsophalangeal joi nt. Administrati on of whi ch of the foll owi ng drugs is the most appropriate initi al therapy?
0A) Allopurinol
0 B) Col chicine
o C) Methotrexate
O D) Probenecicl
O E) Propoxyphene
O F) Tramadol

31. A 22-year-old woman is brought lo the emergency department because of shortness of breath and wheezing for
6 hours. Her symptoms began shortly after visiting her fri end who has cats. She has a history of simil ar
symptoms after exposure to cats. On arrival, she speaks in short sentences. Her respirations are 32/min.
Whi ch of the foll owing sets of resul ts is most likely on pul monary functi on testing?
O A)
O B)
0C)
O D)
O E)
O F)

FEV : FVC

'

Residual
Volume

!
!

Normal
Normal

32. A 78-year-old man comes to the physician because of progressive swelling of the right si de of his face and
neck and his right upper extremity. His right jugul ar vein is engorged. Which of the following di agnoses is most
l ikel y to be confinned on an x- ray of the chest?
O A) Aortic arch aneurysm
0 B) Mass in the upper lobe ofthe right lung
o C) Right cervical rib
o D) Right pneumothorax
0 E) Thoracic duct blockage in the posterior medi astinum

33. Which of the foll owing structures in the schematic of fetal ci rculation eventually causes a continuous munnur if
it fail s to close after birth?

0 A) O B) OC) O D) O E)
34. The resistance of a precapillary arteri ole is increased in a muscl e vascular bed. Which of the following most
l ikely occurs in response to this increase?
0 A)
o B)
OC)
o D)
O E)

Decreased capillary filtrati on rate


Decreased Interstiti al oncotlc pressure
Increased capillary bl ood flow
Increased capillary hydrostatic pressure
Increased vascular conductance

35. A 67-year-old man with severe congestive heart failure and edema has gained 9.1 kg (20 lb). Which of the
following is most likely to contribute to renal sodium retention?
0 A)
O B)
OC)
O D)
O E)

Decreased extracellular fluid volume


Decreased plasma aldosterone concentration
Increased effective arterial blood volume
Increased plasma atrial natriurelic peptide concentration
Increased renal sympathetic nerve activity

comes to the physician because of


36. A 55-year-old man with a 1-year hi story of recurrent ventri cular tachycardia
i
headache, dizziness, and ri nging in hi s ears. He started taking an oral ant arrhythmi c drug for his conditi on 3
3

months ago. Pl atelet count is 80,000/mm Which of the followi ng drugs is most likely responsible for these

adverse effects?

O A) Amiodarone
o B) Mexiletine
OC) Procainamide
0 D) Qui nidine
O E) Sotalol
37. Five members of a family develop severe cramping abdominal pain, vomiti ng, and diarrhea within 24 to 48
hours after eating sushi. An oxidase-positive, gram-negative, curved rod is isolated from the stools of three of
the five patients. Which of the following is the most l ikely causal organism?
O A) Escherichia coli K1
O B) Haemophllus influenzae type b

oC) Helicobacterpylori
o D) Neisseria meningiticlis serogroup B

o E) Salmonella enten1iclis
o F) Salmonella typhi

OG) Shigella sonnei


OH) Streptococcus pneumoniae
o I) Vibrio cholerae

0 J) Vibrio parahaemolyticus
15th
iSOth
}25th

6
9
12
Age (months)

6
9
12
Age (months)

lis, h
i.so,h

125th

38. A 15-month-old boy is brought to the physician because his mother is concerned about his small stature. Hi s
mother says he has had si x respi ratory infections duri ng the past year. He has a normal appeti te, eats finger
foods, and has frequent l oose stools. He wal ks independently. Examination shows a thi n, active toddl er.
Crackles are heard over both lung bases. There is no murmur on cardiac examination. Abdominal examination
shows no abnormalities. His growth curve is shown. Which of the following is the most likel y cause of these
findings?
O A) Cardiomyopathy
O B) Celiac sprue
OC) Cystic fibrosis
OD) Developmental del ay
O E) Lactose intol erance
0 F) Normal growth and devel opment

10

100

Hl staml oo (JJmoVLJ

39. The graph shows gastric acid secretion response versus concentration of histami ne admi nistered before (sol id
curve) and after (dashed curve) the admini strati on of Drug X. Drug Xis most li kel y to be which of the foll owi ng?
0 A) Atropine
O B) Cimeti dine
OC) Diphenhydramine
0 D) Mi soprostol
o E) Omeprazole
o F) Sucral fate
40. During an experiment, the following data were obtained from an anesthetized dog:
Mean glomerular capillary hydrostatic
pressure (HP )

45 mm Hg

6C

Mean Bowman space hydrostatic pressure 12 mm Hg


(HP )
Mean glomerular capillary oncotic pressure23 mm Hg
(TT )
oc

Mean Bowman space oncotic pressure


(TT )

O mm Hg

Coefficient of filtration (K)

3.0 (ml/min)/mm Hg

Renal blood flow


Hematocrit

200 ml/min
40%

'

Which of the foll owing best represents the glomerular filtration rate (in ml/min) in this animal?
O A) 30
0 8) 60
OC) 90
O D) 120
O E) 180

,,:i;.':\i'.'!',.:,;"';/:.'

. ,,. _
,.. - ; , ""..!.._t
.
!v.l'>'
...,.
..
.,. .
.,
,

-.,.,,- ,

.;
,J

'

. 'l
;._ {<.- :
, .

:,.,..<rr

.1
'. .'/. ' :1 " :- '"' ,A:t1
.
r; ,,..

-' '"'""

; f:t
"-
}-
.
,

.. ,i;. t:,::h:.tir&,
1
- ,"-,
}
!ii)l:
:
/. .t;.::2.
.
; /f> l ' J}.''-.:.:
p

'-

.i
> :
. . ;- ,\.:
.:..f;}j
. . ' ,\e._ . ,.

,,. _ : ' ..,,. . ,J., . . ,,.t.


.,,... ,..
Y-- "'' . ;.'e!
&. ,\. 1", .,"I

. .. .
,:..\.. ..

.
41. A previousl y healthy 62-year-old
woman comes to the physician because of bl ood in her urine for the past
week. A CT scan of the abdomen shows a 9-cm, sol id cysti c mass in the right ki dney. A nephrectomy is done; a
histol ogi c section of the mass is shown. Which of the followi ng is the most likely diagnosis?
...,.,r;:.'("}i"
' .):

o A) Angiomyol ipoma
O B) Lei omyosarcoma
OC) Lymphoma
o D) Metastatic small cell carci noma
0 E) Renal cell carcinoma
o F) Transiti onal cell carci noma
42. A 70-year-old man has an 18-month history of hypertension. His past health has been good and he takes no
medications. Blood pressure is 165/1 1 0 mm Hg. There are bruits in the neck and left flank. Serum electrolyte
concentrations are within the reference range, and serum creatinine concentration is 2.5 mgldl. Urinalysis is
negative for glucose, protei n, cells, and casts. Which of the following best reflects the plasma reni n
angiotensin-aldosterone profile in this patient?
O A)
O B)
OC)
OD)
O E)
F)

Renin
l
l

t
t

Angiotensin Aid
osterone
11

l
t
l

t
t
!

43. An 1 8 -year-old woman, gravida 1, para 1 , comes to the physician for a routine postpartum examination. She
says that she often has milk l eakage even when she is not ready to breast-feed her 6-week-ofd son. She
reports that this symptom is particularty severe whenever he begi ns to cry. The physician reassures her that
this is an entirely nonnal response. This reaction is most likely caused by psychogenic sti muli that promote
secretion of which of the following mediators?
o A) Dopamine

o B) Luteinizing hormone

C) Oxytocin
OD) Progesterone
o E) Prolactin

44. A 20-year-old woman who has had ol igomenorrhea for 3 years has the sudden onset of heavy vaginal bleeding.
She is moderately obese and has increased amounts of hair on the face, arms, and legs. Examination of an
endomehial biopsy speci men shows hyperplasia without atypia. Laboratory studies show increased serum
l
concentrati ons of luteinizing hormone, androgens, and estrogens; folli cle-stimu ating hormone concentration is
decreased. Histol ogi c examination of the ovari es is most likely to show whi ch of the foll owi ng?
o A) Mucinous cystadenomas
O B) Mul tipl e persistent corpora lutea
C) Mul tipl e subcortical cysts lined by granulosa cells
O D) Normal histol ogy
0 E) Serous cystadenomas

45. A male newborn delivered to a mother with type 1 diabetes mellitus at 30 weeks' gestation has profound
hypogl ycemia and is di fficult to arouse. He requires a glucose infusion. An umbilical venous catheter is utilized
as a central line for glucose administration wi th the tip directed for placement in the ri ght atrium. In order to
reach its destination, the catheter must pass through whi ch of the following vessels?
0 A) Azojgos vein
0 B) Ductus venosus
O C) Left hepatic artery
00) Right portal vein
0 E) Superior epigastric vein
46. A 60-year-old woman has a serum calci um concentration of 7.0 mg/dl and a serum potassium concentration of
2.8 mEq/L. Her serum parathyroid horm,Jne (PTH) concentration is decreased. The decreased PTH is most
l ikely related to a severe depletion of which of the following?
OA)
O B)
OC)
o D)
0 E)

Iodine
Iron
Magnesi um
Selenium
Zinc

47. A 50-year-old man comes to the physician's office for a routine heal th maintenance examination. He had
radiation therapy for trea.tment of a benign thymi c enlargement when he was 8 days old. Which of the followi ng
aspects of the physi cal examination is most l ikely to identify a long-term compli cation of this therapy?
O A) Auscultation over the cardiac apex
0 B) Auscultation over the cardiac base
OC) Auscultation over the trachea and upper ailways
o 0) Indirect laryngoscopy
o E) Inspection of the hypopharynx
O F) Nasopharyngoscopy
0G) Pal pation of the thyroi d gland
48. A 28-year-ol d woman wi th AIDS has chronic diarrhea of unknown ori gin. After the failure of typical treatment
approaches, subcutaneous injecti ons of octreotide are begun. Which of the followi ng best describes the
therapeutic benefit ofthis structure analogy of somatostatin as compared with nati ve somatostatin?
0 A)
0 B)
O C)
00)
O E)

Decreased lipid solubility


Decreased volume of distri bution
Delayed onset of acti on
Increased bioavailabi lity
Prolonged duration of action

49. A 65-year-old man with chronic obstructive pulllilonary disease has had a "dry mouth" since starting
bronchodilator therapy 2 weeks ago. The sympt,om is not severe enough to discontinue the medication. Whi ch

of the followi ng bronehodilator!. i!. the pati ent rno!.t likely taki ng?

0 A) Albuterol
0 B) lpratropium
OC) lsoproterenol
o D) Terbutaline
o E) Theophylli ne

NBME form 4-2


2. The cystic fibrosi s transmembrane conductance regulator (CFTR) consists of a si ngle peptide chain wi th
transmembrane segments and extracellular and intraceTiular regions. Which of the foOowi ng peptide segments of
CFTR is most likely located in a transmembrane segment?
Thr-Glu-Lys-Glu-lle
O A) Asp-Val-Leu0 B) Glu-Glu-Thr-Glu-Glu-Glu-Val-Gln
OC) lle-Phe-Val-Leu-lle-Tr'frCys- Leu
O D) Lys-lle-Lys-His-Ser-Arg-Gl y-Arg
0 E) Ser-Thr-Gly-Al a - Gly-Lys-Thr-Ser

3. A 10-year-old boy with l ight blond hair, pale ski n, and significant mental retardation (IQ of 50) is most l ikely to
have a defect at which of the followi ng poi nts in the bi ochemical pathway?
Thyroxine (T4)

ta C
l
Homogentlslc Ac d
A

Phenylalanine - Tyrosine - DOPA - Melani n

lE

Acetoacetic Acid

0 A) O B) O C)

OD) O E)

4. During the first meiotic divi sion of human oogenesis, nondisjunction occurs between a pair of homologous
chromosomes. If fertilization occurs, which of the followi ng is the possible number of chromosomes contributed
by the female pronucleus?
OA) 21 or23
o B) 21 or 25
OC) 22 or 23
O D) 22 or 24
0 E) 23 or 24

5. A 44-year-ol d man is brought to the emergency department because of a 5-hour hi story of severe epigastri c pain
that radiates to his back. He drank approximatel y twel ve 1 2 -ounce cans of beer before his pain began.
Abdominal exami nation shows distention wi th generalized tenderness and no masses. Hi s serum amyl ase
activity is 500 U/L, and serum li pase activity is 325 U/L (N=14-280). Which of the followi ng types of necrosis is
most likely in this patient's retroperi toneum?
O A) Caseous
o B) Coagulative
0C) Fat
0 D) Fibrinoi d
0 E) Liquefactive

6. A 17-year-old girl has hemophilia A. Her older brother also has hemophilia A, but neither oftheir parents is
affected. Which of the following is the most likely expl anati on for the development of hemophilia in this gi rl?
o A) Aneuploi dy
0 B) Balanced autosomal translocation
o C) Inheritance of defective factor VII I alleles from both parents
0 D) Skewed X -chromosome inactivation
O E) Unequal recombination

7. Arl 1 8 -year-old man undergoes removal of 75% ofthe li ver following a gunshot wound. In the remaining
hepatocytes, the first change in cell cycle di stribution wi ll be movement of cells between which of the following
phases?

0 A) G to G

O B) G to S

OC) Sto G

'

O D) G to M
O E) Mto G

8. A 26-year-ol d woman has sepsis that resul ted from self-i nflicted slash wounds to the forearm. Some ofthe staff
feel emotionally attached to her; others describe her as a difficul t patient. She is labil e and impul si ve. She has a
history of many psychiatric hospitalizations and abuse of multiple drugs. Which of the following is the most likely
psychiatri c disorder?
0 A) Adjustment disorder
o B) Anxi ety disorder
OC) Bi polar disorder
0 D) Borderli ne personali ty disorder
O E) Schizoid personality disorder


9. A 48-year-ol d man wi th advanced metastatic gastric cancer is admi tted to the hospital because of severe
abdominal pain. High doses of oral analgesi cs, including numerous opioids, nonsteroidal anti-i nflammatory drugs,
and tricyclic agents, have not been successful in managing the pain. Morphi ne admini stered by intravenous
infusi on successfully controls his pain, but the doses needed cause respi ratory depressi on. He has repeatedly
stated that he does not 'Nant cardiopulmonary resuscitation and that he wi shes 'lo die wi th dignity without
unnecessary pain." Which of the following is the most appropri ate course of acti on?
0 A) Administer the lowest dose of morphine that relieves pain, even if it depresses respi rati on
O B) Decrease the dose of morphine and admi nister an anxi ol ytic
OC) Decrease the dose of morphine, explai ning that he wi ll have to tolerate some pain
0 D) Discontinue morphine and try di fferent anal gesi cs
O E) Increase the dose of morphine progress vely until the patient stops breathing
i

10. A healthy 50-year-old man who resides al sea level takes a 45-minute ride in a cable car from near sea level to
a ski resort (altitude 3050 m [10,000 fl)). Which ofthe following label ed points on the diagram best represents
this patient 30 minutes after arrival at the resort?
40

::,SO

30

.. 10

JO

0
20

,0

- -

,.

' '

7,4

pH
A IX/!lI

,. ,..

O A) O B) O C) O D) 0 E) O F) O G)

1 1 . A 70-year-old man begins taking a drug to prevent angina pectoris. N. his next visit, his pulse has decreased
from 70/min to 60/min, and his blood pressure has decreased from 130/90 mm Hg to 1 10/70 mm Hg. The drug
most likely to be responsible for these changes is which of the following?
O A) Captopril
o B) Ni fedipi ne
OC) Nitroglycerin
o D) Prazosin
0 E) Propranolol

-... --.

c.i,

,
.,

'
1,1., d'' ) \

""" ...,..,:l .

."4,
...
---,..
..._":. ......
"'
..
f..
..
. ,, \1,,&' .....
"' ..

....

..
.. ....
-t1-5!'':

)'
..; .
,.
.-.
!!

.., . .. ..

..-:
-..
..
... ) J L, .
12. A 36-year-old woman who underwent a kidney transplantation 2 months ago devel ops fever, malaise, myalgias,
and leukopenia. Medicati ons include prednisone and cyclospori ne. An x-ray of the chest shows bilateral,
interstiti al pulmonary infiltrates. Over the next 2 days, her respiratory status deteri orates, and she is intubated
and mechanically venti lated. Ti ssue obtained on biopsy of the lung is shown. The most appropriate treatment
includes administration of which of the following drugs?

o A) Acyclovir

0 B) Ganciclovir

o C) lnterferon-y

0 D) Lamivudine (3TC)
0 E) Pl econaril

13. Which of the following components of the bacterial cell envel ope exhibits endotoxin activity?
0 A) Lipid A
o B) Lipopeptide
O C) Lipoprotein
OD) Phospholipid

14. A 30-year-old woman has had sexual intercourse on many occasions with men infected with HIV but has not
become infected. Which of the followi ng is the most likely explanation for her resistance to HIV infection?
0 A) Chemokine receptor 5 deficiency
0 B) Congenital asplenia
OC) Hyper lgM syndrome
O D) Paroxysmal nocturnal hemoglobi nuria
o E) Thalassemia

15. Acti vation of cytotoxi c T l ymphocytes to eliminate virus-infected cells requires which of the following?
Interaction of Antigenic
Combined With Signaling
Peptide With
From
O A)
Class I MHC
interferon-y
interl euki n-2 (IL-2)
Class I MHC
0 8)
OC)
Class I MHC
IL-4
Class II MHC
interferon-y
O D)
Class II MHC
O E)
IL-2
Class II MHC
O F)
IL-4

16. After a motorcycle collision, a 23-year-otd man who is being transfused with erythrocytes and fresh frozen
plasma has a systemic anaphylactic reaction. He has a history of recurrent respiratory tract infections. Blood
hemoglobi n concentration is 6.5 g/dl, and circulating B and T lymphocyte counts are normal. Which of the
following is the most likely explanalion of these findings?
O A) AIDS

o B) Common vari able immunodeficiency

o C) Drug-induced immunodeficiency
0 D) lgA defici ency
0 E) X-linked agammaglobul inemia

17. The table reports survival of patients who had an operation for a particular form of cancer.
Number of Patients Number of Patients Percent of Patients
at the Beginning
who Died
Surviving
Interval
of the Interval
During the Interval
This Interval
0-1
115
300
62
year
1-2
185
37
80
years
2-3
24
84
148
years
3-4
85
124
18
years
4-5
25
76
106
years
If a pati ent survives 2 years after the operation, whi ch of the followi ng is the probability of surviving at least 4
years?
OA)
O B)
OC)
0 0)
O E)

.80
.85
.84 x .85
.62 X .80 X .84 X .85
.85- .80

18. An investigator conducts a study to assess the effectiveness of altering school lunches for improving serum
l ipid concentrations in middle school students. A low-fat diet is compared to the standard lunch. In order to
mini mize changes in the lunch habi ts of the students who will be given the low-fat diet (study group), the lower
fat l unch is given to five middle schools on the north si de of the ci ty. The control group consists of five school s
on the south si de of the ci ty where the students will continue to receive the standard lunch. The resul ts indicate
that students in the study group have a significant decrease in their average serum cholesterol concentrati ons
compared to the students in the control group who have an average increase in serum cholesterol
concentrati ons. Which of the foll owi ng raises the most concern about the validity of the study results?
0A) Bi ased measurement
o B) Lack of generali zabi li ty
OC) Lack of a pl acebo
0 D) Lack of randomization
O E) Low power
19.

A healthy person is given intravenous doses of inul in and '"'I-label ed albumin. The inuli n space is calculated as
1 5 L and the '"I-albumin space as 3 L. The interstiti al fluid vol ume in this person (in L) is approximately which of
the following?

O A)
O B)
0C)
O D)
O E)

3
10
12
15
18

20. The sickling of erythrocytes in pati ents with sickle cell disease results from which of the following?
0 A) Aggregation and polymerization of the sickl e cell hemogl obin (HbS)
o B) Inability of HbS to bind heme
O C) Inability of the HbS molecules to form a tetramer
o D) Instability of the HbS molecules

21. A 72-year-old man comes to the emergency department because of a 2-hour episode of loss of vi si on in the left
eye. He has never smoked. Laboratory studies show:
Hemogl obi n
Hematocrit

20 g/dl

62%

Leukocyte count 14,200/mm'


Differential
normal
Mean corpuscular _
87 2 I.Im'
volume
Platelet count

375,000/mm

Whi ch of the following is the most appropriate initi al therapy?


0A) Admi nistration of allopuri nol
OB) Administration of aspirin
OC) Administration of chlorambucil
O D) Administration of hydroxyurea
OE) Administration of warfarin
0 F) Phlebotomy
OG) Splenectomy

22. A 52-year-old woman has medi al strabismus of the right eye; she is unable to move this eye beyond
midposition during attempted horizontal gaze to the right. Which ofthe following cranial nerves labeled on the
ventral surface of the brain stem is most likely to be damaged?

O N 0 0 0 0 0 0 0 0 1) 0

23. A 56-year-old man with alcoholi sm comes to the emergency department because he is unable to raise the
fingers of his left hand. He is afraid that he has had a slroke. He tells the physici an that he fell asleep the night
before al his favorite bar with his arm across the arm of the chair. Which of the following nerves is most likel y
affected in this patient?

o A) Brachia!
O B) Medi an
O C) Musculocutaneous
0D) Radial
O E) Ulnar

24. A 27-year-old woman suddenly develops headache, followed by coma and death. Subarachnoid hemorrhage is
found at autopsy. Whi ch of the followi ng sites in the central nervous system is most likely to be involved?
O A) Cerebral and cerebellar white matter
0 8) Circle of Willis
OC) Dura mater
O D) Internal capsul e
o E) Occipital cortex

25. A 45-year-old man has had four episodes of involuntary twitching of the right foot. Following the last episodes,
he had a tonic-clonic seizure. Which of the following structures on the left is the most likely origin of the
seizure?
OA) Inferior frontal cortex
0 B) Inferior temporal cortex
OC) Insular cortex
0 0) Primary motor cortex
o E) Supplementary motor cortex

26. A 15-year-old boy comes to the physi cian because of nervousness. He tells the physician that he is afraid he is
"going crazy." He describes brief episodes when colors have appeared especially bright and sounds have
seemed louder than usual. He says that several times whi le sitting in cl ass, the desks appeared to move. When
asked, he adds that he frequently skipped school l ast term and had bad grades. He says, "But now l'Ve changed
and I'm attending regularly." Physical examination and laboratory studies show no abnormalities. Which of the
foll owing is the most likely cause ofthis patient's condition?

o A) Alcohol use
O B) Bipolar disorder
OC) Hallucinogen use
O D) Mi graine
0 E) Schizophreni a

27. An experi mental animal is treated with an agent that disrupts desmosomes (maculae adherentes). Which of the
followi ng is most likely to be decreased?

oA) Adhesion of epidermal cells to the basal l amina

0 B) Cohesion between adjacent keratinocytes


OC) Number of melanocytes
O D) Proli feration of keratinocytes
o E) Thickness ofthe stratum comeum

28. A 21-year-old man develops wristdrop afler sustaining a complete fracture of the upper arm associated with
extensi ive ti ssue damage. The fracture was reduced successfully, and the ends of vessels and damaged nerves
were apposed and surgically rej oined. Despite physical therapy, the extensor muscles have a decreased
volume and can contract only weakly. Wound healing and flexion are g,ood. The most li kel y cause of this
condi tion is abnormal function of which ofthe followi ng?

o A) Circul ati on
o B) Glucose transporters

O C) Lymphatic drainage
0 D) Neurotrophic input
0 E) Tendon stretch receptors

29. A 1 2 -year-old boy has had persistent pain in the arm since he fell while playi ng basketball 2 weeks ago. His left
del toid muscle is paralyzed, and he is unabl e to hold his left arm in abduction against resistance. Which of the
following bones is the most likely site of a fracture?

o A) Acromion

O B) Clavicle
OC) Coracoid process
O D) Deltoid tuberosity
O E) Radius
0 F) Shaft of the humerus
0G) Surgical neck of the humerus
O H) Ul na

30. A 77-year-old man has had repealed episodes of difficuHy speaking, facial numbness, and clumsiness ofthe l eft
hand. Doppler ul trasonography of his carotid arteri es shows no abnonnali ties. Which of the followi ng drugs is
the most appropriate therapy to prevent future episodes?

0A) Aspiri n
0 B) Diflunisal
OC) Methyl sali cyl ate
o D) Olsalazi ne
O E) Salicyl ic acid
0 F) Salsalate
OG) Sodium sal icylate
O H) Sul fasalazi ne

,,
...
),
/.. i,

1'
.
.
t't ....
.. Ii} ,.,' \.' .:. .
,,,,

' .. i
..

....,. .

"

'

i!P,'
"':,..

'1P'
...

..

... .

31. Which ofthe following diseases is most likely to affect the primary function of the tissue in the histol ogic
section shown?
O A) Asbestosis
O B) Asthma
0C) Dyski nesis
o D) Lobar pneumonia
O E) Si licosis

32. An otherwise heal thy 45-year-old man has a 2-year hi story of heavy snoring and daytime sleepiness.
Monitoring shows that his arteri al oxygen saturation periodi cally decreases to 87% when he sleeps. Ifthe
man's condition is left untreated, which of the followi ng complications is most likely?
0 A)
O B)
O C)
o D)
0 E)
0 F)

Dilatation of the mitral annulus


Microcyti c hypochromi c anemia
Normocytic normochromic anemia
Respiratory alkalosis
Right ventri cular hypertrophy
Syndrome of inappropri ate ADH (vasopressin) release

33. A 36-year-old man has a generalized tonic- cl oni c seizure after 4 weeks of isoni azid therapy for tuberculosis. He
has no history of seizure di sorders. Which of the following is most likel y responsible for the devel opment of
seizures in this patient?
0 A)
0 B)
0C)
o D)
O E)

Depletion of ascorbic acid from the adrenal gland


Depletion of norepi nephri ne from neurons
Depletion of pyridoxal phosphate from neurons
Nonspecific bl ockade of y-aminobutyric aci d (GABA) receptors
Stimulati on of glutamic acid decarboxylase

!!

1 00

IQ
1.40
Perfugl on Qfeuure
(mm Hg)

34. The graph shows coronary bl ood flow as a function of perfusi on pressure. The solid line indicates the
relationship between bl ood flow and pressure when the coronary arteri es are maximally dilated. Which of the
following pairs of lettered points best illustrates the change that occurs when a healthy person at rest begi ns to
run to catch a bus?
OA) W - Y
O B) X - Y
0 C) X - Z
O D) y ..... w
O E) z __, Y

35. A 78-year-old man with syncope feels heaviness in his chest with exertion and breathlessness when lying
down. A crescendo/decrescendo systolic murmur is heard best at the second right intercostal space with
radiation to the carotid arteries. Which of the followi ng measurements of left ventricular and aortic pressure is
most likely to be found on cardiac catheterization?

Left
Ventricular
OA)
O B)
O C)
00)
O E)

(mm Hg)
100/10
100/18
150/10
150/18
150/18

Aortic

(mm Hg)
150/90
100/50
150/80
100/50
150/80

36. An 18-month-old boy is brought to the physi cian's office for a foll ow-up cardi ac exami nation. He has a palpable
thrill and a harsh, grade 4/6 hol osystol ic murmur that radi ates over the precordium and is heard best at the
l ower left sternal border. Whi ch of the foll,owing is the most l ikely diagnosis?
0A) Mi tral valve regurgitati on
0 B) Ostium primum defect
OC) Ostium secundum defect
o D) Patent ductus arteri osus
O E) Sinus of Valsalva fistula
0 F) Ventricular septa) defect

37. A 74-year-old man with a 5-year history of pernicious anemi a develops a gastric ulcer on the lesser curvature
of the fundus. Bi opsy of the lesion shows adenocarci noma. Microscopic examination of the adjacent gastric
mucosa is most likely to show absence ofwhich of the following cells?

o A) Chief cells
o B) Mucous neck cells

0C) Parietal cells


o D) Small B lymphocytes
O E) Transformed B lymphocytes

38. A 65-year-old woman wi th chronic constipation comes lo the emergency department because of a 3-day history
of abdominal pain. Abdominal examinati on shows distention wi th tenderness in the left lower quadrant.
Sigmoidoscopy shows no tumors, but the scope cannot be inserted beyond 15 cm because of abrupt luminal
narrowing. Which of the followi ng is the most likely di agnosis?

o A) Appendicitis

o B) Diverticulosis
0C) Small-bowel obstruction
O D) Vol vulus of the cecum
0 E) Vol vulus of the sigmoid colon

39. An 87-year-ol d woman who has been taking ibuprofen for the past 5 years for pain associated wi th rheumatoid
arthritis comes lo the physician because of a 3-day history of epi gastri c pain. Her temperature is 37C (98.6
F), pulse is 74/min, and blood pressure is 130/88 mm Hg. Hemoglobin concentration is 13.4 g/dl. There is mild
epigaslric tenderness. Esophagogaslroduodenoscopy shows mild gastritis. She does not want to discontinue
the ibuprofen. Which of the followi ng drugs is most appropri ate to add to her current regimen?

o A) Aluminum hydroxide
o B) Bi smuth subsal icytate

OC) Calcium carbonate


0 D) Fentanyl
o E) Magnesium oxide
0 F) Misoprostol
OG) Nitroglyceri n
0 H) Propylthiouracil

40. The dashed lines in the lettered diagrams show the changes in diameter in afferent and efferent arterioles (solid
lines show a normal diameter). Which ofthe following will result in increased renal plasma flow and increased
glomerularfiltration rate?

O A) O B)

0C)

OD) O E)

41. A 21-year-old woman is evaluated for renal insufficiency. A photomi crograph of a structure seen on uri nalysis is
shown. This finding is most consistent with whi ch of the foll owi ng diagnoses?
0 A) Acute cystitis
o B) Hepatitis B
OC) Mi nimal change di sease
O D) Poslinfecli ous glomerulonephrilis
o E) Renal amyl oidosis

42. A 30-year-old woman comes to the emergency department because of acute sharp flank pain and bl ood in her
urine. An intravenous pyelogram obtained at 30 mi nutes is shown. Which of the following is the most likely
diagnosis?
o A) Appendicitis
o B) Bladder tumor
OC) Cystitis
0 D) Kidney tumor
0 E) Pyel onephritis
0 F) Right ureteral obstruction

43. An obese 52-year-old woman comes to the physi cian because of vaginal bleeding. Her last normal menstrual
period occurred 6 months ago. Biopsy shows endometrial hyperpl asia. This endometrial stimulati on is most
likely due to peri pheral aromatization of which of the following hormones?

o A) Aldosterone

0 B) Androstenedi one
OC) Follicle-stimulating hormone
O D) Luteini zi ng hormone
o E) Progesterone

44. A 56-year-old woman comes to the physician's office for her first heal th maintenance examination in the past 5
years. She has a 2 x 2.5-cm lesion 4 cm from the nippl e in the outer upper quadrant of her left breast. The
l esion is nontender and nonmobil e and has ill -defined borders. There is no discharge on compression of the
nipple. Which of the followi ng is the most likely expl anati on for these findings?
0 A) Adenocarcinoma
o B) Beni gn lymph node enlargement
OC) Fibroadenoma
O D) Fibrocystic disease
O E) Metastatic carcinoma

45. A 20-year-old woman feel s sad, cries occasionall y, and doubts her abil ity lo lake care of her healthy ay-old
newborn. She has started to breastfeed, and her sleep and appetite are normal. Which of the following is the
most li kel y explanation for her feeli ngs?
OA) Adjustment disorder with depressed mood
0 B) Brief reactive psychosis
OC) Major depressive episode
0 D) Normal reaction to bi rth
O E) Postpartum pituitary necrosis

46. A 64-year-old man comes to the physician' s office for a health maintenance examination. He has been unable
to sustain erections during intercourse or masturbation for the past month. He does have spontaneous
nocturnal and morning erections. He had a myocardial infarction 2 months ago. Whi ch of the followi ng is the
most likely explanation for his current erectile disorder?
0 A) Adverse effect of antihypertensive medi cation
o B) Mered body image
OC) Decreased libido
o D) Decreased vascular perfusion
0 E) Fear of sudden death

47. A 54-year-old woman comes to the physician because she has noticed that the front of her neck appears
swoll en. Serum thyroxi ne (T) concentration is 16 g/dl, and serum thyroid-stimulating hormone concentration is
0.1 U/ml. Whi ch of the followi ng is the most l ikely diagnosis?

o A) Cretinism
e B) Diffuse toxic goiter (Graves disease)
OC) Myxedema
0 D) Si mple goiter
o E) Thyroiditis

48. Female pseudohermaphroditism is most often due to a deficiency of which of the following enzymes invol ved in
corti sol bi osynthesis?
o A) 20,22-Desmolase
O B) 17a-Hydroxylase
OC) 18-Hydroxylase
0 0) 2 1 -Hydroxylase
o E) 3()-Hydroxysteroid dehydrogenase

49. An unconscious 1 9 -year-old man is brought to the emergency department by hi s roommate. They had been
smoki ng an "unusual weed" together when he became delirious, then began hallucinating and l ost
consciousness. He cannot be aroused. He has fever, tachycardia, di lated pupils, facial flushing, and dry mouth
and skin. Which ofthe following drugs is most likely to reverse both the central and peri pheral effects in this
patient?
o A)
0 B)
OC)
0 D)
o E)

Hexamethonium
Neostigmine
Physostigmine
Propantheline
Scopolamine

NBME form 4-3

7. A 51-year-ol d man has a chronic cough and night sweats. Examination of lymph node tissue obtained on
transbronchial biopsy shows nodul es containing histiocytes and multi nucl eated giant cells. The abnormal
appearance of these cells is most likely the result oflhe action ofwhich ofthe following?
O A)
0 B)
OC)
O D)
o E)
O F)

Fibroblast growth factor on fibroblasts


lnterferon-y on macrophages
lnter1eukin-1 (IL-1) on macrophages
lnter1eukin-1 (IL-1) on T lymphocytes
lnter1eukin-2 (IL-2) on T lymphocytes
Platelet-derived growth factor on fibroblasts

8. A 32-year-ol d man di es after an episode of massive upper gastrointestinal bleeding. He had episodes of facial
flushing and diarrhea for the past year. A yellow submucosal nodule is identified in the termi nal ileum. The l iver as
seen at autopsy is shown. Which of the following features of the neoplasm's stage is demonstrated?
O A)
0 B)
OC)
0 D)
o E)

Bl oodbome metastases
Direct seeding
Lymphatic spread
Peritoneal implants
Si ze of the primary lesion

II

lll
A1N-1Mfem,1lt
AtWmJlt
U'.<<A:..-C
01.e,:11tm.i1t

9. A couple, 11,2 and 11,3 in the pedigree shown, has just learned that thei r 3-year-old son has fragile X syndrome.
The woman's sister-in-law is currently pregnant, but the couple does not want to tell anyone in their famil y about
the diagnosi s. Which of the following actions by the physici an is most appropriate concerning informing the
sister-in-law?
0 A) Ask for a third party to mediate the communication
o B) Insist that the couple contact the pregnant woman
0C) Keep the diagnosis confidential
0 D) Offer to contact the pregnant woman directly
0 E) Recommend family counseling

15. A dilute disinfectant kills 90% of a bacterial inoculum in 1 0 minutes. Which of the following is the shortest

amount of time (in minutes) required to reduce an inoculum of 1 0 bacteri a to less than 1 0 bacteria?
O A) 1 0
O B) 50
0 C) 100
O D) 140
O E) 180
16. A previousl y healthy 25-year-old woman is brought to the emergency department 30 minutes after a motor
i
vehicle coll sion in which she sustained blunt abdominal trauma. Abdominal examination shows marked
tenderness in the left upper quadrant. During an exploratory operation, the spleen is found to be l acerated and
is removed. Which of the following vaccines shoul d be administered to this patient prior to discharge?
0 A) Diphtheri a-tetanus toxoid
o B) Hepatitis B

o CJ lnfluen..:<1 virus

O D) Measles- mumps-rubella
O E) 23-Valent pneumococcal

Episodes ol uri nary tract


inlections

17. Which of the following percentages of chil dren in this sample had at l east two urinary tract infections?
OA) 10%
O B) 25%
OC) 30%
O D) 35%
0 E) 45%
O F) 75%
OG) 100%

18. Twenty chi ldren are treated in the emergency department for bloody diarrhea. To study this outbreak, the health
department interviews 40 chi ldren wi thout bl oody diarrhea matched by age and neighborhood. The study shows
that 18 of the 20 children wi th bloody diarrhea went to the local county fair and 10 of the 40 children who did not
have bloody diarrhea went to the fair. Which of the following best describes this study design?
0 A) Case-control
o B) Case series
OC) Crossover
O D) Cross-sectional
O E) Hi stori cal cohort
0 F) Prospecti ve cohort
o G) Randomized clinical trial

19. Carbon dioxide is transported from peri pheral tissues to the lungs principally in which of the following forms?
0 A) Bi carbonate
O B) Carbamino groups of hemoglobin
OC) Carbonate
O D) Carbonic acid
O E) Dissol ved carbon dioxide

20. An 1 1-year-ol d boy has a mass in the jaw and cervical lymphadenopathy. A peripheral blood smear shows
abnormal vacuolated lymphoid cells. nssue obtained on biopsy ofthe jaw mass is shown. Which of the
following is the most likely diagnosis?
0 A) Burkitt lymphoma
0 B) Chronic lymphocytic leukemia
O C) Hodgkin disease
0 D) Mycosis fungoides
o E) T-lymphocyte lymphoblastic leukemia
O F) WaldenstrOm macroglobulinemia

21. Two hours after the onset of chest pain, a 48-year-ol d man comes to the emergency department. An ECG
indicates acute myocardial ischemia. He recei ves tissue plasminogen activator as thrombolyli c therapy. Which
of the followi ng adverse effects is most likely?
0 A) Acute allergic reaction
0 B) Hemorrhagic stroke
OC) Hypertensive crisis
O D) Pulmonary embolus
o E) Tonic-clonic sei zure

22. The termination of the pathway that ori ginates in the l ateral part of the left retina and travels through the left
l ateral genicul ate occurs at which of the followi ng labeled cortical areas in the photograph ofthe brain?

O A) O B) OC) OD) O E) 0 F) OG) OH) 0 I) O J)

23. A 34-year-old man has persistent weakness of abduction and external rotati on of the upper extremity 6 weeks
after fracturi ng the surgical neck of the humerus. Whi ch of the followi ng nerves is most li kel y inj ured?
0A) Axillary
o B) Long thoraci c
o C) Musculocutaneous
O D) Radial
o E) Suprascapul ar

24. A 26-year-old woman has had severe intermittent headaches si nce being involved in a motor vehicle colli si on 3
weeks ago. Physical examination is normal. An unenhanced CT scan of the head is shown. Whi ch of the
followi ng is the most l ikely diagnosis?
O A) Epidural hematoma
o B) lntracerebral hemorrhage
OC) Pri mary brain neoplasm
0 D) Subarachnoid hemorrhage
0 E) Subdural hematoma

25. In a patient with a movement disorder, positron emi ssion tomography (PET), using a radioactive ligand for the
dopamine 1 (D) receptor, shows a marked reduction in this receptor in the structure indi cated by the arrow in

'

the MRI of the normal head. Whi ch of the followi ng is the source ofthe innervation of this structure by
dopamine-containing axons?
O A) Globus pallidus
0 B) Motor cortex
OC) Motor thalamus
0 D) Stri atum
O E) Substantia nigra

26. Three days after undergoing an appendectomy, a 49-year-old man becomes agi tated and confused. He is
diaphoretic and disoriented to time and tells the physician that a giant crab is about lo grab him. His
temperature is 37.3C (99.2F), pulse is 1 1 0/min, and bl ood pressure is 176/98 mm Hg. His abdomen is
nontender and there are occasional bowel sounds. Which of the following is the most likely cause of this man' s
condi tion?

o A) Acute stress di sorder

0 B) Cerebral infarction
OC) Del irium tremens
O D) Fat embolism
O E) Peritoni tis

27. Which of the following disorders is the most likely effect of impaired neural crest migration o n cutaneous
development?
0 A) Albinism

o B) Hypoplastic dermal papillae

OC) lchthyosis
o D) Supernumerary sweat glands
o E) Vascular ectasia

28. A 24-year-old man is brought to the emergency department followi ng an automobile collision in which his knee
struck the dashboard with considerable force. On examination, his ti bia moves posteriorly in relation to the
femur. Which of the following ligaments is most likel y to have been tom?
O A) Anterior cruciate
o B) Lateral collateral
OC) Medial collateral
O D) Obl ique popliteal
o E) Patellar
0 F) Posterior cruciate

29. A 71-year-old woman wi th temporal arteri tis has had fever, malaise, and stiffness in her shoul ders and hips
over the past 3 weeks. Which of the following is the most likely diagnosis?
O A)
o B)
OC)
0 D)
o E)

Fibrositis
Polymyalgia rheumatica
Polymyositis
Rheumatoid arthritis
Sjogren syndrome

30. A 54-year-old man comes to the physician because of a 3 -month hi story of severe pain that radiates from his
right thigh to the bottom of his right foot. He has a 1 - year history of chronic low back pain. Use of ibuprofen has
not relieved the pain. Sensation to pinpri ck is decreased over the lateral aspect and pl antar surface of the right
foot. There is diminished strength of right foot plantar flexi on. Which of the followi ng is the most likely cause of
this patient' s condi ti on?
o A) lntervertebral di sc herniati on
o B) Sacrococcygeal teratoma
OC) Sacroil iiti s
O D) Transverse myel itis
0 E) Vertebral body compression fracture
O F) Vitamin B (cobalami n) defici ency

"

31. A 60-year-old man recovers from bronchopneumonia followi ng appropri ate anti biotic treatment. Which of the
foll owing types of cells are most likely responsible for the repair of the alveolar epithelium in this patient?

o A) Alveolar macrophages
O B) Capillary endothelial cells
OC) Goblet cells
o D) Pulmonary interstitial cells
0 E) Type II pneumocytes

32. A 19-year-old man is brought to the emergency department because of the sudden onset of sharp chest pain
and difficul ty breathing 3 hours a,go. Temperature is 37C (98.6"F), pulse is 1 1 0/min, and respi rations are
36/min and shallow. He is slightly cyanotic. Examination shows deviation ofthe trachea to the right and
subcutaneous crepitus. Which of the following findings is most likely in this patient?
0 A)
0 B)
OC)
O D)
0 E)
o F)

Dullness on percussion on the right


Hyperresonance on percussion on the left
Increased breath sounds on the l eft
Increased breath sounds on the right
Increased vocal fremi tus on the left
Whispered pectoril oquy on the right

33. A 26-year-old woman with stable asthma wheezes when exposed to cats. Which of the following is the most
appropriate prophylactic therapy?
0 A)
o B)
OC)
O D)
o E)

Inhaled cromolyn
Inhaled ipratropium
Oral cimetidine
Oral corti costeroids
Oral theophylli ne

34. The largest proporti onal increase in left ventricular afterl oad and myocardial oxygen consumpti on is most likely
to occur following a 20% increase in whi ch of the followi ng?
0 A) Aortic pressure
0 B) Coronary bl ood flow
o C) Heart rate
o D) Right atrial pressure
O E) Stroke volume

35. A 56-year-old woman dies in a motor vehicle accident. At autopsy, a stenotic mitral valve with shortened,
thickened chordae tendineae is found. This finding is most likely to be due to a past infection with which of the
following?
O A}
0 B)
O C)
OD)
O E)

Coxsackie B vi rus
Group A l3-hemolytic streptococcus
Staphylococcus aureus
Treponema pallidum
Viridans streptococci

36. A 52-year-old man has intermittent episodes of supraventricular tachycardi a. During these episodes, he has a
pul se of 190 to 200/min and a systolic blood pressure of 60 to 80 mm Hg. Which of the foll owing is the most
likely cause of the hypotension?
O A)
0 B)
o C)
00)
O E)

Depressed myocardial contractility


Erroneous bl ood pressure readings
Inadequate sympathetic tone
Inadequate ventricular filling
lntravascular volume depleti on

37. A 75-year-old man has intermi ttent crampi ng pain in his right groin. Exami nation shows a large inguinal herni a
that extends into the scrotum. Repair is recommended. During the operation, which ofthe following arteries is
the most useful landmark for locating the deep inguinal ring?
O A)
O B)
0C)
O D)
O E)

External iliac
Femoral
Inferior epigastric
Obturator
Superior epigastric

38. A 25-year-old woman bri ngs her 1-month-old son to the physi cian because he has been vomiting with
increasing forcefulness during or immediatel y after his feedings for the past week. The infant has no fever and
seems relatively content but hungry after vomiting. He has not had a bowel movement for the past 2 days. The
mother is concerned because the infant's father had a si milar problem at this age and had to have an operation.
Physical examination shows normal chest sounds and a palpable ovoid mass in the upper abdomen. Which of
the following is the most likely cause of his condition?
o A) Achalasia
O B) Congenital duodenal or jejunaI atresia
OC) Congenital megacolon (Hirschsprung disease)
0 D) Congenital tracheoesophageal fistula
O E) Diaphragmatic herniati on
0 F) Pyloric stenosis

39. A 17-year-old boy comes to the physician because of intermittent nausea and vomiting of undigested, bile
stained food for the past 3 months. He has a 10-year history oftype 1 diabetes mellitus. Findings on physical
examination are unremarkable. Hemoglobin A is 18%, and fasting serum glucose concentration is 180 mg/dl.
,c
Which of the following is the most l ikely cause of hi s symptoms?
0 A) Autonomic dysfuncti on
O B) Chronic gastri tis

o C) Gastric carcinoma

o D) Microangiopathy
O E) Peptic ulcer disease

40. Autopsy of a female newborn who died 1 day after birth shows the absence of ki dneys. Rudimentary ureters
are present bilaterally. Ul trasonography at 28 weeks' gestation would most likel y have shown which of the
following?
0 A) Absence ofthe bl adder
0 B) Bilateral gonadal dysgenesis
OC) Multipl e renal arteries bilaterally
0 DJ Oligohydramnios
O E) Polyhydramnios

41. Serum osmolality, uri ne osmolality, and urine sodium concentrati on are measured in six patients who have
identi cal height and weight. Which of the following sets of values was most likel y obtained from a patient having
inappropriate ADH (vasopressin) secretion?

0 A)
O B)
OC)
O D)
O E)
O F)

Serum
Urine
Osmolali ty
Osmolality
(mOsmol/kg) (mOsmol/kg)
270
270
275
275
300
350

50
550
50
550
280
400

Urine Na'
(mEq/L)
60
60
10
10
60
10

42. A 50-year-old man has had intermittent flank pain and hematuria over the past year. An x-ray of the abdomen
shows an abnormality in the pelvi s of the left kidney. The surgically resected ki dney is shown. Which of the
following is most likely associated with these findi ngs?
0 A) Aflatoxin-B exposure
o B) Escherichia coli infecti on
OC) Hypercalcemia
o D) Hyperuricemia
0 E) -Naphthylami ne exposure
0 F) Proteus infection

43. A 25-year-old primigravi d woman at 34 weeks' gestation comes to the physician for a routine prenatal
examination. She just noticed an enlargement of a small pigmented bump she has had since birth. Physical
examination shows a 6-mm, tan projection on the chest just below the mammary crease in the midclavicular
line. The structure has regular margins and is not tender. The structure is most likely which of the following?
O A) Accessory nipple
o B) Epidermal cyst
0 C) Lentigo
O D) Mel anoma
0 E) Pilar cyst

N B M E form 4-4

7. Some oncogenes are mutants of normal cell-surface receptors that have tyrosine protein kinase moti fs in their
cytoplasmi c domains. The type of mutation of such a receptor that is most likely to result i n cell transformation is
a mutant protei n wi th which of the foll owing characteristics?
0A)
0 B)
o C)
o D)
O E)

Constitutive activation of cell ul ar transcription


Del etion ofthe cytopl asmic domain
Failure to become phosphorylaled upon receiving the signal
Failure to receive the signal that normally activates cellul ar transcri ption
Recognition of a different signal than usual

8. A 50-year - ol d surgeon wi th al cohol on his breath enters the surgical suite to do a procedure. Pri or lo the
procedure, he i s asked lo leave. Later that day, he is confronted by the hospital's physi cian's assi stance
committee. He confesses that he has been dri nki ng alcohol heavi ly on a daily basis. Which of the following long
term plans is most likely to help the physician mai ntain obriety?
0 A) Cognitive therapy for depression
0 B) Participation in Al cohol ics Anonymous
o C) Therapy wilh an antidepressant
O D) Therapy wilh a y-ami nobutyri c acid (GABA) agonisl
O E) Therapy wilh disulfiram
0 F) Therapy wilh an opiate agonisl

ooc,ocCc0o
(co .tJ o o
e
r, O,io" oO d
o . o o O

ic

o"'

0o

, ooo

o
o OO ooe
o(Jf' 0

j
9. A 70-year-old man has easy faligability, malaise, and an ataxic gait. He is pale and has a sore tongue, absent
vibratory sense up to his knees, and bilateral extensor pl antar reflexes. A peri pheral blood smear is shown.
Which of the following is the most likely cause of this patient's disorder?
0 A) Acquired deficiency of intri nsic factor
0 B) Carcinoma of the colon
o C) Congenital absence of transcobalamin II
o D) Congenital malabsorption ofvitami n B (cobalamin)
O E) Hookworm infestation

,,

',/1 100

g 75

:sV so

J 25 i.....,...;.....--,----.-10

100

1000

(Subsiance) nM
10. The effects of administration of drug X only and norepinephri ne only are tested on an isolated muscle. The
graph shows the maximal contraction of the muscle as a function of the concentrations of drug X and
norepinephline. Both drugs act through the same receptor. Which of the foll owing best describes drug X?
O A) Competitive antagonist
o B) Full agonist
OC) Full antagonist
o D) Inverse agonist
o E) Noncompetitive antagonist
0 F) Partial agonist

1 1 . A 40-year-old woman wi th rapid cycli ng bi polar disorder is being treated with valproic acid. One month after she
starts taking a second drug, her serum valproic acid concentration is decreased by 20%. Which of the following
is the most likely second drug?
0 A) Carbamazepine
o B) Diazepam
o C) Haloperidol
o D) Hydrochl orothiazide
o EJ lmipramine
o F) Nifedipi ne
OG) Propranolol

30

60
90
nne(mll'I)

1 20

1 40

12. The graph shows serum penicillin G concentrations after patients X and Y are given peni cillin G, 250 mg orally.
Pati ent Xis an otherwi se healthy 30-year-old man. Which of the following best explai ns the findings for patient

Y?

O A) Achlorhydri a
0 B) Concomi tant use of cimetidine
OC) Decreased protein binding
o D) Intestinal malabsorption
O E) Renal insufficiency

13. A 1 6 -year-old girl is febri le and incoherent but able to indi cate that she has severe lower abdomi nal pain. Pel vi c
examination shows a foul-smelling cervi covaginal di scharge, and a septic abortion is suspected. Which of the
following procedures is most appropriate for identifying the mi crobial agents causing her infection?
0 A) Inoculation of anaerobi c transport media at the bedside
0 B) Inoculation ofthe speci men into tissue cul ture

OC) Quantitation ofthe bacteria in the discharge by using a cal ibrated inoculating loop

o D) Streaki ng a swab specimen onto Thayer-Martin media at the bedside


O E) Use of Lowenstei n-Jensen and Sabouraud media for acid-fast bacteria and fungi, respectively

14. Ten days after returning from Kenya, a 40-year-old woman has temperatures to 40C (104F), headache,
spl enomegaly, and ri gors. Over the next several days she develops paroxysms of fever and chills. These
symptoms most l ikely occur when which of the following happens?

o A) Gametocytes enter the blood

0 B) Mature schizonts rupture erythrocytes


o C) Merozoites enter the hepatocytes
o D) Sporozoites enter the blood
O E) Trophozoites leave the hepatocytes

15. A 50-year-old man undergoes allogeneic bone marrow transpl antation for relapsed acute lymphocyti c leukemi a
that includes pretreatment with total body irradiation. His HLA profile is HLA-A1, -A10, HLA-B7, -B18, and HLA
Cw1-D. The donor bone marrow cells have the followi ng HLA profile: HLA-A3, -A9, HLA-B7, -B8, HLA-Cw2, Cw3. Three months after the transplant, the patient is di agnosed with graft-versus-host disease (GVHD); he
dies 1 month later. Which of the following best describes the pri nci pal immune mechanism responsible for
GVHD in this patient?
O A) Activati on of donor cell-deri ved natural kill er cells
O B) Donor cell-deri ved innate immune responses directed againstt disparate MHC antigens
OC) Donor T-lymphocyte-mediated suppression of bone marrow hematopoiesis
0 D) Generation of donor B -lymphocyte antibody responses directed agai nst di sparate host MHC antigens
0 E) Generation of donor T-lymphocyte responses directed agai nst di sparate MHC antigens
16. A 42-year-old man has a temperature of 39.2C (102.6F) and mild jaundice 2 days after transfusion of packed
red cells for the treatment of chroni c anemia. Laboratory studies show hyperbilirubinemia and a mild increase in
serum lactate dehydrogenase activi ty. Which of the followi ng findings will most likely be observed on analysis of
peripheral bl ood cells?
0 A) Compl ement protein C3bi bound to monocytes
o B) Degranulated neutrophi ls
O C) Erythrocyte ghosts
O D) HLA-DR upregulation in circul ating CD3+ T lymphocytes
0 E) lmmunoglobul in bound to erythrocytes
0 F) Nucleated erythrocytes

17. A screening test for col on cancer is administered to 1000 people wi th bi opsy-proven colon cancer and lo 1 000
people wi thout colon cancer. The lest resul ts are positi ve for 250 of the proven cases and 100 of those without
colon cancer. The screening test is now to be used on a population of 100,000 people with a known prevalence
rate of colon cancer of 80 per 100,000. Which of the following is the expected number of true positi ves and
false posi tives in this population of 100,000 peopl e?
A)
O B)
OC)
O D)
O E)
O F)
OG)
O H)

True Positive False Positive


9992
20
89,928
20
40
9992
50
89,928
60
9992
60
89,928
80
9992
80
89,928

18. A 3-year-old boy with nephroblastoma (Wilms tumor) is brought to the physician for a follow-up exami nation.
Physical exami nation shows no abnomialities. A CT scan ofthe chest is shown. Which of the following normal
structures is indicated by the arrow?
o A) Brachiocephalic artery
O B) Left brachiocephalic vein
OC) Superiorvena cava
0 D) Thymus
0 E) Thyroid gland

An 18-year-ol d woman has had pharyngitis, fever, and lethargy for 3 days. Leukocyte count is 1 1 ,500/mm'. The
photomicrograph shows the type of cell that is predominant in her peripheral blood smear. Which of the
followi ng is the most likely diagnosis?
O A) Acute lymphoblastic leukemi a
O B) Acute monocytic leukemia
OC) Acute myelogenous leukemia
0 D) Bacterial pharyngiti s
0 E) Infectious mononucl eosis

20. During a clinical study, a researcher investi gates methods of accelerating marrow engraftment followi ng bone
marrow transplantation in cancer patients. A synthetic glycoprotein, si milar in function to granulocyte colony
stimulating factor (G-CSF), is designed. VJhich of the following is a characteri stic of G-CSF activi ty that must
be taken into account to assure similar biologic function?
0 A) Activati on of a specific cell-surface receptor
o B) Display of cross-species functi onal ity
O C) Downregul ation of cell growth
O D) Production by endothelial cells
0 E) Stimulati on of megakaryocytic cell production
21. Dorsal root gangl ia and the sensory ganglia of the trigeminal, facial, veslibul ocochlear, gl ossopharyngeal, and
vagus nerves are deri ved from whi ch of the following?
O A)
O B)
0 C)
OD)
o E)

Neural crest cells


Neuroblasts derived from the neural tube
Neuroepithelial cells of the neural tube
Somites
Undifferenti ated mesenchyme

22. A 10-year-old boy is evaluated for progressive headaches over the past 3 months. An MRI of the head shows a
lesion in the structure indicated by the arrow in the section of a normal brain. Which of the following is the most
likely associated symptom?
0 A) Contral ateral dysmetria
O B) Contral ateral gaze-evoked nystagmus
OC) lpsilateral hypertonia
O D) lpsilateraJ intention tremor
o E) Truncal ataxi a
23. A 3-year-ol d girl is brought to the physi cian because of fever and pain in her right ear for 8 hours. Her
temperature is 39.4C (1 02.9F). Examination of the ri ght ear shows perforation of the tympanic membrane and
a purulent exudate. A Gram stain of the exudate shows gram-positive di plococci. Which of the followi ng is the
most li kel y causal organism?

o A) Haemophi/us influenzae
o B) Moraxe//a catarrhalis
o C) Neisseria meningitidis

O D) Sf.aphy/ococcus aureus
0 E) Streptococcus pneumoniae

24. A 1 -day-old newborn has had bil ious vomiting that began 1 hour after birth. Physical examination shows upward
sl anting eyes, short small fingers, and a wide space between the first and second toes. Peristattic waves are
seen, but there is no abdominal distention. Which of the foll owing is the most likely additional finding in thi s
patient?

o A) Congenital aganglioni c megacolon


0 B) Duodenal atresia
OC) Meconium ileus
o D) Pyl ori c stenosis
O E) Tracheoesophageal fistula

25. A 6-year-ol d boy is brought to the physi cian by his parents because his behavior has been different over the
past 2 months. His mother says, "He seems fine during the day, but he wakes up once or twice a week
screami ng and maki ng no sense. His eyes are open during an episode, but he doesn't recognize me or his
father. It seems as though he's looking ri ght through us. After about 10 minutes, he calms down and falls back to
sleep."' Which of the followi ng responses by the physician is most appropriate?
o A) "I woul d like child protecti ve services to evaluate your family."
O B) "It is best for your son to undergo an EEG."
OC) "It is best for your son to undergo evaluation for a psychiatric disorder. "
O D) "It is best for your son to undergo an MRI of the brain."
0 E) 'Thi s most likely is a common benign sleep disorder that will resol ve on its own."

C
0
' iii

j
- 50
E
::,

.E

1;\
:i:

,--

,i

[ NE]
26. Norepinephri ne (NE) is administered in increasing doses to a smooth muscle preparation rich in a -adrenergic
receptors. The graph shows the response of muscle tension with NE alone (solid curve) and with NE in the
presence of an a-adrenergic blocking agent (dashed curve). Which of the following a-adrenergic blocking
agents is most likely being used?
o A)
0 B)
O C)
OD)

o E)

Doxazosin
Phenoxybenzamine
Phentolamine
Prazosin
Tolazoline

27. A 22-year-old man comes to the emergency department because of the sudden onset of excruciating chest pain
that radiates to his back. His pulse is weak and rapi d. A CT scan of the chest shows a fluid density wi thin the
aortic wall. A defect of which of the foll owi ng proteins is most likely in this patient?
OA) El asti n
0 B) Fibrillin
OC) Fibronectin
O D) lntegrin
OE) Laminin

28. A healthy 25-year-ol d man who has been traini ng for a marathon duri ng the past 6 months runs 1 5 mil es daily.
Endurance training of normal skeletal muscle l eads to which ofthe following muscle adaptations?
oA)
0 B)
OC)
0 D)
o E)

Fiber-si ze reduction
Fiber-type conversion from type I (slow) to type II (fast)
Hyperplasia ofmyocytes
Increased capil lary-to-fiber ratio
Increased glucose 6-phosphatase activity

29. A 67-year-old woman comes to the physician because of an 1 8 -month hi story of progressi ve persistent left hip
pain. The pain is characterized as bori ng and achi ng. I t is not relieved by rest and worsens with ambul ation.
She walks wi th a li mp because of the pain. She has become "hard of hearing" over the past 5 years and has
passed two ki dney stones in the last 6 months. She has pain when di rect pressure is applied over the greater
trochanter of the left femur and femoral shaft. There is also pain and li mited range of moti on of the left hip wi th
flexi on and external rotati on. Serum calci um and phosphorus concentrations are within the reference range,
serum al kaline phosphatase activi ty is markedly increased, and uri nary hydroxyproline excreti on is increased.
Whi ch of the following is the most l ikely diagnosis?
0 A) Osteitis deformans (Paget disease)
0 B) Osteoarthriti s
OC) Osteomalacia
o D) Osteomyel iti s
O E) Osteoporosis
O F) Rheumatoid arthriti s
30. A 78-year-old woman has had temporal headaches, blurred vision, jaw claudication, and achi ng in proxi mal
muscles for the past 3 weeks. Erythrocyte sedimentation rate is 1 1 0 mm/h. Which of the followi ng drugs is the
most appropriate treatment?
0 A) Allopurinol
O B) Aspirin
O C) Capsaici n
OD) Cyclosporine
O E) Gold
o F) lndomethacin
o G) Methotrexate
0 H) Prednisone
O I) Probenecid
O J) Zafirlukast
31. A 38-year-old man has a 9-month history of progressive dyspnea. X-ray of the chest shows di ffuse interstitial
pulmonary infiltrates. Examination of tissue obtained on lung biopsy shows chronic interstitial inflammatory
infiltrates wi th hyperplasia of type II pneumocytes and irregular areas of collagen deposition. Which of the
following is the most likely diagnosis?
0 A) Acute respiratory distress syndrome
o B) Bronchial asthma
OC) Bronchiectasi s
o D) Emphysema
0 E) Idiopathic pulmonary fibrosis

32. A 34-year-old woman is brought lo the emergency department after she collapsed when she returned from a
nonstop flight from New Delhi lo New York City. She has smoked 1 pack of cigarettes daily for 1 6 years. She is
sexually active and uses an oral contraceptive. Her pulse is 125/mi n, and blood pressure is 75/50 mm Hg.
Physical examinati on shows jugul ar venous di stention. The lungs are clear to auscul tation. Pulmonary artery
calheterizalion shows a pulmonary systoli c/diastolic arterial pressure of 50/25 mm Hg (N=15- 30/3- 12) and a
pulmonary artery wedge pressure of 5 mm Hg (N=B-16). Which ofthe followi ng is the most likely cause of her
pulmonary hypertension?
OA) Decreased cardiac output
0 B) Increased left atri al pressure
OC) Increased left ventricular diastolic pressure
O D) Increased pul monary bl ood flow
o E) Increased pul monary venous resistance
O F) Occlusi on of left ventricular outtlow tract
0G) Occl usion of pulmonary arteri es
o H) Pericardia! tamponade
33. A left ventricular pressure-vol ume loop descri bi ng one cardiac cycle is shown. Mitral valve opening occurs at
which of the following l abeled points?
B

C
D
60

1 00

l.t wnt11c..,e1 .,..

O A)

O B)

O C)

O D)

160

0 E)

34. In a patient who is hemormaging, which of the following is an adaptive response that helps to maintain mean
arterial pressure?
0 A) Increased encl-systol ic volume
O B) Increased venous compliance
0 C) Inhibition of secretion of ADH (vasopressin)
0 D) A fluid shi ft from intersti tial to vascular compartments
o E) Release of atrial natriuretic pepti de
35. A 57-year-old man is brought to the emergency department because of the sudden onset of severe left-sided
chest pain during exercise. Laboratory findings and an ECG suggest a large, transmural, anterior wall
myocardial infarction. Which of the following microscopic changes are most likely to be present in the injured
myocardium 36 hours after the beginning of the infarcti on?
0 A)
o B)
OC)
O D)
O E)

Coagulation necrosis and neutrophilic inflammation


Granulation tissue fom1ation
Interstitial and perivascul ar lymphocytes
Macrophagi c phagocytosis of degenerated myocytes
Necrolizing arterilis of coronary vessels

36. A 48-year-old man with coronary artery disease and renal insufficiency starts treatment wi th atorvastatin to
decrease hi s serum cholesterol concentration. If he needs additional hypocholesterolemic therapy, which of the
foll owing would be safest to avoid rhabdomyolysis?
0 A)
0 B)
O C)
00)

Cholestyramine
Fenofibrate
Fluvaslatin
Gemfibrozi l

37. A 30-year-old woman has a 1 2 -hour history of chills and abdominal pai n. Her last menstrual peri od was 2 weeks
ago. Her temperature is 38.8C (101.8F), pul se is 120/min, respirations are 18/min, and bl ood pressure is
132/82 mm Hg. Abdominal and pel vic examinations show tenderness on palpation of the right lower quadrant,
pain in the ri ght lower quadrant after pressure is quickl y released from the l eft lower quadrant. pain in the ri ght
l ower quadrant when the ri ght hi p is flexed agai nst resistance, and tenderness in the ri ght rectal wall. Bowel
sounds are absent. Test of stool for occul t bl ood is negati ve. Whi ch of the followi ng is the most likely
diagnosis?
0 A) Appendicitis
O B) Cystiti s
0 C) Diverticulitis
O D) Ectopi c pregnancy
0 E) Pelvi c inflammatory di sease
38. A 57-year-old man comes to the physician because of a 3 -month hi story of increased abdominal gi rth,
hemorrhoids, and swelli ng of his feet after peri ods of standi ng or sitting. He has a 20-year hi story of alcohol ism
and hypertensi on. Current medi cati ons include hydrochlorothiazi de (25 mg dail y). Physical exami nation shows
scleral icterus and jaundice. The abdomen is round and distended, with a fluid wave and a palpable spleen ti p.
i
Which of the foll ow ng best explains the patient's spl enomegaly?
O A) Ascites
o B) Bl ockage of afferent splenic lymphatics
O C) Decreased osmotic pressure in the hepatic veins
O D) Hyperacti vity of spleni c macrophages
O E) Hyperplasia in the periarteri al lymphatic sheaths
O F) Hyperplasia in the peripheral whi te pulp
0G) Increased hydrostatic pressure in the splenic vein
O H) Stenosis of the splenic artery
39. A 5-year-old boy is brought to the physi cian by his parents because 'lle is constantly scratching his bottom."
Exami nation shows small, thread-like worms in the perianal region. Which of the followi ng drugs is the most
appropriate treatment?
0 A)
0 B)
OC)
OD)
o E)

Diethylcarbamazi ne
Mebendazole
Metronidazole
Pentamidi ne
Praziquantel

40. A 6-year-ol d girl who is recovering from a severe "strep" throat is brought to the physician because her uri ne
has had a "smoky" appearance several times in the past week. Urinalysis shows trace amounts of blood and
protein. Which of the followi ng cell types initi ates the repair mechanism necessary to resolve the proteinuria
and hematuria?

o A)
O B)
OC)
o D)
o E)
0 F)

Erythrocytes
Juxtaglomerular cells
Mesangi al cells
Plasma cells
Polkissen
Proximal tubule epithelial cell s

41. A 1 7 -year-old boy has chronic renal insufficiency. His serum urea ni trogen (BUN) and creatinine concentrations
are increased, and his serum calcium concentration is decreased. Whi ch of the followi ng is most l ikely
responsible for the devel opment of hypocalcemia in this pati ent?
0 A)
0 B)
0C)
OD)
O E)

42.

Decreased secreti on of parathyroid hormone


High concentration of parathyroid hormone-related peptide
Hyperphosphatemi a
Increased secreti on of calcitonin
Vitamin D toxicity

A 26-year-old woman comes to the physician because of a 2-year hi story of recurrent episodes of cystitis that
are associated with sexual intercourse. Her symptoms began after she marri ed her husband. Vital signs are
normal. Physical exami nati on and urinalysis show no abnormal iti es. Vvhich of the following practices is most
l ikely to decrease the frequency ofthese infections?

0 A) Douching on a weekl y basis

o B) Drinki ng 8 ounces offlui ds immediately prior to intercourse

0C) Havi ng her husband use a condom

o D) Using a diaphragm for contraception


O E) Voiding soon after intercourse

43. An otherwi se healthy 76-year-old woman comes to the physician's office because of uncontrollabl e voi ding of
small amounts of uri ne when she coughs or laughs. This also happens sometimes when she walks. Postvoi d
volumes are small, and there is no nocturnal incontinence. Focused examination shows no abnormalities except
for some laxity of the pel vic floor musculature. Whi ch of the foll owing is the most likely diagnosis?
0 A)
o B)
OC)
0 D)
O E)

Bl adder tumor causing partial outl et obstruction


Detrusor overacti vity
Fibrosis of detrusor muscle
Outlet incompetence
Small infarct of pontine micturiti on center

44.

Genetic screening shows that a newborn with a 46,XY karyotype has a disabling mutation of the 5a-reductase
gene. Which of the following best describes this newborn's genital system?

o A) Female external genitalia and female genital ducts


0 B) Female external genitalia and male genital ducts
OC) Female external genitalia and male and femal e genital ducts
OD) Male external genitalia and female genital ducts
o E) Male external genitalia and male genital ducts
O F) Male external genitalia and male and female genital ducts
45. A 31-year-old man comes to the physician because of a decreased l ibi do. He had viral orchitis 6 months ago.
Physical exami nation shows gynecomastia and testicul ar atrophy. Which of the followi ng drugs is most
appropriate for long-term management of this condition?
OA) Danazol
0 B) Gonadotropi n-releasing hormone
OC) Human chorionic gonadotropin
o D) Sildenafil
0 E) Testosterone

46. A 45-year-old man is diagnosed with a somaloslalin-producing tumor. Which of the foll owi ng sets of physi ologic
changes is most likel y in this man?
Insulin Secretion Gastric Motility

t
t

O A)
O B)
OC)
O D)
O E)
O F)
OG)
0 H)

Glucagon

Secretion

!
!

1
!
!

!
1
!
t

47. A 19-year-old woman is brought lo the emergency department by her roommates because of progressive
l ethargy and confusion during the past 12 hours. Her roommates say that she had a 1 0-kg (22-lb) weight l oss
duriing the past month despite substantially increasing her food and liquid intake. On arrival, she is obtunded.
Her temperature is 36.2C (97.2F), pulse is 92/min, respirati ons are 22/min, and blood pressure is 90/40 mm
Hg. Physical exami nation shows no other abnormali ties. Serum studies show:
151
mEq/L

Na

5.0 mEq/L
118
mEq/L

K'

er
HCO

8 mEq/L

Glucose
Creati nine

491 mgldl
1.4 mgldl

Without appropri ate treatment, she is at greatest risk for a sinonasal infection wi th which of the following causal
organisms?

o A) Cryptococcus neoformans
O B) Histoplasma capsulatum
0 C) Mucor circinelloides

o D) Nocardia brasi/iensis

o E) Trichophyton rubrum

48. Tri iodothyroni ne (T) is more potent than thyroxine (T ) for the treatment of hypothyroidism because of which of
the following?

o A) Active transport into target cells


o B) Greater accumulation in mitochondria

0 C) Greater receptor affinity


O D) Higher oral bioavailability
O E) Longer plasma hal f-l ife

49. A 75-year-old man is brought lo the physician by his family because of akinesia, ri gidity, postural instabili ty, and
tremor. Symptoms improve wi th levodopa therapy, but he develops anorexi a, nausea, vomiting, and mil d
orthostalic hypotension. A drug wi th which of the foll owing mechanisms of action is most appropriate to
decrease the gastrointestinal symptoms and hypotension?
0 A)
o B)
o C)
O D)
O E)

Inhibi tion of catechol- 0 -methyl transferase


Inhibi tion of monoamine oxi dase B
Inhibi tion of peripheral dopa decarboxylase
Stimulati on of cholinergic receptors
Stimulati on of dopamine receptors

Block 1
1... zinc finger motif is a D N A binding domain for steriod receptor so A is right answer
2 M c A rdle disease G lycogen is there in the ms but cant be used bec. of deficievy of G lycogen phospholylase enzyme
3- Discuss
4- T ight junction are present between the cells and when they are damaged the + discussion
5- C F is A R disease ... the child with diease is homozygous for C F gene.. car rier are heterogygous for the mutation...
For the parent to produce a homozygous child they must both be car rier of the C F . discuss
6- W e are transplanting gene in the bone mar row so is cant be structural gene and the probable answer could be B
7- here its cant be sarcoma becasue vimentin antibodies reaction are negative... only clue is keratin antibodies as in
squamous cell ca there is keratin pearls so E seem to be right
8 B E SU R E T H A T Y O U U N D E RST A N D T H E PR O B L E M so its better to get more info rather going to fast...........
9-He is without food from the last one week... note he consumed his food in the first two weeks. A is wrong as no diet
is there hepatic glycogen store are for the first few hours.skeletal muscle glycogen cant be used because of abscence of
glucose 6 phosphatase enzyme in muscle D is the cor rect answer .. muscle protein are being used by the
gluconeogenesis process urea nitrogen cant help
10- D is the cor rect 11-Bleomycin causes pulmonary fibrosis mean a restrictive lung disease in restrictive lung diease
either there is increase in F E V1/F V C ratio or there is no change in it..... total lung volume decrease as there is
restriction in filling the lungs to there capacity please add what will be the effect of diffusing capacity C is the answer
12-A Z T is reverse transtriptase inhibitor so E is right
13- urease is the answer but what is the substrate of urease and what the product??
14-B road base point to blastomycosis
15-these are sign of acute cholecystitis so E is right
16-this is acute rejection as occur after weeks of transplantations... so T lymphocyte play there role
17-need explanations
18 need explanations
19-T N F is produced in response to E ndotoxin and then T N F causes release of N O which is the main mediator of
shock and hypotension

20-D dimer are specific for D I C , D dimer can only assay the cross linked fibrin monomer while F DP assay detect all
the product of plasmin cleave of fibrenogen and insloluble fibrin clots
21-H lateral corticospinal tract....what is at G position?
22-diagram is not clear any way they terminate at thalamus
23- A neostigmine inhibiot the enzyme
24- B is right D M causes accumulation of sorbitol in the schwann cell becuase of A L D OSE R E D U C T ASE E N Z Y M E
which convert glucose into sorbitol and lead to osmotic damage to the shwaan cell and defective myelination of
peripheral nerve and so peripheral neuropathy..........
25-need explanations from net 26-C anti psychotic drug SE
27-what is the diagnosis?????????
28-iliopsoas
29-need net help
30-B
31-case of asthma which is an obstructive lung diseas F E V1. F V C is typically decreased and residual volume is
increased
32-superior vena cava syndrome due to small cell lung carcinoma causeing obstruction of the vena cava and so
puffiness of the face and neck
33- Patent ductus arterosis connecting aorta with left pulmonary vein
34-as we are increasing the precapillary resistance when flow will dec and the filtration will decrease .... 35-Beta 1
increase renin secretion but alpha 1 decrease renin secretion.
36...
37 J
38-C ystic fibrosis patient have short stature
39-Did you noticed that when we add the drug X we need more histamine to get the same response mean there is a
revercible competitive inhibitor there
40- its 30
41-discuss net ................
42- becuase there are bruit its mean there is atherosclerosis of the renal artery .. less blood goes to the kidney and
renin is secreted which convert Ag1 into Ag 2 and which in turn increases ald. and increase Na absortion and so H T N.
43- O xcytocin but what is the 2nd messenger of O xcitocin?
44-P C OS clue is decreased FSH
45-B
46-net search today
47-W e know that radiation induced thyroid papillary ca is most common but can it be L aryngeal C A????
48-E but see notes
49-M blocker block secretion and so cause dry mouth ipratropium

50-the oral drugs pioglitazone and rosiglitazone can cause or exacerbate heart failure and pulmonary edema and
should be avoided in patients with left ventricular dysfunction (impaired pumping ability of the heart) or chronic
renal insufficiency.

BLOCK TWO
1-B seem to be right... when we add antibiotic resistance plasmid, we can get the colonies who are resistance to a
particular drug and can eliminate the drug that are sensitive ...
2- C seem to be right as we see that in C F there is a defect mutation and 508 codon is missing coding for
Phenylalanine amino acid .. in C only phenylalanine is shown there
3-Phenylalanine H ydroxylase deficiency leading to P T U....
4-Nondisjunction will produce either an egg with 24 chromosome or with 22 chromsomes..
5-Acute pancreatitis so enzymatic fat necrosis
6-X linked recessive disease more common in male and if female are symptomatic then its because of inactivation of
more maternal chromsome than paternal... female are homozygouys for the abnormal chromosome.
7-A
8-Unstable mood , impulsiveness.. female more than male
9-here is a point never assist the patient to die .... passively is ok but actively is not ok.... i think C seem to be right
10-D??
11-E
12-transplant patient often get C M V infection so G ancyclovir
13-A
14-C C R5 i % of US population give protection to H I V infection
15-B
16-D because IgG protect respiratory mucosa from infection 17-listen steven daughtery
18-D lack of ramdomization need explanations
19-see physio
20-A
21-polycythemia so F
22-c abducent nevre
23-Radial nevre
24-circle of willus
25-D should know from where various seizure arise in the brain
26-E can any body explain?

27- Cohesion B W adjacent cells ......... and held them togethers


28-D never problem
29-G
30-what is the diagnosis
31-D seem to be right
32-what is the diagnosis?
33-C
34-During exercise the heart rate increases and when H R reach 180 it compromise the coronary blood supply .. O K
now pumping action increases the heart metabolism and increases metabolite causes coronary vasodilation..... and so
increases blood flow. Now in exercise increase wor k volume occur at same pressure so C seem to be right
35- Aortic pressure will be less than ventricular pressure so D seem to be right
36-D but need to check
37-Parietal cell 38-E
39-F
40-c
41- see picture of renal ameyloidosis from google
42-F
43-B
44-history is relevent to adenocarcimona
45-Babies Blue Normal reaction to birth
46-E
47-D need to check lab value
48-D
49-C seem to be right ... physiostigmine enter C NS , neostigmine not so when you have confusion about which enter
then think N is for N O T ..
50- C
BLOCK 3
1- A/T rich region are where H istone bind to D N A, Nucleosome are formed by the histone octamer plus D N A wound
round it. O ne nucleosome is connected to the other by H1 histone. Need net search or discussion..
2-C release factor
3-A lanine is converted to pyruvate for G luconeogenesis
4-F 5-reperfusion injury by free redical
6-A utosomal recessive
7-G amma interferon secreted by T h1 cell causes M acrophages activation and activated macrophages become fused
to form multinucleated giant cell

8-A
9-C but not sure need forum discussion
10-V it. C
11-E
12-A
13-A lkylating agent
14-D as this virus is segmented so produces a number of band need discussion
15-discussion
16- Pneumococcus encapsulated bacteria
17-this is a bar graph ok first you count how many are total children so 25+30+35+10=100 now how many children
have two episode of U T I? answer 10 now out of hundred children 10 have two episode of U T I and will be be 10
percent .
18-C ase cantrol study
19-A
20-Aferican type of Bur kit lymphoma classic picture of star ry sky appearance associated with c myc oncongene..
21-B
22-F
23-A
24-E
25-D
26-C
27-A melanocyte are derivative of neural crest cell
28-Posterior cruciate ligament T ibia move posterioly while for anterior cruciate ligament tibia move anteriorly
29-B polumyalgia rheumatica is associated with Temporal arteritis
30-A???
31-T ype11 form new type 1 cells
32-Pneumothorax there will be H yper resonant, deviation of tracha to opposite plus decreased breath sound and
absent fremitus
33-A
34-A
35-B
36-D but need discussion
37-C
38-F for hurshsprung disease symptoms appear soon after birht

39-A but how need explanation


40-A bsent of kidney lead to oligohydoamnious
41- A D H causes increase water reabsorption so serum osmolarity would Dec , U rine osmolarity inc and urine sodium
conc will be high
42-F 43-A accessory nipple got inlarged due to Pregnency
44-G uljan says that
age M C tumor of gynae
45 Cervical
55 endometrial
65 O varians Note C E O alphabat come accordingly in the Q stem ist mention that she has P C OS mean has high
estrogen plus H eight is normal mean she dont have tuner syndrome which point to ovarian syndrome ... but its
endometrial C a post menopausal bleeding
45-E
46-D PI D
47-B
48-D due to parathyroid removal
49-need discussion
50-D but need to know whether is this a brush border enzyme??

BLOCK 4
1-A
2-D need clarification
3-Need to see Y H biology notes C?
4-A action potential travel to the interior of myocyte due to T tubule
5-its X linked recessive mean only diease gene on X chromosome and if that male parent would have defective gene
on his X chromosome then he would have the disease and he dont have its mean that he is not car rier of this disease
6-B
7-need clarification from Y H biology notes
8-B??
9-A not congenital because he is 70 year old
10-Partial agonist as at low dose its acting as agonist but at high dose its acting as antagonist.
11-A
12-Penecillin excreted by kid. so when renal function are compromised then sr. drug level increse as this y patient
13-D

14-B
15-E
16-C but what is ghost cell?
17-Need mastering of Biostatistics 18-T hymus
19-E
20-A but need to check
21-A
22-Cerebellum and in cerebellum has ipsilateral signs
23-E
24-Duodenal atresia and ths diagnosis seem to be hurshsprug disease
25-E
26-B but need to check this nusty Q
27-B
28- D increase blood circulation
29-need to discuss A?
30-H
31-E 32-G D V T
33-E
34-D
35-A but need to memorise the time sequence of infarction
36-A
37-A but how to differentiate between the rest of conditions
38-cir rhosis and increase in splenic vein pressure due to portal hypertension
39-B need to revise all the antihelmeitic drugs
40-what is the main function of mesengial cell....
41-C
42-i think E but some says c
43-B because for outlet incontenence there is continouse voiding without a stress factor
44-5 alpha reductase convert testosterone into D H T which is necessary for the development of external ganitalia
including prostate when D H T is not formed female external feature are formed but internal male system is
developed because testosterone is required to form normal internal male struture .
45-E
46-A n inhibitory hormone so inhibit all the function
47- Diabetic ketoacidosis mucor specific

48-C
49-C
50-Spironolactone aldosterone antagonist...

NBME
Form 3 Section 1

6. A 75-year-old woman wi th metastatic small cell carcinoma of the lung is brought to the emergency department
after coll apsing. On arrival, she is pale, sweati ng, and lethargic. Her temperature is 37.2C (99F), pul se is
130/mi n and weak, respirations are 25/min, and bl ood pressure is 90/60 mm Hg. Breath sounds are decreased
bilaterall y. An x-ray of the chest shows a tumor mass in the hi lar area and an addi tional radiodensity in the left
lower l obe of the lung. She dies within a few hours despi te supporti ve care and antimi crobi al therapy. Cul tures of
blood collected at the ti me of admissi on grow Klebsiella pneumoniae. Whi ch of the following is the most li kel y
mediator released from macrophages that ini tiated the process that led to her death?
0A) a -Anti trypsin
'
o B) Complement C3b
OC) C-reactive protei n

OD) Factor XII (Hageman factor)


O E) lnterferon-y
O F) lnterleukin-8 (IL-8)
OG) Pl atel et-activati ng factor
OH) Tumor necrosis factor
7. A 45-year-old woman has the recent onset of shortness of breath. She has had a cough and a 14-kg (30-l b)
weight loss over the past 6 months. An x-ray of the chest shows a 4-cm mass in the mi ddl e lobe of the right l ung.
Exami nation of tissue obtai ned on biopsy shows a small cell, neuroendocri ne lung carci noma. Whi ch of the
following abnormaliti es is most li kel y to develop in this pati ent?
O A) Hypocalcemia
0 B) Hypoglycemia
O C) Hyponatremi a
OD) Pol ycythemia

8. A 45-year-old woman has increasi ng dyspnea and fatigue. She is pal e. Hemogl obi n concentrati on is 7.5 g/dl,
leukocyte count is 4500/mm', and platel et count is 89,000/mm'. A peripheral bl ood smear is shown. Based on
these findings, whi ch of the following addi ti onal findi ngs is most li kel y?
O A) Antiparietal cell antibodi es in serum
o B) Decreased serum concentration of iron
O C) Increased hemogl obin A on hemoglobi n el ectrophoresis
'
OD) Increased osmotic fragili ty of the erythrocytes
O E) Positive direct antigl obuli n test
9. The epi dermal growth factor receptor and the platel et-deri ved growth factor receptor are both classified as which
of the followi ng?
0 A) Calcium-dependent protein ki nases
o B) cGMP receptors
OC) JAK/STAT kinases
OD) Receptor guanyl yl cyclases
o E) Receptor serine/threonine
ki nases
O F) Receptor tyrosine kinases

20. A 59-year-old man comes to the emergency department because of fever and shaki ng chills for 24 hours. He
compl eted a course offluorouracil for a neopl asm 1 week ago. Hi s temperature is 39C (102.2F), and bl ood
pressure is 85/60 mm Hg. Physi cal examination shows no other abnormaliti es. His hematocrit is 30%,
leukocyte count is 400/mm', and platel et count is 100,000/mm'. Which of the followi ng agents will most
specifically compl ement anti biotic therapy in this patient?
oA) Erythropoietin
O B) Granulocyte col ony-sti mul ating factor
OC) lnterteukin-1 (IL-1)
O D) IL-2
0 E) Leucovorin
21. An acute extensive bilateral lesi on in the reticul ar formation of the tegmentum of the midbrain is most likely to
resul t in which of the followi ng?
O A) Insomni a
o B) Intractabl e pai n
OC) Loss of consci ousness
OD) Narcolepsy
o E) Rapi d eye movements
22. Followi ng a traumati c injury, a 25-year-ol d man has l oss of abducti on, l ateral rotation, and flexi on of the arm;
loss of flexi on at the elbow; and l oss of supination of the foreann. X-rays show no fractures. A lesion of which
ofthe foll owi ng labeled structures on the diagram best expl ains these findi ngs?

OA) OB) OC) OD) O E)


23. A 10-year-old boy is sent to the princi pal's office by his teacher. While waiti ng outside the office, he becomes
very anxi ous, starts to hyperventil ate, and faints. A decrease in which of the followi ng is the most l ikel y cause
ofthe decrease in cerebral bl ood flow?
o A) Arterial Pco,
0 B) Arterial pH
OC) Arterial Po,
OD) Cardiac output
O E) Pul se
24. A 4366- g (9 l b 1 0 oz) male newborn is del ivered at term. The cephali c vaginal del ivery was difficul t and
compl icated by severe depression of the right shoulder. An MRI shows avulsion of a si ngle root of the brachi a!
pl exus. The ventral ramus of which of the followi ng spinal nerves has most likely been injured?
OA) CS
O B) C6
OC) C7
O D) C8
O E) T1

29. Whi ch of the following actions is most li kely to be severel y impaired if a nerve is injured by a fracture at site X
In the x-ray'?
O A) Dorsi flexion of the foot
o B) Extensi on of the leg
OC) Flexion of the l eg
OD) Invers on of the foot
o E) Plantari flexi on of the foot
30. Two brothers, aged 26 and 28 years, have simil ar disorders. Both of their hi stori es include blue scl erae at birth,
joint hypermobi lity, easy bruisi ng, and moderate bi lateral hearing loss. Each has had about 20 fractures si nce
bi rth, all ofwhich have heal ed rapidl y and without deformi ty. Thei r symptoms are most likely caused by a defect
in the synthesis of which of the foll owi ng compounds?
OA) Hydroxyapatite
0 B) Hydroxyproli ne
OC) Procollegen
0 D) Prothrombin
31. A 66-year-old woman has fluid in the pl eural cavi ty. To obtain a sample of fluid, a needl e is inserted at the
midaxi llary line in the si xth intercostal space. After piercing the skin, which ofthe followi ng is the correct order
(superficial to deep) of structures that wi ll be pierced in order to enter the pl eural cavity?
o A) Costal pleura, visceral pleura
0 B) lntercostal muscles, costal pleura
OC) lntercostal muscl es, mediastinal pl eura
o D) lntercostal muscl es, visceral pleura
O E) Vi sceral pl eura, costal pleura
32. A 28-year-old man comes to the physici an because of a 1 - month history of severe shortness of breath with
exertion. He has smoked 1 pack of cigarettes daily for 10 years. His father and his uncl e died in thei r 40s of
lung di sease. The l ungs are hyperresonant to percussi on. Breath sounds are di ffusel y decreased. Pulse
oximetry on room air whil e the patient is at rest shows a normal oxygen saturation that decreases to less than
86% when the patient cl imbs one flight of stairs. Pulmonary functi on tests show an FEV of 1.0 L/min (25% of
'
predicted) and an FVC of 3.3 L/mi n (70% of predicted). Five years ago, his lung function was normal. Histol ogi c
examination of tissue obtained on bi opsy is most likel y to show whi ch of the following?
0 A) Bronchopulmonary dysplasi a
o B) Chronic bronchi tis
OC) Interstiti al fibrosis
o D) Obl iterative bronchi ol itis
O E) Panacinar emphysema

33. An 8-year-ol d boy is brought to the physici an by his parents because of epi sodes of shortness of breath,
nonproducti ve cough, and wheezi ng that last 5 to 10 minutes and occur more frequentl y during the winter. The
incidence and severity of these epi sodes is most likel y to be decreased by selective inhi bilion of whi ch of the
following enzymes?
O A) Cyclooxygenase-1
0 B) Cyclooxygenase-2
OC) Epoxygenase
OD) 5-Lipoxygenase
o E) Phosphol ipase A'
0 F) Thromboxane synthase
34. Under control condi ti ons, flow through a bl ood vessel is 100 ml/mi n under a pressure gradi ent of 100 mm Hg.
The di ameter of the vessel is increased twofol d and the pressure gradi ent is maintained at 100 mm Hg. Whi ch
of the foll owi ng is the current flow (in ml/mi n)?
OA) 100
O B) 200
OC) 400
O D) 800
O E) 1600
35. A 17-year-old gi rl comes to the physici an because of shortness of breath. A systol ic murmur is heard best in the
left fifth intercostal space in the midclavi cul ar li ne. Whi ch of the following val vul ar abnormali ti es is the most
likely cause ofthe murmur?
oA) Aorti c insufficiency
O B) Aorti c stenosis
OC) M tral insufficiency
o D) Mii traI stenosis
O E) Pulmonary insuffici ency
o F) Pulmonary stenosi s
OG) Tricuspi d insufficiency
OH) Tricuspid stenosis
36. A 61-year-old woman with a l ong history of poorl y controll ed hypertensi on suddenl y has excruci ati ng anteri or
upper sternal pain radiating to the neck and back. On admissi on to the emergency department, her bl ood
pressure is 210/11 O mm Hg. Fifteen minutes l ater, while lyi ng down, i t is 1 10/64 mm Hg. Transesophageal
echocardiography shows aortic insuffici ency, a suggestion of a double l umen of the ascending aorta, and a
pericardia! effusi on. She dies suddenl y while awaiting an operati on. Whi ch ofthe following is the most l ikely
cause of death?

oA) Acute hemopericardium


O B) Acute myocardial infarct
OC) Cerebral infarct
OD) Congesti ve heart failure
0 E) Massive retroperi toneal hemorrhage
37. A 45-year-old man wi th increased serum LDL concentrati ons is treated wi th cholestyrami ne, which binds bil e
sal ts in the small intestine. The most li kel y resul t is preventi on of whi ch of the foll owing?
0 A) Conversion of I Dls to LDLs in the bloodstream
o B) Formation of chylomicrons in intestinal epithelial cells
OC) Resorption and redi stri bution of bile sal ts to the li ver
0 D) Synthesis ofcholesterol in the liver

Efflux rrom pancreati c fi stul a

Vdume

Protei n

con1en1

38. During a physiological study of pancreatic function, a researcher studies a fasted experimental animal with a
chronic pancreatic fistula. The researcher administers a variety of substances intravenously to mimic
postprandial level s. The volume and composition of pancreatic secretions are then measured. The results from
two such experiments are shown in the graph. Which of the following substances was most likely admir-.stered
to obtain the results shown?
O A) Acetylchollne
0 B) Cholecystokinin
OC) Gastrin-releasing peptide
OD) Secretin
O E) Somatostatin
0 F) Vasoactive Intestinal polypeptide
39. A 32-year-old man has dysphagia and odynophagia after ingesting a drain cleaner containing sodium hydroxide
pellets.Which of the following is the most likely associated finding 3 months later?
0 A) Aspiration pneumonia
o B) Blindness
OC) Esophageal stricture
OD) Gastric perforation
o E) Hemolytic anemia
O F) Renal fai lure
OG) Systemic acid-base imbal ance

40. A 49-year-old man comes to the physician because of fatigue, lightheadedness on standing or s itting up rapidly,
and dark-colored stools for the past 4 days. He moved to the USA from southeastern Asia 2 years ago. Hi s
history includes chronc hepatitis B infection. Examination shows pale mucous membranes, an enlarged liver, a
prominent periumbitical venous pattern, and nonbleeding internal hemorrhoids. Hemog lobin concentration is 7.8
g/dl. Upper endoscopy shows circumferentially prominent tortuous veins with an adherent clot at the
gastroesophageal junction. Which of the foll owing mechanisms best expl ains the blood loss in this patient?
o A) Acute inflammation of the gastroesophageal mucosa
0 B) Arteriovenous fistutae at the gastroesophageal junction
OC) Hepatic artery obstruction with increased celiac pressure
0 D) Increased flow through collateral hepatic veins
0 E) Increased pressure in submucosal esophageal veins
41. During normal micturition, pressure within the bladder increases in response to which of the following changes?
o A) Contraction of the external sphincter
0 B) Parasympathet ic outflow from the sacral spinal cord
OC) Somatic efferent outflow from the sacral spinal cord
OD) Sympathetic outflow from the lumbar spinal cord
0 E) Voluntary rel axation of the external sphincter

42. A 45-year-old woman comes to the physi cian because of a mass in her left flank. Two months ago she had a
hysterectomy for cervi cal cancer that had invaded beyond the serosa of the uterus. The mass is largel y
compri sed of a mar1<edl y di lated renal pel vi c and calyceal system. Which of the followi ng is the most likel y
di agnosis?
OA) Hydronephrosi s
0 B) Hypernephroma
O C) Nephroscl eros s
i
OD) Pyelonephrosis
o E) Vesicoureteral reflux

43. A 1 7 -year-old gi rl with chronic renal insufficiency undergoes a unilateral nephrectomy. A photograph of the
resected ki dney is shown. Which of the followi ng is the most l ikely cause of thi s patient's renal insuffici ency?
0 A) Cervical cancer

o B) Di sseminated intravascul ar coagul ation

OC) Endometri osis


OD) Essential hypertensi on
O E) Fibromuscul ar dyspl asia of the renal artery
O F) lgA nephropathy
OG) Poststreptococcal gl omerulonephritis
OH) Preecl ampsi a
O I) Type 1 di abetes mellitus
o J) Vesicoureteral
reflux
44. A 35-year-old man comes to the physici an because of a 3-week hi story of pain with urination and bl ood in his

uri ne at the end of urination. He is a fishery consul tant wi th the Peace Corps and returned to the USA 1 month

ago following a 2-year assi gnment in Africa. He frequently came in contact with l ake water while in Africa, ate
locally prepared foods, and was sexually acti ve with two women whi le usi ng condoms consi stently. Physical
i
examinat on shows no abnormaliti es. Ultrasonography of the urinary tract shows bil ateral mi ld hydronephrosis
and nodules in the bl adder. Exposure to which of the followi ng types of organi sms is the most likely cause of
thi s patient's condi tion?

O A) Bacteria
O B) Fungus
O C) Mycobacteri um
OD) Parasite
O E) Vi rus

45. A 23-year-old woman comes to the physi cian because of heavy watery vaginal discharge, itching, and burni ng
for 2 weeks. She is sexually acti ve with one partner. Specul um examination shows profuse, foamy, yellow
di scharge. A wet mount preparati on shows motile organisms and numerous leukocytes. Whi ch of the following
is the most li kel y diagnosis?
O A) Bacterial vaginosis
0 B) Chancroid
O C) Chlamydia trachomalis infecti on
OD) Trichomoniasis

46. A previously healthy 29-year-old woman has the sudden onset of dyspnea and becomes unresponsive after the
vaginal delivery of a male newborn who weighs 3570 g (7 lb 14 oz). Pregnancy was uneventful. Oxytocin was
administered during labor to augment contractions. Examination shows no pulse and no spontaneous
respirations. Which of the following is the most likely diagnosis?
O A) Acute myocardial infarction
O B) Amniotic fluid embolism
OC) Eclampsla
0 D) Hemorrhagic cerebral stroke
o E) Oxytocin overdose
47. A 22-year-old man with a nonseminomatous germ eel tumor is scheduled to undergo a course of chemotherapy
with ifosfamide, carboplatin, and etoposide. Mesna should be added to his regimen to prevent which of the
following potential toxic effects of the antineoplastic drugs?
oA) Cardiomyopathy
O B) Drug-induced leukemia
OC) Hemorrhagic cystitis
O D) Hyperglycemia
0 E) Pulmonary fibrosis
48. A 28-year-old woman is brought to the emergency department after being involved in an automobi le collision.
She has lost an estimated 400 ml of blood. Pl asma renin activity is five times greater than the reference range.
The most likely explanation for this increase is an increase in which of the following?
o A) Arterial pressure
O B) Renal sympathetic neNe activity
OC) Serum angiotensin II concentration
oD) Serum atrial natriuretic peptide concentrat ion
0 E) Sodium del very
to the macula densa
i
49. A 22-year-old man comes to the physician's office because of fatigue, blurred vision, and increased thirst for
the past 2 months. During this time, he has awakened several times a night to urinate, and urinates 8 to 10
times dai ly. He takes no medications. Serum sodium concentration is 160 mEq/L and ur ine osmolality is 1 65
mOsmol/kg H 0. Which of the following is the most l ikely diagnosis?
'
O A) Acute renal fai lure
o B) Aldosterone-secreting neoplasm
OC) Congestive heart fai lure
OD) Diabetes insipidus
o E) Syndrome of inappropriate ADH (vasopressin)
50. A 60-year-old woman develops anemia several years after a subtotal gastrectorny. Which of the following is the
most likely finding on a per ipheral blood smear?
O A) Basophi lic stippling
O B) Helmet cells
OC) Macrocytosi s of erythrocytes
O 0) Microcytosis of erythrocytes
o E) Reticul ocytosis
1B

2D
3E
4A
5E
611
7C
8A
9F
JOA
J IB
12A
13C
l4E
150

16C
17E
1813
19 Jntectious mononucleosis
201l
21C

318

32E

330

34E

35C

22A

36A

23A
24A
25E
26C
27C
280
291\
30C

38D
39C
40E

3/C

418

2A
31

440

450
468
47C
488
49D

soc

NBME
Form 3 Section 2

4. Cultured ceffs are treated with an experimental agent that leads to rapid increases in cytoplasmic calcium
concentrati ons and an increase in cellular di acylglycerol concentrations. This experi mental agent most likely
stimul ates whi ch of the following?
oA) Adenyl yl cycl ase
O B) GTPase-activating protein
OC) Phosphollpase C
o D) Protein kinase A
O E) Protein kinase C

5. A 54-year-old man comes to the physici an because of an intermi ttent rash and joi nt pai n during the past 9
months. Physical examination shows erythematous, scaly plaques on the extensor surfaces of the upper and
lower extremiti es. Removal of the scales resul ts in small bleedi ng points in the epidermi s. A photograph of the
pati ent's knees is shown. A bi opsy specimen of one of the lesions shows neutrophils collecting wi thi n the stratum
corneum, parakeratosi s, and el ongated dermal papi llae. Which of the followi ng cellul ar adaptations is most likely
in the epidermis ofaffected skin in this patient?
OA) Atrophy
o B) Dysplasi a
OC) Hyperpl asia
O 0) Hypertrophy
O E) Metapl asia
O F) Neopl asia

ONAfl(Ct>e

6. Southern bl otting is used to di stingui sh maternal and paternal copies of chromosome 7 in a si ngle person. The
diagram on the top indicates the DNA regi on that is amplified and the predi cted sites of di gestion with restriction
endonucl eases. The site of hybridization of the labeled probe is also indicated. The gel pattern obtained is shown
on the bottom. This pattern is most l ikel y indi cative of whi ch of the following?
O A) Chromosome walking
O B) Mei oti c segregation
o C) Nondisjuncti on ofchromosomes duri ng meiosis
O 0) Recombination
o E) Restriction fragment l ength polymorphi sm
7. A neopl asm in the col on invades through the basement membrane into the lami na propria. Which of the following

proteins is most li kely to be di rectly involved in the process of invasion?


0 A) APC gene product
o B) p53 protei n
OC) ras gene product
OD) Rb gene product
o E) Type IV collagenase

8. A 33-year-old man of Hispanic descent comes to the emergency department INith his 6-year-old daughter. The
pati ent, who speaks only Spanish, appears In mil d pain and says "dol or" while pointi ng to hi s abdomen. His
daughter speaks both Engl ish and Spanish. The physi cian does not speak Spanish. Which of the follolNing initial
acti ons by the physi cian is most appropriate?
0A) Ask a hospital housekeeper who speaks Spanish to interpret for the pati ent
0 B) Ask the pati ent's daughter to serve as an Interpreter between the patient and the physi cian
o C) Have a professional interpreter present while the patient is bei ng examined
00) Use a medical English-Spanish di ctionary to aid in understandi ng the patient
o E) Use pictures to aid in understanding the patient
9. Whi ch of the following is the rationale for advi sing a decrease in dietary intake of sodi um chl ori de in patients wi th
hepati c disease who devel op edema?
o A)
Edema flui d has a high concentration of Ci'
o B) Gl omerul ar fil tration rate is increased
)
OC Injured hepatocytes are more permeabl e to Na'
o D) Secondary hyperal dosteronism is present
0 E) Serum globul in concentration is increased
10. A 34-year-ol d African American man comes to the physician because of muscle aches and reddish brown urine
for 12 hours. He began treatment INith tri methopri m -sulfamethoxazol e 1 day ago for otitls media. He is allergic to
penicilli n. A sample of bl ood after centri fugation shows red serum. A deficiency of which of the followi ng
enzymes is the most l ikel y cause of these adverse effects?
0 A) Acetyltransferase
O B) Gl ucose-6-phosphate dehydrogenase
OC) y-Glutamyl transpepti dase
0 0) Gl utathione reductase
o E) 1 a-Hydroxylase

"

Tl lnt

..

1 1 . During an experiment in vitro, the effects of Drugs X and Yon bacteria are studied. The effects of Drug X al one,
Drug Y alone, and the combinati on of Drugs X and Yon bacteri al cell number after 12 hours are depicted on the
graph shown. Which ofthe followi ng pairs of drugs bes! represents the combination of Drugs X and Y?
o A) Amoxiclllin and cefepime
0 B) Ceftazidime and erythromycin (l ow dose)
oC) Ciprotloxacin and minocycline
OD) lsoni azi d and fluconazol e
O E) Minocycl ine and erythromycin (high dose)
o F) Pi peraci llin and ceftazidi me
OG) Tobramycln and amplcl llin

12. A 4-year-old boy is brought to the physician because of a 1-day hi story ofrestlessness, irri tability, and intense
perirectal itchi ng. He attends a large day-care facir.ty. Examination of the perirectal regi on shows small worms
that glow under bright light. A photomicrograph of a cellophane tape test i s shown. Which of the followi ng is the
most appropriate pharmacotherapy?
o A) Mebendazole
O B) Metronidazole
OC) Mitomyci n C
O D) Pentamidi ne
O E) Tetracycli ne
13. A 22-year-old woman comes to the physician because of abdominal cramping and bloody d iarrhea for 24 hours.
She works as a ki ndergarten teacher, and several of the children at her school have had a gastroi ntesti nal
illness in the past week. She takes no medi cati ons. Physical exami nation shows increased bowel sounds.
Whi ch of the following is the most likely causal organi sm?
O A) Bacillus anthracis
O B) Bacillus subti/is
OC) C/ostridium difficile
OD) C/ostridium perfringens
o E) Listeria monocytogenes
O F) Salmonella typhi
OG) Shigel/a flexneri
14. A 40-year-old man has evidence of chronic heart failure. Cl inical si gns of both ri ght and left heart failure are
present. His heart is dilated. When he was a child, his family settled new1y cleared land in the Brazifian Amazon
forest, and he lived in a primitive shelter buil t of mud, wood, and stone for 12 years. Which of the followi ng is
the most likel y causal organism?
o A) En/amoeba histolytica
o B) Leishman/a braziliensis
OC) Schistosoma mansoni
OD) Trypanosoma cruzi
O E) Wucherer/a bancroffi
15. A 24-year-old woman is brought to the emergency department because of a 3-day h istory of migratory
polyarthri tis. Over the past 24 hours, she has become unable to walk because of a tenosynovitis ofthe right
patella. Examination of fluid from the right kneejoint shows 50,000 leukocytes/mm'. A Gram stain of this fluid
and a cervical exudate show gram-negative diplococci . Analysis of her serum for complement acti vity indicates
that a total hemolytic complement activity is only 5% of normal. A defici ency in which o f the followi ng
complement proteins enhanced the likelihood of this infection in this patient?
0A) Activating enzymes (C1 r, C1s, C2b, Bb, D)
OB) Complement receptors (CR1, CR2, CR3, CR4, C1qR)
OC) Membrane attack complex (C5b, C6, C7, ca, C9)
OD) Membrane-binding proteins (C4b, C3b)
o E) Peptide mediators of inflammation (C5a, C3a, C4a)

16. The tabl e shows survi val information for patients v.,t,o had an operation for a parti cular form of cancer:
Patients
at the
Beginning

Patients
Who Died
During

Interval of the Interval the Interval


0 - 1 year
1-2
years
2-3
years
3-4
years

Patients
Surviving
This Interval

500

100

(%)

400

50

87.5

350

35

90

315

31

90

80

Based on this informati on, if a patient survives for 1 year, which of the following is the probability that he or she
will survive for 4 years?

0A) (0.875 - 0.8) x 0.9 = 0.0675


0 B) 0.8 x 0.875 x 0.9 x 0.9 = 0.567
OC) 0.875 x 0.9 = 0.7875
OD) 0.875 x 0.9 x 0.9 = 0.7088
O E) 0.9
0 F) 0.9 - 0.875 = 0.025

17. In a cohort study, 10,000 men who consume al cohol were compared with 10,000 men who do not consume
al cohol. The results show that men who consume alcohol have an increased ri sk for cancer of the bl adder that
Is stati sticall y si gnificant at the 5% level. Whi ch of the following is the most appropriate interpretati on of thi s
result?
0 A) Al least 5% of the bladder cancers can be attributed to consumpti on of alcohol
0 B) Al least 5% of consumers of alcohol wi ll develop cancer of the bl adder
OC) Such an effect woul d occur by chance in less than 5% of repeated samples ofthis size
OD) There is at l east a 5% increased risk for bl adder cancer among consumers ofalcohol
O E) Men who drink alcohol are 5 ti mes more l ikel y to devel op bl adder cancer than those who do not drink alcohol
18. A 2-year-old boy wi th normal size, wei ght, and development is brought to the physi cian because of a painful,
swollen left knee for the past week. He has had two epi sodes of asymptomatic hematuri a during the past 6
months. Both parents are heal thy. He has achieved developmental mi lestones at appropriate ages. Hi s height
and weight are at the 70th percenti le. Exami nati on shows brui ses and resolving hematomas on both el bows and
si gns of cutaneous hemorrhages at the site of a deep spl inter. Laboratory studies show:
Hemoglobi n
Erythrocyte count
Leukocyte count
Bl eeding ti me
Partial thrombopl astin
ti me

12 g/dl
normal
normal
normal
increased

The most likel y di agnosis can be proven by the therapeutic response to a single injection of which of the
following material s?

O A) Factor I (fibri nogen)


o B) Factor VI II (antihemophili c factor)
OC) Factor IX (pl asma thromboplastin component)
OD) Folic acid
o E) Vi tami n K

19. A 4-year-old gir l requires a splenectomy after an automobile collision. Which of the following i s most likely to be
a l ong-term compl ication in this patient?
OA) Cytomegalovirus infection
oB) Lymphopenia
OC) Non-Hodgkin lymphoma
00) streplococcal infection
o E) Thrombocytopenia
20. A 58-year-old man has had progressive weakness over the past 8 months. Exam ination shows weakness and
muscle atrophy in the upper and lower extremities. Muscle stretch reflexes are decreased in the upper
extrem i ties and increased in the lower extremities. He has no sensory deficits. Which of the following findings is
most l ikely?
O A) Cystic lesion invoMng midline structures and both ventral horns in the cervical enlargement
0 B) Demyel ination of the left corticospinal tract at all levels of the cord
O C) Lesion of the left anterolateral quadrant a t C-2
0D) Loss of myelin in the dorsal col umn white matter
o E) Loss of myelin in the lateral col umn while matter and loss of ventral horn motoneurons
21. A newborn has strabismus. Which of the following is the rationale for surgically correcting this problem during
earty childhood?
0 A) Cataract formation is more l ikely in m isaligned eyes
O B) Neither eye will develop normal monocular vision
OC) Normal binocular vision will not develop
OD) Retinal ganglion cells in the temporal hem iretinas will degenerate
O E) Separate representations of each reti na will develop on the visual cortex
22. During an experimental study of the mechanisms of nerve conduction, a researcher isolates a nerve and uses a
microelectrode to measure its membrane potential distal to a site of electrical stimulation. The resting potential
of the nerve is found to be -90 mV. The following membrane potentials are then recorded at various points in
time after stimulation:
Time After Stimulation
(msec)
0
0.5
1
2

Membrane Potential
-90mV
+25mV
-100 mV
-90mV

The change in membrane potential from the resting potential occurring between 0.5 and 1 milliseconds is most
likel y caused by increased permeability to which of the following ions?
OA) Ca'
OB) Cf
OC) K'
00) Na
OE) po

23. A 37-year-otd man is brought to the emergency department after falling down a flight of stairs. Physical
examination shows acute alcohol intoxication, a bruise over the left temp le, and a swollen, painful wrist. Over
the next 30 minutes, his level of consciousness deteriorates, and the left pup il becomes dilated. Which of the
following is the most likely cause of the neurologic findings?
0A) Cerebral contusion
0B) Compression of the facial nerve
OC) Laceration of the m iddle meningeal artery
00) Laceration ofthe superior cerebral (bridging) veins
o E) Laceration of the temporal artery
0 F) Occul t brain tumor unmasked by the fall

24. A 10-year-old gi rl is brought to the physici an because of several 1- to 3-cm, li ght brown macul es on her trunk as
shown in the photograph. Physical examination shows axi llary freckl ing. Neurologi c exami nati on shows no focal
findings. Whi ch ofthe following is the most likely di agnosis?
0 A) Dysplasti c nevus syndrome
o B) Neurofibromatosis
OC) Osl er-Weber-Rendu di sease
O 0) Tuberous scl erosi s
O E) von Hippel-Lindau disease

25. A 10-year-old boy is brought to the emergency department by his parents 1 hour after he fell down the steps in
the mi ddle of the night. Hi s mother reports that he has a history of wanderi ng around the house at night, but he
never seems to recall it afterwards. During these episodes, he does not scream or appear frightened. He is
otherwi se heal thy. and he takes no medi cati ons. Physi cal exami nation shows a 2-cm area of edema and
ecchymosi s over the forehead. Pol ysomnography is most l ikely to show an abnormal ity in which of the foll owi ng
stages of sleep?
OA) Stage 1
O B) Stage 2
OC) Slow-wave (del ta)
OD) REM
26. A 69-year-ol d man has been taking a dopami ne agoni st for 4 weeks for Parkinson di sease. He has had some
benefit from this treatment and decides on his own to increase his dosage. Which of the following adverse
effects Is most likel y to result from this increase in dosage?
o A) Depressi on
O B) Exaggerated tremor
OC) Generalized hypotonla
OD) Psychosis
O E) Tardi ve dyskinesia

27. A 4-year-old chil d develops a fever, sore throat, ulcerated lesions on the oral mucosa, and a rash on the hands
and feet. Whi ch of the following is the most li kely viral cause?
0A) Adenovirus type 12
o B) Coxsackievirus
OC) Cytomegal ovirus
0D) Influenza virus
OE) Marburg virus
O F) Mumps virus
OG) Parvovirus B19
oH) Pol iovirus type 1
o I) Rhi novirus type 8

28. Which of the following label ed structures in the electron micrograph is present in cardiac muscle but not in
skeletal muscl e?

O A) OB) OC) OD) O E)


29. A 73-y-ear-old man is brought to the emergency department because of excruciating low back pain thal began
while he was painting his garage 1 day ago. He vi sits the physician every 6 months for monitoring of
hypertension well controlled with a diuretic. An x-ray of the spine shows several osteoblastic lesions In the
vertebrae. Which of the following is the most likely diagnosis?
O A) Malignant lymphoma
O B) Metastatic lung carcinoma
oC) Metastatic prostate car,clnoma
0 D) Multiple myeloma
o E) Osteosarcoma

30. A 9-year-old gir l with cystic fibrosis is brought to the emergency department because of a 12-hour history of
fever, severe shortness of breath, and cough productive of thick sputum. Her temperature is 39c (102.2F),
pulse is 80/min, respi rations are 30/min, and blood pressure is 90/60 mm Hg. Diffuse wheezing and scattered
rhonchi are heard on auscultation of the chest. The CT scan of the chest shown depicts multiple dilated
structures which are a result of multiple infections (as indicated by the arrows). The dilated structures are most
likely which of the following?
O A) Bronchi
o B) Interlobular septae
OC) Pulmonary arteries
OD) Pulmonary veins
o E) Subpleural septae

31. A 25-year-old man comes to the emergency department because of a productive cough, fever, night sweats,
shortness of breath, malaise, weight loss, and poor appetite that have progressivel y worsened over the past 3
weeks. He has worked in construction in the USA since emigrating from Thailand 1 year ago. His temperature i s
38.3C (101 F) and respirations are 30/min. He a.ppears emaciated and has prominent needle tracks on hi s
forearms. Which of the following i s the most likely diagnosi s?
0 A) Acute respiratory distress syndrome
o B) Atyp ical viral pneumonia
OC) Di sseminated tuberculosis
o D) Hypersensitivity pneumoniti s
O E) Metastatic carcinoma of the lung
0 F) Mycop lasmal pneumonia
32. An 80-year-old woman dies shortly after admission to the hospital because of a day history of progressive ly
increasing shortness of breath. She had a history ofheart fai lure. Examination of the lungs dur ing autopsy
shows alveolar damage with distended lymphatics, congested alveo lar capillaries, and pink-staining
homogenous proteinaceous hyaline material in a lveoli. Which of the following is the most likel y cause of these
findings?
O A) Amyloidosi s
O B) Aspiration pneumonitis
OC) Emphysema
O D) Pneum onia
o E) Pulmonary edema
o F) Septic shock
33. An 8year-old woman comes to the physician for an annual examination. The physician hears a new murmur
that i s indicative of aortic stenosis. Which of the following diagrams best depicts the murmur?
Sf

S2

AI..dm1111n11.I

$1

lumnuu,,..I
cI
lmm111...I
lmmmmuml
e! I111111n111111ll
0

OA) O B) OC) 00) O E)


34. A 14-year-ol d girl is admitted to the hospital because of palpitations, an irregular pulse, migratory polyarthritis,
and a low-grade fever . She had acute tonsillitis 3 weeks ago. Assays for anti streptococcal antibodies and C
reactive proteins are positive. Her 1 0-day stay in the hospital is uneventful, and her ECG changes have
resolved. Prophylactic anti streptococcal therapy is used for prevention of which of the following long-term
sequel ae?
o A) Aschoff bodies
o B) Fibrinous pericarditis
OC) Mitra! valve stenosis
o0) Polyarthritis
O E) Sydenham chorea

35. Coronary bl ood flow is studied in an ani mal model of ischemic heart di sease. In thi s model, coronary bl ood flow
is normal when the ani mal is at rest, but when the ani mal becomes acti ve coronary vasoconstri ction occurs and
coronary bl ood flow decreases. The most l ikely cause of the observed vasoconstri ction is the action of which
of the foll owi ng on vascular smooth muscle?
O A) Adenosi ne
0 B) Nitri c oxi de
oC) Norepinephri ne
OD) Oxygen
o E) Prostacyclin
36. A 10-year-old gi rl with a rare form offamili al dysautonomi a is brought to a uni versi ty medical center that
speciali zes in treating patients with her condition. She has a debilitating orthostati c hypotension that is most
severe in the morning despi te treatment wi th fludrocortisone. After eval uation, a new oral medi cation is
prescribed to increase her bl ood pressure. Thi s new drug most likely bel ongs to which of the followi ng classes?
OA) Pure a -adrenergi c agonist
0 B) Pure [3-adrenergi c agonist
OC) Pure [3 -adrenergi c antagonist
OD) Pure cholinergic muscari nic agonist
o E) Pure chol inergic ni coti nic antagonist
37. A 4-year-old boy has had recurrent watery diarrhea, abdominal di stenti on, and increased borborygmi for 3
months. The elimination of most dairy products from his diet causes his symptoms to abate. In which of the
folowing portions of the gastrointestinal tract is an enzyme deficiency most l ikely?
OA) Col on
O B) Jejunum
OC) liver
O D) Pancreas
O E) Stomach
38. A 42-year-old man who is a resident of the USA comes to the physi cian because of greasy fousmelling stool s
and a 5.5-kg (12-l b) wei ght loss over the past 6 months. He has a 6-year history of chronic abdominal and back
pai n. The epi gastri um is tender on deep pal pation. A CT scan of the abdomen shows pancreatic atrophy and
calcificati ons. Whi ch of the following is the most li kely cause of hi s disease?
O A) Alcohol dependence
0 B) Congeni tal pancreati c cyst
o C) Di abetes mellitus
OD) Gallstones
o E) Heredity
O F) Pancreas di visum
OG) Protei n-cal ori e mal nutrition
39. A 5S-year-old man comes to the physici an because of a 2 -month history of intermi ttent, severe, sharp
abdominal pai n that occurs pri mari ly after meal s and lasts about 30 to 60 minutes. There is no fever, diarrhea,
or weight loss. Physical examination shows no abnormal ities. Which of the following i s most l ikely responsible

forthese symptoms?

O A) Acute cholecysti tis


O B) Ascendi ng cholangi tis
OC) Bil iary col ic
OD) Pri mary bi liary ci rrhosis
o E) Scl erosing chol angi ti s

40. A 22-year-old woman is brought to the emergency department unconscious 30 mi nutes after sustaini ng multipl e
injuri es in a motor vehicl e collisi on. Her temperature is 36C (96.8F), pul se is 82/mi n, respi rations are 6/mi n,
and blood pressure is 110175 mm Hg. Physical examination shows si gnificant head trauma with a depressed
segment of parietal bone. Arteri al bl ood gas anal ysis on room air shows:
pH
Pco,
Po,
HCO

7.17
80mm
Hg
50mm
Hg
28
mEq/L

Whi ch of the following renal compensatory responses will occur in this patient?
O A) Decreased H. secretion into the tubu e
l
o B) Decreased proximal tubule HCO reabsorpti on

OC) Decreased ti tratabl e acid excreti on


0 D)
Increased HCO excretion

0 E) Increased NH Cl excreti on

41. Two weeks after an upper respiratory illness, a 6-year-ol d boy devel ops peri orbi tal and peripheral edema. Other
physical findi ngs are normal; bl ood pressure is 110/60 mm Hg. Laboratory studies show:
Serum
Creatinine
Protein
Antistreptol ysi n 0
ti ter
Urine
Protein
Erythrocytes

0.8 mg/dl
4.0 g/dl
80 Todd units (N=12 -166)
4+
rare

Whi ch of the following is the most likely di agnosis?


O A) Al port syndrome
0 B) Goodpasture syndrome
O C) Membranoprol iferative glomerul onephritis
OD) Minimal change di sease
O E) Poststreptococcal glomerul onephritis
42. A 34-year-ol d man has multicentric capillary hemangi oblastomas withi n the cerebellum, retina, and spi nal cord.
He is at increased risk for developing which of the following neoplasms in the kidney?
O A) Angi omyofipoma
o B) Nephroblastoma (Wi lms tumor)
OC) Oncocytoma
OD) Renal cell carci noma
O E) Rhabdomyosarcoma

43. A 32-year-ol d man with hyper tension comes to the physician for a follow-up examination. He has been treated
wit h a p-adrenergic blocking agent for the past 6 weeks. His bl ood pressure now is 145/90 mm Hg. Physical
examination shows no abnormalities. Hydrochloroth iazide is added to the medication regimen. Which of the
following best describes the renal mechanism of action of hydrochlorothiazide in this patient?
O A) Blocks the mlneralocortlcoid receptor in the late distal tubule and collecting duct
0 B)
Blocks the Na-K-2cf symporter
OC) Blocks the sodium channel in the late distal tubule and collecting duct
O D)
Inhibits the Na'-cf symporter
o E) Produces osmotic diuresis
O F) Stimulates carbonic anhydrase In the proximal tubule

1e--I

H)POthl ITfJi

r1,. ., ;,

e _!...;c.,-,

r.:-,;;:-::,c,_

ISes1 oli eens(

y
J_

(t) :c u1Stw1Qo:oryct1cc1(1n tt.elo1go:1 c:clfl:iuuc


t.dies11.:{l:'litih:bt!orye:=,l cnl ltt'! -11l cd/'11 ::1;e

44. During an exper imental study of male reproduct ive function. a researcher administers var ious neutral izing
antibodies to male experimental animals in order to map regul atory factor s that influence testicul ar function. A
model of pos itive and negative regulatory pathways is developed and shown in the diagram. If the arrow labeled
X represents testosterone, which of the following hormones most likely mediated the effect indicated by the
arrow labeled Y?
O A) Follicle-stimulating hormone
O B) Gonadotropin-rel easing hormone
O C) lnhibi n
OD) Luteinizing hormone
o E) Testosterone
45. A 32-year-old nulligravid woman comes to the physician because she has not been able to conceive for 2
years. She has a 3-year hi story of pelvic pain and dyspareunia. PeMc examination shows enlargement of the
left adnexa and nodularity of the cude-sac. Laparoscopy shows dark areas 3 to 5 mm in d iameter throughout
the cul- d e -sac. The left adnexa are covered by adhesions and cannot be vi sualized. A biopsy speci men of the
cude-sac lesion is most likely to show which of lhe foll owing?
0 A) Adenocarcinoma
o B) Endomelr ial glands and stroma
OC) Ep ithelial inclusion cysts
OD) Granuloma
O E) Mesothelial tumor
46. A 32-year-old man who is a bodybuilder is being evaluated for infertil ity. He has two children who were born
when he ,vas in hi s eay 20s. He has been taking a supplement for the past 3 year s to increase muscle mass
and strength. He takes no prescr ibed medications. He appears to be healthy and is muscular. Physical
examinati on shows testicul ar atrophy. Evaluation shows azoospermia. This patient is most likely taking which of
the following substances?
O A) Creati ne
o B) Erythropoietin
OC) Grev.th hormone
o D) Prednisone
O E) Progesterone
0 F) Testosterone

NBME
Form 3 Section 3

NBME
Form 3 Section 4

5. In a cl ini cal study, 100 subjects are exami ned for a parti cular geneti c alterati on hypothesized to be related to
prostate cancer. The results are shown in the table.
Genotype

AA
AB
BB

Number of
Controls

30
25
45

If A is one allel e, and Bis the other, which of the foll owi ng is the frequency of the B allel e?
OA) 25/200
O B) 45/200
OC) 50/200
OD) 90/200
O E) 1 15/200


6. A 30-year-old woman devel ops serum sickness (immune compl ex formation) following parenteral admini stration
of peni cillin. A pl asma sampl e from thi s pati ent is found to be strongl y chemotactic for neutrophi ls, monocytes,
eosi nophils, and basophils. Which of the followi ng mediators is most l ikely responsible for the chemotacti c
acti vity in thi s patient's plasma?
OA) C3a
O B) C3b
oc) C5a
OD) C5b
O E) C5-C9


7. A 15-year-old girl is brought to the physi cian for a wel child examination. She is upset because her ri ght breast is
larger than the left, and she will not attend physical education cl asses because she is embarrassed to undress.
Breast development began 2 years ago. Menarche has not yet occurred. Exami nation shows the right breast to
be twi ce as large as the left; the right breast is Tanner stage 4, and the left breast is Tanner stage 3. Which of
the following is the most appropriate inteNenti on?
O A) Reassurance
o B) Psychological evaluation
OC) Estrogen therapy
o D) Progesterone therapy
O E) Operati ve treatment

8. A 21-year-old man has the sudden onset of fever, sweating, cyanosis, tachycardia, tachypnea, unstable bl ood
pressure, and muscle rigidi ty after admini stration of halothane and succi nyl chol ine for an appendectomy. An
abnormal ity in whi ch of the following is the most li kely cause of this reaction?
0 A) ca'' re easing channe
ls
l
O B) Camitine palmi toyl transferase
OC) Mitochondrial ATPase
OD)
Na- K ATPase
o E) Phosphorylase -kinase

.i

lO

.; 10

5
2

toA

2.ao 21----<-

-+ - -+- - +---1

: 0115
iJ:::::::l==::::l:==::t:==:t::::::I
J

,..__. _,_ _ ....._ _ ..__,


3

6
9
Ti me (hou1s)

12

9. The graph shows the pl asma concentration of an antibiotic as a functi on of time after an intravenous bol us. If the
anti biotic is admini stered continuously by intravenous infusion, approxi matel y how many hours will it take for the
plasma concentration of the drug to first achieve almost 97% of i ts steady-state concentration?
OA) 3
O B) 9
OC) 15
OD) 21
O E) 27

10. A 61-year-old woman has been treated for the past 10 years with warfari n anti coagulant therapy. Her
prothrombin ti me (INR) has remained stabl e. Whi ch of the following curves shows the most likel y change in her
prothrombin time followi ng addition of cimetidine to her regi men?

---- -e
,,,--- --c
ci-- t----t -------D
,.--

---A

Q.'--. ,- e<I - - - - Cimirf!n9 sa't


Time (d";ls)-

OA) OB) OC) OD) OE)

1 1 . A 58-year-old man devel ops esophageal candidiasi s 2 days after completi ng antibiotic therapy for si nusiti s. An
oral antifungal drug is initiated. Medi cations include glipi zi de for type 2 diabetes mellitus and warfarin because
ofprevious cardiac valve replacement. Si x days later, his candidiasi s has improved, but his serum gl ucose
concentration has decreased from 120 mg/dl to 80 mg/dl, and hi s prothrombi n ti me has increased from 30
seconds (INR=2.5) to 40 seconds (INR=3.3). Whi ch of the following mechanisms of action explains the
anti fungal activi ty of thi s drug?
oA) Decreasing hepatic bl ood flow
O B) Decreasing squal ene epoxidase
OC) Decreasing synthesis of ergosterol
OD) Increasing thymi dyl ate biosynthesis
0 E) Preventing mitosi s by bindi ng tubul in

12. A 6S-year-old woman has fever, back pain, and di zzi ness on standing. Her blood pressure is recorded as
70/0 mm Hg in the emergency department. Bl ood and uri ne cultures subsequentl y grow Escherichia coli. Whi ch
of the foll owi ng bacteri al structures is most li kely responsi bl e for the hypotensi on?
oA) Capsular pol ysacchari de
O B) Fimbri a
OC) Lipid A
oD) o antigen
O E) Pori n

13. A 3-year-old boy is evaluated for bi lateral conjunctivi ti s and a dry cough. He attends a day-care center, and
most of hi s cl assmates have simi lar symptoms. Physical exami nati on shows intense bilateral conjuncti val
erythema and a mild serous exudate. The lungs are clear to auscul tation. Which of the following viruses is the
most li kely cause of this chil d' s illness?
O A) Adenovi rus
0 B) Mumps virus
O C) Parvovi rus
OD) Respi ratory syncyti al virus
o E) Rhi novirus
o F) Rotavl rus

14. Cells from a pancreatic carcinoma express very small amounts of class I MHC molecul es and are resistant to
recogniti on and killing by CDS+ T l ymphocytes isol ated from the host indivi dual. Substanti al tumor cell lysis is
observed when these cell s are cul tured in the presence of a mixed leukocyte popul ation isolated from the
peripheral bl ood of a normal (cancer-free) indi vidual. Whi ch of the following leukocytes in the bl ood of the
nonnal indi vidual are most l ikel y responsi bl e for the in vitro lysis of the tumor cell s?
0 A) B-lymphocyte effector (pl asma) cells
o B) CD4+ T lymphocytes
OC) CDS+ T lymphocytes
o D) Eosinophi ls
O E) Macrophages
0 F) Natural killer cells
OG) Neutrophils

15. A 4-year-old boy devel ops uncontrolled bleedi ng after mi nor trauma. He has a hi story of recurrent
si nopulmonary i nfections and eczematous rashes. Whi ch of the following predisposi ng conditi ons is most l ikel y
present?

oA) Bruton agammaglobul inemia

O B) Common vari able immunodefici ency


OC) Sel ective lgA deficiency
O D) Thymic hypop as a
! i
0 E) Wi skott-Aldri ch syndrome


16. A 3-year study is pl anned to assess the i ncidence and etiol ogy of respiratory di sease in 574 indi viduals greater
than age 60. The study popul ati on consists oftwo groups: one group cares for chi ldren under the age of 5; the
second group does not care for chil dren. At the onset of respi ratory symptoms. cul tures and serol ogic studi es
will be performed. Whi ch of the following best describes thi s study desi gn?
O A) Case-control
O B) Case series
O C) Cl ini cal trial
OD) Cohort
o E) Communi ty survey

17. The graph shows the oxygen di ssociation curve of hemoglobi n A for a healthy indi vi dual. Which of the followi ng
curves best represents the di ssociation curve for the same indi vidual after extensi ve physical training at hi gh
al titudes?

D
40

eo

120

P02 (mmHg)

1so

----- l'P.alll'Yyi n:fMduat

OA) O B) OC) OD)

18. A 68-year-old woman has a pathologi c fracture of the hip. A photomicrograph of tissue obtained on aspi ration of
bone marrow is shown. Which of the followi ng is the most likel y di agnosis?
O A) Acute myel ogenous l eukemi a
O B) Chronic lymphocytic leukemia
OC) Langerhans cell hi stiocytosis
OD) Metastatic adenocarcinoma
o E) Multi ple myel oma

19. A 40-year-old woman comes to the physi cian because of 'too much stress in my l ife now." She descri bes
feel ing depressed and tense because of a recent marital separation, a job change, the need to relocate to
another city, and the worsening of her 8-year-ol d son's asthma. Physical examination shows no abnormali ties.
Laboratory studies are most likely to show which of the followi ng abnormaliti es?
O A) Decreased l ymphocyte reacti vi ty to anti gens
0 B) Decreased serum prolactin concentrati on
o C) Increased serum cal cium concentration
OD) Increased serum thyroid-stimul ating honnone concentrati on
o E) Mil d normochromi c, normocytic anemia

20. A 74-year-old woman has a stroke that resul ts in weakness of the left hand and forearm. Whi ch of the following
arteries is most likel y occluded?
O A) Left anteri or cerebral
o B) Left mi ddl e cerebral
OC) Right anterior cerebral
0 D) Ri ght middl e cerebral

20. A 74-year-old woman has a stroke that resul ts in weakness of the left hand and forearm. Whi ch of the following
arteries is most li kel y occluded?
O A) Left anteri or cerebral
o B) Left mi ddl e cerebral
OC) Right anterior cerebral
0 D) Ri ght middl e cerebral

21. A 45-year-old woman has weakness in the anterior thi gh and is unabl e to extend her leg at the knee. Fl exi on of
the hi p and knee is nonnal. Sensati on is absent on the medial si de of the leg. Whi ch of the following nerves is
most likely injured?
oA) Common peroneal
O B) Femoral
OC) Lumbosacral trunk
o D) Obturator
O E) Ti bial

22. An uni mmunized 2-year-old gi rl is brought to the emergency department after havi ng a generali zed tonic-clonic
sei zure. She has had a fever and an upper respiratory tract infection for the past 4 days. A Gram stain of
cerebrospinal fluid shows numerous gram-negative coccobacilli. Whi ch of the following vaccines would most
likel y have prevented this infecti on?
OA) DTP
O B) Hib
O C) Meningococcal
OD) Pneumococcal
o E) Sal monella!

23. A 6-year-old boy falls off hi s bicycl e and hits hi s head; he has a transient loss of consci ousness. After a brief
recovery, he becomes progressively more somnol ent. On examination, he has weakness in the right si de of the
face and right upper extremity. CT scan ofthe head is shown. The injury at the site l abel ed X is a consequence
of rupture of which of the followi ng arteri es?
oA) Anteri or cerebral
O B) Anteri or choroi dal
OC) Anter or communicati ng
o D) Middl ei cerebral
O E) Middl e meningeal

24. A 45-year-old man has had general ized jerki ng movements in his extremiti es, progressi ve dementia, and
psychotic behavi or for the past year. His mother died of a simi lar illness when she was 50 years ol d. A CT scan
of the head is most likel y to show atrophy of whi ch of the following areas of the brai n?
oA) Caudate nucl eus
0 B) Gl obus pallidus
OC) Hippocampus
OD) Putamen
O E) Substantia nigra

25. A 22-year-old woman is brought to the emergency department by pol ice after they found her walki ng naked
down the mi ddl e of the street. She is screaming that the poli ce are planni ng to hurt her and that they are
members of a secret soci ety that has been following her for months. She is not cooperative wi th hi story taki ng
and says quickly, " Plants areni often watered. Those doorknobs need polishi ng. Where's the merry-go-round?"
She is emaciated, dirty, and disheveled. Laboratory studi es show mi ld microcytic anemi a. Urine toxi cology
screening is negative. Whi ch of the following is the most li kel y diagnosis?
oA) Adjustment di sorder wi th disturbance of conduct
0 B) Di ssociati ve fugue
OC) Schizophreniform disorder
OD) Schizotypal personal ity disorder
0 E) Substance-induced mood disorder

26. A 3-year-old girl is brought to the emergency department by her parents 1 D mi nutes after they found her playing
wi th an empty medication bottle. The ibottle had contained 60 tablets of diazepam. Physi cal examination and
laboratory studies show no abnormal ti es. Thi s pati ent i s at greatest risk for which ofthe followi ng
compl ications?
O A) Acute tubul ar necrosis
0 B) Cardiac arrflythmia
O C) Hepati c fai lure
0 D) Respi ratory depressi on
o E) Sei zures

27. A 16-year-old boy is brought to the physi cian because he is much shorter than his peers. He is proportionally
devel oped and otherwise healthy. An x-ray of the wri st is recommended to assess bone age. Which of the
following radiographic findings will be an indication that his growth i s compl ete?
o A) Absence of the epiphyseal cartilaginous pl ate
O B) Absence of the pri mary ossification center
OC) Presence of calc fied mater al in the diaphysis
o D) Presence of calcii fied materii al in the epiphysis
O E) Presence of li nes of arrested growth

28. A 30-year-old woman who is menstruating devel ops tenderness of the right knee. She has had arthral gias
involvi ng her wrists and knees for the past 3 days. She has no urinary tract symptoms or vaginal di scharge.
The right knee is swollen and warm to the touch. Approximately 1 D necrotic pustules are present on her upper
and lower extremi ties. A Gram stai n of scrapi ngs from the cervical os is shown. Which ofthe followi ng is the
most likely causal organism?
o A) Ca/ymmatobacterium donovani
O B) Chlamydia trachomatis
OC) Haemophi/us ducreyi
o D) Neisseria gonorrhoeae
O E) Neisseria meningitidis

29. A 48-year-old man wi th chroni c pancreati c insuffici ency devel ops greasy, fousmelling stools. He has lost 12 kg
(26 lb) over the past 6 months. Which of the followi ng is most likel y to be decreased in thi s man?
O A) Chondrocyte formati on oftype I collagen

o B) Mineral ization of bone matrix

OC) Osteocl ast production of carbonic anhydrase II


0 D) Osteocyte production of alkaline phosphatase
o E) Uri nary excretion of hydroxyproli ne

30. A 10-year-old boy is being eval uated because of poor growth, recurrent episodes of sinusitis, and a chronic
cough. An xray of the chest shows evi dence of bronchiectasi s. A rare inheri ted di sorder causing immoti le,
structurally abnormal ci li a in the respiratory epithelium is diagnosed. This pati ent's impaired mucociliary
clearance is most li kel y to predispose hi m to pulmonary infecti ons wi th which of the following infectious agents?
O A) Candida albicans

O B) Cryplococcus neoformans
O C) Cytomegal ovirus
0 D) Mycobacterium avium- intracel/u/are
o E) Staphylococcus aureus

31. A previ ously heal thy 30-year-ol d man comes to the emergency department because of the sudden onset of ri ght
shoulder pain and shortness of breath. Breath sounds are absent over the ri ght lung. Hi s cervi cal trachea at the
superi or sternal border is shi fted to the left. The most likely cause ofhi s condi tion is rupture ofwhi ch ofthe
following?
O A) Anteri or hernia
0 B) Api cal pul monary bl eb
O C) Bronchial di verticul um
OD) Cystic adenomatoid mal formati on
o E) Posterolateral herni a

32. A 40-year-old man who works in a gravel pit is brought to the emergency department after bei ng hit in the chest
by a sl ab of cement. He does not smoke and does not have a cough. An xray of the chest does not show any
fractures, but mul ti pl e small nodul es are seen, some of which are cal ci fied, i n the upper lobes. A PPD skin test
is negative. Whi ch of the following is the most li kely diagnosis?
OA) Asbestosi s
0 B) Berylliosis
O C) Byssi nosis
OD) Industrial bronchi ti s
o E) Sil icosis

33. A 60-year-old man comes to the physici an for a follow-up exami nati on. He had a myocardial infarcti on ofthe
right ventricl e 3 weeks ago. Physi cal examination shows an accumulati on of free fluid in the peritoneal space.
Whi ch ofthe foll owing mechanisms i s the most l ikel y cause ofthi s findi ng?
oA) Decreased capill ary colloid osmotic pressure
O B) Increased capillary hydrostatic pressure
OC) Increased capillary permeability
o D) Increased l ymphati c colloid osmotic pressure
O E) Lymphatic obstruction

34. A 65-year-old man comes to the physici an because of recurrent li ghtheadedness. His ECG is shown. Whi ch of
the following is most li kel y responsi ble for the abnormal ECG pattern in this pati ent?
oA) Atri oventri cular conduction di sease
0 B) Atri oventri cular re-entrant pathway
OC) Ectopic focus in the atria
0 D) Ectopic focus in the ventricle
O E) Sinus node di sease

35. An 18-month-ol d boy is brought to the physici an' s office for a foll ow-up cardiac exami nation. He has a
conti nuous, grade 3/6, rumbl ing, machi nery-l ike murmur that is heard best at the mi d to upper left sternal border.
Whi ch of the foll owing is the most likely di agnosis?
o A) Mitra! valve regurgitati on
O B) Osti um pri mum defect
OC) Ostium secundum defect
o D) Patent ductus arteri osus
O E) Sinus of Valsalva fistula
o F) Ventri cular septa! defect

36. A previ ously heal thy 65-year-ol d man is brought to the emergency department because of severe chest pain for
1 hour. His pul se is 95/min, and bl ood pressure is 160/110 mm Hg. The lungs are clear to auscultation. An S

'

gallop and a grade 216 systol ic murmur are heard al the apex. An ECG shows 3 mm of ST-segment elevation in
leads V through V . The most appropri ate treatment includes i mmedi ate administration of whi ch of the
'
following?
o A) Captopril
O B) Di goxi n
OC) Furosemide
OD) Magnesium
O E) Ti ssue pl asminogen activator

37. Whi ch of the foll owing hormones, produced by the duodenal and proxi mal jejuna! mucosa, sti mul ates the release
of pancreati c jui ce ri ch in digesti ve enzymes?
O A) Chol ecystoki nin
o B) Gl ucagon
OC) Pancreatic pol ypeptide
0 D) Secretin
o E) Vasoactive intestinal polypeptide

38. An asymptomati c 56-year-old woman has occul t blood in her stool. Barium studies of her transverse colon show
an intral uminal mass. The surgical specimen is shown. Which of the following is the most li kel y diagnosi s?
O A) Adenocarcinoma
O B) Carcinoid tumor
OC) Pol ypoi d leiomyoma
OD) Tubul ar adenoma
adenoma
o E) Villous

44. A 1-year-old chil d has unilateral cryptorchidi sm. Even if thi s probl em is surgi cally corrected, the pati ent wi ll be al
increased risk for development of which of the followi ng in the cryptorchid testis?
O A) Germ cell neoplasm
o B) Leydi g cell tumor
OC) Recurrent orchiti s
0 D) Spermatocele
o E) Testi cular torsion


45. A 33-year-old woman, gravida 2, para 1 , at term gi ves bi rth to a 4167-g (9 lb 3 oz) male newborn by a
spontaneous 30-minute vagi nal del ivery. Shortl y after deli very of the pl acenta, there is a brisk flow of bl ood
from the uterus. Manual pal pati on of the uteri ne cavity shows a large fragment of retained placental materi al. A
curettage shows a fibrous nodul e of placental ti ssue that is firml y adherent to the uterine wall . Whi ch of the
following is the most li kel y cause of the bleedi ng in thi s patient?
0 A) Abruptio placentae
o B) Choriocarcinoma
OC) Factor XIII (transgl utaminase) deficiency
o D) Granulomatous endometritis
O E) Placenta accreta
0 F) Placenta previa

46. An 8-year-ol d boy is brought to the pedi atrici an's office because of a painful lump on his neck for the past 3
days. Exami nati on shows a 3-cm, tender, rounded, cysti c mass in the anteri or mi dl ine, midway between the
chin and sternal notch. The mass el evates with swallowing. Which ofthe foll owi ng best expl ains thi s finding?
o A) Failed differentiation of the third pharyngeal pouch
O B) Hypertrophy of the lingual tonsils
OC) Incompl ete atrophy of the thyrogl ossal duct
o D) Mal devel opment of the first branchial arch
0 E) Persi stence ofthe second branchial groove

47. A 5S-year-old man has had decreasing interest in sex over the past 2 years and impotence for the past 6
months. He has frequent headaches. Examination shows bi temporal visual fiel d defects, sparse faci al and pubi c
hai r, and testicul ar atrophy. MRI shows a 3-cm pitui tary tumor. Whi ch of the following sets of serum hormone
concentrations is most li kel y?
OA)
O B)
OC)
OD)
O E)

Luteinizing
Hormone

!
!

Normal

1
1

Testosterone

!
t

normal

!
t

48. A 6-month-ol d mal e infant is sluggish and appears edematous. He has coarse immature facies and poor muscle
tone. Which of the followi ng is the most l ikely diagnosi s?
0 A) Achondrop asia
o B) Congeni tal l hypothyroidi sm
OC) Di astrophi c dwarfism
OD) Multi ple carti lagi nous exostoses
o E) Muscul ar dystrophy
0 F) Myasthenia gravi s
OG) Osteogenesi s imperfecta
OH) Tay-Sachs di sease
0 I) Vi tami n 0-defici ency ri ckets

NBME
Form 2 Section 1

6. A 54-year-ol d woman wi th type 2 diabetes melli tus well contro lled with oral hypogl ycemic agents comes to the
physician because of a 2-week hi story of swelli ng i n her left knee. She tells the physi ci an that her knee is very
stiff when she awakens in the morning
. and at bedti me. The pain and sti ffness worsen wi th exerti on. Low-dose
ibuprofen resul ts in incompl ete rel ief Which of the foll owi ng best expl ai ns why a larger dose of ibuprofen is more
appropri ate?
o A) The anti-inflammatory dose is nearl y twi ce the anal gesi c dose
o B) The drug is excreted at a fast rate because of renal compl ications of diabetes mellitus
OC) Increasi ng the dose extends the hal f-l ife of the drug
o D) Menopausal women metabol ize the drug at a fasIer rate than premenopausal women
o E) Oral hypoglycemic agents interfere wi th protein binding of ibuprofen
7. A 37-year-ol d man develops a spi king fever in the intensi ve care unit after an operation to control an abdomi nal
hemorrhage. Eval uation does not show a focal infecti on. Therapy v.ith a broad-spectrum cephal osporin is
ini tiated. After 3 days of antibioti c therapy, the pati ent has abdomi nal pai n and profuse watery diarrhea.
Examination of stool for ova and parasi tes is negative. stool tests posi ti ve for C/ostridium difficile antigen. The
most appropri ate therapy is

8. The photomi crograph i s of lung tissue as seen at autopsy of a pati ent with an occupational exposure to
o A) asbestos
O B) berylli um
OC) coal dust
OD) iron
O E) si li ca
9. A 35-year-old marri ed man comes to the physi ci an because of a 2-day history of burning wi th urinati on. He tells
the physi ci an that he had an affai r whil e on a business tri p 2 weeks ago. The next day, the pati ent's wi fe
tel ephones the physician, stating, 'My husband saw you yesterday. Does he have a sexua lly transmi tted
disease?" Which of the following responses by the physi ci an is most appropri ate?
o A) "I recommend that you see your gynecol ogist as soon as possible."
o B) "I suggest that you ask your husband if he has had any sexual relations outside the marriage."
OC) "I'm sorry, but I will need your husband's permi ssi on to di scuss his care v.ilh you."

OD) "I'm sure everythi ng is going to be all ri ght. "

o E) "Perhaps you and your husband should seek marital counsel ing."
10. A 25-year- ol d man swallows 100 ml of carbon tetrachlori de in a sui ci de attempt. Which of the following is the
most likel y outcome?
o A) Central nervous system fail ure as a consequence of DNA damage in neurons
o B) Gastri c toxi ci ty rel ated to erosi ve gastritis
OC) Heart failure as a result of lipi d peroxidation of cardiac myocytes
o D) No toxi ci ty because the kidneys will excrete the carbon tetrachl ori de
o E) Severe hepatoloxi ci ty as a consequence of damage to the pl asma membranes

1 1 . A genetic el ement is found to contai n several genes and to catal yze its own movement withi n and betv,een
chromosomes. Inverted 3nucl eoti de repeats are present at the ends of thi s geneti c element. This geneti c
el ement i n hepatocytes is incapabl e of repl icati ng independently. Based on thi s description, thi s geneti c
el ement is best categori zed as whi ch of the followi ng?
o A) Compl ex transposon
o B) Insertion sequence
OC) Pl asmi d
OD) Regul on
12. A 55-year-ol d man with di abetes mellitus has burni ng dysesthesias over the l ateral aspect of his left lower leg.
Capsaicin-containing cream is effective in allevi ating the symptoms. The substance most l ikel y to have caused
the dysesthesias is
o A) ADH (vasopressi n)
o B) angiotensin II
OC) dopami ne
o D) inositol
o E) neuropeptide Y
o F) ni tric oxi de
OG) norepi nephri ne
o H) substance P

13. A 2-day-ol d mal e neonate is transferred to the neonatal intensi ve care unit because of tachypnea and intense
cyanosi s. There is a prominent l ift and a soft, systol ic murmur is heard al ong the left sternal border. S is loud
1
at the lower left sternum. X-ray fil m of the chest shows a normal heart size and pul monary vascul ar pattern.
Echocardi ography shows the aorta lyi ng anterior and to the right of the pul monary artery. Whi ch of the followi ng
is the most l ikely diagnosi s?
0 A) Aorti c coarctati on
0 B) Kartagenes syndrome
OC) Patent ductus arteri osus
0 D) Transposition of the great vessels
0 E) Ventri cul ar septal defect
14. A previ ousl y healthy 32-year-ol d woman has the onset of recurrent headaches. She devel ops nausea, vomi ting,
weakness, abdominal pain, and easy brui sability. Her temperature is 38.5 C (101 .3 F). Laboratory studies
show:
Hemogl obin
L eukocyte count

Mean corpuscul ar vol ume


Pl atel et count

6.9 g/dL

3
15,000/mm

78 m3

3
30,000/mm

She i s immediately admi tted to the hospi tal. Over the next few days, she devel ops progressi ve lethargy and
lapses i nto a coma. A peripheral bl ood smear is most likely to show
o A) macrocytes
o B) schi stocytes
OC) si ckle cells
o D) spherocytes
o E) target cells
15. The recombi nant vacci ne used to immunize against hepatiti s B infections el icits anti bodies against which of the
following vi ral proteins?
O A) HBcAg
O B) HBeAg
OC) HBsAg
o D) Pol ymerase

16. A 15-year-ol d gi rl has fever, nausea, and acute abdomi nal pai n and tenderness. Laboratory studies show a
neutrophi lic leukemoid reaction. A laparotomy and appendectomy are done. Whi ch of the followi ng hi stologic
findi ngs will establi sh the di agnosi s of acute appendicitis?
o A) Folli cul ar lymphoid hyperpl asi a
o B) Hi sti ocyti c infil trate of the l ami na propri a
OC) Lymphoi d infil trate of the mucosa
o D) MucosaI ul ceration
o E) Neutrophi lic infil trate of the muscularis
17. Duri ng the second and third tri mesters of pregnancy, women have increased l evel s of ci rculating
gl ucocorticoi ds but have few of the catabol ic responses associ ated wi th excess gl ucocorticoids. The most
li kel y explanation of this findi ng is increased
o A) autoanti bodies to steroi ds
o B) metaboli c cl earance of steroi ds
OC) steroid-binding protei ns
o D) steroid receptors
o E) steroid uptake mechanisms
18. A 59-year-ol d man comes to the physi ci an because he has had mal ai se, fati gue, and anorexi a since losing hi s
job 3 months ago. Serum y-gl utamyl transferase (GGT) acti vi ty is increased. Which of the following questi ons is
most appropri ate to initi ate a di scussion about hi s use of alcohol?
o A) "Do you thi nk you are a probl em dri nker?"
o B) "How much do you dri nk in a typical week?"
OC) "Is there something you have not tol d me so far?"
o D) "So, do you l ike to hit the bottl e sometimes?"
o E) 'What is your favori te drink?"
o F) "You are not a dri nker, are you?"
19. The preval ence of a disease is hal f as great in town A than it is in town B, but the inci dence of the disease is
no different in town A than it is in town B. Whi ch of the followi ng best expl ai ns these findings?
o A) The case fatali ty rate is twi ce as hi gh in town A
o B) The duration of the disease is twice as long in town B
OC) The number of new cases in town A i s twi ce as many as those in town B
o D) Peopl e in town A use medi cal care faci lities half as often as those in town B
o E) The proportion of asymptomatic cases is twice as much in town B than in town A
20. A 58-year-ol d man comes to the physi ci an because of bl ood in hi s uri ne. He had a nephrectomy 1o years ago
because of injuri es sustai ned in a motor vehicl e collision. Eval uation shows renal cell carcinoma in his
remaining kidney and pulmonary metastases. lnterl euki n-2 (IL-2) therapy is started. Three weeks l ater, CT scan
confirms regressi on of the tumor. Whi ch of the followi ng most l ikel y caused the regression?
o A) An effect of IL-2 on tumor vasculari zati on
o B) Cytotoxic effects of IL-2 on tumor cells
OC) Increased natural killer cell acti vi ty
OD) Induction of antitumor antibodi es by IL-2
o E) Induction of B -l ymphocyte prol iferation
o F) Spontaneous remi ssi on

21. A 41-year-ol d woman remains hospi tal ized and in bed for 10 days because of compl icati ons following a
hysterectomy for uteri ne lei omyomas. On the tenth day, she suddenl y devel ops severe respiratory distress and
collapses. She is resusci tated, but di es 2 days l ater. A l ung as seen at autopsy is shown. Which of the
following is the most l ikel y di agnosi s?
O A) Bronchopneumonia
0 B) Intersti ti al pneumonia
OC) L ung abscess
0 D) Pul monary infarct
0 E) Pul monary metastases
22. A 42-year-ol d man begins drug therapy for hypertension. Three months later, he has a chronic nonproducti ve
cough. Fi ndi ngs on x-ray fil ms of the chest are normal. He is most li kely taking which of the followi ng
antihypertensi ve drugs?
o A) Enalapri l
o B) Hydrochlorothi azide
OC) Prazosi n
o D) Propranol ol
o E) Verapamil
23. Which of the followi ng best descri bes how C/ostridi11m perfringens evades destructi on by neutrophil s in
patients wi th cl ostri di al myonecrosis?
o A) Damaging neutrophi l membranes by means of a membrane-damagi ng toxi n
o B) Preventi ng contact wi th neutrophi ls by means of a pol ysaccharide capsul e
OC) Preventi ng phagocytosi s by disruption of neutrophil acti n pol ymerization
o D) Survi val in the cytopl asm of a neutrophil
o E) Survi val in the phagosome of a macrophage
24. After 6 hours of excruci ating pain, a 26-year-old man passes a 3-mm cal cul us. The cal cul us is most l ikel y to be
composed primaril y of
OA) bilirubi n
o B) cal cii um sal ts
OC) cyst ne
o D) triple phosphate
o E) urate
25. A 53-year-ol d obese woman who has hypertensi on is brought to the emergency department after coll apsing.
Priorto admission, she had severe excruciating stabbi ng-l ike chest pain radi ating to her left shoul der. She is
anxi ous and in severe pain. Pul se is 110/min, and bl ood pressure is 160/80 mm Hg. On auscultati on, heart
sounds are di stant; pul sus paradoxus is present. ECG shows l ow vol tage and ST-T wave changes. An x-ray
film of the chest shows wi deni ng ofthe medi astinum. The most likel y diagnosis i s
o A) aortic di ssecti on wi th cardi ac tamponade
o B) hypertensi ve crisis
OC) massi ve pul monary thromboembol ism
o D) myocardial infarct wi th congestive fai lure
o E) rupture of the papillary muscle

26. A 7 -month-ol d femal e infant is brought to the physician' s office after having a tonic-cl onic sei zure. Fi ndi ngs
incl ude ataxi a, hypotoni a, heari ng loss, hai r loss, and a rash. Laboratory studi es show normal pl asma ami no
acid concentrations but abnormal uri nary organic aci d concentrations consi stent wi th multipl e carboxyl ase
deficiency. The administration of which of the foll owi ng is the most appropri ate treatment of thi s pati ent?
OA) Bi otin
o B) Vitami n B (ri boflavi n)
2
OC) Vitami n B (pyridoxine)
6
OD) Vitami n B (cyanocobal amin)
12
OE) Vitami n C
27. An examiner strokes a v,isp of cotton across a patient's left cornea. There is no response. The examiner then
strokes the cotton across the pati ent's ri ght ey-e; both eyes bl ink. The most likel y expl anati on ofthese findi ngs
is damage to wnich of the followi ng cranial nerves on the left?
OA) Opti c
o B) Ocul omotor
OC) Tri geminal
o D) Abducens
O E) Faci al
28. A 15-year-ol d gi rl has had a sore throat and temperatures to 37.7 C (1 00 F) for 3 days. Physi cal exami nation
3
shows erythematous pharyngeal mucosa and cervical lymphadenopathy. Leukocyte count is 12,000/mm with
60% lymphocytes. A sl ide aggl uti nation test is positi ve for anti bodies against Epstei r>-Barr vi rus. Thi s vi rus is
most likel y to repl icate initially in whi ch of the following cells?
0 A) Peri pheral blood B lymphocytes
0 B) Peri pheral blood monocytes
OC) Peri pheral blood T l ymphocytes
0 D) Pharyngeal B lymphocytes
0 E) Pharyngeal monocytes
0 F) Pharyngeal T lymphocytes
29. A mother reports that her 1 - year-old mal e infant is less acti ve. Weakness is progressi ve. The mother says
that even though her son is weak, his muscles appear larger than before. The most likely diagnosis is
o A) central core disease
o B) Duchenne' s muscular dystrophy
OC) li ml>-gi rdl e dystrophy
o D) myotoni c dystrophy
o E) nemal ine myopathy

30. The di agram shows a l iver l obul e. Lon!t-term destruction of the hepatocytes, collapse of the hi stologi c
archi tecture, and production of fibrous materi al in the areas indi cated by the arrow is most l ikel y to resul t in
which ofthe following?
0 A) Decreased intestinal motility
0 B) Gallstones
OC) Increased central venous pressure
0 D) Increased hepati c venous pressure
0 E) Increased portal pressure

31. A previ ousl y healthy 2-year-ol d boy has a 1x2-cm mass at the mi dl ine of the neck immediately above the
larynx. The mass is not tender and moves when he swallows. The mass is most li kel y to be
o A) a branchi al cl eft cyst
o B) lymphadenopathy
OC) a submental abscess
o D) a thyrogl ossal duct cyst
o E) the thyroid gl and
32. The ini ti al vacci nes agai nst Haemophi/us inf/uenzae type B prepared wi th capsular pol ysaccharides were not
very effecti ve in chil dren l ess than 2 years ol d. Subsequent vacci nes have been greatly improved by the
conjugati on of the capsular pol ysaccharides to one of several protei ns. Why does thi s conjugation improve
immunogeni ci ty?
o A) CD4+ T lymphocytes are activated
o B) Cytotoxic T lymphocyte responses are increased
OC) Direct activation of B lymphocytes by the vaccine is increased
o D) Nonimmune cl earance of the vacci ne is decreased
o E) Phagocytosis of the vaccine is increased
33. A 45-year-ol d man has a myocardial infarction. A coronary stent restores blood flow, but reperfusion causes a
paradoxi cal increase in damage. Whi ch of the followi ng is the most likely mechanism of the new myocardial
damage?
o A) Direct poisoni ng of mitochondrial enzymes
o B) Increase in lipid peroxi dalion of membranes
OC) Inhi bi ti on of ATPase-dependent transport
o D) Loss of intracellul ar amino aci ds
o E) Preventi on of cross-l inki ng of l abil e ami no acids
34. A 12-year-ol d boy is brought to the emergency department because of vomi ti ng and rapi d breathi ng. Over the
past 2 weeks, he has been drinki ng fluids and urinating excessivel y. He has l ost wei ght. He is thi n, afebri le,
and drowsy. Respi rati ons are rapi d and deep, and there is a frui ty odor on his breath. Laboratory studies
show:
Arterial bl ood pH
Serum
Gl ucose
Urea ni trogen (BUN)
Creatinine

7.0
575 mg/dL
74 mg/dL
2.0 mg/dL

Which of the foll owi ng is most li kel y to be increased in thi s pati ent's bl ood?
OA) Ammoni a
o B) Camitine
OC) !3-Hydroxybutyrate
o D) lsovaleric acid
O E) Sodium
35. A 42-year-ol d woman has a 3-month hi story of an intermi ttent reddi sh-brown di scharge from the ni ppl e of her
ri ght breast. No masses are pal pable in either breast. There is no pai n or tenderness. Whi ch of the followi ng is
the most likel y diagnosis?
o A) Fibroadenoma
o B) Fibrocystic disease
OC) lntraductal papilloma
o D) Lobul ar carci noma
o E) Masliti s
o F) Traumati c fat necrosis

36. An 82-year-ol d woman has become increasi ngl y di sori ented over the past year. She is pl easant and attempts
to be cooperative. She performs poorl y on a Mi ni -Mental State Examinati on. Pul se i s 80/min, and bl ood
pressure is 160/90 mm Hg. MRI of the head shows small l acunar lesi ons predomi nantly in the lenti cul ostri ate
arteri es of the internal capsules and in other wi del y disparate regions of both cerebral cortices. Which of the
following is the most l ikel y expl anati on for these findings?
o A) Dementia, Alzhei mer' s type
o B) Huntington' s disease
OC) Mul ti -infarct dementi a
o D) Mul ti pl e scl erosis
o E) Parki nsonism

-10

10

20

30

Fleu1el pressure (cmH,O)

37. The graph shows an isovol umetric pressure-flow curve obtai ned at 25% of total l ung capacity. An increase in
which of the following is most l ikel y to i ncrease the plateau value of the expiratory flow rate?
o A) ;>jrway resistance
o B) Amount of surfactant i n the alveol ar space
OC) Cil iary beating frequency
o D) Expi ratory effort
O E) Lung vol ume
o F) Parasympathetic activity of fibers innervating the airways
38. A 20-year-ol d woman takes ibuprofen and misoprosto) for rheumatoi d arthritis. She is planning to become
pregnant. Her physician advi ses her to stop misoprostol to avoi d whi ch of the followi ng?
o A) Fetal mal formations
o B) Intrauteri ne growth retardation
OC) Premature cl osure of the fetal ductus arteriosus
o D) Spontaneous aborti on
39. A 72-year-ol d woman comes to the physi ci an because of an episode of acute substernal chest pain that
occurred whil e she was rushi ng to catch a bus. She has basi lar crackles at both l ung bases. S, i s normal and S,
is decreased. A grade 3/6 systol ic murmur is heard best at the upper right sternal border and radi ates to the
neck. The foll owi ng data obtained during cardiac catheteri zati on were taken before and duri ng exerci se:
Aortic

pressure
(mm Hg)

Control
Exerci se

130/70
160/80

Left ventricul ar vol ume


End-diastolic EndHeart rate
(ml)
systolic
(/min)
(ml)
140
165

50
58

85
120

Whil e exercisi ng, the pati ent has shortness of brea.th but no chest pain. Which ofthe foll owi ng is the most li kely
cause of the shortness of breath?
0 A) Decreased cardi ac output duri ng exerci se
0 B) Decreased pul monary bl ood flow
OC) Increased aortic pressure
0 D) Increased end-diastol ic pressure of the left ventricl e during exercise
0 E) Tachycardia

40. A 3-year-ol d girl has mul tipl e bone fractures, mi ld hearing l oss, and blue sclerae. Her mother also has bl ue
sclerae and has near-total bil ateral hearing loss. These findi ngs are most likel y to be due to a defect in which
ofthe following protei ns?
o A) Dystrophi n
o B) El astin
OC) Fibri llin
OD) Myosi n
o E) Type I co llagen
41. A 23-year-ol d woman has prol onged epi sodes of wheezing, dyspnea, and anxiety that are reli eved by Jl adrenergi c agoni sts. The major mediator of her condi tion is
o A) bradyki ni n
o B) eosi nophil chemolactic factors
OC) leukotri enes
o D) prostagl andi n E
o E) thromboxane
42. In pati ents wi th bronchi al asthma, which of the foll owi ng eosinophi l products is most l ikel y to be responsible for
damage to the epithel ium?
OA) Heparin
o B) Leukotri ene C
4
OC) Major basi c protein
0 D) NADPH oxidase
0 E) Pl atelet acti vating factor
43. In a student l aboratory sessi on, a male medi cal student finds erythrocytes in hi s uri ne. This finding is confirmed
by the hospi tal l aboratory. He deni es any hi story of renal probl ems or recent i llness. Vital si gns are wi thin
normal l imits. Antistreptol ysin O titer i s less than 1:100. Mi croscopic examination of urine is unremarkabl e
except for rare red cell casts. There is no protei nuria. The most likely diagnosi s is
o A) di abeti c gl omeruloscl erosi s
o B) lgA nephropathy
OC) lupus nephritis
o D) membranous glomerul onephri tis
o E) poststreptococcal gl omerul onephri ti s
44. A 45-year-ol d woman says that her feet feel hot. The skin of her feet is pi nk, dry, and hot. Whi ch of the
following fibers are most li kel y to be di srupted?
o A) Autonomi c afferent
o B) Postgangli onic sympathetic
OC) Pregangl ioni c parasympathetic
o D) Somati c cutaneous afferent
45. After several courses of treatment for non-Hodgkin's l ymphoma, a 33-year-ol d man devel ops gross hematuri a.
Which of the followi ng drugs is most likely lo be responsible for this adverse effect?
o A) Cannusti ne (BCNU)
o B) Cyclophosphami de
OC) Daunorubicin
o D) Methotrexate
o E) Vincri sline

NBME
Form 2 Section 2

9. A 65-year-old man is brought to the emergency department because of a 1 -week history of confusi on,
disori entati on, talki ng to hi msel f, del usi onal ideas about his landl ord, and inconti nence. Mental status exami nation
on arri val is difficul t because he does not stay awake. Whi ch of the following is the most li kel y diagnosis?
oA) Bipol ar I disorder
O B) Delirium
OC) Dementi a
oD) Mal ingering
O E) Schizophrenia

10. A 9-year-otd boy has had the lesions shown for several months. Whi ch of the following hi stologi c changes
would be most l ikel y if this lesion were biopsi ed?
oA) Cytoi d bodies and l oss of adhesi on to adjacent cells
O B) Epidermal acanthosi s wi th subepidermal bl ister
OC) Epidermal thi nning wi th marked inflammatory dermal infil trate
O D) Marked squamous cell dyspl asi a wi th thi nned stratum corneum
O E) Thi ckened stratum comeum and epi dermi s and vacuol ated squamous cells
1 1 . A case-control study is conducted to determine if women who del ay childbeari ng are at increased ri sk for
havi ng chi ldren wi th low birth wei ght. A group of women who were at least 35 years ol d during thei r first
pregnancy are compared wi th a group of women who were between the ages of 20 and 24 years during their
first pregnancy. The followi ng data are obtained:
e tr-'9

,..,,
lOW
e:llhv.<!ig?r.
rcmol

""

..
...
,,

'"'

1<$

,.,

7S
1149

m,

What is the odds rati o for an ol der woman (relati ve to that of a younger woman) to have an infant with a low
bi rth weight?
0A) (21 I 425) I (54 I 799) = 0.73
0 B) (54 I 799) I (21 I 425) = 1 .37
OC) (54 I 745) I (21 I 404) = 1 .39
OD) (54 I 799) X 100 = 6.8
0 E) (54 I 75) I (745 I 1149) = 11.1

12. In the bi osynthesis of gl ucose from pyruvate by the l iver, the first intermediate formed from pyruvate is
o A) acetyl CoA
O B) al anine
OC) lactate
OD) methylmal onyl CoA
O E) oxaloacetate
13. A 52-year-old woman comes to the physi cian because of the onset of headaches 1 month ago. MRI of the head
shows a tumor in the posteri or l obe of the pituitary gland. Thi s tumor is most l ikely to affect secreti on of which
of the foll owi ng hormones?
o A) Adrenocorticotropic hormone
0 B) ADH (vasopressi n)
OC) Cortisol
o D) Growth hormone
0 E) Mel atonin
o F) Thyroxine (T )
4
14. A 3-year-old boy has a 5-week history of weight l oss and bul ky, frothy, foul-smelli ng stool s. Di etary modi fication
resul ts in cl inical improvement. Which of the followi ng is the most likel y findi ng on exami nation of a bi opsy
specimen from the small intesti ne?
oA) Atrophy ofvi lli and microvilli
0 B) Degenerati ve changes of the myenteri c plexus
OC) Multi ple outpouchi ngs through the muscle coat
o D) Numerous ulcerations and pseudopol yps
0 E) Pseudomembrane coveri ng the luminal surface
15. A 3-year-old boy wi th cystic fibrosis is eval uated for the acute onset of an uncontrollable nosebl eed. There is
no fami ly hi story of a bleeding disorder. A few large brui ses are seen on the legs; no otheri abnormali ti es are
noted. Prothrombi n and parti al thrombopl asti n times are prolonged. The patient's abnormal ti es are likely to
improve wi th treatment using a cofactor for whi ch process?
O A) Carboxyl ation of prothrombin
0 B) Conversion of homocystei ne to methi oni ne
O C) Farnesylation of ras
OD) Hydroxyl ati on of procollagen
o E) Phosphorylation of pyruvate dehydrogenase
16. A young coupl e has been unabl e to achieve a viable pregnancy. Semen analysi s shows normal qual ity and
quantity of sperm. The woman has regular menstrual
cycl es and takes no contracepti ves. Biopsy of her nasal
j
si nus mucosa shows epitheli al cells with the ma ori ty of cilia bent at hal f their l ength. The most l ikel y functional
cause for this coupl e's infertili ty is a defici ency in wtiich of the followi ng?
O A) Capaci tati on of spermatozoa
0 B) Completion of meiosi s by ovul ated oocytes
O C) Implantati on into the uterus
OD) Migrati on of conceptus
o E) Mobil ity of spermatozoa
17. A 45-year-old woman has a 7-month history of moderate abdominal pai n that is exacerbated by eating. She has
used al cohol heavi ly for the past 10 years. X-ray films of the abdomen show diffuse retroperi toneal
cal cificati ons. The most likely di agnosis is
o A) cholangi tis
0 B) chroni c pancreati ti s

OC) pancreatic islet cell neopl asm


o D) perforated gastric ulcer
O E) ulcerative coli ti s

18. Acting bi laterally, the external obl ique muscles produce whi ch movement of the vertebral col umn?
o A) Extensi on
O B) Fl exion
OC) Lateral flexion
OD) Rotation
19. A 40-year-old man has the recent onset of right-sided chest pain, difficulty breathi ng, and fever. On auscultation
he has crackl es, decreased respiratory sounds, and evi dence of a ri ght-sided pl eural effusion. Whi ch of the
following best descri bes the expected type of inflammatory process invol ving the l ung?
oA) Fibrobl asts, collagen deposition, repl acement of i ntri nsic tissue
O B) Macrophages, l ymphocytes, increased concentrations of immunogl obuli ns
OC) Mast cells, histami ne, smooth muscl e contracti on
OD) Neutrophi ls, increased vascul ar permeabili ty, edema
0 E) Vascul ar budding, fibroblasts, edema, proteoglycans
20. A 35-year-old woman has a 1 - month history of intermi ttent ptosi s and dipl opia. She has li mited abducti on and
el evati on of the right eye and fati gable ptosis of the left. After 2 days of pyridostigmine therapy, her ocul ar
symptoms resolve. One week later she devel ops nausea and abdominal cramping that are most severe 30
minutes after her dose of pyridostigmine. Admini strati on of which of the followi ng medicati ons is most likel y to
amel iorate her gastrointestinal symptoms?
0A) Atropi ne
o B) Bromocripti ne
OC) Neosti gmi ne
OD) Phenoxybenzami ne
O E) Propranolol

21. A 55-year-old man is hospi tali zed after having a stroke. The MRI of the head is obtained 3 days later. The
pati ent most likely has whi ch of the following si gns on the ri ght?
O A) Homonymous hemianopia and numbness of the face
O B) Weakness of the upper li mb and aphasi a
OC) Weakness of the upper and lower limbs
OD) Weakness of the face, upper and lower li mbs
0 E) Weakness of the face and dysarthri a

26. Two months after a renal transpl ant, a 47-year-ol d man has increasi ng dyspnea. X-ray film of the chest shows
an interstiti al infil trate. A photomi crograph of l ung ti ssue obtained on biopsy is shown. Whi ch of the following is
the most likel y causal organi sm?
OA) Cytomegal ovirus
o B) Epstein -Barr virus
OC) Hepati tis B vi rus
OD) HTLV-1
o E) Papillomavi rus
O F) Parvovi rus B19

27. A 68-year-old woman is admi tted to the hospi tal for eval uati on of gross hematuria. Two days after admi ssion
she devel ops aphasia and right hemi pl egi a, followed shortl y by coma and death. The cut surface of one of her
kidneys as seen at autopsy is shown. Which ofthe foll owi ng was the most likely cause ofthe hematuri a?
O A) Metastatic cancer
0 B) Papill ary necrosi s
OC) Pol ycystic ki dney disease
OD) Renal abscess
o E) Renal cell carci noma
O F) Renal infarct
28. A 4-year-old girl has had increasing di fficul ty climbi ng stairs over the past 3 months. She has a normal identical
twi n si ster and an 8-year-old brother wi th simi lar probl ems; he has been diagnosed wi th Duchenne' s muscul ar
dystrophy. Which of the following best expl ains the di scordance in the identical twi ns?

oA) Paternal genomic impri nting

O B) Skewed X inacti vati on


OC) SRY gene transl ocati on
OD) Trinucleotide repeat expansion
0 E) Uniparental disomy
29. Basi c fibroblast growth factor is appli ed to an experimentally induced dermal wound. A histol ogic section is
prepared 72 hours later. Compared wi th an untreated wound, the most likel y alteration in this wound is
O A) decreased numbers of fibroblasts

o B) increased breakdown of matri x

OC) increased numbers of bl ood vessel s


0 D) increased numbers of eosinophi s
o E) Increased numbers of giant cellsl

39. Whi ch of the foll owing modi fications of an endogenous pharmacologi call y active peptide would be most likel y to
sl ow the metabol ism of the peptide, thus decreasing its clearance and increasi ng its hal f-li fe?
O A) Administering the peptide with an enteri c coati ng
o B) Decreasing the number of amino acids in the peptide
OC) Ensuring a free amino group at the N termi nal
0 D) Ensuring a free carboxyl group at the C terminal
o E) Substituti ng a o for an L amino acid
40. At a gi ven P0 , more o can be unl oaded from hemoglobi n when whi ch of the following is decreased in
2
2
erythrocytes?
0 A) 2, 3-Bisphosphoglycerate
O B) pH
OC) PC0
2
oD) Temperature
41. A 27-year-old man who uses heroin intravenously develops fever followed several days l ater by dyspnea and
pl euritic chest pain. X-ray films ofthe chest obtained on successive days show continuous appearance of new
bil ateral cavitary infil trates. Echocardi ography discl oses a 1.5-cm vegetation on the tricuspid val ve. Cul tures of
bl ood and sputum are most likel y to grow
O A) Enterococcus faecium
0 B) Mycobaclerium tuberculosis
O C) Staphylococcus aureus
OD) Streptococcus pneumoniae
o E) viridans streptococci
42. In the di agram of a blood vessel, what is the princi pal si te of nitric oxi de acti on in producing vasodi lation?

O N 0 0 0 O E) O F)
43. A neonate has a murmur at the apex, whi ch is a normal transient findi ng in up to 50% of newborns. Which of the
following acti ons concerni ng informing the parents about the murmur is most appropri ate?
O A) Defer telling the parents and order an echocardi ogram

o B) Defer telling the parents because heart murmurs often resol ve

OC) Defer telli ng the parents but order a cardi ol ogy consultation
0 D) Tell the parents and examine the infant again in 24 hours
o E) Tell the parents that thi s findi ng is normal but offer a cardiology consultation

44. Over a peri od of 3 weeks, a 60-year-ol d woman devel ops anorexi a, restl essness, irritabili ty, and confusi on.
Serum sodi um concentration is 125 mEq/L; uri ne osmolali ty is increased. Assumi ng the pati ent's di sorder
represents a paraneopl astic syndrome, whi ch of the foll owing di agnosti c procedures is most li kely to result in an
abnonnal finding?
O A) Exami nation of bone marrow
0 B) Liver scan
OC) Mammography
0 D) Measurement of urinary excretion of 5 -hydroxyindol eaceti c acid
o E) X-ray fil m of the chest
45. A 2-year-old boy is brought to the physici an' s office for a wel chi ld vi si t. He was treated in the hospital at birth
for moderate hyperbil irubinemi a. He has had three epi sodes of otiti s media. He has a vocabul ary of 100 words
and is starting to use two-word sentences. He can run without falling but cannot ride a tri cycle. Which of the
following best descri bes his motor and language devel opment?
Motor
OA)
O B)
OC)
OD)

Delayed
Delayed
Normal
Nonnal

Language
del ayed
nonnal
del ayed
nonnal

46. Nonna! cycles of cili ogenesi s in ovi duct epi theli al cells depend di rectly on which of the following acti vities?
o A) Biogenesis of additional mitochondri a
0 B) Chromosomal DNA repli cati on
OC) Gl ycosylati ons wi thin the Gol gi body
OD) Multi pli cati ons of centrioles
0 E) Synthesis of cytokeratin
47. A 45-year-old woman wno has acute di verti culi tis devel ops severe hypotensi on, tachypnea, and tachycardi a.
She subsequently develops cyanosi s and hypoxemia unresponsive to oxygen therapy. X-ray films of the chest
show a diffuse al veol ar infiltrate. Bl ood cul ture grows Escherichia coli. A major factor in the pathogenesis of
thi s syndrome is
O A) degranul ation of mast cells
0 B) delayed (type IV) hypersensi tivi ty to bacteri al products
O C) increased surfactant production by type II pneumocytes
OD) producti on of antibodi es to al veol ar basement membranes
o E) rel ease of oxygen free radicals from sequestered neutrophi ls
48. A 65-year-old man wi th ingui nal adenopathy undergoes a lymph node biopsy. Light microscopy shows
repl acement of nodal ti ssue by cells that form well-defined, uni fonn gl andul ar structures. The nuclei are
relativel y small and nucl eol i are inconspicuous. An immunohi stochemical stain for prostate-speci fic antigen is
posi tive. These findings are most consistent with which of the followi ng?
O A) Higtrgrade, hi gh-stage neoplasm
O B) Higlrgrade, low-stage neoplasm
OC) Low-grade, higtrstage neopl asm
0 D) Low-grade, l ow-stage neopl asm

NBME
Form 2 Section 3

4. A 45-year-old man who is undergoing di alysi s for chronic renal failure starts recei ving erythropoi eti n therapy. He
has a hi story of type 1 diabetes mellitus, peripheral neuropathy, and welcontrolled hypertensi on. Exacerbati on
of which of the followi ng is most likely to occur in this pati ent?
oA) Chronic renal insuffici ency
O B) Di abetes mellitus
OC) Hypertensi on
o D) Ostei tis fibrosa cysti ca
0 E) Peri pheral neuropathy
5. A 2-year-ol d boy has a unil ateral cl eft lip and cleft palate. He has no other malformations and is otherwi se
heal thy. He has no famil y history of bi rth defects. His parents are not rel ated. Which of the foll owi ng genetic
mechanisms is most likely to have caused these congenital abnormal ities?
o A) Autosomal dominant inheri tance
O B) Autosomal recessi ve inheri tance
OC) Chromosome aneupl oidy
OD) Chromosome deleti on
O E) Mitochondrial DNA mutation
o F) Multi factorial inheri tance
OG) X -l inked recessi ve inheri tance
6. A 73-year-old man with a history of angi na is undergoi ng cardiac catheterization vi a the ri ght femoral artery. As
the pi g-tail catheter is advanced into the apex of the l eft ventricle, whi ch of the foll owing val ves wi ll be crossed?
OA) Aorti c
O B) Mitral
OC) Pulmonary
OD) Tricuspid
7. A 45-year-old woman has a 3-month history of a persistent cough and increasi ng shortness of breath. She
denies chest pain or other illnesses. Bl ood pressure is 1 10170 mm Hg, and pul se is 1 10/min. She has di ffuse
basil ar crackl es and an s gallop, but no murmur. ECG shows sinus tachycardia and nonspecific ST-T wave
3
changes. Echocardiography shows a di lated l eft ventricle and a di ffusel y hypoki netic heart. The most likely
diagnosis is
o A) acute pericardi tis wi th effusi on
O B) dil ated cardiomyopathy
OC) hypertrophic cardiomyopathy
o D) severe mi tral val ve prol apse
O E) sil ent subendocardi al infarct
8. A 45-year-old man develops a fever, a cough productive of rusty sputum, and chest pain. Sputum cul tures are
positive for Streptococcus pneumoniae. He is treated wi th anti biotics and recovers uneventfull y. Whi ch of the
followi ng cell types is most likel y responsibl e for regenerati on of the injured tissue?
oA) Cl ara cell
O B) Fibrobl ast
OC) Macrophage
o D) Type I pneumocyte
O E) Type II pneumocyte
9. An 18-year-ol d man has nausea, vomiti ng, headache, a temperature to 39.4 C (103 F). and painful swelling of
the parotid glands. He i s at increased risk for
0 A) appendici ti s
o B) arthritis
OC) cystiti s
OD) epi scl eri ti s
o E) orchi ti s

10. A 45-year-old woman with rheumatoid arthri ti s calls the physi cian on a Saturday morning because she cannot
tum the bathroom faucet due to pain and weakness in her hands and wrists. She is tearful and says, "My
husband has al ready l eft for work and my hands are too weak to turn the water on. It hurts too much and now I
can' t even take a bath." Which of the following is the most appropri ate response?

OA) " Come to the office right away and I will gi ve you a cortisone shot."
0 B) "I know thi s must be difficult. Perhaps a pl umber may be abl e to hel p you."
O C) "I sympathize wi th you. Unfortunately, it looks like your onl y opti on is to wait unti l your husband comes home."
0 D) " Many peopl e wi th rheumatoid arthri tis have difficulty turning faucets. Perhaps you would be interested in
joini ng a support group."
0 E) "People wi th rheumatoid arthri ti s often feel that their independence has been l ost because oftheir pain.
Perhaps using a tool with a l ong handl e woul d hel p:

1 1 . A 24-year-old man dies suddenl y. At autopsy, there is evidence of cocaine abuse but no evidence of gross or
microscopic structural alterati ons of the heart. Which of the followi ng is the most likel y cause of the cardi ac
malfuncti on in this man?
2
0 A)
Blockade of ca + channels
O B) Blockade of dopami ne synthesis
OC) Potenti ati on of norepi nephrine and bl ocki ng of its presynapti c uptake
OD) Potenti ati on of serotoni n at postsynaptic sites in the atri oventri cular node
0 E) Stroke due to mal ignant hypertension
12. A child born wi thout limbs is found to have a mutation in a receptorfor a fibrobl ast growth factor. The absence
of l imbs most li kel y resul ted from a disrupti on of the processes controlling which of the followi ng?
O A) Apoptosis
o B) Di fferentiation
OC) Epi thel ial-mesenchymal conversi ons
0 D) Prol iferation
13. A 38-year-old woman has noticed the gradual devel opment of a goi ter. Thyroi d function studi es show serum
thyroxi ne (T ) and tri iodothyronine (T ) concentrations to be wi thin the reference range. Serum concentration
3
4
of thyroid-stimul ating hormone is sl ightl y increased. The serum contains a high level of antimi crosomal
(antiperoxidase) anti bodies, but no thyroi d-stimul ating hormone receptor anti bodies are detected. The most
likely di agnosis is
o A) autoi mmune (Hashi moto's) thyroi ditis
O B) di ffuse toxi c goi ter (Graves' disease)
OC) pri mary atrophy of the thyroid gl and
o D) Ri edel's struma
O E) subacute thyroi di tis
13. A 38-year-old woman has noticed the gradual devel opment of a goi ter. Thyroi d function studi es show serum
thyroxi ne (T ) and tri iodothyronine (T ) concentrations to be wi thin the reference range. Serum concentration
3
4
of thyroid-stimul ating hormone is sl ightl y increased. The serum contains a high level of antimi crosomal
(antiperoxidase) anti bodies, but no thyroi d-stimul ating hormone receptor anti bodies are detected. The most
likely di agnosis is
o A) autoi mmune (Hashi moto's) thyroi ditis
O B) diffuse toxi c goi ter (Graves' disease)
OC) pri mary atrophy of the thyroid gl and
o D) Ri edel's struma
O E) subacute thyroi di tis

14. A 2-year-old boy has bri ttl e depigmented hair, cerebral atrophy, delayed myeli nati on, motor delay, and mental
retardation. Lysyl oxidase activi ty is decreased. The function of which of the followi ng substances is most
likel y to be defecti ve in this patient?
OA) Al bumi n
O B) Cerul opl asmi n
OC) Chondroi ti n sulfate
O D) Collagen
15. Alveol itis wi th intersti ti al edema, inflammatory cell accumul ation, and type II epi thel ial cell hypertrophy and
hyperpl asia is bel ieved to be an earl y and central event in the pathogenesis of
O A) anthracosis
O B) asthma
OC) centri aci nar emphysema
0 D) chroni c bronchitis
o E) pul monary
fibrosi s
16. A previ ously heal thy 19-year-ol d woman comes to the medi cal cl ini c because of severe abdominal pain for the
past 18 hours. She has had increasi ngly frequent painful urination over the past 24 hours. She is not sexually
active. Temperature is 38.5 C (101.3 F), pulse is 90/min, respirati ons are 14/min, and bl ood pressure is 1 15/76
mm Hg. The suprapubi c area is tender on pal pati on. Mi croscopic exami nation of uri ne shows pyuri a, hematuria,
and bacteriuria. Whi ch of the following organisms is the most l ikel y cause of these symptoms?
O A) Chlamydia trachomalis
o B) Escherichia coli
OC) Neisseria gonorrhoeae
OD) Pseudomonas aeruginosa
0 E) Staphylococcus aureus
17. Ami no aci ds that contri bute at l east one nitrogen or carbon to the purine ri ng during the bi osynthesis of purine
nucl eotides are
O A) al anine, asparagi ne, and lysi ne
o B) al anine, methi oni ne, and histidi ne
OC) aspartate, argini ne, and hi stidi ne
OD) aspartate, gl yci ne, and glutamine
o E) glycine, seri ne, and tyrosine
18. A 58-year-old man develops an acute coronary thrombosis with resultant myocardial infarcti on. The swelli ng of
the myocardi al cells borderi ng the infarct is most li kely to be due to
O A) depletion of ATP
2+
O B)
efflux of Ca
+
OC)
efflux of Na
o D) increase of intracell ul ar pH
O E) producti on of H 0
2 2

24. A 26-year-old woman has a Trichomonas vagina/is infection and begi ns metroni dazol e therapy. The patient
should be advi sed to avoid whi ch of the following duri ng therapy?
OA) Cheese
O B) Milk
OC) Sunl ight
OD) Tobacco
O E) Wi ne
25. A 63-year-old woman has chest pain whil e climbing the stairs of her office building. She si ts down and takes
some medication, after which she is able to continue. Which of the followi ng i s the mechanism of acti on of the
drug?

oA) Decreased afterl oad

O B)
OC)
o D)
O E)

Decreased prel oad


Increased afterl oad and prel oad
Increased preload
Increased contracti lity

26. The absorption of calci um from the intesti nal tract resul ts pri ncipally from the direct action of whi ch of the
following?
O A) Cal citonin
O B) Cycli c AMP
OC) 1,25-Di hydroxycholecalci ferol
0 D) 25-Hydroxychol ecalci ferol

o E) Parathyroi d hormone

27. A 23-year-old woman has a fever and a cough producti ve of rust-col ored sputum. A gram-posi tive coccus
isolated from the sputum is sensitive to optochin. Which of the followi ng is the most l ikely causal organism?
O A) Enterococcus faecium
o B) Streptococcus agalactiae (group B)
OC) Streptococcus pyogenes (group A)
OD) Streptococcus pneumoniae
o E) Streptococcus sanguis
28. A 38-year-old man comes to the physici an because of invol untary movements of the arms and legs for the past
month and progressi ve faci al twi tching and gri macing over the past year. Neurol ogi c exami nati on shows
athetoid and choreiform movements of the arms and legs, dystoni c movements of the tongue, dysarthria,
impairment in immedi ate and short-term memory, and dyscalcul ia. Which of the following is the most li kel y

diagnosis?

O A) Amyotrophic lateral sclerosis


o B) Dementi a, Alzhei mer's type
OC) Huntington' s di sease
o D) Korsakoff's syndrome
O E) Multi -i nfarct dementia
O F) Parkinson's disease
OG) Werni cke's encephal opathy
29. A 28-year-old woman is undergoi ng evaluation at 37 weeks' gestation. Bl ood pressure is 120/75 mm Hg sitting,
115/70 mm Hg standi ng, and 90/60 mm Hg supi ne. The most likel y expl anation for these findi ngs is
0 A) compression of the vena cava
o B) gestati onal diabetes mellitus
OC) peri partum cardiomyopathy
0 D) preecl ampsi a
o E) uterine contraction

30. Endothel ial cells of the postcapi llary venules increase expression of intercell ular adhesion molecul e-1 (ICAM-1)
in response to cytoki nes. The increased expressi on of ICAM-1 resul ts directly in which of the following
neutrophil functions?

o A) Activation

O B) Adherence
OC) Chemotaxis
o D) Demargination
0 E) Transmi grati on
31. A 3-week-ol d infant has mil k intolerance due to a defect in the enzymes that di gest lactose. Thi s enzyme is
located at which of the followi ng sites in enterocytes?
O A) At the apical surfaces
o B) At the basolateral surfaces
OC) Wi thin the Gol gi compl ex
OD) Wi thin l ysosomes
o E) Wi thin peroxisomes
32. A 78-year-old woman has a 2-month history of flail ing movements of the l imbs on one side. There is no fami ly
hi story of a simi lar movement di sorder. Which of the following structures is most l ikely to be involved?
O A) Cerebell ar hemi sphere
o B) Ci ngulate cortex
OC) Pri mary motor cortex
0 D) Stri atum
o E) Substantia
nigra
O F) Subthal amic nucl eus
33. A 16-year-old gi rl is eval uated because she has not yet had a menstrual period. She is 147 cm (4 ft 10 in) tall
and weighs 42 kg (93 lb). Breast development is Tanner stage 1 of 5 and pubic hair growth is Tanner stage 2 of
5, respectively. She has a l ow hairl ine, short 4th metacarpal s bi laterally, and multi ple pi gmented nevi. A cervi x
is seen on pel vi c examinati on. Laboratory studies show an increased serum folli cl e-stimulating hormone
concentration. Serum thyroi d-sti mul ating hormone concentration and serum prol acti n concentration are wi thi n
the reference ranges. Which of the followi ng is the most likel y di agnosi s?
o A) Androgen insensi tivi ty syndrome
O B) Gonadal dysgenesis 45,X (Turner' s syndrome)
OC) Gonadotropi n -secreting pi tuitary tumor
OD) Isol ated gonadotropin deficiency
O E) Paramesonephric (mullerian) aplasi a
34. A 30-year-old woman is brought to the emergency department because of pain in her upper arm after falling off
a horse. X-ray films show a cl osed simpl e fracture of the upper humerus. Serum studies 3 weeks later are most
likely to show an increased concentration of
o A) al kal ine phosphatase
O B) calci um
OC) creatine ki nase
o D) y-glutamyltransferase
O E) phosphorus
35. An 18-year-old man has mul tipl e bil ateral angi omyoli pomas. Whi ch of the following lesions is most l ikely to be
found wi thi n the central nervous system?
O A) Adenocarcinoma
o B) Cortical hamartoma
OC) Gl iobl astoma multi forme
0 D) Metastatic angi omyol ipoma
o E) Subependymal meni ngioma

36. A 28-year-old man reports extremel y familiar thoughts (deja vu) just before he has a seizure. Which labeled
area in the di agram is the most li kely site of the causal lesi on?

om

ON 0

0 OE) O F)

37. A 16-year-old gi rl has generali zed weakness and nocturnal leg cramps. She is not taki ng any medications.
Laboratory studies show:
Serum
135 mEq/L

Na+

Cl
+

108 mEq/L
2.8 mEq/L

21 mEq/L

HC0
3
Urine pH

7.0

Whi ch of the foll owing is the most likely di agnosis?


oA) Adrenal insufficiency
O B) Hyporeni nemic hypoal dosteroni sm
OC) Nephrogeni c diabetes insi pidus
OD) Renal tubul ar acidosi s
0 E) Syndrome of inappropriate ADH (vasopressi n)
38. The graph shows the populati on di stri bution for values obtained from a test on a group of heal thy and di seased
persons. A, B, C, D, and E represent various deci sion points for determini ng disti nctions between diseased
and normal popul ati ons. Whi ch cut-off point will give a specifici ty of 100%?
Norm.".11

Numbe<ot
i ndHidual s
tested

BC D E

Rangeofval ues tor aparti cul arteat

OA) O B) OC) OD) O E)

43. Eukaryoti c proteins that exi st as a nonmembrane-associ ated, gl ycosyl ated, di sul fide-l inked dimer in the mature
form are generall y found in which of the followi ng?
O A) Cytopl asm
o B) Extracell ular space
OC) Mi tochondrial matrix
0 D) Nucl ear matri x
O E) Nuclear pore

44. Serum LDL-chol esterol concentrations are measured in blood samples collected from 100 healthy vol unteers.
The data follow a normal di stri bution. The mean and standard devi ation for thi s group is 130 mg/dl and 20
mg/dl, respectivel y. The standard error of the mean is 5.0. The 95% confidence interval about the mean is
120 to 140 mg/dl. If bl ood samples are collected from 25 heal thy vol unteers instead of 100, which of the
following best expresses the expected impact on the standard error and the 95% confidence interval about the
mean?
Standard
error
OA)
O B)
OC)
OD)
O E)
O F)

Wi dth of 95% confidence


interval

l
!
l

1
1
1

No
change
OH)
No
change
OG)

no change

!
I

no change

l
1

45. A 31-year-old bl ack man wi th acute sinusi tis requires antibiotic therapy. He says that several family members
have developed hemol ysis after taki ng drugs. Whi ch of the following anti microbial drugs should be avoided in
thi s patient?

oA) Ci profloxacin
O B) Erythromycin
OC) Penicilli n
oD) Sulfisoxazole
O E) Tetracycl ine

46. A 14-year-old boy devel ops a pruri ti c vesicul ar rash on his face and hands 3 days after hiking in the woods.
Whi ch of the following are most likel y to be invol ved in hi s illness?
O A) Antibody, compl ement C5-9
o B) Ci rcul ating immune compl exes, compl ement C5a, neutrophils
OC) Cytotoxi c T lymphocytes
OD) lgE, mast cells, histamine, cytoki nes
o E) Lymphocytes. cytokines, macrophages

47. A 20-year-old man has a painless 2-cm nodul e in hi s ri ght scrotum. The nodule is di fficul t to di stingui sh from the
right testis and does not transil luminate. There is no additional swelling or inguinal adenopathy. The most likely
di agnosis is

oA) cryptorchi dism

O B) hydrocel e
OC) scrotal cyst
o D) testi cul ar cancer
0 E) testi cul ar torsi on
o F) vari cocel e

48. A ma le neonate has a palpab le midabdominal mass. Abdominal ultrasonography shows no kidneys in the normal
anatom ic position and a single U-shaped renal mass with two collecting syst ems near the aortic bifurca tion .
Which of the following embryo logic events best exp lains this finding?
o A) Divis ion of the metanephr ic diverticulum
o B) Failed devel opment of the ureter io bud
OC) Fai led differentiation oflhe urogenital sinus
o D) Fus ion of the metanephric masses
0 E) Normal development of two ureter ic buds

---

01 -

----

8 -

- - - -

49. A 4-year-ol d boy is diagnosed by biochemical testing as having medium-chain acyl-CoA dehydrogenase
(MCAD) deficiency, an autosomal recessive di sorder offatty acid oxidat ion. A po lymerase chain reaction
(PCR) amplificat ion of the region of the gene containing the common mutat ion is done. PCR of both the mutant
and normal genes yields an 87-base pair (bp) product. After Ncol digestion, however, the mutant PCR product
yields 61 and 26-bp fragments, while the normal gene product i s not cleaved. Resul ts of this test on a DNA
sample from the boy and his fami ly are shown. On the basis of this assay, which of the following conclusions
about s ibling 3 is most likely?
O A) The MCAD status of this sibli ng is indeterminate
o B) This sibling does not have MCAD deficiency
OC) This sibli ng has MCAD deficiency
o D) This sibl ing must have been the product of nonpaternity

--"'i'",...--@--,,,,
0

Aceta1TT10phen

50. Which of the following metabolic processes shares the same melaboic pathway as that of the reaction shown?
O A) Deam inat ion of o-amphetamine
O B) Formation of minoxidil sulfate
OC) Hydrolysis of acetylsalicylic acid
OD) ,-...Oea lkytation of desipramine
o E) t.'-Hydroxytat ion of meperidine

IC
28
38
4C

SF

6A
78
8E
9E
J OA
I IC
1 20
13A
l4D
15E
168

l7D
1 8A
198
20D
2 1D
228
23D
24E
258
26C
27D
28C
29A
308
31 A
32F

38E
398
40C
41 C
42A
438
44E
45D
46E
47D
48D
49C
508

NBME
Form 2 Section 4

6. Whi ch of the following tests best di stingui shes central diabetes insi pidus from nephrogenic di abetes insipidus?
oA) Injection of ADH (vasopressi n)
0 B) Injection of hypertonic sal ine
OC) Injection of insuli n
OD) Water deprivation
0 E) Water loading
7. An el derl y woman has had general ized bone pain and a loss of height over the last 5 years. Complete bl ood
count is normal. Serum cal cium, phosphate, and creatini ne concentrations and serum al kal ine phosphatase
acti vity are withi n the reference range. Serum protei n el ectrophoresis shows no abnormal ities. X-ray fil ms of the
spine show generalized loss of bone density and a compressi on fracture of T-8. Which of the followi ng is most
l ikely to be seen in tissue obtained on bi opsy of the ili ac crest?
O A) Cysts filled with fibroblasts and gi ant cell s
o B) Di sorderly osteobl asti c and osteocl astic acti vity
OC) Sheets of pl asma cells
OD) Thin trabecul ae wi th low osteocl astic activi ty
8. A protei n contains a 30-amino acid structural moti f wi th four cysteine residues coordinated to a zinc atom. In
whi ch of the following is this structured motif most l ikely to be found?
O A) Nucl eosome
o B) Ri bonuclease
OC) Ribosome
OD) Spliceosome
o E) Transcription factor
9. The maj ori ty of cellular proteins and RNA are synthesized throughout the cell cycl e and are only interrupted
bri efly during M phase. Synthesis of whi ch of the following proteins occurs predominantl y in S phase of the cell
cycle?
OA) Cycli ns
O B) DNases
OC) Histones
o D) RNA polymerases
O E) snRNPs (small nucl ear ribonucl eoproteins)
o F) Tubul ins

10. A 46-year-old man is killed in an automobile accident. The photograph shows a ki dney at autopsy. Whi ch of the
following would have been expected in this pati ent?
oA) Cerebral arteri al aneurysms
O B) Hypertensi ve heart disease
OC) Multi ple hepatic cysts
o D) Normal renal function
O E) Vesicoureteral reflux

16. A 62-year-old man develops numerous superfici al bl isters over the scalp, face, groi n, and trunk. New lesions
devel op over areas subjected to mini mal trauma. Biopsy ofone ofthe lesions shows an intraepi dermal bl ister
wi th suprabasal acanthol ysi s. Serol ogi c studi es are likely to show an autoantibody directed against whi ch of
the following?
O A) Anchoring fil aments
0 B) Basement membrane proteins
OC) Desmosomal protei ns
OD) Hemidesmosomal protei ns
o E) lntegri ns
17. Fetal and maternal erythrocytes have the same concentrati on of 2,3-bi sphosphogl ycerate. However, compared
wi th maternal hemoglobi n (HbA), fetal hemoglobin (HbF) has a hi gher oxygen affinity, which facilitates
transplacental diffusi on of oxygen. The presence of whi ch of the following pol ypeptide chai ns in HbF accounts
for this findi ng?
O A) a Chains alone
O B) J3 Chains al one
O C) y Chains
OD) Both a and J3 chains
18. Whi ch group ofregulators invoMng inflammatory or immedi ate (type I) hypersensiti vi ty reactions is capable of
causing increased vascul ar penneability and attracti on of l eukocytes?
O A) Essential fatty acids
o B) Leukotri enes
OC) Prostaglandins
0 D) Thromboxanes
19. A 14-year-old gi rl is brought to the physici an's office by her mother who is concerned that the girl has not begun
to menstruate. Physical exami nati on shows normal breast devel opment, no pubic hai r, nonnal female external
geni tal ia, bilateral inguinal hernias, a vagi na that ends i n a bli nd pouch, and no pal pable uterus or ovari es.
Ul trasound exami nati on confirms the absence of internal female geni tal ia. The underlying problem in this pati ent
is rel ated to which of the foll owi ng?
0 A) Defective androgen receptors
o B) Excess producti on of fetal estrogen
OC) Excess producti on offetal progesterone
o D) Insuffici ent fetal testosterone production
0 E) Insufficient producti on of paramesonephri c (mOlleri an) inhibitory factor during fetal devel opment
20. A 5S-year-old man wi th mul tiple pri mary li ver abscesses caused by Escherichia coli and anaerobic bacteri a has
fever and shaki ng chills. Whi ch ofthe following is the most likely cause ofthese symptoms?
O A) Associ ated abscess in the hypothalamus
o B) Hypothalami c response to leukocytosi s
OC) Leukocyte inflammation of the hypothalamus
0 D) Release of fragments of damaged hepatocytes into the ci rcul ation
o E) Release of l eukocyte cytoki nes into the circulation
21. A 27-year-old man wi th type 1 di abetes mell i tus is brought to the emergency depar1ment because of stupor and
di abeti c ketoacidosi s. Hi s diabetes is usuall y well controlled. His wi fe says he has had a fever and productive
cough for several days and has been taki ng an over-the-counter cough syrup. Whi ch of the folloWing is the
most li kely cause of the ketoacidosi s?
O A) Cough syrup
0 B) Dehydration
O C) Fail ure to take insul in
OD) Infecti on
o E) Lack of exerci se

22. An increase in the acti vity of which of the followi ng enzymes is most likely to increase the rate of metabol ism of
gl ucose 6-phosphate through gl ycol ysi s in the li ver?
OA) Fructose 1,6-bisphosphatase
o B) Fructose 1,6-bisphosphate al dolase (al dol ase A)
OC) Fructose 2,6-bisphosphatase
OD) Gl ucoki nase
o E) 6-Phosphofructo-1-ki nase
23. A 5S-year-old man being treated for congesti ve heart fail ure becomes confused about his medications and
takes an overdose of di goxi n. Five hours later, he reports anorexia, nausea, vomiting, and vi sual di sturbances.
Serum di goxi n concentration is 1 O ng/mL (toxi c > 2.5 ng/mL); serum potassi um concentration is 7.2 mEq/L.
Whi ch of the following is the most likely cause of the hyperkal emia?
OA) Hyporeni nemic hypoaldosteroni sm
0 B) Increased renal tubul ar absorption of potassi um
+ +
O C)
Inhibi ti on of Na _K ATPase
0 D) lntravascul ar hemol ysi s
o E) Rhabdomyol ysis

24 During wh ch phase of the ventricu ar action potential shown does norepinephr ne influence the influx of Ca2+?
l
i
i
OA)
O B)
OC)
OD)
O E)

25. A 32-year-old man comes to the physici an for a routine exami nation. He tells the physici an that he has gained
weight, but that he doesnt enjoy eati ng or many other activi ti es. He says he needs at l east 1 1 hours of sl eep,
but still feel s fatigued most days and has difficul ty concentrating. He is 178 cm (5 ft 10 i n) tall and weighs 90 kg
(198 lb). Whi ch of the following psychiatri c disorders is most l ikely to be present in this man?
O A) Antisoci al personal ity di sorder
0 B) Major depressi ve di sorder
O C) Obsessi ve-compul sive disorder
OD) Post-traumatic stress disorder
o E) Somatizati on disorder

26. A 67-year-old man is admi tted to the hospital because of worseni ng dyspnea over the past 6 hours. He was
di agnosed with hypertension 10 years ago. Temperature is 36 C (96.8 F), pul se is 100/min, respirations are
22/mi n, and blood pressure is 136/87 mm Hg. Physical exami nati on shows an s gall op by cardi ac auscul tati on
3
and bil ateral coarse ral es halfvvay up the l ungs bilaterally. Which of the following is the most appropriate di etary
order for thi s patient?
oA) Cl ear l iqui d diet
O B) Enteral feedi ng via a nasoenteric tube
OC) Full liqui d diet
o D) Hign-fiber diet
0 E) Low-residue di et
o F) Parenteral nutrition via a central venous access
OG) Protei n-restri cted diet
0 H) Regul ar hospital di et
o I) Sodium-restri cted di et
27. Whi ch of the foll owing features of the inactivated poli ovirus vaccine best explains why it prevents paral yti c
pol io but does not prevent repli cati on of the vi rulent pol iovi rus in the enteric tract?
0 A) It does not eli cit cell-mediated immunity to vi ral capsid protei ns
o B) It does not eli cit neutral izing anti body to virulent poliovirus
OC) It el icits lgG but not secretory lgA
0 D) It onl y protects against one serotype of vi rul ent poli ovi rus
28. What is the major consequence of a hereditary defect in the producti on of active pyruvate ki nase in the li ver
and in erythrocytes?
O A) Decreased capacity to fix CO
2
o B) Decreased gluconeogenesi s from alanine
OC) Decreased oxidation of pyruvate
OD) Decreased producti on of ATP from gl ucose
O E) Increased affinity of hemogl obin for oxygen
29. A heal thy 25-year-old woman who is at 24 weeks' gestati on devel ops heartburn withi n an hour after eati ng.
Whi ch of the following is the most likely cause?
0A) Decreased competency of the lower esophageal sphincter
o B) Decreased esophageal motil i ty
OC) Increased gastric acid producti on
0 D) Increased gastric emptyi ng
o E) Increased gastric motility
30. Whi ch of the foll owing immunosuppressive agents acts in the lymphocyte by compl exing to and inacti vating the
lymphocyte-specific phosphatase cal cineurin?
O A) Antilymphocyte globul in
o B) Azathioprine
OC) Corticosteroids
OD) Cyclosporine
o E) OKT3 monocl onal antibody
31. After sampl ing red wine and cheese, a 55-year-ol d man has a severe pounding headache. Pul se is 1 10/min,
and blood pressure is 230/145 mm Hg. He is most likel y taki ng which ofthe foll owi ng drugs?
O A) Di phenhydramine
o B) Enalapril
OC) lmipramine
0 D) Nitrogl ycerin
o E) Tranyl cypromine

32. A 34-year-old woman has had fatigue, irri tabil ity, intermittent li ghtheadedness, and heavy menstrual periods for
the past 6 months. Laboratory eval uation is most likely to show which of the followi ng?
OA) Decreased serum vitamin B

o B) Hypercal cemi a
OC) Hypocalcemia
OD) Increased serum vi tamin B
O E) Megal obl asti c anemia

12

12

(cyanocobal ami n) concentration

(cyanocobal amin) concentration

o F) Microcytic anemia

33. A 10-year-old boy wi th asthma has been receivi ng oral hydrocortisone dail y in increasing doses for the past 2
years. He is obese and has growth retardati on and edema of hi s lower limbs. Whi ch of the following is the most
likely explanation for the edema?
o A) The drug bi nds lo both glucocorticoid and mineralocorti coid receptors
O B) The drug is converted to a mineralocorti coi d in the l iver
OC) Pu monary producti on of angiotensi n is increased
o D) Renl i n acti vity is i ncreased
because of stress
0 E) Secretion of ADH (vasopressi n) is increased because of stress
34. Beni gn prostati c enl argement causes a characteristic impression on the uri nary bl adder in whi ch area of the
cystogram?

OA) O B) OC) OD)


35. A 48-year-old man comes to his physician because hi s wi fe noticed that hi s ri ght pupil is "small." Physi cal
examination shows that the ri ght pupil is constricted and does not react to li ght. His l eft pupi l is normal. These
findings are most li kel y due to a lesion invol ving whi ch of the following structures on the right?
o A) Cervical spinal cord
O B) Frontal eye fiel ds
OC) Lateral genicul ate nucleus
o D) Optic tract
O E) Vi sual cortex

36. A 64-year-old woman comes to the physi cian' s office because of gradually worseni ng abdomi nal cramps,
nausea, di arrhea, flushing of the skin, and heart pal pitations. Twenty-four hour urinary 5-hydroxyi ndoleacetic
aci d excreti on is increased. The most likely cause of these findi ngs is a neopl asm ori ginating in whi ch of the
following structures?
O A) Adrenal gl and
O B) Esophagus
O C) Kidney
OD) Small intesti ne
O E) Spl een
O F) Thyroid
37. When a cytogenetic abnormality is suspected in an infant wi th mul tiple congenital anomalies, chromosomes
from ci rcul ating leukocytes are analyzed. These chromosomes are most condensed and easiest to eval uate in
whi ch ofthe following states?
o A) Anaphase
O B) lnterphase
OC) Metaphase
o D) Prophase
O E) Tel ophase

38. An afebri le 8-year-old girl is eval uated for mild edema and a "puffy" face. She recovered from an untreated
sore throat and fever 3 weeks ago. Urinal ysis shows moderate proteinuria and hematuria wi th occasional red
cell casts. A photomicrograph of renal tissue obtained on bi opsy is shown. Which of the foll owi ng is most likel y
responsi ble for the cellul ar infittrate shown?
0 A) Bradykini n
O B) C5a
OC) Histami ne
o D) N-formyl ated pepti des
O E) Serotonin
39. A hyperacti ve 6-year-old gi rl develops microcytic anemi a, cerebral edema, and ataxia. Determination of whi ch
of the foll owi ng serum enzyme activi ti es is most likel y to confirm the diagnosis?
0A) 5-Aminolevuli nic acid (i5-ALA) dehydrogenase
o B) Gl ucose 6-phosphate dehydrogenase
OC) Gl utathi one transferase
0 D) Lactate dehydrogenase
o E) Pyruvate kinase
40. Whi ch of the foll owing drugs produces vasoconstriction that is not bl ocked by prazosin and bradycardia that is
bl ocked by scopolamine?
O A) Amphetamine
o B) Angi otensi n
OC) Norepinephri ne
0 D) Phenyl ephri ne

41. An obese 44-year-ol d man wi th type 2 di abetes melli tus is unabl e to achieve appropriate glycemic control with
di et and exerci se. The physician consequentl y prescribes a sulfonylurea. Thi s class of drugs promotes the
release of i nsul in from pancreatic 13 cells through which of the following factors?
+
0 A)
Inhibi ti on of ATP-sensi tive K channels
o B) Inhib t on of Na+-K+ ATPase
ii
2
O C)
Inhibi ti on of vol tage-sensi ti ve ca + channels
0 D) Stimul ation of guanylylate cycl ase
o E) Stimul ation of phosphodiesterase
42. A 4-cm bronchogeni c cyst is an incidental finding at autopsy of a 5:,-year-ol d man. The cyst most li kely arose
from which of the foll owi ng embryologi c structures?
O A) Allantois
O B) Foregut
OC) Mesonephri c (wotffian) duct
OD) Metanephric duct
o E) Omphalomesenteric duct
43. A 19-monttrold girl dies after ingesting a bottl e of her mothe(s iron sul fate suppl ements. At autopsy,
microscopic examination of hepati c tissue shows lysosomal degradation. Which of the followi ng is the most
likely mechanism of thi s finding?
oA) Activation of gl utathione peroxi dase acti vity
O B) Inhibi ti on ofgl utathione-S-transferase activi ty
OC) Peroxi dati on of DNA
OD) Peroxi dati on of membrane lipi ds
44. Ei ght hours after ingesting 30 g of acetaminophen in a sui cide attempt, a 17-year-ol d boy is admitted to the
hospital with nausea and vomi ting. If left untreated, the compl ication most likel y to develop in this patient is
O A) cardi ac arrhythmi a
o B) gastrointestinal bleedi ng
OC) hepati c necrosi s
0 D) respi ratory depression
o E) toni c-cloni c sei zures
45. A 3-week-ol d female infant wi th dehydrati on devel ops bilateral renal vein thrombosi s, but only the right kidney
infarcts. Collateral drainage vi a which of the followi ng veins most likely prevented infarction of the left kidney?
O A) Left ovari an
o B) Left third l umbar
OC) Spl eni c
0 D) Superi or mesenteric

46. A 20-year-old man ingests 1 L of water over 1 hour. Whi ch of the following sets of changes is most li kely at the
end of the hour?
Pl asma
Uri ne
ADH
(vasopressi n) osmolali ty
OA)
O B)
OC)
OD)
O E)
O F)
OG)
OH)

T
T
T
T
!
!
!
!

Urine
flow

T
T
!
l
!

l
!
!

47. Phenylketonuri a is an autosomal recessi ve disease. What is the probability that the asymptomatic brother or
si ster of a chil d wi th the disease is a heterozygous carrier?
OA) 0
O B) 1/4
OC) 1/2
OD) 2/3
O E) 3/4

48. A SO-year-old woman comes to the physi cian because of swell ing of her left cheek for 2 weeks. An elevation
wi th a small openi ng in the area shown by the arrow is noted on examination of the oral mucosa. This area is
most l ikely the openi ng of whi ch of the following structures?
OA) Draini ng abscess
O B) Fistul a
O C) Parotid gl and duct
OD) Sublingual gland duct
o E) Submandibular gl and duct

Das könnte Ihnen auch gefallen